Hesi Review 1

¡Supera tus tareas y exámenes ahora con Quizwiz!

The CKMB level would peak within _________ hours after an MI.

12 - 20

Sodium

135 - 145

Rapid acting: onset

15 mins

Hepatitis A: incubation period

15-50 days

A client is currently receiving an infusion labeled Heparin Sodium 25,00 units in 5% Dextrose Injection 500 ml at 14 ml/hour. A prescription is received to change the rate of the infusion to 900 units of Heparin per hour. The nurse should set the infusion pump to deliver how many ml.hour?

18 ml/hour

Rule of Nines: Front trunk

18%

OA's: sulfonylureas

1st gen =Tolbutamide (orinase), chlorpropamide (diabinese). 2nd gen = Glipizide (glucotrol), glimepiride (amaryl)

Long acting: duration

20-26 hr

HCO3

21-28

Blood clots are expected for the first __________ hours following a BPH-correcting surgery

24 - 36

Blood clots are normal for the first _____ to _____ hours after a TURP

24 to 36 Hours

Rapid acting: duration

3 - 5 hrs

Potassium

3.5 - 4.7

Fibrocystic changes are most common between

35 and 50 years old

Diet for DM

45% to 50% carbs 15% to 20% protein 30% or less fat Foods high in complex carbs, high in fiber, low in fat

Platelets

5,000 - 10,000

Clients should void within _____ hours of removal of catheter

6 Hours

A BPH surgical patient is expected to urinate within _____ after removal of catheter

6 hours

To reduce risks for CAD, saturated fats should be limited to ______ of total daily calories.

7%

pH

7.35-7.45

Facial weakness and paralysis could indicate damage to

7th cranial nerve

Intermediate: peak

8 hr

Up to ______ of women experience an intermittent nipple discharge

80%

Calcium

9 - 10.5

Rules of Nines: head/neck

9%

Rule of Nines: upper extremities

9% each 4.5% anterior&posterior

Normal calcium level

9.0-10.5

Lymphocyte

A blood cell that produces antibodies.

Disc

A piece of cartilage between backbones.

Furosemide is prescribed for a 4-year-old who has ventricular septal defect. Which outcome indicates to the nurse that this pharmacological intervention was effective? A. Daily weight decrease of 2 pounds B. Blood urea nitrogen (BUN) increase from 8 to 12 mg/dl C. urine specific gravity change from 1.021 to 1.031 D. urinary output decrease of 5 ml/hour

A. Daily weight decrease of 2 pounds

The mother reports to the nurse that the thick honey-colored crusts on her child's legs began as flat red spots. This is highly indicative of what condition? A. Impetigo B. Eczema C. Ringworm D. Psoriasis

A. Impetigo

Which activity is best for the nurse to initiate with a depressed client? A. Make cut-out cookies B. Play chess C. Play volleyball D. Watch television

A. Make cut-out cookies

The DASH (Dietary Approaches to Stop Hypertension) diet is prescribed for a client with uncontrolled hypertension. Which dietary choices should the nurse instruct the client to eat? A. Shredded wheat B. Avocado salad with olives C. Cheddar cheese D. Canned roasted almonds

A. Shredded wheat

When asked to prioritize nursing actions use the _______ rule

ABC RULE: -Airway -Breathing -Circulation

Symptoms of Crohn's disease

Abdominal pain right LOWER quadrant fatty diarrheal stools constant fluid loss low-grade fever weight-loss

Acute renal failure

Abrupt deterioration of the renal system

OA's: alpha-glucose inhibitors

Acarbose (precose) Miglitol (glyset)

Cholecystitis

Acute inflammation of the gallbladder

Instruct patients with ________ insufficiency to wear a medical alert bracelet and to carry _______ ___________ with them at all times

Adrenal; Simple Carbohydrates

Foods to avoid with pancreatitis

Alcohol caffeine rich foods Fatty or spicy foods

Skin cell

An epithelial cell is a (an)

Total urine output for adults

Approximately 1500 to 2000 ML

A type of breast-conserving surgery for breast cancer

Axillary Node Dissection

In monitoring a client receiving propylthiouracil (PTU) for hyperthyroidism, an increase in which finding indicates that the medication is producing the desired effect? A. Urinary output B. Body weight C. Pulse rate D. Blood pressure

B. Body weight

24. A nurse is preparing to establish oxygen therapy for a patient with COPD, and the physician's prescription reads "oxygen per nasal cannula at 5 L per minute." Which of the following actions should the nurse take? A. Administer the oxygen as prescribed. B. Call the physician and question the correct flow rate of the oxygen. C. Establish the oxygen as prescribed and obtain an ABG. D. Change the delivery device from a nasal cannula to a simple oxygen mask.

B. Call the physician and question the correct flow rate of the oxygen. The nurse should call the physician immediately and question the flow rate for delivery of the oxygen before implementation. Oxygen is used cautiously in patients with COPD because of longstanding hypoxemia serving as the respiratory drive mechanism. If high levels of oxygen are administered, the respiratory drive can be obliterated. Changing the device to a simple oxygen mask may alter the oxygen concentration being delivered to the patient and will further enhance the obliteration of the patient's respiratory drive. Obtaining an ABG sample is not a priority at this time, and the action does not address the validity of the prescribed oxygen dosing for the patient.

At 1000 the healthcare provider prescribes an increase dose in the dosage of a client's loop diuretic from 40 mg to 80 mg a day. The nurse has already administered today's 40 mg dose of the loop diuretic at 0600. Which action should the nurse implement? A. Schedule the second 40 mg of the diuretic for administration at 1800 that evening B. Clarify the start date of the new dose with the healthcare provider C. Administer an additional 40 mg of the diuretic after checking the serum potassium level D. Implement the increase in the prescribed dosage the following morning

B. Clarify the start date of the new dose with the healthcare provider

The nurse administers an oral antiviral to a client with shingles. Which finding is most important for the nurse to report to the healthcare provider? A. elevated liver function tests B. Decreased white blood cell count C. Vomiting and diarrhea D. Pruritus and muscle aches

B. Decreased white blood cell count

Which assessment is most important for the nurse to include in the daily plan of care for a client with a burned extremity? A. Range of motion B. Distal pulse intensity C. Extremity sensation D. Presence of exudate

B. Distal pulse intensity

When completing a mental health assessment, the nurse wishes to obtain information about a client's insight and judgement. What interview technique is most useful in assessing these mental abilities? A. Ask the client to describe what is meant by the proverb, "A penny saved is a penny earned." B. Have the client describe what should be done if a child was observed alone in a locked car on a hot day C. Show the client a list of 3 words a the beginning of the interview, and ask for word recall later. D. Request that the client complete a mathematical operation, such as counting backward by 9s form 100

B. Have the client describe what should be done if a child was observed alone in a locked car on a hot day

The nurse is triaging clients in an urgent car clinic. The client with which symptoms should be referred to the health care provider immediately? A. High fever, skin rash, and a productive cough B. Headache, photophobia, and nuchal rigidity C. Nausea, vomiting, and poor skin turgor D. Malaise, fever, and stiff, swollen joints

B. Headache, photophobia, and nuchal rigidity

In assessing a client at 34-weeks gestation, the nurse notes that she has a slightly elevated total T4 with a slightly enlarged thyroid, a hematocrit of 28%, a heart rate of 92 beats per minute, and a systolic murmur. Which finding requires follow-up? A.Elevated parathyroid hormone level B. Hematocrit of 28% C. Heart rate of 92 beats per minute D. Systolic murmur

B. Hematocrit of 28%

In monitoring a client's respiratory stauts, which symptom is characteristic of early acute (adult) respiratory distress syndrome (ARDS)? A. Coarse breath sounds B. Increased respiratory rate C. Intercostal retractions D. Pleural friction rubs

B. Increased respiratory rate

Sublingual nitroglycerin is administer to a male client with unstable angina who complains of crushing chest pain. Five minutes later the client becomes nauseated and his blood pressure drops to 60/40. Which intervention should the nurse implement. A. Administer a second dose of nitroglycerin B. Infuse a rapid IV normal saline bolus C. Begin external chest compressions D. give a PRN antiemetic medication

B. Infuse a rapid IV normal saline bolus When chest pain is treated with a vasodilator, such as nitroglycerin, and the blood pressure falls to a critical level, a right ventricular infraction may have occurred which requires immediate infusion of IV fluid

8. The nurse receives a physician's order to transfuse fresh frozen plasma to a patient suffering from an acute blood loss. Which of the following procedures is most appropriate for infusing this blood product? A. Hand the fresh frozen plasma as a piggyback to a new bag of primary IV solution without KCl. B. Infuse the fresh frozen plasma as rapidly as the patient will tolerate. C. Hang the fresh frozen plasma as a piggyback to the primary IV solution. D. Infuse the fresh frozen plasma as a piggyback to a primary solution of normal saline.

B. Infuse the fresh frozen plasma as rapidly as the patient will tolerate. The fresh frozen plasma should be administered as rapidly as possible and should be used within 2 hours of thawing. Fresh frozen plasma is infused using any straight-line infusion set. Any existing IV should be interrupted while the fresh frozen plasma is infused, unless a second IV line has been started for the transfusion.

The nurse receives a physician's order to transfuse fresh frozen plasma to a patient suffering from an acute blood loss. Which of the following procedures is most appropriate for infusing this blood product? A. Hand the fresh frozen plasma as a piggyback to a new bag of primary IV solution without KCl. B. Infuse the fresh frozen plasma as rapidly as the patient will tolerate. C. Hang the fresh frozen plasma as a piggyback to the primary IV solution. D. Infuse the fresh frozen plasma as a piggyback to a primary solution of normal saline.

B. Infuse the fresh frozen plasma as rapidly as the patient will tolerate. The fresh frozen plasma should be administered as rapidly as possible and should be used within 2 hours of thawing. Fresh frozen plasma is infused using any straight-line infusion set. Any existing IV should be interrupted while the fresh frozen plasma is infused, unless a second IV line has been started for the transfusion.

When assessing a client who had a supratentorial craniotomy, what action should the nurse implement when determining the client's Glasgow coma scale (GCS) rating? A. Inject cold water into the client's ear. B. Instruct the client to raise an arm C. Determine the intracranial pressure D. Check the patellar and radial reflexes

B. Instruct the client to raise an arm

The nurse is planning discharge care for a male client with metastatic cancer. The client tells the nurse that he plans to return to work despite pain, fatigue, and impending death. Which goal is most important to include in this client's plan of care? A. Implement decisions about future hospice services within the next 3 months B. Maintains pain level below 4 when implementing outpatient pain clinic strategies C. Requests home health care if independence becomes compromised for 5 days D. Arranges for short term counseling if stressors impact work schedule for 2 weeks

B. Maintains pain level below 4 when implementing outpatient pain clinic strategies

The nurse makes a supervisory home visit to observes an unlicensed assistive personnel (UAP) who is providing personal care for a client with Alzheimer's Disease. The nurse observes that whenever the client gets upset, the UAP changes the subject. What action should the nurse take in response to this observation? A. Affirm that the UAP is using an effective strategy to reduce the client's anxiety B. Meet with the UAP later to role model more assertive communication techniques C. Assume care of the client to ensure that the effective communication is maintained D. Tell the UAP to offer more choices during the personal care to prevent anxiety

B. Meet with the UAP later to role model more assertive communication techniques

A male client with hepatitis A is admitted with elevated hepatic enzymes and jaundice. Which intervention should the nurse implement? A. Provide cloth gowns for the client to wear B. Meticulous hand washing after each client contact C. Place the client in strict airborne precautions D. Use plastic utensils with each meal tray

B. Meticulous hand washing after each client contact

Which abnormal lab finding indicates that a client with diabetes needs further evaluation for diabetic nephropathy? A. Hypokalemia B. Microalbuminauria C. Elevated serum lipids D. Ketonuria

B. Microalbuminuria is the earliest sign of nephropathy and indicates the need for follow-up evaluation. Hyperkalemia (A) is associated with end stage renal disease caused by diabetic nephropathy. (C) may be elevated in end stage renal disease. (D) may signal the onset of DKA.

The nurse learns that a client in a semi-private room has a postoperative wound that is colonized with a multi-drug resistant organism. What action should the nurse implement? A. Maintain standard precautions and place the roommate in protective isolation B. Move the client to a private room and implement contact precautions for the client C. Initiate airborne precautions for both clients, leaving the in the semi-private room D. Begin droplet precautions for both clients before moving them to private rooms

B. Move the client to a private room and implement contact precautions for the client

A client with superficial burn to the face, neck. and hands resulting from a house fire is admitted to the burn unit. Which assessment finding indicated to the nurse that the client should be monitored for carbon monoxide poisoning? A. Expiratory stridor and nasal flaring B. Mucus membrane cherry red color C. Carbonaceous particles in sputum D. Pulse oximetry reading of 80 percent

B. Mucus membrane cherry red color

A male client with pneumonia is diaphoretic and confused. The cardiac monitor indicates tachycardia with frequent premature ventricular beats. Atrial blood gas (ABG) results are: pH 7.24, PaCO2 65 mmHg, HCO3 24 mEq/L. Which intervention is most important for the nurse to include in the client's plan of care? A. Obtain a 12 lead electrocardiogram (ECG) daily B. Observe frequently for signs of hypoventilation C. Assess lungs for increasing pulmonary secretions D. Maintain patent IV catheter for antibiotic therapy

B. Observe frequently for signs of hypoventilation Hypoventilation is a critical sign that of pending respiratory failure. The ABG results indicate respiratory acidosis as evidenced by a decrease of pH, elevated PaCO2 and normal HCO3. Management of severe respiratory acidosis is correction of the underlying cause for hypoventilation, which may require mechanical ventilation

The nurse observes that a postoperative client with a continuous bladder irrigation has a large blood clot in the urinary drainage tubing. What action should the nurse perform first? A. Determine the client's blood pressure and apical pulse rate B. Observe the amount of urine in the client's drainage bag C. Obtain a pulse oximeter to assess the client's oxygen saturation D. Review the medication record for recently administered medications

B. Observe the amount of urine in the client's drainage bag

A client with a peripherally inserted central venous catheter (PICC) line has a fever. What client assessment is most important for the nurse to perform? A. Gently palpate the neck for tenderness B. Observe the antecubital fossa for inflammation C. Check for drainage in the subclavian area D. Measure for any increase in abdominal girth

B. Observe the antecubital fossa for inflammation Peripherally inserted central catheters are threaded into the basilic vein at the antecubital area. If the client develops a fever, the nurse should assess for signs of an infection at the insertion site of the catheter

The nurse is assessing an infant on admission to the newborn nursery and finds that both brachial pulses are bounding, but bilateral femoral pulses are only slightly palpable. Which assessment should the nurse implement next? A. Assess heart sounds for a murmur B. Obtain blood pressures in all extremities C. Listen to the lung fields for fine crackles D. Elevate the legs for evidence of edema

B. Obtain blood pressures in all extremities

The nurse manager is concerned about the number of falls that have occurred on the unit in the last month. Which action is most likely to decrease the number of falls? A. Place all clients on the unit regardless of age at risk for falls B. Obtain the evidence based practice guidelines for fall prevention C. Determine if pain medication is related to those who fell D. Inquire about what other units are doing to prevent falls

B. Obtain the evidence based practice guidelines for fall prevention

The nurse is using a straight urinary catheter kit to collect a sterile urine specimen firm a female client. After positioning and prepping the client, rank the actions in the sequence they should be implemented. A. Don sterile gloves and prepare the sterile field. B. Open the sterile catheter kit close to the client's perineum C. Cleanse the urinary meatus using the solution, swabs, and forceps provided D. Place distal end of the catheter in sterile specimen cup and insert catheter into meatus

B. Open the sterile catheter kit close to the client's perineum A. Don sterile gloves and prepare the sterile field. C. Cleanse the urinary meatus using the solution, swabs, and forceps provided D. Place distal end of the catheter in sterile specimen cup and insert catheter into meatus

A client with metastatic cancer who was taking hydromorphone (Dilaudid) PO at home is now receiving the medication IV while in the hospital. To evaluate if the client is receiving an equianalgesic dose of the Dilaudid, what assessment should the nurse complete? A. Respiratory rate B. Pain scale C. Level of consciousness D. Blood pressure

B. Pain scale

The nurse has received funding to design a health promotion project for African-American women who are at risk for developing breast cancer. Which resource is most important in designing this program? A. A lasting of African-American women who live in the community B. Participation of community leaders in planning the program C. Morbidity data for breast cancer in women of all races D. Techical assistacne to produce a video on breast self-examination

B. Participation of community leaders in planning the program

A clients morning laboratory test results include leukocytes 3,500/mm^3 or 3.5x10^9/L (SI). Based on the laboratory result, which complaint is this client most likely to report to the nurse? A. Inability to walk without shortness of breath B. Persistent cough with yellow-colored sputum C. Superficial cuts do not readily stop bleeding D. A red streak and pain in right calf muscle

B. Persistent cough with yellow-colored sputum The leukocyte level is below normal (WBC 5,000-10,000). WBC are integral to the body's response to infection, so the client's report of yellow-colored sputum is characteristic of purulent sputum, a sign of infection

When administering medications, when is the last opportunity for the nurse to discover a near-miss medication error? A. During medication administration documentation B. When determining the client'd identity at the bedside C. After the dose is dispensed by the computer-controlled system D. As the prescription is verified with the client's medical record

B. When determining the client'd identity at the bedside

18. Nursing interventions for the patient with aplastic anemia are directed toward the prevention of the complications of A. fatigue and dyspnea. B. hemorrhage and infection. C. thromboemboli and gangrene. D. cardiac arrhythmias and heart failure.

B. hemorrhage and infection. Hemorrhage from thrombocytopenia and infection from neutropenia are the greatest risks for the patient with aplastic anemia. The patient will experience fatigue from anemia, but bleeding and infection are the major causes of death in aplastic anemia.

A client has been taking oral corticosteroids for the past five days because of seasonal allergies. Which assessment finding is of most concern to the nurse? A) White blood count of 10,000 mm3. B) Serum glucose of 115 mg/dl. C) Purulent sputum. D) Excessive hunger.

C) Purulent sputum. Steroids cause immunosuppression, and a purulent sputum (C) is an indication of infection, so this symptom is of greatest concern. Oral steroids may increase (A) and often cause (D). (B) may remain normal, borderline, or increase while taking oral steroids.

What is the correct location for the placement of the hand for manual chest compressions during CPR on the adult client. A. Just above the xiphoid process on the upper third of the sternum. B. Below the xiphoid process midway between the sternum and the umbilicus. C. Just about the xiphoid process on the lower third of the sternum. D. Below the xiphoid process midway between the sternum and the first rib.

C.

A home health nurse is visiting a client with a history of heart failure (HF). When interviewing the client, which question provides the most useful information for the nurse? A. "Have you been weighing yourself once a month?" B. "Have you had any headaches lately?" C. "How many pillows do you sleep on at night?" D. "How much caffeine are you drinking?"

C. "How many pillows do you sleep on at night?"

The nurse is evaluating the health teaching of a female client with condyloma acuminata. Which statement by the client indicates that teaching has been effective? A. "These warts are caused by a fungus" B. "Early treatment is very effective" C. "I need to have regular pap smears" D. "I will clean my hot tub better"

C. "I need to have regular pap smears"

A male client with bipolar disorder has difficulty concentrating and plans to attend group for the first time. He tells the nurse that he will try to stay for the music relaxation group. After 20 minutes in the group, he becomes restless and begins to leave. What should the nurse do? A. Ask the client to stay until the end B. Encourage the client to go to another group C. Allow the client to leave the group D. Offer the client an antianxiety medication

C. Allow the client to leave the group

During assessment of a 2-year-old infant, the nurse notices a bluish-black discoloration over the lumbosacral area. Which action should the nurse take? A. Ask the mother about the discoloration B. Report possible child abuse to protective services C. Document the finding in the report D. Gently rub the area with skin cream to promote healing

C. Document the finding in the report

While completing an admission assessment for a client with unstable angina, which closed ended question should the nurse ask about the client's chest pain? A. Tell me about the activities that cause your pain? B. When did you first notice the pain in your chest? C. Does your pain occur when walking short decrease? D. How do you feel when the pain becomes noticeable?

C. Does your pain occur when walking short decrease?

A client admitted with an acute myocardial infraction receives a cardiac diet with sodium restriction and complains that his hamburger is flavorless. Which condiment should the nurse offer? A. Pickle relish B. Steak sauce C. Fresh horseradish D. Tomato ketchup

C. Fresh horseradish

The nurse and a social worker are talking when a male client with psychosis angrily shouts at the nurse, "Stop talking about me." The nurse should document the client is exhibiting which symptom? A. Auditory hallucinations B. Visual hallucinations C. Ideas of reference D. Thought broadcasting

C. Ideas of reference

To assess for the presence of diaphragmatic breathing, what action should the nurse take? A. Attach an apnea monitor to the chest wall B. Auscultate the lung bases anteriorly C. Observe the movement of the abdomen D. Note any gaps between respirations

C. Observe the movement of the abdomen

The nurse determines that a patient is experiencing common adverse effects from the inhaled corticosteroid beclomethasone (Beclovent) after noting which of the following? A. Adrenocortical dysfunction and hyperglycemia B. Elevation of blood glucose and calcium levels C. Oropharyngeal candidiasis and hoarseness D. Hypertension and pulmonary edema

C. Oropharyngeal candidiasis and hoarseness Oropharyngeal candidiasis and hoarseness are common adverse effects from the use of inhaled corticosteroids because the medication can lead to overgrowth of organisms and local irritation if the patient does not rinse the mouth following each dose.

A man who has a known problem with alcohol is accused of stealing from his employer. When he returns home that evening, he accuses his son of stealing from school, and physically abuses the child for what the father describes as the child's dishonest behavior. Which two defense mechanisms are being used by the father? A. Sublimation and displacement B. Denial and sublimation C. Projection and displacement D. Projection and denial

C. Projection and displacement

26. Which of the following conditions is manifested by unexplained shortness of breath and a high mortality rate? A. Bleeding ulcer B. Transient ischemia C. Pulmonary embolism D. MI

C. Pulmonary embolism A high mortality rate is associated with a pulmonary embolism. A pulmonary embolism is an obstruction of the pulmonary artery caused by an embolus. It presents with hypoxia, anxiety, restlessness, and shortness of breath. Bleeding ulcers, MI, and transient ischemia are not associated with such a high mortality rate.

Which of the following conditions is manifested by unexplained shortness of breath and a high mortality rate? A. Bleeding ulcer B. Transient ischemia C. Pulmonary embolism D. MI

C. Pulmonary embolism A high mortality rate is associated with a pulmonary embolism. A pulmonary embolism is an obstruction of the pulmonary artery caused by an embolus. It presents with hypoxia, anxiety, restlessness, and shortness of breath. Bleeding ulcers, MI, and transient ischemia are not associated with such a high mortality rate.

The nurse is preparing a 50 ml dose of 50% Dextrose IV for a client with insulin shock. How should the nurse administer the medication? A. Dilute Dextrose in one liter of 0.9% Normal Saline solution B. Mix Dextrose in a 50 ml piggyback for a total volume of 100 ml. C. Push the undiluted Dextrose slowly through the currently infusing IV D. Ask the pharmacist to add the Dextrose to a TPN solution

C. Push the undiluted Dextrose slowly through the currently infusing IV

The nurse administers the opioid antagonist naloxone HCL (Narcan) to a young adult client who overdosed on hydromorphone (Dilaudid). Which assessment data indicates that the naloxone is effective? A. Babinski reflex changes from negative to positive B. Pupil size increases from 4 mm to 6 mm C. Respiration increases from 8/minute to 12/minute D. Blood pressure decrease from 140/94 to 120/78

C. Respiration increases from 8/minute to 12/minute

A client with chronic kidney disease is being discharged with continuous ambulatory peritoneal dialysis (CAPD). What is the priority nursing diagnosis the nurse should use when developing a discharge teaching plan for this client? A. Altered nutriton B. Impaired mobility C. Risk for sepsis D. Risk for injury

C. Risk for sepsis

A client is admitted to a medical unit with a diagnosis of gastritis and chronic heavy alcohol abuse. What should the nurse administer to prevent the development of Wernicke's syndrome? A. Lorazepam (Ativan) B. Famotidine (Pepcid) C. Thiamine (Vitamin B1) D. Atenolol (Tenormin)

C. Thiamine (Vitamin B1)

A client with a new diagnosis of Raynaud's disease lives alone. Which instruction should the nurse include in the client's discharge teaching plan? A. have a caregiver for 8 hours daily. B. develop a walking exercise routine C. keep room temperature at 80F D. Wear TED stockings at night

C. keep room temperature at 80F keeping the environment warm may minimize vasoconstriction, which decreases blood flow and causes the pain associated with Raynaud's disease.

The nurse is preparing an older male client with Parkinson's disease for discharge to home. Which instruction should the nurse provide to promote independence and reduce risk for injury? (select all that apply) A. use a long stride when walking B. try to swing arms while walking C. take scheduled medication on time D. place small rugs over tile flooring E. consider use of recliner lift chair

C. take scheduled medication on time E. consider use of recliner lift chair

14. Upon entering the room of a patient who has just returned from surgery for total laryngectomy and radical neck dissection, a nurse should recognize a need for intervention when finding A. a gastrostomy tube that is clamped. B. the patient coughing blood-tinged secretions from the tracheostomy. C. the patient positioned in a lateral position with the head of the bed flat. D. 200 ml of serosanguineous drainage in the patient's portable drainage device.

C. the patient positioned in a lateral position with the head of the bed flat. After total laryngectomy and radical neck dissection, a patient should be placed in a semi-Fowler's position to decrease edema and limit tension on the suture line.

Irritability and restlessness are early signs of:

Cerebral Hypoxia

PEG Tube Care

Check tube placement and residual volume q4h

Protein must also be restricted in?

Chronic Renal Failure

Tissue destruction results from

Coagulation protein denaturation ionization of cellular contents

Ulcerative Colitis

Colon only Tenesmus Pain Big Cancer Risk Continuous Lesions Diarrhea & Anemia common Surgical Cure

Hepatitis C: source of infection

Contaminated blood products, contaminated needles, IV drug use, dialysis

Hepatitis B: source of infection

Contaminated blood products, mother to child at birth, contaminated needles

Hepatitis A: source of infection

Contaminated food, water or shellfish

Fluid-filled Movable 2-3 cm

Cysts

A client has undergone insertion of a permanent pacemaker. When developing a discharge teaching plan, the nurse writes a goal of, "The client will verbalize symptoms of pacemaker failure." Which symptoms are most important to teach the client? A) Facial flushing. B) Fever. C) Pounding headache. D) Feelings of dizziness.

D) Feelings of dizziness. Feelings of dizziness may occur as the result of a decreased heart rate, leading to decreased cardiac output (D). (A and C) will not occur as the result of pacemaker failure. (B) may be an indication of infection postoperatively, but is not an indication of pacemaker failure.

While the school nurse is teaching a group of 14-year-olds, one of the participants remarks, "You are too young to be our teacher! You're not much older than we are!" How should the nurse respond? A. "I think I am qualified to teach this group." B. "How old do you think I am?" C. "Do you think you can teach it any better?" D. "We need to stay focused on the topic."

D. "We need to stay focused on the topic."

The nurse assesses the perineum of a client who is complaining of perineal pain 6 hours after a normal delivery, and finds small perineal (vulvar) hematomas. Based on this assessment finding, which treatment should the nurse implement? A. Cleanse the area with warm water B. Prepare for surgical excision C. Spray topical analgesic to the perineum D. Apply ice packs on the perineum

D. Apply ice packs on the perineum

39. The nurse is scheduled to give a dose of ipratropium bromide by metered dose inhaler. The nurse would administer the right drug by selecting the inhaler with which of the following trade names? A. Vanceril B. Pulmicort C. AeroBid D. Atrovent

D. Atrovent The trade or brand name for ipratropium bromide, an anticholinergic medication, is Atrovent.

Dinoprostone (Prostin E-2) is prescribed for primigravida who had a missed spontaneous abortion. An increase in which finding should the nurse expect? A. Maternal temperature B. Rh antibody production C. Hemoglobin (Hgb) levels D. Contractions of the uterus

D. Contractions of the uterus

A client who is admitted to the hospital is suspected of having meningitis. The nurse should plan to prepare the client for which diagnostic test? A. CT scan of brain B. Electroencephalogram (EEG) C. Synovial fluid analysis D. Lumbar puncture

D. Lumbar puncture

A female client who has been taking diclofenac (Zipsor) for the past month is admitted with right upper quadrant tenderness, jaundice, and flu-like symptoms. She is also complaining of fatigue, diarrhea, and pruritus. Which intervention is most important to include in this client's plan of care? A. Review results of serum protein electrolytes B. Determine frequently of indigestion C. Evaluate intake and output ratios D. Monitor serum bilirubin levels

D. Monitor serum bilirubin levels

The nurse is assessing a 9-year-old boy who is experiencing an acute asthma attack. When auscultating this child's breaths sounds, which finding is the nurse most likely to obtain? A. Diminished breath sounds heard throughout all areas B. Fine crackles upon inspiration C. Louder breath sounds over the lower lung fields D. Musical sounds upon expiration

D. Musical sounds upon expiration

Following laser trabeculoplasty surgery for open-angle glaucoma, the client reports acute pain deep within the eye. What action should the nurse take? A. Apply bilateral eye shields to reduce photosensitivity B. Administer an antiemetic to prevent vomiting C. Begin postoperative prophylactic antibiotics D. Report the complaint of eye pain to the surgeon

D. Report the complaint of eye pain to the surgeon

Symptoms of G.I. bleeding

Dark tarry stool's coffee ground emesis bright red rectal bleeding fatigue Pallor Severe abdominal pain Decreased BP rapid pulse

Necr/o

Death

Cirrhosis

Degeneration of liver tissue causing enlargement, fibrosis and scarring

Acute pancreatitis

Digestion of the pancreas by it's own enzymes, primarily trypsin

Treatment for cholelithiasis

Dissolution therapy Endoscopic retrograde cholangiopancreatography Lithotripsy

The first type of drugs for managing HTN

Diuretics

Hyponatremia: causes

Diuretics GI fluid loss diaphoresis

Diverticular disease

Diverticulosis diverticulitis

Common symptoms of endometriosis

Dysmenorrhea Dyspareunia Diff/Painful Defecation Infertility

HESI hint

Electrolytes are profoundly affected by kidney problems. Sodium and chloride extracellular ions. potassium and phosphate intracellular ions.

Lab data for cirrhosis

Elevated bilirubin, AST, AL T, alkaline phosphatase, PT and ammonium. Decreased Hgb, HCT, electrolytes and Albumin

Topical substances that soften the skin; Examples: Mineral Oil, Cetyl Alcohol, Stearyl Alcohol, Lubriderm

Emollients

Blue is to chronic bronchitis as Pink is to _____________.

Emphysema

Pink Puffer

Emphysema

When a cancer patient receiving chemo, complains of stomatitis, the nurse should

Encourage frequent rinsing with salt or saline Never alcohol-based products because it's very irritating

Acromegaly

Enlargement of extremities after puberty due to pituitary gland problem.

Benign prostatic hyperplasia

Enlargement or hyper trophy of the prostate.

Leukocyte

Eosinophil is a (an)

The tumors for pheochromosytoma produce, store, and release:

Epinephrine and Norepinephrine

These type of burns destroy the entire dermal layer

Full-Thickness

DNA

Genes are composed of?

PSA levels can be elevated with

Gland enlargement and/or cancer

Manifestations of BPH include:

Hesitancy when starting urination Decrease in size/force of stream Frequency and Nocturia

Hyperglycemia

High blood glucose happens when the body has too little insulin or when the body can't use insulin properly.

Diet for hyperthyroidism

High calorie High protein low caffeine Low Fiber

Hyperthermia, Goiter, Exophthalmos, Agitation, Anxiety---- are just some of the manifestations of

Hyperthyroidism

Respiratory alkalosis: cause

Hyperventilation response to acidosis bacteremia thyrotoxicosis fever Hepatic failure hysteria

Craniotomy

Incision of the skull.

Intermediate acting insulin

Isophane insulin

What is the point of restricting salt for a patient experiencing exophthalmos related to hyperthyroidism?

It prevents periorbital edema

It's not uncommon for a patient with SIADH to manifest the following symptoms

Loss of thirst Weight Gain Irritability Muscle Weakness Decreased LOC

Diagnosis of hypothyroidism

Low T3 below 70 Low T4 below 5 Presence of T4 antibody

Supine

Lying on the back.

Hyperkalemia: signs and symptoms

Muscle weakness bradycardia dysrhythmias Flaccid paralysis K > 5.0

Vaccine for hepatitis

Only hep A & B

Hepatitis A: route of infection

Oral Fecal

The most predictive factors for development of breast cancer are

Over 40 years old Positive family hx

If a patient has severe anemia, HGB <6, the nurse should anticipate an order for

PRBC transfusion

When a patient is taking hyperlipidemia meds, like Questran, it's important to monitor:

PT/INR to alert of Vitamin K deficiencies

The first factor a nurse should assess before teaching

Patient's willingness to learn

Antagonist of heparin:

Protamine Sulfate

Liver

RUQ contain the _______.

Urine output in first 72 hours after injury

Radically decreased with increased specific gravity

Urine output 72 hours to two weeks after injury

Radically increased

Histamine 2 antagonists

Ranitidine (Zantac) Cimetidine (Tagamet) Famotidine (Pepcid)

Type 2 DM

Rare development of ketoacidosis with extreme hyperglycemia HHNKS develops

The nurse initiates neurologic checks for a client who is at risk for neurologic compromise. Which manifestation typically provides the first indication of altered neurologic function? A. Change in level of consciousness B. Increasing muscular weakness C. Changes in pupil size bilaterally D. Progressive nuchal rigidity

Rationale: A A decrease or change in the level of consciousness (A) is usually the first indication of neurologic deterioration. (B and C) may also occur but are much less likely to be the first sign of neurologic compromise. (D) is often a sign of meningitis.

Which consideration is most important when the nurse is assigning a room for a client being admitted with progressive systemic sclerosis (scleroderma)? A. Provide a room that can be kept warm. B. Make sure that the room can be kept dark. C. Keep the client close to the nursing unit. D. Select a room that is visible from the nurses' desk.

Rationale: A Abnormal blood flow in response to cold (Raynaud's phenomenon) is precipitated (A) in clients with scleroderma. (B) is not a significant factor. Stress can also precipitate the severe pain of Raynaud's phenomenon, so a quiet environment is preferred to (C), which is often very noisy. (D) is not necessary.

The nurse is performing hourly neurologic checks for a client with a head injury. Which new assessment finding warrants immediate intervention by the nurse? A. A unilateral pupil that is dilated and nonreactive to light B. Client cries out when awakened by a verbal stimulus C. Client demonstrates a loss of memory of the events leading up to the injury D. Onset of nausea, headache, and vertigo

Rationale: A Any change in pupil size and reactivity is an indication of increasing intracranial pressure and should be reported to the health care provider immediately (A). (B) is a normal response to being awakened. (C and D) are common manifestations of head injury and are of less immediacy than (A).

A client is being discharged following radioactive seed implantation for prostate cancer. What is the most important information that the nurse should provide to this client's family? A. Follow exposure precautions. B. Encourage regular meals. C. Collect all urine. D. Avoid touching the client.

Rationale: A Clients being treated for prostate cancer with radioactive seed implants should be instructed regarding the amount of time and distance needed to prevent excessive exposure (A) that would pose a hazard to others. (B) is a good suggestion to promote adequate nutrition but is not as important as (A). (C) is unnecessary. Contact with the client (D) is permitted but should be brief to limit radiation exposure

The nurse is caring for a client who is 1 day post-acute myocardial infarction. The client is receiving oxygen at 2 L/min via nasal cannula and has a peripheral saline lock. The nurse notes that the client is having eight premature ventricular contractions (PVCs) per minute. Which intervention should the nurse implement first? A. Obtain an IV pump for antiarrhythmic infusion. B. Increase the client's oxygen flow rate. C. Prepare for immediate countershock. D. Gather equipment for endotracheal intubation.

Rationale: B Increasing the oxygen flow rate (B) provides more oxygen to the client's myocardium and may decrease myocardial irritability as manifested by the frequent PVCs. (A) can be delegated and is a lower priority action than (B). Defibrillation may eventually be necessary, but (C) is not the immediate treatment for frequent PVCs. (D) may become necessary if the client stops breathing, but is not indicated at this time.

Which condition should the nurse anticipate as a potential problem in a female client with a neurogenic bladder? A. Stress incontinence B. Infection C. Painless gross hematuria D. Peritonitis

Rationale: B Infection (B) is the major complication resulting from stasis of urine and subsequent catheterization. (A) is the involuntary loss of urine through an intact urethra as a result of a sudden increase in intraabdominal pressure. (C) is the most common symptom of bladder cancer. (D) is the most common and serious complication of peritoneal dialysis.

Short acting insulin

Regular insulin

Critical systems affected include

Respiratory integumentary cardiovascular renal G.I. neurologic

Type 1 DM

Results from B cell destruction

-stomy

Surgical creation of a permanent opening to the outside of the body.

Catabolism

The process by which food is burned to realease energy.

Treatment for hyperthyroidism

Thyroid ablation by medication, radiation, thyroidectomy, adenectomy of portion of anterior pituitary where TSH producing tumor is located = hormone replacement

Addison crisis

Vascular collapse hypoglycemia Essential to reversing the crisis aldosterone replacement

deviated septum

a deflection of the normally straight nasal septum.

wheezes

a form of rhonchus characterized by continuous high-pitched squeaking sound caused by rapid vibration of bronchial walls.

pneumoconiosis

a general term for lung diseases caused by inhalation and retention of dust particles.

surfactant

a lipoprotein that lowers the surface tension in the alveoli, reduces the amount of pressure needed to inflate the alveoli, and decreases the tendency of the alveoli to collapse.

The inflammation associated with diverticulitis would likely cause

a low grade fever

community-acquired pneumonia

a lower respiratory tract infection of the lung parenchyma with onset in the community or during the first 2 days of hospitalization.

hemothorax

accumulation of blood in the pleural space.

To determine a woman's fertile period, the nurse would require

an accurate record of the # of days of cycle for the past 6 months

PSA can detect

cancer, but is not a definitive diagnosis

pentobarbital (Neubatal sodium)

contriandicated with liver damage

As people age, the specific gravity of urine tends to

decline

Those that take oral contraceptives are at increased risk for

developing blood clots

esophagogastromy

esophageal cancer risk for infection = meticulious oral care should be provided several times a day prior to surgery

Treatment decisions are based on whether a breast tumor has

estrogen receptors

Corticosteroids may cause

excessive hunger

hyperresponsiveness

excessive or exaggerated response to a stimulus; in asthma leads to bronchoconstriction in response to physical, chemical, or pharmacologic stimuli.

Check placement of an NG Tube by

expiring upon insertion

Primary Radiation uses ________ beams.

external

Colostomy has

formed stool, and smells

diverticulitits

hard ridgid abdomen & elevated WBC = peritonitis = medical emergency should be reported to PCP immediately s/s left lower quadrant pain; elevated temperature; refusing to eat; nausea

Hepatitis B

health care providers should have Hep B vaccine; transmitted by fecal/oral contamination

PVD peripheral vascular disease

help client dangle legs

Steroids cause

immunosuppression

Because of the diuretic effect of Lasix, a patient should be instructed to take it

in the morning, to avoid nocturia

A sign of ascites

increase in abdominal girth

The primary cause of hepatic encephalopathy

increase in serum ammonia level

To reduce cardiac risk factors, a patient should

increase intake of soluble fiber to 10-25 grams per day

This is contraindicated in ACUTE prostatitis patients

insertion of a catheter

flail chest

instability of the chest wall resulting from multiple rib fractures.

NG Tube

no drainage in 2 hours client nausated = reposition client on side

hypercalcemia

positive trousseau sign = carpal spasm

trigeminal neuralgia (5th crainal nerve)

sudden stabbing severe pain over the lip and chin

Trigeminal nerve damage would manifest as

sudden, stabbing, severe pain over the lip and chin

telemetry

ventrical fibrillation = life threatening start CPR

A client receiving cholestyramine (Questran) for hyperlipidemia should be evaluated for what vitamin deficiency? A) K. B) B12. C) B6. D) C.

A) K. Clients should be monitored for an increased prothrombin time and prolonged bleeding times which would alert the nurse to a vitamin K deficiency (A). These drugs reduce absorption of the fat soluble (lipid) vitamins A, D, E, and K. (B, C, and D) are not fat soluble vitamins.

A client is admitted to the hospital with a diagnosis of severe acute diverticulitis. Which assessment finding should the nurse expect this client to exhibit? A) Lower left quadrant pain and a low-grade fever. B) Severe pain at McBurney's point and nausea. C) Abdominal pain and intermittent tenesmus. D) Exacerbations of severe diarrhea.

A) Lower left quadrant pain and a low-grade fever. Left lower quadrant pain occurs with diverticulitis because the sigmoid colon is the most common area for diverticula, and the inflammation of diverticula causes a low-grade fever (A). (B) would be indicative of appendicitis. (C and D) are symptoms exhibited with ulcerative colitis.

During assessment of a client with amyotrophic lateral sclerosis (ALS), which finding should the nurse identify when planning care for this client? A) Muscle weakness. B) Urinary frequency. C) Abnormal involuntary movements. D) A decline in cognitive function.

A) Muscle weakness. Amyotrophic lateral sclerosis (ALS) is characterized by a degeneration of motor neurons in the brainstem and spinal cord and are manifested by muscle weakness (A) and wasting. ALS does not manifest (B and C). In ALS, the client remains cognitively intact, not (D), while the physical status deteriorates.

Which assessment finding by the nurse during a client's clinical breast examination requires follow-up? A) Newly retracted nipple. B) A thickened area where the skin folds under the breast. C) Whitish nipple discharge. D) Tender lumpiness noted bilaterally throughout the breasts.

A) Newly retracted nipple. A newly retracted nipple (A), compared to a life-long finding, may be an indication of breast cancer and requires additional follow-up. The inframammary ridge (B) is a normal anatomic finding. Up to 80% of women may experience an intermittent nipple discharge (C), especially related to recent stimulation, and in most cases, nipple discharge is not related to malignancy. (D) is a classic finding for fibrocystic breast disease, a benign condition.

When teaching diaphragmatic breathing to a client with chronic obstructive pulmonary disease (COPD), which information should the nurse provide? A) Place a small book or magazine on the abdomen and make it rise while inhaling deeply. B) Purse the lips while inhaling as deeply as possible and then exhale through the nose. C) Wrap a towel around the abdomen and push against the towel while forcefully exhaling. D) Place one hand on the chest, one hand the abdomen and make both hands move outward.

A) Place a small book or magazine on the abdomen and make it rise while inhaling deeply. Diaphragmatic or abdominal breathing uses the diaphragm instead of accessory muscles to achieve maximum inhalation and to slow the respiratory rate. The client should protrude the abdomen on inhalation and contract it with exhalation, so (A) helps the client visualize the rise and fall of the abdomen. The client should purse the lips while exhaling, not (B). (C and D) are ineffective.

A splint is prescribed for nighttime use by a client with rheumatoid arthritis. Which statement by the nurse provides the most accurate explanation for use of the splints? A) Prevention of deformities. B) Avoidance of joint trauma. C) Relief of joint inflammation. D) Improvement in joint strength.

A) Prevention of deformities. Splints may be used at night by clients with rheumatoid arthritis to prevent deformities (A) caused by muscle spasms and contractures. Splints are not used for (B). (C) is usually treated with medications, particularly those classified as non-steroidal antiinflammatory drugs (NSAIDs). For (D), a prescribed exercise program is indicated.

A client with heart disease is on a continuous telemetry monitor and has developed sinus bradycardia. In determining the possible cause of the bradycardia, the nurse assesses the client's medication record. Which medication is most likely the cause of the bradycardia? A) Propanolol (Inderal). B) Captopril (Capoten). C) Furosemide (Lasix). D) Dobutamine (Dobutrex).

A) Propanolol (Inderal). Inderal (A) is a beta adrenergic blocking agent, which causes decreased heart rate and decreased contractility. Neither (B), an ACE inhibitor, nor (C), a loop diuretic, causes bradycardia. (D) is a sympathomimetic, direct acting cardiac stimulant, which would increase the heart rate.

Small bowel obstruction is a condition characterized by which finding? A) Severe fluid and electrolyte imbalances. B) Metabolic acidosis. C) Ribbon-like stools. D) Intermittent lower abdominal cramping.

A) Severe fluid and electrolyte imbalances. Among the findings characteristic of a small bowel obstruction is the presence of severe fluid and electrolyte imbalances (A). (B, C, and D) are findings associated with large bowel obstruction.

When assessing a patient's sleep-rest pattern related to respiratory health, the nurse would ask if the patient: (Select all that apply.) A. Has trouble falling asleep B. Awakens abruptly during the night C. Sleeps more than 8 hours per night D. Has to sleep with the head elevated

A,B,D The patient with sleep apnea may have insomnia and/or abrupt awakenings. Patients with cardiovascular disease (e.g., heart failure that may affect respiratory health) may need to sleep with the head elevated on several pillows (orthopnea). Sleeping more than 8 hours per night is not indicative of impaired respiratory health.

When admitting a 45-year-old female with a diagnosis of pulmonary embolism, the nurse will assess the patient for which of the following risk factors? (Select all that apply.) A. Obesity B. Pneumonia C. Hypertension D. Cigarette smoking

A,C,D Research has demonstrated an increased risk of pulmonary embolism in women associated with obesity, heavy cigarette smoking, and hypertension. Other risk factors include immobilization, surgery within the last 3 months, stroke, history of DVT, and malignancy.

Which of the following instructions are most appropriate in the home management of a patient who has undergone surgery for oral cancer? A. "You should drink plenty of fluids and eat foods you enjoy." B. "It is normal to have some leakage of saliva from the suture line." C. "Lying in a prone position helps decrease swelling at the suture line." D. "You should avoid foods high in protein while your suture line is healing."

A. "You should drink plenty of fluids and eat foods you enjoy." For patients who have undergone treatment for head and neck cancers, maintaining adequate nutrition is a challenge. The nurse encourages the patient to increase fluids to prevent dehydration and liquefy secretions. These patients are more likely to eat foods that they enjoy and can tolerate.

The nurse is observing an unlicensed assistive personnel (UPA) who is performing morning care for a bedfast client with Huntington disease. Which care measure is most important for the nurse to supervise? A. Oral care B. Bathing C. Foot care D. Catheter care

A. A client with Huntington disease experiences problems with motor skills such as swallowing and is at high risk for aspiration. (B, C, D) do not pose life-threatening consequences.

34. The patient has an order for each of the following inhalers. Which of the following should the nurse offer to the patient at the onset of an asthma attack? A. Albuterol (Proventil) B. Beclomethasone (Beclovent) C. Ipratropium bromide (Atrovent) D. Salmeterol (Serevent)

A. Albuterol (Proventil) Albuterol is a short-acting bronchodilator that should initially be given when the patient experiences an asthma attack.

Following a patient's bone marrow aspiration, which of the following nursing interventions should a nurse anticipate? A. Application of firm pressure to the site B. Positioning the patient in a prone position C. Positioning the patient in a supine position D. Application of a warm, moist compress to the site

A. Application of firm pressure to the site After a bone marrow aspiration procedure, a nurse should apply pressure to the aspiration site until bleeding stops. Application of a warm, moist compress will not alter the potential for bleeding. Positioning the patient to assume a supine or prone position also will not address the need to control bleeding from the aspiration site.

An elderly male client tells the nurse that he often wakes up during the night. What action should the nurse implement first? A. Ask the client to describe what happens when he awakens B. Encourage the client to describe his bedtime routines and habits C. Instruct the client to keep a daily sleep journal for one week D. Reassure the client that sleep needs often decrease with age

A. Ask the client to describe what happens when he awakens

An unconscious client is admitted to the intensive care unit and is placed on a ventilator. The ventilator alarms continuously and the client's oxygen saturation level is 62%. What action should the nurse take first? A. Call respiratory therapy B. Begin manual ventilation immediately C. Monitor oxygen saturation levels q5 minutes D. Silence the alarm and call the technician

A. Call respiratory therapy

During morning rounds, the nurse finds a client who has no spontaneous respirations and does not respond to shaking. The nurse activates the "Code-Blue" system. While waiting for the code team to arrive, what action should the nurse implement? A. Check the pulse for 10 seconds and begin chest compressions B. Check the airway for a foreign body and remove it if visualized C. Assess blood pressure and pupillary response to light D. Seal the face mask of an Ambu bag over the client's mouth and nose

A. Check the pulse for 10 seconds and begin chest compressions

The nurse plans to obtain a urine specimen for culture from a client's indwelling catheter. The nurse enters the room with the syringe and notes that there is 100 ml of urine in the drainage bag, but no urine is in the tubing. What action should the nurse take? A. Clamp the tubing until urine is observed in the tubing B. Obtain sterile normal saline to irrigate the catheter C. Separate the tubing from the catheter and withdraw a urine specimen D. Remove the urine specimen from the drainage bag

A. Clamp the tubing until urine is observed in the tubing

One year after diagnosed with Pneumocystis carinii pneumonia, a client is admitted with respiratory failure. Respirations are shallow with periods of apnea. After the healthcare provider delivers a grim prognosis to the client's family, which intervention should the nurse implement first? A. Clarify client's end of life wishes B. Review client's CD4 cell count C. Obtain arterial blood gases D. Assist with insertion of an airway

A. Clarify client's end of life wishes

When initially teaching a patient the supraglottic swallow following a radical neck dissection, with which of the following foods should the nurse begin? A. Cola B. Applesauce C. French fries D. White grape juice

A. ColaWhen learning the supraglottic swallow, it may be helpful to start with carbonated beverages because the effervescence provides clues about the liquid's position. Thin, watery fluids should be avoided because they are difficult to swallow and increase the risk of aspiration. Nonpourable pureed foods, such as applesauce, would decrease the risk of aspiration, but carbonated beverages are the better choice to start with.

A client who is 3 weeks postpartum is brought to the mental health unit by her husband for admission because she has been verbalizing that baby is evil. After an assessment interview, the nurse determines the client thinks that the baby is going to bring harm to the other children. How should the nurse document the client's altered thought process? A. Delusional thoughts B. Visual hallucinations C. Ideas of reference D. Nihilistic ideas

A. Delusional thoughts

3. When caring for a patient with metastatic cancer, the nurse notes a hemoglobin level of 8.7 g/dl and hematocrit of 26%. The nurse would place highest priority on initiating interventions that will reduce which of the following? A. Fatigue B. Thirst C. Headache D. Abdominal pain

A. Fatigue The patient with a low hemoglobin and hematocrit (normal values approximately 13.5% to 17% and 40% to 54%, respectively) is anemic and would be most likely to experience fatigue. This symptom develops because of the lowered oxygen-carrying capacity that leads to reduced tissue oxygenation to carry out cellular functions.

The nurse is caring for a patient admitted to the hospital with pneumonia. Upon assessment, the nurse notes a temperature of 101.4° F, a productive cough with yellow sputum and a respiratory rate of 20. Which of the following nursing diagnosis is most appropriate based upon this assessment? A. Hyperthermia related to infectious illness B. Ineffective thermoregulation related to chilling C. Ineffective breathing pattern related to pneumonia D. Ineffective airway clearance related to thick secretions

A. Hyperthermia related to infectious illness Because the patient has spiked a temperature and has a diagnosis of pneumonia, the logical nursing diagnosis is hyperthermia related to infectious illness. There is no evidence of a chill, and her breathing pattern is within normal limits at 20 breaths per minute. There is no evidence of ineffective airway clearance from the information given because the patient is expectorating sputum.

49. The nurse evaluates that a patient is experiencing the expected beneficial effects of ipratropium (Atrovent) after noting which of the following assessment findings? A. Increased peak flow readings B. Increased level of consciousness C. Decreased sputum production D. Increased respiratory rate

A. Increased peak flow readings. Ipratropium is a bronchodilator that should lead to increased PEFRs.

In caring for a patient with a PCA infusion of morphine sulfate through the right cephalic vein, the nurse assess that client is lethargic, with a blood pressure of 90/60, pulse rate of 118 beats/minute, and a respiratory rate of 8 breaths/minute. What assessment should the nurse perform next? A. Observe the amount and dose of morphine in the PCA pump syringe B. Note the appearance and patency of the client's peripheral IV site C. Palpate the volume of the client's right radial pulse D. Auscultate the client's breath sounds bilaterally

A. Observe the amount and dose of morphine in the PCA pump syringe

A 15-year-old client with a spinal cord injury develops spastic leg tremors, sweating, and a headache. Which action should the nurse implement? A. Palpate the bladder for distention B. Obtain an oxygen saturation level C. Administer a prescribed analgesic D. Encourage dorsiflexion of the feet

A. Palpate the bladder for distention

8. While obtaining the admission assessment data, which of the following characteristics would a nurse expect a patient with anemia to report? A. Palpitations B. Blurred vision C. Increased appetite D. Feeling of warm flushing sensation

A. Palpitations Patients experiencing moderate anemia (hemoglobin [Hb] 6 to 10 g/dL) may experience dyspnea (shortness of breath), palpitations, diaphoresis (profound perspiration) with exertion, and chronic fatigue. Blurred vision is associated in patients experiencing profound anemia states. Anorexia is common in patients with severe anemia, as well. Patients with anemia often appear pale and complain of feeling cold because of compensatory vasoconstriction of the subcutaneous capillaries.

While obtaining the admission assessment data, which of the following characteristics would a nurse expect a patient with anemia to report? A. Palpitations B. Blurred vision C. Increased appetite D. Feeling of warm flushing sensation

A. Palpitations Patients experiencing moderate anemia (hemoglobin [Hb] 6 to 10 g/dL) may experience dyspnea (shortness of breath), palpitations, diaphoresis (profound perspiration) with exertion, and chronic fatigue. Blurred vision is associated in patients experiencing profound anemia states. Anorexia is common in patients with severe anemia, as well. Patients with anemia often appear pale and complain of feeling cold because of compensatory vasoconstriction of the subcutaneous capillaries.

An older male adult resident of an extended care facility receives a prescription for diphenhydramine (Benadryl) 25 mg PO to treat generalized pruritus. Two hours after administration of the drug, he continues to experience itching, is confused, and has an unsteady gait. What action should the nurse implement first? A. Place the client on fall precautions B. Apply soft limb restraints to extremities C. Give a second dose of Benadryl D. Lubricate the skin with an emollient

A. Place the client on fall precautions

45. The nurse reviews pursed lip breathing with a patient newly diagnosed with emphysema. The nurse reinforces that this technique will assist respiration by which of the following mechanisms? A. Preventing bronchial collapse and air trapping in the lungs during exhalation B. Increasing the respiratory rate and giving the patient control of respiratory patterns C. Loosening secretions so that they may be coughed up more easily D. Promoting maximal inhalation for better oxygenation of the lungs

A. Preventing bronchial collapse and air trapping in the lungs during exhalation The focus of pursed lip breathing is to slow down the exhalation phase of respiration, which decreases bronchial collapse and subsequent air trapping in the lungs during exhalation.

What modification is most important for the nurse to recommend to a client with high cholesterol? A. Replace beed with fish or poultry B. Include six servings of fruits and vegetables daily C. Limit portion sizes using the "Plate Method" D. Use vegetables oils in food preparation

A. Replace beed with fish or poultry

A client who had a below-the-knee amputation is experiencing severe phantom limb pain (PLP) and asks the nurse if mirror therapy will make the pain stop. Which response by the nurse is likely to be most helpful? A. Research indicated that mirror therapy is effective in reducing phantom limb pain B. You can try mirror therapy, but do not expect complete elimination of the pain C. Transcutaneous electrical nerve stimulation (TENS) has been found to be more effective D. Where did you learn about the use of mirror therapy in treating phantom limb pain?

A. Research indicated that mirror therapy is effective in reducing phantom limb pain

The nurse is caring for a postoperative patient with sudden onset of respiratory distress. The physician orders a STAT ventilation-perfusion scan. Which of the following explanations should the nurse provide to the patient about the procedure? A. This test involves injection of a radioisotope to outline the blood vessels in the lungs, followed by inhalation of a radioisotope gas. B. This test will use special technology to examine cross sections of the chest with use of a contrast dye. C. This test will use magnetic fields to produce images of the lungs and chest. D. This test involves injecting contrast dye into a blood vessel to outline the blood vessels of the lungs.

A. This test involves injection of a radioisotope to outline the blood vessels in the lungs, followed by inhalation of a radioisotope gas.A ventilation-perfusion scan has two parts. In the perfusion portion, a radioisotope is injected into the blood and the pulmonary vasculature is outlined. In the ventilation part, the patient inhales a radioactive gas that outlines the alveoli.

The nurse is caring for a client with a chest tube to water seal drainage that was inserted 10 days ago because of a ruptured bullae and pneumothorax. Which finding should the nurse report to the healthcare provider before the chest tube is removed? A. Tidal of water in the water seal chamber B. Bilateral muffled breath sounds at bases C. Temperature of 101 degrees F D. Absence of chest tube drainage for 2 days.

A. Tidal in the water seal chamber should be reported to the HPC to show that the chest tube is working properly. (B) may indicate hypoventilation from the chest tube and usually improves when the tube is removed. (C) indicates infection (D) is an expected finding.

A client in septic shock jas a double lumen central venous catheter with one liter of 0.9% Normal Saline solution infusing at 1 ml/hour through one lumen and Total Parenteral Nutrition (TPN) Infusing at 50 ml/hour through one port. The nurse prepares newly prescribed IV antibiotics that should take 45 minutes to infuse. What intervention should the nurse implement? A. Use a secondary port of the Normal Saline solution to administer the antibiotic B. Add the antibiotic to the TPN solution, and continue the normal saline solution C. Stop the TPN infusion for the time needed to administer the prescribed antibiotic D. Add the antibiotic to the Normal Saline solution and continue both infusions

A. Use a secondary port of the Normal Saline solution to administer the antibiotic

27. A patient with acute exacerbation of COPD needs to receive precise amounts of oxygen. Which of the following types of equipment should the nurse prepare to use? A. Venturi mask B. Partial non-rebreather mask C. Oxygen tent D. Nasal cannula

A. Venturi mask The Venturi mask delivers precise concentrations of oxygen and should be selected whenever this is a priority concern. The other methods are less precise in terms of amount of oxygen delivered.

The nurse is investigating a client injury that occurred when a mechanical lift malfunctioned while moving a client from the bed to a orthopedic-chair. Which question should the nurse ask first? A. Was the equipment used according to policy? B. Which day of the week did the injury occur? C. When was the lift last checked by maintenance? D. What mitigating circumstances led to the injury?

A. Was the equipment used according to policy?

6. A nurse is working on a respiratory care unit where many of the patients are affected by asthma. Which of the following actions by the nurse would most likely increase respiratory difficulty for the patients? A. Wearing perfume to work B. Encouraging patients to ambulate daily C. Allowing the patients to eat green leafy vegetables D. Withholding antibiotic therapy until cultures are obtained

A. Wearing perfume to work People with asthma should avoid extrinsic allergens and irritants (e.g., dust, pollen, smoke, certain foods, colognes and perfumes, certain types of medications) because their airways become inflamed, producing shortness of breath, chest tightness, and wheezing. Many green leafy vegetables are rich in vitamins, minerals, and proteins, which incorporate healthy lifestyle patterns into the patients' daily living routines. Routine exercise is a part of a prudent lifestyle, and for patients with asthma the physical and psychosocial effects of ambulation can incorporate feelings of well-being, strength, and enhancement of physical endurance. Antibiotic therapy is always initiated after cultures are obtained so that the sensitivity to the organism can be readily identified.

24. The nurse is caring for a patient with an acute exacerbation of asthma. Following initial treatment, which of the following findings indicates to the nurse that the patient's respiratory status is improving? A. Wheezing becomes louder B. Vesicular breath sounds decrease C. Aerosol bronchodilators stimulate coughing D. The cough remains nonproductive

A. Wheezing becomes louder The primary problem during an exacerbation of asthma is narrowing of the airway and subsequent diminished air exchange. As the airways begin to dilate, wheezing gets louder because of better air exchange.

When caring for a 2-day-old infant, the nurse observes that the babies legs are flexed with limited abduction. Based on the finding, what action should the nurse take next? A. notify the healthcare provider B. continue care since this is a normal finding C. document the finding in the record D. perform range of motion to the joint

A. notify the healthcare provider limited abduction could indicate developmental hip dysplasia and the HCP should be notified

3. The nurse notices clear nasal drainage in a patient newly admitted with facial trauma, including a nasal fracture. The nurse should: A. test the drainage for the presence of glucose. B. suction the nose to maintain airway clearance. C. document the findings and continue monitoring. D. apply a drip pad and reassure the patient this is normal.

A. test the drainage for the presence of glucose. Clear nasal drainage suggests leakage of cerebrospinal fluid (CSF). The drainage should be tested for the presence of glucose, which would indicate the presence of CSF.

The only lung cancer NOT related to cigarette smoking

Adenocarcinoma

The patient with pheochromocytoma's HTN is stabilized with what type of medications?

Adrenergic Blocking Agents

An adolescent receives a prescription for an injection of S-matriptan succinate, 4 mg subcutaneously for a migraine headache. Using a vial labeled, 6 mg/0.5 ml, how many ml should the nurse administer? (nearest hundredth)

0.33 ml

A school-aged child who weighs 42 pounds receives a post-tonsillectomy prescription for promethazine (Phenergan) 0.5 mg/kg IM to prevent postoperative nausea. The medication is available in 25 mg/ml ampules. How many ml should the nurse administer?

0.4 ml

Diagnosis of Addison disease

ACTH stimulation test

Hypertonic IV solution

5% dextrose in lactated ringer's 5% dextrose in 0.45% saline 5% dextrose in 0.9% saline 10% dextrose in water

Short acting: duration

5-8 hrs

Addison's Disease is usually not detected until about

90% of the adrenal cortex is already destroyed

Ulcerative colitis

Affects superficial mucosa of the large intestines and rectum causing the bowel to eventually narrow shorten and thicken do to muscular hypertrophy

Cause of acute pancreatitis

Alcohol ingestion and biliary tract disease

Medication for Crohn's disease

Aminosalicylates Antimicrobials Corticosteroids Immunosuppressants

When checking for bleeding in a post thyroidectomy patient, it's important to check

Anterior and Posterior dressings

Monitor level of arterial pressure and counteract as rise in arterial pressure through vagally mediated cardiac slowing and vasodilation with decreased sympathetic tone, therefore reflex control of circulation elevates the systemic arterial pressure when it falls and lower it when it rises

Arterial Baroreceptors

Nursing Interventions for Exophthalmos

Artificial Tears Salt Restriction Sit Upright as much as possible Dark Glasses Corticosteroids if severe

Complications from cirrhosis

Ascites edema portal hypertension Esophageal varices encephalopathy respiratory distress coagulation defects

Pacemaker fires at a constant rate

Asynchronous or Fixed Pacemaker

Hepatitis C: incubation period

Average 14-180 days

Which milestone indicates to the nurse successful achievement of young adulthood? A) Demonstrates a conceptualization of death and dying. B) Completes education and becomes self-supporting. C) Creates a new definition of self and roles with others. D) Develops a strong need for parental support and approval.

B) Completes education and becomes self-supporting. Transitioning through young adulthood is characterized by establishing independence as an adult, and includes developmental tasks such as completing education, beginning a career, and becoming self-supporting (B). (A and C) are characteristic of adolescence. Although strong bonds with parents are an expected finding for this age group, the need for support and approval (D) indicates dependency, which is a developmental delay.

Seconal 0.1 gram PRN at bedtime is prescribed for rest. The scored tablets are labeled grain 1.5 per tablet. How many tablets should the nurse plan to administer? A. 1/2 tablet B. 1 tablet C. 1 1/2 tablet D. 2 tablets

B. 15 gr = 1 g, 0.1 x 15 = 1.5 grains

Thirty-six hours after cesarean delivery, a client complains of nausea and bloating. Assessment reveals a distended abdomen and no bowel movement since delivery. What intervention should the nurse implement first? A. Increase fiber in diet B. Auscultate the abdomen C. Insert a rectal tube D. Encourage ambulation

B. Auscultate the abdomen

The nurse caring for a client with dysphagia is attempting to insert a nasogastric tube (NGT), but the client will not swallow and is not gagging. What action should the nurse implement to facilitate the NGT passage into the esophagus? A. Offer the client sips of water or ice and coax to swallow B. Flex the client's head with chin to the chest and insert C. Elevate the bed 90 degrees and hyperextend the head D. Push the NGT beyond the oropharynx gently yet swiftly

B. Flex the client's head with chin to the chest and insert

A nurse call the nurse to report that at 0900 she administered a PO dose of digoxin to her 4-month-old infant, but at 0920 the baby vomited the medicine. What instruction should the nurse provide to his mother? A. give another dose B. withhold this dose C. administer a half dose now D. mix the next dose with food

B. withhold this dose

Symptoms of peptic ulcer disease

Belching bloating epigastric pain radiating to the back and relieved by antacids (not associated with food eaten)

A client with a serum sodium level of 125 mEq/ml or mmol/L (SI)should benefit most from the administration of which intervention solution? A. 10% Dextrose in 0.45% sodium chloride B. 5% Dextrose in 0.2% sodium chloride C. 0.9% sodium chloride solution (normal saline) D. 0.45% sodium chloride solution (half normal saline)

C. 0.9% sodium chloride solution (normal saline)

Chronic

Continuing over a long period of time.

Medications for ulcerative colitis

Corticosteroids Antidiarrheals Sulfasalazine (azulfidine) Mesalamine Infliximab (remicade)

A client who is fully awake after a gastroscopy asks the nurse for something to drink. After confirming that liquids are allowed, which assessment action should the nurse consider a priority? A) Listen to bilateral lung and bowel sounds. B) Obtain the client's pulse and blood pressure. C) Assist the client to the bathroom to void. D) Check the client's gag and swallow reflexes.

D) Check the client's gag and swallow reflexes. Following gastroscopy, a client should remain nothing by mouth until the effects of local anesthesia have dissipated and the airway's protective reflexes, gag and swallow reflexes, have returned (D). (A, B, and C) are not the priority before reintroducing oral fluids after a gastroscopy.

What is the correct procedure for performing an ophthalmoscopic examination on a client's right retina? A) Instruct the client to look at examiner's nose and not move his/her eyes during the exam. B) Set ophthalmoscope on the plus 2 to 3 lens and hold it in front of the examiner's right eye. C) From a distance of 8 to 12 inches and slightly to the side, shine the light into the client's pupil. D) For optimum visualization, keep the ophthalmoscope at least 3 inches from the client's eye.

D) For optimum visualization, keep the ophthalmoscope at least 3 inches from the client's eye. The client should focus on a distant object in order to promote pupil dilation. The ophthalmoscope should be set on the 0 lens to begin (creates no correction at the beginning of the exam), and should be held in front of the examiner's left eye when examining the client's right eye. For optimum visualization, the ophthalmoscope should be kept within one to three inches of the client's eye (D). (A and B) describe incorrect methods for conducting an ophthalmoscopic examination. (C) should illicit a red reflex as the light travels through the crystalline lens to the retina.

The nurse is preparing a client for discharge from the hospital following a liver transplant. Which intervention is most important for the nurse to include in this client's discharge teaching plan? A. Keep a record of daily urinary output B. Report the onset of scleral jaundice C. Measure the abdominal girth daily D. Monitor for an elevated temperature

D. Monitor for an elevated temperature

With the client's eyes closed, the nurse places a common object in the client's hand and asks the client to describe the object. The client accurately names the object. How should the nurse document this assessment finding? A. No paraesthesia present B. Short term memory intact C. Active range of motion D. Positive for stereognosis

D. Positive for stereognosis

The nurse witnesses a baseball player receive a blunt trauma to the back of the head with a softball. What assessment data should the nurse collect immediately? A. Reactivity of deep tendon reflexes, comparing upper to lower extremities. B. Vital signs readings, excluding blood pressure if need equipment is unavailable. C. Memory of events that occurred before and after the blow to the head. D. Ability to spontaneously open the eyes before any tactile stimuli are given.

D. The LOC should be immediately established immediately after the head injury has occurred. Spontaneous eye opening (D) is a simple measure of LOC. (A) is not the best indicator of LOC. (B) is important but not the best indicator of LOC. (C) can be assessed after LOC has been established by assessing eye opening.

Hyperthyroidism

Excessive activity of thyroid gland. graves disease or goiter.

Hypertrophy

Excessive devolopment.

Cystocele

Hernia of the urinary bladder.

Hiatal hernia

Herniation of the esophagogastric junction and a portion of the stomach into the chest through the esophageal hiatus of the diaphragm

Rapid acting insulin

HumaLog NovoLog Apidra

Tonsillitis

Inflammation of lymph tissue in the throat.

The most aggressive form of breast malignancies

Inflammatory with peau d'orange

Diverticulitis

Inflammed diverticula which may cause obstruction infection and hemorrhage

Action of heparin:

Inhibits clot formation

Kegel Exercises can be described like this:

It's like trying to stop and start stream of urine while you're peeing

blunt trama to back of head

LOC assessment most important

Hemoglobin

M: 14 - 18 F: 12 - 16

OA's: biguanides

Metformin (glucophage)

What does a patient need to know when taking Proscar for BPH?

Monitor LFTs frequently Significant change in BPH expected in 1 month Meds should not be handled by women or children

What am I describing here: Numbness of the extremities Loss of balance Visual disturbances

Multiple Sclerosis

Common adverse effects of chemotherapy

Nausea, vomiting, alopecia, and bone marrow depression

A deficiency in vitamin A cause can:

Night Blindness

How can PID pelvic pain be treated?

OTC Meds Birth Control Pills TCAs

Malignant lung cancers associated with cigarette smoking:

Oat-cell carcinoma Squamous-cell carcinoma

Absence of bowel sounds indicate

Paralytic ileus

Neurogenic causes of intestinal obstruction

Paralytic ileus spinal cord lesions

Signs and symptoms of hyper glycemia

Polydipsia polyuria polyphagia blurred vision weakness weight-loss

Bowel preparation for surgery

Polyethylene glycol (GoLYTELY)

Pain management

Provide pain medication before dressing change

A client is admitted to the hospital with a diagnosis of severe acute diverticulitis. Which nursing intervention has the highest priority? A. Place the client on NPO status. B. Assess the client's temperature. C. Obtain a stool specimen. D. Administer IV fluids.

Rationale: A A client with acute severe diverticulitis is at risk for peritonitis and intestinal obstruction and should be made NPO (A) to reduce risk of intestinal rupture. (B, C, and D) are important but are less of a priority than (A), which is implemented to prevent a severe complication.

During a health fair, a male client with emphysema tells the nurse that he fatigues easily. Assessment reveals marked clubbing of the fingernails and an increased anteroposterior chest diameter. Which instruction is best to provide the client? A. "Pace your activities and schedule rest periods." B. "Increase the amount of oxygen you use at night." C. "Obtain medical evaluation for antibiotic therapy." D. "Reduce your intake of fluids containing caffeine."

Rationale: A Manifestations of emphysema include an increase in AP diameter (referred to as a barrel chest), nail bed clubbing, and fatigue. The nurse can provide instructions to promote energy management, such as pacing activities and scheduling rest periods (A). (B) may result in a decreased drive to breathe. The client is not exhibiting any symptoms of infection, so (C) is not necessary. (D) is less beneficial than (A).

The nurse is counseling a healthy 30-year-old female client regarding osteoporosis prevention. Which activity would be most beneficial in achieving the client's goal of osteoporosis prevention? A. Cross-country skiing B. Scuba diving C. Horseback riding D. Kayaking

Rationale: A Weight-bearing exercise is an important measure to reduce the risk of osteoporosis. Of the activities listed, cross-country skiing (A) includes the most weight-bearing, whereas (B, C, and D) involve less.

Seconal, 0.1 g PRN at bedtime, is prescribed for rest. The scored tablets are labeled grain 1.5 per tablet. How many tablets should the nurse plan to administer? A. ½ tablet B. 1 tablet C. 1½ tablets D. 2 tablets

Rationale: B 15 gr = 1 g. Converting the prescribed dose of 0.1 g to grains requires multiplying 0.1 × 15 = 1.5 grains. The tablets come in 1.5 grains; therefore, the nurse should plan to administer one tablet (B). (A, C, and D) are incorrect.

The nurse observes ventricular fibrillation on telemetry and, on entering the client's bathroom, finds the client unconscious on the floor. Which intervention should the nurse implement first? A. Administer an antidysrhythmic medication. B. Start cardiopulmonary resuscitation. C. Defibrillate the client at 200 J. D. Assess the client's pulse oximetry.

Rationale: B Ventricular fibrillation is a life-threatening dysrhythmia, and CPR should be started immediately (B). (A and C) are appropriate, but CPR is the priority action. The client is dying, and (D) does not address the seriousness of this situation.

In assessing a client diagnosed with primary aldosteronism, the nurse expects the laboratory test results to indicate a decreased serum level of which substance? A. Sodium B. Phosphate C. Potassium D. Glucose

Rationale: C* Clients with primary aldosteronism exhibit a profound decline in serum levels of potassium (C); hypokalemia; hypertension is the most prominent and universal sign. The serum sodium level is normal or elevated, depending on the amount of water resorbed with the sodium (A). (B) is influenced by parathyroid hormone (PTH). (D) is not affected by primary aldosteronism.

The nurse receives the client's next scheduled bag of TPN labeled with the additive NPH insulin. Which action should the nurse implement? A. Hang the solution at the current rate. B. Refrigerate the solution until needed. C. Prepare the solution with new tubing. D. Return the solution to the pharmacy.

Rationale: D Only regular insulin is administered by the IV route, so the TPN solution containing NPH insulin should be returned to the pharmacy (D). (A, B, and C) are not indicated because the solution should not be administered.

Hypocalcemia: causes

Renal failure hypoparathyroidism malabsorption pancreatitis

Three stages of burn care

Resuscitative/emergent acute rehabilitation

Metabolic alkalosis: cause

Retention of base or removal of acid from body fluids vomiting Burns potassium depletion

Severity is determined by:

Rule of nines, Lund and browder method

A person with a small bowel obstruction is at risk for

SEVERE fluid and electrolyte imbalances

STEMI:

ST-Elevation Myocardial Infarction

Medications to avoid with peptic ulcer's

Salicylates NSAID's Corticosteroids Anticoagulants

The presence of ________ can indicate glaucoma or optic nerve and visual pathway disorders

Scotomas

Characteristics of diabetic ketoacidosis

Serum glucose of 250 ketonurea in large amounts arterial pH of less than 7.30 nausea/vomiting dehydration abdominal pain Kushmaul reparations

Diagnostic test for ulcerative colitis

Sigmoidoscopy and colonoscopy

Arteriole

Small artery.

Diet for peptic ulcers

Small frequent meals high-protein high fat Low carb

Greatest chance for continence

Stoma created from the sigmoid colon on the left side of the abdomen

Metabolism

Sum of the chemical processes in a cell.

1st° burn

Superficial partial thickness, injury to the epidermis, leaves skin pink or red but no blisters, dry, painful, slight edema, no scarring or skin graft required

Coccyx

Tailbone.

Mediastinum

The space in the chest between the lungs.

With cancer of the larynx:

The tongue, mouth appear white, gray, dark brown, black and patchy

Hypochondriac regions

The upper lateral regions of the abdomen, beneath the ribs.

Categories of Burns

Thermal radiation electrical chemical

Action of mucosal healing agent

Treatment of peptic ulcer's

Myelogram

X-ray record of the spinal cord.

Recommendations for breast screening

Yearly mammo after age 40 Breast self-exams

Tender breast lumpiness, bilaterally----- finding of:

a benign condition: fibrocystic breast disease

thoracentesis

a surgical procedure done to remove fluid from the pleural space.

acute bronchitis

an inflammation of the lower respiratory tract that is usually due to infection.

Myxedema is a

medical emergency

Metabolic acidosis

pH: low PCO2: normal HCO3: low

Elevating the head of a patient with exophthalmos will

promote fluid drainage from the periorbital areas

A lot of the most common antihyperglycemic agents are

sulfa drugs

After a thyroidectomy, the nurse should be sure to

support the patients neck with pillows avoid flexion of neck avoid tension on suture lines Position in semi-fowler's

thoracotomy

surgical opening into the thoracic cavity.

tracheostomy

surgical opening into the trachea through which an indwelling tube may be inserted.

When examining a client with emphysema, what physical findings is the nurse likely to see?

-Barrel Chest -Dry/productive cough -Decreased breath sounds -Dyspnea -Crackles in lung fields

Bacteria gain access in one of the 3 ways:

-Blood -CSF Contamination (Surgery/Catheters) -Skull (Sneezing, Coughing, Kissing, Droplets)

What symptoms of pneumonia might the nurse expect to see in an older client?

-Confusion -Lethargy -Anorexia -Rapid Respiratory Rate

State (4) nursing interventions for assisting the client to cough productively:

-Encourage deep breathing -Increase fluid intake to 3 L/day -Use humidity to loosen secretions -Suction airway to stimulate coughing

Acute onset symptoms of bacterial meningitis:

-HA -Fever -Stiff Neck -Vomiting

Symptoms of TURP syndrome are:

-Increased BP -Nausea -Confusion

During mechanical ventilation, what are the (3) major nursing interventions?

-Monitor respiratory status and secure connection -Establish a communication mechanism with the client -Keep airway clear by coughing and suctioning

MONA for patients with myocardial infarction:

-Morphine -Oxygen -Nitroglycerin -Aspirin

Digoxin Toxicity Symptoms:

-Nause/Vomiting -Anorexia -Visual Disturbances -Restlessness -HA -Cardiac Dysrhythmias -Pulse <60 bpm

In patients with acute renal failure, limit what (3) things?

-Protein -Salt -Fluids

List (4) common symptoms of pneumonia the nurse might note on physical examination:

-Tachypnea -Fever with chills -Productive cough -Bronchial breath sounds

Four control systems play a major role in maintaining blood pressure:

-The arterial baroreceptor system -Regulation of body fluid volume -The renin-angiotensin/aldosterone system -Vascular autoregulation

What are the (3) basic type of diuretics used to decrease blood volume and lower BP:

-Thiazides -Loop -Potassium-Sparing

Hypotonic IV solution

0.5% normal saline 2.5% dextrose in 0.45% NS

Normal Creatinine:

0.6 to 1.2 mg/dL

Completed strokes usually produce neurologic deficits within

1 hour

Oral iron should be taken

1 hour before meals

Long acting: onset

1 hr

Rule of Nines: perineal area

1% for adults

Intermediate: onset

1-3 hr

The healthcare provider prescribes Morphine Sulfate Oral Solution 38 mg PO q4 hours for a client who is opioid-tolerant. The available 30 ml bottle is labeled, 100 mg/5 ml (20 mg/ml), and is packed with a calibrated oral syringe to provide accurate dose measurements. How many ml should the nurse administer?

1.9 ml

Normal BUN:

10 to 20 mg/dL

A client is receiving an IV solution labeled Heparin Sodium 20,000 units in 5% Dextrose Injection 500 ml at 25 ml/hour. How many units of heparin is the client receiving each hour?

1000 units/hour

The healthcare provider prescribes diltiazem (Cardizem) for a child with hypertension who weighs 66 pounds. Based on the recommended dose of 3.5 mg/kg/day, how many mg should the child receive per day?

105 mg/day

Residual catheterization at an outpatient clinic would probably use a

12 French straight catheter

Intermediate: duration

12-16 hr

Hepatitis B: incubation period

14-180 days

Rule of Nines: back trunk

18%

Rule of Nines: lower extremities

18% each 9% anterior/posterior

Cholesterol should be limited to ________ mg per day

180

A catheter is kept in a BPH-surgery post-op patient for

2 - 4 days following surgery

A client receives a prescription for bacitracin 20,000 units every 12 hours IM. The medication is available in a vial that contains 50,000 units and includes reconstitution instructions: "Use 4.8 ml diluent to yield a total volume of 5 ml." How many ml should the nurse administer?

2 ml

A woman's fertile period occurs

2 weeks prior to onset of menses

Short acting: peak

2-4 hrs

A client with a psychotic disorder is receiving haloperidol (Haldol) 3 mg IM q30 minutes x 3 hours for agitation control. The medication is available in 5 mg/ml. How many ml will the client receive over the next 3 hours?

3.6 ml

Short acting: onset

30-60 min

Rapid acting: peak

30-90 mins

Sodium nitroprusside (Nipride) at 0.8 mcg/kg/minute is prescribed for a client who weighs 65 kg. The available IV solution is labeled Nipride 50 mg in 500 ml D5W. The nurse should program the infusion pump to deliver how many ml/hour?

31 ml/hour

PCO2

35-45

A TB test is effective __________ after exposure.

4 - 6 weeks

The average age when menstruation stops is between

52 and 54

Sagittal

A plane that divides the body into right and left portions.

A male client who has never smoked but has had COPD for the past 5 years is now being assessed for cancer of the lung. The nurse knows that he is most likely to develop which type of lung cancer? A) Adenocarcinoma. B) Oat-cell carcinoma. C) Malignant melanoma. D) Squamous-cell carcinoma.

A) Adenocarcinoma. Adenocarcinoma is the only lung cancer not related to cigarette smoking (A). It has been found to be directly related to lung scarring and fibrosis from preexisting pulmonary disease such as TB or COPD. Both (B and D) are malignant lung cancers related to cigarette smoking. (C) is a skin cancer and is related to exposure to sunlight, not to lung problems.

A client who is receiving chemotherapy asks the nurse, "Why is so much of my hair falling out each day?" Which response by the nurse best explains the reason for alopecia? A) Chemotherapy affects the cells of the body that grow rapidly, both normal and malignant. B) Alopecia is a common side effect you will experience during long-term steroid therapy. C) Your hair will grow back completely after your course of chemotherapy is completed. D) The chemotherapy causes permanent alterations in your hair follicles that lead to hair loss.

A) Chemotherapy affects the cells of the body that grow rapidly, both normal and malignant. The common adverse effects of chemotherapy (nausea, vomiting, alopecia, bone marrow depression) are due to chemotherapy's effect on the rapidly reproducing cells, both normal and malignant (A). (B and D) do not provide correct information about chemotherapy-induced alopecia. Although (D) is a true statement, it does not effectively answer the client's question.

The nurse is assessing a client with bacterial meningitis. Which assessment finding indicates the client may have developed septic emboli? A) Cyanosis of the fingertips. B) Bradycardia and bradypnea. C) Presence of S3 and S4 heart sounds. D) 3+ pitting edema of the lower extremities.

A) Cyanosis of the fingertips. Septic emboli secondary to meningitis commonly lodge in the small arterioles of the extremities, causing a decrease in circulation to the hands (A) which may lead to gangrene. (B, C, and D) are abnormal findings, but do not indicate the development of septic emboli.

The nurse is interviewing a male client with hypertension. Which additional medical diagnosis in the client's history presents the greatest risk for developing a cerebral vascular accident (CVA)? A) Diabetes mellitus. B) Hypothyroidism. C) Parkinson's disease. D) Recurring pneumonia.

A) Diabetes mellitus. A history of diabetes mellitus poses the greatest risk for developing a CVA (A). (B, C, and D) may place the client at some risk due to immobility, but do not present a risk as great as (A).

A postmenopausal client asks the nurse why she is experiencing discomfort during intercourse. What response is best for the nurse to provide? A) Estrogen deficiency causes the vaginal tissues to become dry and thinner. B) Infrequent intercourse results in the vaginal tissues losing their elasticity. C) Dehydration from inadequate fluid intake causes vulva tissue dryness. D) Lack of adequate stimulation is the most common reason for dyspareunia.

A) Estrogen deficiency causes the vaginal tissues to become dry and thinner. Estrogen deprivation decreases the moisture-secreting capacity of vaginal cells, so vaginal tissues tend to become thinner, drier (A), and the rugae become smoother which reduces vaginal stretching that contributes to dyspareunia. Dyspareunia is not related to (B or C). While (D) can contribute to discomfort during intercourse, the primary cause is hormone-related.

A middle-aged male client with diabetes continues to eat an abundance of foods that are high in sugar and fat. According to the Health Belief Model, which event is most likely to increase the client's willingness to become compliant with the prescribed diet? A) He visits his diabetic brother who just had surgery to amputate an infected foot. B) He is provided with the most current information about the dangers of untreated diabetes. C) He comments on the community service announcements about preventing complications associated with diabetes. D) His wife expresses a sincere willingness to prepare meals that are within his prescribed diet.

A) He visits his diabetic brother who just had surgery to amputate an infected foot. The loss of a limb by a family member (A) will be the strongest event or "cue to action" and is most likely to increase the perceived seriousness of the disease. (B, C, and D) may influence his behavior but do not have the personal impact of (A).

An 81-year-old male client has emphysema. He lives at home with his cat and manages self-care with no difficulty. When making a home visit, the nurse notices that his tongue is somewhat cracked and his eyeballs are sunken into his head. What nursing intervention is indicated? A) Help the client to determine ways to increase his fluid intake. B) Obtain an appointment for the client to see an ear, nose, and throat specialist. C) Schedule an appointment with an allergist to determine if the client is allergic to the cat. D) Encourage the client to slightly increase his use of oxygen at night and to always use humidified oxygen.

A) Help the client to determine ways to increase his fluid intake. The nurse should suggest creative methods to increase the intake of fluids (A), such as having disposable fruit juices readily available. Clients with COPD should have at least three liters of fluids a day. These clients often reduce fluid intake because of shortness of breath. (B) is not indicated. These symptoms are not indicative of an allergy (C). Many elderly depend on their pets for socialization and self-esteem. Humidified oxygen will not relieve these symptoms and increased oxygen levels will stifle the COPD client's trigger to breathe (D).

A client is placed on a respirator following a cerebral hemorrhage, and vecuronium bromide (Norcuron) 0.04 mg/kg q12h IV is prescribed. Which nursing diagnosis is the priority for this client? A) Impaired communication related to paralysis of skeletal muscles. B) High risk for infection related to increased intracranial pressure. C) Potential for injury related to impaired lung expansion. D) Social isolation related to inability to communicate.

A) Impaired communication related to paralysis of skeletal muscles. To increase the client's tolerance of endotracheal intubation and/or mechanical ventilation, a skeletal-muscle relaxant such as vecuronium is usually prescribed. Impaired communication (A) is a serious outcome because the client cannot communicate his/her needs. Although this client might also experience (D), it is not a priority when compared to (A). Infection is not related to increased intracranial pressure (B). The respirator will ensure that the lungs are expanded (C).

A 51-year-old truck driver who smokes two packs of cigarettes a day and is 30 pounds overweight is diagnosed with having a gastric ulcer. What content is most important for the nurse to include in the discharge teaching for this client? A) Information about smoking cessation. B) Diet instructions for a low-residue diet. C) Instructions on a weight-loss program. D) The importance of increasing milk in the diet.

A) Information about smoking cessation. Smoking has been associated with ulcer formation, and stopping or decreasing the number of cigarettes smoked per day is an important aspect of ulcer management (A). Diet management includes a reduction in high-fiber/high-roughage foods as well as spicy foods. (B) would be indicated for inflammatory bowel disease. Sodium and caloric intake are not the key elements in an ulcer diet. Although this client does need (C), the management of his ulcer is the key factor at this point. (D) would actually increase gastric acid production.

The nurse is caring for a client with syndrome of inappropriate antidiuretic hormone (SIADH), which is manifested by which symptoms? A) Loss of thirst, weight gain. B) Dependent edema, fever. C) Polydipsia, polyuria. D) Hypernatremia, tachypnea.

A) Loss of thirst, weight gain. SIADH occurs when the posterior pituitary gland releases too much ADH, causing water retention, a urine output of less than 20 ml/hour, and dilutional hyponatremia. Other indications of SIADH are loss of thirst, weight gain (A), irritability, muscle weakness, and decreased level of consciousness. (B) is not associated with SIADH. (C) is a finding associated with diabetes insipidus (a water metabolism problem caused by an ADH deficiency), not SIADH. The increase in plasma volume causes an increase in the glomerular filtration rate that inhibits the release of rennin and aldosterone, which results in an increased sodium loss in urine, leading to greater hyponatremia, not (D).

A client is admitted for further testing to confirm sarcoidosis. Which diagnostic test provides definitive information that the nurse should report to the healthcare provider? A) Lung tissue biopsy. B) Positive blood cultures. C) Magnetic resonance imaging (MRI). D) Computerized tomography (CT) of the thorax.

A) Lung tissue biopsy. Sarcoidosis is an inflammatory condition that is characterized by the formation of widespread granulomatous lesions involving a pulmonary primary site. Although chest radiography identifies sarcoidosis, lung tissue biopsy (A) obtained by bronchoscopy or bronchoalveolar lavage provides definitive confirmation. (B) does not provide results for sarcoidosis. Although MRI and CT identify pulmonary lesions, the (C and D) are not necessary and do not provide definitive confirmation.

The nurse is assessing a client with chronic renal failure (CRF). Which finding is most important for the nurse to respond to first? A) Potassium 6.0 mEq. B) Daily urine output of 400 ml. C) Peripheral neuropathy. D) Uremic fetor.

A) Potassium 6.0 mEq. Hyperkalemia (normal serum level, 3.5 to 5.5 mEq) is a serious electrolyte disorder that can cause fatal arrhythmias, so (A) is the nursing priority. (B) is an expected finding associated with renal tubular destruction. In CRF, an increase in serum nitrogenous waste products, electrolyte imbalances, and demyelination of the nerve fibers contribute to the development of (C). (D) is a urinous odor of the breath related to the accumulation of blood urea nitrogen and is a common complication of CRF, but not as significant as hyperkalemia.

An adult client is admitted to the hospital burn unit with partial-thickness and full-thickness burns over 40% of the body surface area. In assessing the potential for skin regeneration, what should the nurse remember about full-thickness burns? A) Regenerative function of the skin is absent because the dermal layer has been destroyed. B) Tissue regeneration will begin several days following return of normal circulation. C) Debridement of eschar will delay the body's ability to regenerate normal tissue. D) Normal tissue formation will be preceded by scar formation for the first year.

A) Regenerative function of the skin is absent because the dermal layer has been destroyed. Full-thickness burns destroy the entire dermal layer. Included in this destruction is the regenerative tissue. For this reason, tissue regeneration does not occur, and skin grafting is necessary (A). (B, C, and D) are simply false.

After checking the urinary drainage system for kinks in the tubing, the nurse determines that a client who has returned from the post-anesthesia care has a dark, concentrated urinary output of 54 ml for the last 2 hours. What priority nursing action should be implemented? A) Report the findings to the surgeon. B) Irrigate the indwelling urinary catheter. C) Apply manual pressure to the bladder. D) Increase the IV flow rate for 15 minutes.

A) Report the findings to the surgeon. An adult who weighs 132 pounds (60 kg) should produce about 60 ml of urine hourly (1 ml/kg/hour). Dark, concentrated, and low volume of urine output should be reported to the surgeon. Although other actions (B, C, and D) may be indicated, the assessment findings should be reported to the healthcare provider.

A client is brought to the Emergency Center after a snow-skiing accident. Which intervention is most important for the nurse to implement? A) Review the electrocardiogram tracing. B) Obtain blood for coagulation studies. C) Apply a warming blanket. D) Provide heated PO fluids.

A) Review the electrocardiogram tracing. Airway, breathing, and circulation are priorities in client assessment and treatment. Continuous cardiac monitoring is indicated (A) because hypothermic clients have an increased risk for dysrhythmias. Coagulations studies (A) and re-warming procedures (C and D) can be initiated after a review of the ECG tracing (A).

A 46-year-old female client is admitted for acute renal failure secondary to diabetes and hypertension. Which test is the best indicator of adequate glomerular filtration? A) Serum creatinine. B) Blood Urea Nitrogen (BUN). C) Sedimentation rate. D) Urine specific gravity.

A) Serum creatinine. Creatinine (A) is a product of muscle metabolism that is filtered by the glomerulus, and blood levels of this substance are not affected by dietary or fluid intake. An elevated creatinine strongly indicates nephron loss, reducing filtration. (B) is also an indicator of renal activity, but it can be affected by non-renal factors such as hypovolemia and increased protein intake. (C) is a nonspecific test for acute or chronic inflammatory processes. (D) is useful in assessing hydration status, but not as useful in assessing glomerular function.

In preparing to administer intravenous albumin to a client following surgery, what is the priority nursing intervention? (Select all that apply.) A) Set the infusion pump to infuse the albumin within four hours. B) Compare the client's blood type with the label on the albumin. C) Assign a UAP to monitor blood pressure q15 minutes. D) Administer through a large gauge catheter. E) Monitor hemoglobin and hematocrit levels. F) Assess for increased bleeding after administration.

A) Set the infusion pump to infuse the albumin within four hours. D) Administer through a large gauge catheter. E) Monitor hemoglobin and hematocrit levels. F) Assess for increased bleeding after administration. (A, D, E, and F) are the correct selections. Albumin should be infused within four hours because it does not contain any preservatives. Any fluid remaining after four hours should be discarded (A). Albumin administration does not require blood typing (B). Vital signs should be monitored periodically to assess for fluid volume overload, but every 15 minutes is not necessary (C). This frequency is often used during the first hour of a blood transfusion. A large gauge catheter (D) allows for fast infusion rate, which may be necessary. Hemodilution may decrease hemoglobin and hematocrit levels (E), while increased blood volume and blood pressure may cause bleeding (F).

The nurse working in a postoperative surgical clinic is assessing a woman who had a left radical mastectomy for breast cancer. Which factor puts this client at greatest risk for developing lymphedema? A) She sustained an insect bite to her left arm yesterday. B) She has lost twenty pounds since the surgery. C) Her healthcare provider now prescribes a calcium channel blocker for hypertension. D) Her hobby is playing classical music on the piano.

A) She sustained an insect bite to her left arm yesterday. A radical mastectomy interrupts lymph flow, and the increased lymph flow that occurs in response to the insect bite increases the risk for the occurrence of lymphedema (A). (B) is not a factor. Lymphedema is not significantly related to vascular circulation (C). Only overuse of the arm, such as weight-lifting, would cause lymphedema--(D) would not.

A female client receiving IV vasopressin (Pitressin) for esophageal varice rupture reports to the nurse that she feels substernal tightness and pressure across her chest. Which PRN protocol should the nurse initiate? A) Start an IV nitroglycerin infusion. B) Nasogastric lavage with cool saline. C) Increase the vasopressin infusion. D) Prepare for endotracheal intubation.

A) Start an IV nitroglycerin infusion. Vasopressin is used to promote vasoconstriction, thereby reducing bleeding. Vasoconstriction of the coronary arteries can lead to angina and myocardial infarction, and should be counteracted by IV nitroglycerin per prescribed protocol (A). (B) will not resolve the cardiac problem. (C) will worsen the problem. Endotracheal intubation may be needed if respiratory distress occurs (D).

The nurse is preparing a teaching plan for a client who is newly diagnosed with Type 1 diabetes mellitus. Which signs and symptoms should the nurse describe when teaching the client about hypoglycemia? A) Sweating, trembling, tachycardia. B) Polyuria, polydipsia, polyphagia. C) Nausea, vomiting, anorexia. D) Fruity breath, tachypnea, chest pain.

A) Sweating, trembling, tachycardia. Sweating, dizziness, and trembling are signs of hypoglycemic reactions related to the release of epinephrine as a compensatory response to the low blood sugar (A). (B, C, and D) do not describe common symptoms of hypoglycemia.

The nurse formulates the nursing diagnosis of, Urinary retention related to sensorimotor deficit for a client with multiple sclerosis. Which nursing intervention should the nurse implement? A) Teach the client techniques of intermittent self-catheterization. B) Decrease fluid intake to prevent over distention of the bladder. C) Use incontinence briefs to maintain hygiene with urinary dribbling. D) Explain that anticholinergic drugs will decrease muscle spasticity.

A) Teach the client techniques of intermittent self-catheterization. Bladder control is a common problem for clients with multiple sclerosis. A client with urinary retention should receive instructions about self-catheterization (A) to prevent bladder distention. Adequate hydration, not (B), is important to reduce the risk of urinary tract infections by promoting elimination which reduces the time microorganisms spend in the bladder and by diluting the number of microorganisms in the bladder. Self-catheterization helps prevent dribbling, so (C) is unnecessary. Cholinergic drugs improve bladder muscle tone and help with bladder emptying, not (D).

The nurse is teaching a female client about the best time to plan sexual intercourse in order to conceive. Which information should the nurse provide? A) Two weeks before menstruation. B) Vaginal mucous discharge is thick. C) Low basal temperature. D) First thing in the morning.

A) Two weeks before menstruation. Ovulation typically occurs 14 days before menstruation begins (A), and sexual intercourse should occur within 24 hours of ovulation for conception to occur. High estrogen levels occur during ovulation and increase the vaginal mucous membrane characteristics, which become more "slippery" and stretchy, not (B). A rise in basal temperature, not (C), signals ovulation. The timing during the day is not as significant in determining conception as the day before and after ovulation (D).

A client who is sexually active with several partners requests an intrauterine device (IUD) as a contraceptive method. Which information should the nurse provide? A) Using an IUD offers no protection against sexually transmitted diseases (STD), which increase the risk for pelvic inflammatory disease (PID). B) Getting pregnant while using an IUD is common and is not the best contraceptive choice. C) Relying on an IUD may be a safer choice for monogamous partners, but a barrier method provides a better option in preventing STD transmission. D) Selecting a contraceptive device should consider choosing a successful method used in the past.

A) Using an IUD offers no protection against sexually transmitted diseases (STD), which increase the risk for pelvic inflammatory disease (PID). The use of an IUD provides the client with no protection from STDs (A). While pregnancy rates with the use of an IUD are somewhat higher, (B) is not therapeutic, but judgmental. (C) is judgmental and does not provide the client any information about use of an IUD. While talking about contraceptives may include (D), it is does not provide the best information to maintain the client's health.

57. When admitting a patient with the diagnosis of asthma exacerbation, the nurse will assess for which of the following potential triggers? (Select all that apply.) A. Exercise B. Allergies C. Emotional stress D. Decreased humidity

A,B,C Although the exact mechanism of asthma is unknown, there are several triggers that may precipitate an attack. These include allergens, exercise, air pollutants, respiratory infections, drug and food additives, psychologic factors, and GERD.

When admitting a patient with the diagnosis of asthma exacerbation, the nurse will assess for which of the following potential triggers? (Select all that apply.) A. Exercise B. Allergies C. Emotional stress D. Decreased humidity

A,B,C Although the exact mechanism of asthma is unknown, there are several triggers that may precipitate an attack. These include allergens, exercise, air pollutants, respiratory infections, drug and food additives, psychologic factors, and GERD.

55. During admission of a patient diagnosed with non-small cell carcinoma of the lung, the nurse questions the patient related to a history of which of the following risk factors for this type of cancer? (Select all that apply.) A. Asbestos exposure B. Cigarette smoking C. Exposure to uranium D. Chronic interstitial fibrosis

A,B,C Non-small carcinoma is associated with cigarette smoking and exposure to environmental carcinogens, including asbestos and uranium. Chronic interstitial fibrosis is associated with the development of adenocarcinoma of the lung.

During admission of a patient diagnosed with non-small cell carcinoma of the lung, the nurse questions the patient related to a history of which of the following risk factors for this type of cancer? (Select all that apply.) A. Asbestos exposure B. Cigarette smoking C. Exposure to uranium D. Chronic interstitial fibrosis

A,B,C Non-small carcinoma is associated with cigarette smoking and exposure to environmental carcinogens, including asbestos and uranium. Chronic interstitial fibrosis is associated with the development of adenocarcinoma of the lung.

Which content about self-care should the nurse include in the teaching plan of a client who has genital herpes? (Select all that apply.) A. Encourage annual physical and Pap smear. B. Take antiviral medication as prescribed. C. Use condoms to avoid transmission to others. D. Warm sitz baths may relieve itching. E. Use Nystatin suppositories to control itching. F. Douche with weak vinegar solutions to decrease itching.

A,B,C,D. (E) is specific for Candida infections and (F) is used to treat Trichomonas.

52. When assessing a patient's sleep-rest pattern related to respiratory health, the nurse would ask if the patient: (Select all that apply.) A. Has trouble falling asleep B. Awakens abruptly during the night C. Sleeps more than 8 hours per night D. Has to sleep with the head elevated

A,B,D The patient with sleep apnea may have insomnia and/or abrupt awakenings. Patients with cardiovascular disease (e.g., heart failure that may affect respiratory health) may need to sleep with the head elevated on several pillows (orthopnea). Sleeping more than 8 hours per night is not indicative of impaired respiratory health.

56. When admitting a 45-year-old female with a diagnosis of pulmonary embolism, the nurse will assess the patient for which of the following risk factors? (Select all that apply.) A. Obesity B. Pneumonia C. Hypertension D. Cigarette smoking

A,C,D Research has demonstrated an increased risk of pulmonary embolism in women associated with obesity, heavy cigarette smoking, and hypertension. Other risk factors include immobilization, surgery within the last 3 months, stroke, history of DVT, and malignancy.

54. To promote airway clearance in a patient with pneumonia, the nurse instructs the patient to do which of the following? (Select all that apply.) A. Splint the chest when coughing B. Maintain a semi-Fowler's position C. Maintain adequate fluid intake D. Instruct patient to cough at end of exhalation

A,C,D The nurse should instruct the patient to splint the chest while coughing. This will reduce discomfort and allow for a more effective cough. Maintaining adequate fluid intake liquefies secretions, allowing easier expectoration. Coughing at the end of exhalation promotes a more effective cough. The patient should be positioned in an upright sitting position (high-Fowler's) with head slightly flexed.

To promote airway clearance in a patient with pneumonia, the nurse instructs the patient to do which of the following? (Select all that apply.) A. Splint the chest when coughing B. Maintain a semi-Fowler's position C. Maintain adequate fluid intake D. Instruct patient to cough at end of exhalation

A,C,D The nurse should instruct the patient to splint the chest while coughing. This will reduce discomfort and allow for a more effective cough. Maintaining adequate fluid intake liquefies secretions, allowing easier expectoration. Coughing at the end of exhalation promotes a more effective cough. The patient should be positioned in an upright sitting position (high-Fowler's) with head slightly flexed.

After diagnosis and initial treatment of a 3-year-old child with cystic fibrosis, the nurse provides home care instruction to the mother. Which statement by the child's mother indicates that she understands home care treatment to promote pulmonary functions? A. "Chest physiotherapy should be performed twice a day before a meal." B. "Administer a cough suppressant every 8 hours.' C. "Maintain supplemental oxygen at 4 to 6 L/minute." D. "Energy should be conserved by scheduling minimally strenuous activities."

A. "Chest physiotherapy should be performed twice a day before a meal."

29. The physician has prescribed salmeterol (Serevent) for a patient with asthma. In reviewing the use of dry powder inhalers (DPIs) with the patient, the nurse should provide which of the following instructions? A. "Close lips tightly around the mouthpiece and breathe in deeply and quickly." B. "To administer a DPI, you must use a spacer that holds the medicine so that you can inhale it." C. "Hold the inhaler several inches in front of your mouth and breathe in slowly, holding the medicine as long as possible." D. "You will know you have correctly used the DPI when you taste or sense the medicine going into your lungs."

A. "Close lips tightly around the mouthpiece and breathe in deeply and quickly." Dry powder inhalers do not require spacer devices. The patient should be instructed to breathe in deeply and quickly to ensure medicine moves down deeply into lungs. The patient may not taste or sense the medicine going into the lungs.

The physician has prescribed salmeterol (Serevent) for a patient with asthma. In reviewing the use of dry powder inhalers (DPIs) with the patient, the nurse should provide which of the following instructions? A. "Close lips tightly around the mouthpiece and breathe in deeply and quickly." B. "To administer a DPI, you must use a spacer that holds the medicine so that you can inhale it." C. "Hold the inhaler several inches in front of your mouth and breathe in slowly, holding the medicine as long as possible." D. "You will know you have correctly used the DPI when you taste or sense the medicine going into your lungs."

A. "Close lips tightly around the mouthpiece and breathe in deeply and quickly." Dry powder inhalers do not require spacer devices. The patient should be instructed to breathe in deeply and quickly to ensure medicine moves down deeply into lungs. The patient may not taste or sense the medicine going into the lungs.

A female client is seen in the Emergency Department with a broken arm. She is visibly anxious and tells the nurse "I am afraid my husband is going to kill me. The beatings are getting worse, and everyone says I should leave him, but I am afraid of what he will do to me and the children if I do leave." What question is most important for the nurse to ask the client? A. "Have you thought about what to do when you are in an unsafe situation?" B. "Have you considered attending family violence programs sponsored by the police department?" C. "Do you know a clergy person you can talk to?" D. "Did you experience violence in your family of origin?"

A. "Have you thought about what to do when you are in an unsafe situation?"

29. Which of the following instructions are most appropriate in the home management of a patient who has undergone surgery for oral cancer? A. "You should drink plenty of fluids and eat foods you enjoy." B. "It is normal to have some leakage of saliva from the suture line." C. "Lying in a prone position helps decrease swelling at the suture line." D. "You should avoid foods high in protein while your suture line is healing."

A. "You should drink plenty of fluids and eat foods you enjoy." For patients who have undergone treatment for head and neck cancers, maintaining adequate nutrition is a challenge. The nurse encourages the patient to increase fluids to prevent dehydration and liquefy secretions. These patients are more likely to eat foods that they enjoy and can tolerate.

A male client with arterial peripheral vascular disease (PVD) complains of pain in his feet. Which instruction should the nurse give to the UPA to quickly relieve the client's pain? A. Help the client to dangle his legs. B. Apply compression stockings. C. Assist with passive leg exercises. D. Ambulate three times daily.

A. A client who has arterial PVD may benefit from a dependent position which can be achieved by dangling by improving blood flow and relieving pain. (B) is indicated for venous insufficiency and (C) is indicated for bed rest. (D) is indicated to facilitate collateral circulation and may improve long term complaints of pain.

32. Which of the following test results identify that a patient with an asthma attack is responding to treatment? A. A decreased exhaled nitric oxide B. An increase in CO2 levels C. A decrease in white blood cell count D. An increase in serum bicarbonate levels

A. A decreased exhaled nitric oxide. Nitric oxide levels are increased in the breath of people with asthma. A decrease in the exhaled nitric oxide concentration suggests that the treatment may be decreasing the lung inflammation associated with asthma.

Which of the following test results identify that a patient with an asthma attack is responding to treatment? A. A decreased exhaled nitric oxide B. An increase in CO2 levels C. A decrease in white blood cell count D. An increase in serum bicarbonate levels

A. A decreased exhaled nitric oxide. Nitric oxide levels are increased in the breath of people with asthma. A decrease in the exhaled nitric oxide concentration suggests that the treatment may be decreasing the lung inflammation associated with asthma.

The nurse on a busy surgical unit is assigning client care to a registered nurse (RN) and a practical nurse (PN). Which client is best to assign to the PN? A. A preoperative client who has developed urinary retention and need a urinary catheter inserted B. A postoperative client who is receiving a unit of packed red blood cells C. A preoperative client who is fearful and anxious about the impending surgery D. A postoperative client who is febrile, has a productive cough, and is complaining of pain

A. A preoperative client who has developed urinary retention and need a urinary catheter inserted

The nurse is assessing several clients at a homeless shelter. Which client(s) should the nurse refer for involuntary admission to a psychiatric facility? (select all that apply) A. A young adult who has lost his appetite because the communists are asking him to kill all conservative Catholics B. An adult male who has not eaten for three days and says he is Jesus Christ and that God loves all people C. An older woman who used to be a prostitute and resides in the shelter about three nights weekly D. A middle-aged woman who sits for long periods of time and mumbles to herself about wanting to die E. An adolescent male who works on the streets as a beggar to purchase marijuana, food, and shelter

A. A young adult who has lost his appetite because the communists are asking him to kill all conservative Catholics B. An adult male who has not eaten for three days and says he is Jesus Christ and that God loves all people D. A middle-aged woman who sits for long periods of time and mumbles to herself about wanting to die

23. A patient is having inspiratory stridor (crowing respiration) and the nurse suspects he is experiencing a laryngospasm. Which of the following would be most appropriate to implement for a patient experiencing a laryngospasm? A. Administer 100% oxygen. B. Position the patient in high Fowler's position. C. Insert a 16-gauge (large-bore) IV needle. D. Activate the emergency response team (code blue team) to the patient's room.

A. Administer 100% oxygen.A nurse should immediately administer 100% oxygen to the patient until the airway is fully reestablished, the larynx relaxes, and the spasms stop. Activating the emergency response team is not an immediate nursing action at this time because the nurse can administer the oxygen without the assistance of others. Positioning the patient in high Fowler's will not address the patient's need for immediate reoxygenation because of the patient's compromised respiratory state. Insertion of an IV device is not the first priority response but should be implemented after the nurse has assessed that the airway is stable.

A patient is having inspiratory stridor (crowing respiration) and the nurse suspects he is experiencing a laryngospasm. Which of the following would be most appropriate to implement for a patient experiencing a laryngospasm? A. Administer 100% oxygen. B. Position the patient in high Fowler's position. C. Insert a 16-gauge (large-bore) IV needle. D. Activate the emergency response team (code blue team) to the patient's room.

A. Administer 100% oxygen.A nurse should immediately administer 100% oxygen to the patient until the airway is fully reestablished, the larynx relaxes, and the spasms stop. Activating the emergency response team is not an immediate nursing action at this time because the nurse can administer the oxygen without the assistance of others. Positioning the patient in high Fowler's will not address the patient's need for immediate reoxygenation because of the patient's compromised respiratory state. Insertion of an IV device is not the first priority response but should be implemented after the nurse has assessed that the airway is stable.

The nurse makes a supervisory home visit to observe an unlicensed assistive personnel (UAP) who is providing personal care for a client with Alzheimer's Disease. The nurse observes that whatever the client gets upset, the UAP changes the subject. What action should the nurse take in response to this observation? A. Affirm that the UAP is using an effective strategy to reduce the client's anxiety B. Meet with the UAP later to role model more assertive communication techniques C. Assume care of the client to ensure that effective communication is maintained D. Tell the UAP to offer more choices during the personal care to prevent anxiety

A. Affirm that the UAP is using an effective strategy to reduce the client's anxiety

4. When caring for a patient who is 3 hours postoperative laryngectomy, the nurse's highest priority assessment would be: A. Airway patency B. Patient comfort C. Incisional drainage D. Blood pressure and heart rate

A. Airway patency Remember ABCs with prioritization. Airway patency is always the highest priority and is essential for a patient undergoing surgery surrounding the upper respiratory system.

When caring for a patient who is 3 hours postoperative laryngectomy, the nurse's highest priority assessment would be: A. Airway patency B. Patient comfort C. Incisional drainage D. Blood pressure and heart rate

A. Airway patency Remember ABCs with prioritization. Airway patency is always the highest priority and is essential for a patient undergoing surgery surrounding the upper respiratory system.

The patient has an order for each of the following inhalers. Which of the following should the nurse offer to the patient at the onset of an asthma attack? A. Albuterol (Proventil) B. Beclomethasone (Beclovent) C. Ipratropium bromide (Atrovent) D. Salmeterol (Serevent)

A. Albuterol (Proventil) Albuterol is a short-acting bronchodilator that should initially be given when the patient experiences an asthma attack.

A charge nurse is establishing priorities of client needs at the beginning of the morning shift. Which client situation has the highest priority for immediate care? A. An elderly client who is trying to climb over the side rails B. A diabetic client who requires insulin before breakfast C. A postoperative client with unremitting nausea D. A 5-year-old child who is crying because the parents left

A. An elderly client who is trying to climb over the side rails

21. A 45-year-old man with asthma is brought to the emergency department by automobile. He is short of breath and appears frightened. During the initial nursing assessment, which of the following clinical manifestations might be present as an early symptom during an exacerbation of asthma? A. Anxiety B. Cyanosis C. Hypercapnia D. Bradycardia

A. Anxiety An early symptom during an asthma attack is anxiety because he is acutely aware of the inability to get sufficient air to breathe. He will be hypoxic early on with decreased PaCO2 and increased pH as he is hyperventilating.

A 45-year-old man with asthma is brought to the emergency department by automobile. He is short of breath and appears frightened. During the initial nursing assessment, which of the following clinical manifestations might be present as an early symptom during an exacerbation of asthma? A. Anxiety B. Cyanosis C. Hypercapnia D. Bradycardia

A. Anxiety An early symptom during an asthma attack is anxiety because he is acutely aware of the inability to get sufficient air to breathe. He will be hypoxic early on with decreased PaCO2 and increased pH as he is hyperventilating.

The nurse is performing hourly neurological check for a client with a head injury. Which new assessment finding warrants the most immediate intervention by the nurse? A. A unilateral pupil that is dilated and nonreactive to light. B. Client cries out when awakened by a verbal stimulus. C. Client demonstrates a loss of memory to the events leading up to the injury. D. Onset of nausea, headache, and vertigo.

A. Any changes in pupil size and reactivity is an indication of increasing ICP and should be reported immediately. (B) is normal for being awakened. (C & D) are common manifestations of head injury and less of an immediacy than (A).

3. Following a patient's bone marrow aspiration, which of the following nursing interventions should a nurse anticipate? A. Application of firm pressure to the site B. Positioning the patient in a prone position C. Positioning the patient in a supine position D. Application of a warm, moist compress to the site

A. Application of firm pressure to the site After a bone marrow aspiration procedure, a nurse should apply pressure to the aspiration site until bleeding stops. Application of a warm, moist compress will not alter the potential for bleeding. Positioning the patient to assume a supine or prone position also will not address the need to control bleeding from the aspiration site.

A client diagnosed with angina pectoris complains of chest pain while ambulating in the hallway. Which action should the nurse implement first? A. Support the client to a sitting position. B. Ask the client to walk slowly back to the room. C. Administer a sublingual nitroglycerin tablet. D. Provide oxygen via nasal cannula.

A. Assist in safely repositioning and then administer (C & D). Then the client can be escorted back to the room via wheelchair or stretcher (B).

What nursing intervention is particularly indicated for the second stage of labor? A. Assisting the client to push effectively so that expulsion of the fetus can be achieved B. Providing pain medication to increase the client's tolerance of labor pains C. Assessing the fetal heart rate and pattern for signs of fetal distress D. Monitoring effects of oxytocin administration to help achieve cervical dilation

A. Assisting the client to push effectively so that expulsion of the fetus can be achieved

The home care nurse provided self-care instructions for a client with chronic venous insufficiency caused y deep vein thrombosis. Which instructions should the nurse include in the client's discharge teaching plan? (select all that apply) A. Avoid prolonged standing or sitting B. Maintain the bed flat while sleeping C. Cross legs at knee but not at ankle D. Continue wearing elastic stockings E. Use recliner for long periods of sitting

A. Avoid prolonged standing or sitting D. Continue wearing elastic stockings E. Use recliner for long periods of sitting

Before administering a parenteral nutrition through a central vein, the should confirm information from which sources? (select all that apply) A. Client's identification band B. Healthcare providers prescription C. Dietician's progress notes D. Solution label E. Measured residual volume F. Medication administration record

A. Client's identification band B. Healthcare providers prescription D. Solution label F. Medication administration record

5. When initially teaching a patient the supraglottic swallow following a radical neck dissection, with which of the following foods should the nurse begin? A. Cola B. Applesauce C. French fries D. White grape juice

A. ColaWhen learning the supraglottic swallow, it may be helpful to start with carbonated beverages because the effervescence provides clues about the liquid's position. Thin, watery fluids should be avoided because they are difficult to swallow and increase the risk of aspiration. Nonpourable pureed foods, such as applesauce, would decrease the risk of aspiration, but carbonated beverages are the better choice to start with.

A male client with chronic asthma tells the nurse that he is having more episodes of bronchoconstriction and increased mucous production. Which action should the nurse implement? A. Determine if rescue inhaler is being used first during an acute episodes. B. Inquire about effectiveness of corticosteroid inhaler during pollen days C. Auscultate lungs for adventitious sounds consistent with fluid accumulation D. Ask if an environmental trigger is associated with the asthmatic episodes

A. Determine if rescue inhaler is being used first during an acute episodes.

The register nurse (RN) is observing a newly hired practical nurse (PN) give a newborn a vitamin K (AquaMEPHYTON) injection. The PN uses a filter need to draw 0.25 ml of AquaMEPHYTON into the syringe, cleanses the thigh with alcohol in a circular motion, and prepares to inject the needle at a 90 degree angle in the left vastus lateralis. What action should the RN take? A. Direct the PN to change the filter needle to a 1/2 inch needle B. Confirm that the correct injection technique is being used C. Suggest injecting the medication into the dorsogluteal muscle D. Recommend administration of the injection at a 45 degree angle

A. Direct the PN to change the filter needle to a 1/2 inch needle

The nurse is preparing to administer 1.6 ml of medication IM to a 4-month-old infant. Which intervention should the nurse implement? A. Divide the medication into two injections with volumes under 1 ml B. Inject into the center one-third of the medical aspect of the thigh C. Administer into the ventrogluteal muscle with child prone D. Apply a topical anesthetic ointment prior to the injection

A. Divide the medication into two injections with volumes under 1 ml

In assessing a client 48 hours following a fracture, the nurse observes ecchymosis at the fracture site, and recognizes that hematoma formation at the bone fragment site has occurred. What action should the nurse implement? A. Document the extent of he bruising in the medical record B. Assign a UAP to take vital sign measurements q1h C. Advise the client that anticoagulation therapy may be needed. D. Call the lab to obtain a stat APTT and prothrombin time

A. Document the extent of he bruising in the medical record

To prevent infection by autocontamination during the acute phase of recovery from multiple burns, which intervention is most important for the nurse to implement? A. Dress each wound separately B. Avoid sharing equipment between multiple clients C. Use gown, mask, and gloves with dressing change D. Implement protective isolation

A. Dress each wound separately

The husband of an older woman, diagnosed with pernicious anemia, calls the clinic to report that his wife still has memory loss and some confusion since she received the first dose of nasal cyanocobalamin two days ago. He tells the nurse that he is worried that she may be getting Alzheimer's disease. What action should the nurse take? A. Explain that memory loss and confusion are common with Vitamin B12 deficiency B. Ask if the client is experiencing any change in bowel habits C. Determine if the client is taking iron and folic aid supplements D. Encourage the husband to bring the client to the clinic for a complete blood count

A. Explain that memory loss and confusion are common with Vitamin B12 deficiency

When caring for a patient with metastatic cancer, the nurse notes a hemoglobin level of 8.7 g/dl and hematocrit of 26%. The nurse would place highest priority on initiating interventions that will reduce which of the following? A. Fatigue B. Thirst C. Headache D. Abdominal pain

A. Fatigue The patient with a low hemoglobin and hematocrit (normal values approximately 13.5% to 17% and 40% to 54%, respectively) is anemic and would be most likely to experience fatigue. This symptom develops because of the lowered oxygen-carrying capacity that leads to reduced tissue oxygenation to carry out cellular functions.

A middle-aged woman, diagnosed with Graves' disease, asks the nurse about this condition. Which etiological pathology should the nurse include in the teaching plan about hyperthyroidism? (select all that apply) A. Graves' disease, an autoimmune condition, affects thyroid stimulating hormone receptors B. Large protruding eyeballs are a sign of hyperthyroid function C. Early treatment includes levothyroxine D. T3 and T4 hormone levels are increased E. Weight gain is a common complaint in hyperthyroidism

A. Graves' disease, an autoimmune condition, affects thyroid stimulating hormone receptors B. Large protruding eyeballs are a sign of hyperthyroid function D. T3 and T4 hormone levels are increased

Which menu selection by a male client indicates to the nurse that he understands the dietary management of Crohn's disease? A. Grilled chicken sandwich and pasta B. Tossed green salad and breadsticks C. Cheese enchiladas and beans D. Hamburger and French fries

A. Grilled chicken sandwich and pasta

6. The nurse is caring for a patient admitted to the hospital with pneumonia. Upon assessment, the nurse notes a temperature of 101.4° F, a productive cough with yellow sputum and a respiratory rate of 20. Which of the following nursing diagnosis is most appropriate based upon this assessment? A. Hyperthermia related to infectious illness B. Ineffective thermoregulation related to chilling C. Ineffective breathing pattern related to pneumonia D. Ineffective airway clearance related to thick secretions

A. Hyperthermia related to infectious illness Because the patient has spiked a temperature and has a diagnosis of pneumonia, the logical nursing diagnosis is hyperthermia related to infectious illness. There is no evidence of a chill, and her breathing pattern is within normal limits at 20 breaths per minute. There is no evidence of ineffective airway clearance from the information given because the patient is expectorating sputum.

The nurse evaluates that a patient is experiencing the expected beneficial effects of ipratropium (Atrovent) after noting which of the following assessment findings? A. Increased peak flow readings B. Increased level of consciousness C. Decreased sputum production D. Increased respiratory rate

A. Increased peak flow readings. Ipratropium is a bronchodilator that should lead to increased PEFRs.

A client is admitted voluntarily to the in-patient psychiatric unit for multi-substance dependency. While the client is in a group therapy, a peace officer approaches the nurse's station demanding to arrest the client. How should the nurse respond? A. Inform the officer that client information cannot be released B. Ensure safety for other client's by handcuffing the client during removal C. Refer the peace officer to the client's healthcare provider D. Call the unit security code and have the peace officer escorted off the unit.

A. Inform the officer that client information cannot be released

A nurse is admitting a 4-month-old who has respiratory syncytial virus (RIS). What intervention should the nurse implement first? A. Initiate contact precautions B. Place the infant under a mist tent C. Elevate aretial blood gases D. Obtain the infants vital signs

A. Initiate contact precautions

1. When assessing a patient's nutritional-metabolic pattern related to hematologic health, the nurse would: A. Inspect the skin for petechiae. B. Ask the patient about joint pain. C. Assess for vitamin C deficiency. D. Determine if the patient can perform ADLs.

A. Inspect the skin for petechiae. Any changes in the skin's texture or color should be explored when assessing the patient's nutritional-metabolic pattern related to hematologic health. The presences of petechiae or ecchymotic areas could be indicative of hematologic deficiencies related to poor nutritional intake or related causes.

When assessing a patient's nutritional-metabolic pattern related to hematologic health, the nurse would: A. Inspect the skin for petechiae. B. Ask the patient about joint pain. C. Assess for vitamin C deficiency. D. Determine if the patient can perform ADLs.

A. Inspect the skin for petechiae. Any changes in the skin's texture or color should be explored when assessing the patient's nutritional-metabolic pattern related to hematologic health. The presences of petechiae or ecchymotic areas could be indicative of hematologic deficiencies related to poor nutritional intake or related causes.

A male client with major depression who is taking fluoxetine calls the psychiatric clinic complaining of be more agitated, irritable, and anxious than usual. Which intervention should the nurse implement? A. Instruct the client to seek medical attention immediately. B. Explain that these are common side effects of Prozac. C. Encourage him to take the fluoxetine at night with a snack. D. Tell the client to have a complete blood count (CBC) drawn.

A. Instruct the client to seek medical attention immediately. Agitation, irritability, anxiety, and mania are early signs of serotonin syndrome, a rare but fatal reaction to SSRIs, so the client should seek medical attention immediately.

35. The nurse who has administered a first dose of oral prednisone (Deltasone) to the patient with asthma writes on the care plan to begin monitoring which of the following patient parameters? A. Intake and output B. Bowel sounds C. Apical pulse D. Deep tendon reflexes

A. Intake and output Corticosteroids such as prednisone can lead to fluid retention. For this reason, it is important to monitor the patient's intake and output.

The nurse who has administered a first dose of oral prednisone (Deltasone) to the patient with asthma writes on the care plan to begin monitoring which of the following patient parameters? A. Intake and output B. Bowel sounds C. Apical pulse D. Deep tendon reflexes

A. Intake and output Corticosteroids such as prednisone can lead to fluid retention. For this reason, it is important to monitor the patient's intake and output.

Which medication should the nurse anticipate administering to a client who is diagnosed with myxedema coma? A. Intravenous administration of thyroid hormones B. Oral administration of hypnotic agents C. Intravenous bolus of hydrocortisone D. Subcutaneous administration of vitamin K

A. Intravenous administration of thyroid hormones

The school nurse is planning to begin an obesity screening program in a school system. It is best to begin the screening program with which group? A. Kindergarden B. Third grade C. High school D. Onset of puberty

A. Kindergarden

An older female who ambulates with a quad-cane prefers to use a wheelchair because she has a halting and unsteady gait at times. Which interventions should the nurse implement? (select all that apply) A. Move personal items within client's reach B. Lower bed to the lowest possible position C. Raise all bed rails when client is resting D. Give directions to call for assistance E. Assist client to the bathroom q 2 hours F. Encourage the use of the wheelchair

A. Move personal items within client's reach B. Lower bed to the lowest possible position D. Give directions to call for assistance E. Assist client to the bathroom q 2 hours

The nurse is interviewing a client with schizophrenia. Which client behavior requires immediate intervention? A. Muscle spasms of the back and neck B. Rocks back and forth in the chair C. Shuffling gait and stooped posture D. Lip smacking and frequent eye blinking

A. Muscle spasms of the back and neck

While caring for a client with a new onset of diabetes mellitus, which intervention is most important for the nurse to include in the client's plan of care? A. Observe client's glucose self-monitoring technique B. Provide written diabetic diet instructions C. Teach client how to read food labels D. Check accuracy of glucose monitoring equipment

A. Observe client's glucose self-monitoring technique

A client with an electrical burn is admitted to the emergency department on a backboard with a cervical collar. Which intervention should the nurse implement? A. Place the client on cardiac telemetry B. Obtain STAT arterial blood gases C. Flush the burned area with sterile normal saline D. Elevate the client's head of bed to 45 degrees

A. Place the client on cardiac telemetry

The nurse is caring for a one-week infant who has a ventriculoperitoneal (VP) shunt that was placed 2 days after birth. Which findings are an indication of a postoperative complication? (select all that apply) A. Poor feeding and vomiting B. Leakage of CSF form the incisional site C. Hyperactive bowel sounds D. Abdominal distension E. White blood cell count of 10, 000/mm^3

A. Poor feeding and vomiting B. Leakage of CSF form the incisional site D. Abdominal distension

The nurse reviews pursed lip breathing with a patient newly diagnosed with emphysema. The nurse reinforces that this technique will assist respiration by which of the following mechanisms? A. Preventing bronchial collapse and air trapping in the lungs during exhalation B. Increasing the respiratory rate and giving the patient control of respiratory patterns C. Loosening secretions so that they may be coughed up more easily D. Promoting maximal inhalation for better oxygenation of the lungs

A. Preventing bronchial collapse and air trapping in the lungs during exhalation The focus of pursed lip breathing is to slow down the exhalation phase of respiration, which decreases bronchial collapse and subsequent air trapping in the lungs during exhalation.

While assessing a client's chest tube (CT), the nurse discovers bubbling in the water seal chamber of the chest tube collections device. The client's vital sign are: blood pressure 80/40 mmHg, heart rate 120 beats/minute, respiratory rate 32 breaths/minute, oxygen saturation 88%. Which interventions should the nurse implement? (select all that apply) A. Provide supplemental oxygen B. Auscultate bilateral lung fields C. Administer a nebulizer treatment D. Reinforce occlusive CT dressing E. Give PRN dose of pain medication

A. Provide supplemental oxygen B. Auscultate bilateral lung fields D. Reinforce occlusive CT dressing

The nurse observes an unlicensed assistive personnel (UAP) applying an alcohol-based hand rub while leaving a client's room after taking vital signs. What action should the nurse take? A. Remind the UAP to continue rubbing the hands together until they dry B. Supervise the UAP in the next client's room to evaluate hand hygiene C. Instruct the UAP to return to the clients room to perform hand washing D. Advise the UAP to wear gloves when obtaining vital signs for all clients

A. Remind the UAP to continue rubbing the hands together until they dry

The legs of a client who is receiving hospice care have become mottled in appearance. When the nurse observes the unlicensed assistive personnel (UAP) place a heating pad on the mottled areas, what action should the nurse take? A. Remove the heating pads and pace a soft blanket over the client's legs and feet B. Advise the UAP to observe the client's skin while the heating pads are in place C. Evaluate the client's feet on a pillow and monitor the client's pedal pulses frequently D. Instruct the UAP to reposition the heating pads to the sides of the legs and feet

A. Remove the heating pads and pace a soft blanket over the client's legs and feet

The scrub nurse places the fenestrated drapes to expose the operative area for a client who is having a hepatic tumor removed. The scrub nurse should assist with applying the sterile impermeable adhesive drape or surgical skin barrier to which area? A. Right subcostal B. Right lumbar C. Low transverse D. Midline abdominal

A. Right subcostal

10. If a nurse is caring for an 80-year-old patient with a temperature of 100.4° F, crackles at the right lung base, pain with deep inspiration, and dyspnea, which of the following orders is the nurse's priority? A. Sputum specimen for culture and sensitivity B. Codeine 15 mg orally every 6 hours as needed C. Incentive spirometer every 2 hours while awake D. Amoxicillin (Amoxil) 500 mg orally 4 times a day

A. Sputum specimen for culture and sensitivity The patient presents with signs of a respiratory infection. To initiate the most effective therapy, the health care prescriber must know the pathogen causing the infection. Therefore, the sputum specimen is the nurse's priority. If the antibiotic is administered before the specimen is obtained, the results of the culture might not be as accurate and could impair the effectiveness of therapy. After the specimen is obtained, the nurse can administer codeine for coughing and begin the incentive spirometry to mobilize secretions and improve the patient's ability to expectorate the secretions.

If a nurse is caring for an 80-year-old patient with a temperature of 100.4° F, crackles at the right lung base, pain with deep inspiration, and dyspnea, which of the following orders is the nurse's priority? A. Sputum specimen for culture and sensitivity B. Codeine 15 mg orally every 6 hours as needed C. Incentive spirometer every 2 hours while awake D. Amoxicillin (Amoxil) 500 mg orally 4 times a day

A. Sputum specimen for culture and sensitivity The patient presents with signs of a respiratory infection. To initiate the most effective therapy, the health care prescriber must know the pathogen causing the infection. Therefore, the sputum specimen is the nurse's priority. If the antibiotic is administered before the specimen is obtained, the results of the culture might not be as accurate and could impair the effectiveness of therapy. After the specimen is obtained, the nurse can administer codeine for coughing and begin the incentive spirometry to mobilize secretions and improve the patient's ability to expectorate the secretions.

A nurse stops at the site of a motorcycle accident and finds a young adult male lying face down in the road in a puddle of water. It is raining, no one is available to send for help, and the cell phone is in the car about 50 feet away. What action should the nurse take first? A. Stabilize the victim's neck and roll over to evaluate his status B. Examine the victim's body surfaces for arterial bleeding C. Open the airway and initiate resuscitative measures D. Return to the car to call emergency response 911 for help

A. Stabilize the victim's neck and roll over to evaluate his status

A young woman is preparing to leave for a 7-day boat trip. She requests a prescription for motion sickness, so the healthcare provider prescribes meclizine (Antivert). Which instruction should the nurse include in this client's teaching? A. Suck on hard candy for a dry mouth while taking this drug B. Sit upright for at least 30 minutes after taken this drug C. Avoid eating shellfish for 24 hours after taking this drug D. Do not drink caffeinated beverages while taking this drug

A. Suck on hard candy for a dry mouth while taking this drug

In assessing an older client with dementia for sundowning syndrome, what assessment technique is best for the nurse to use? A. Observe for tiredness at the end of the day. B. Perform a neurologic exam and mental status exam. C. Monitor for medication side effects. D. Assess for decreased gross motor movement.

A. Sundowning syndrome is a pattern of agitated behavior in the evening, believed to be associated with tiredness at the end of the day combined with fewer orienting stimuli, such as activities and interactions. (B, C, & D) with not provide information about this syndrome.

A client with Type 1 diabetes mellitus (DM) is admitted for an emergency cholecystectomy. To prevent diabetic ketoacidosis (DKA), which intervention is most important for the nurse to implement? A. Supplement insulin needs using a sliding scale B. Monitor the client for polyuria and dehydration C. Measure urine ketones when blood glucose exceeds 300 mg/dl D. Teach the client to monitor blood glucose levels frequently when ill

A. Supplement insulin needs using a sliding scale

A client with hypertension has been receiving ramipril (Altace) 5 mg PO daily for 2 weeks and is scheduled to receive a dose at 0900. At 0830 the client's blood pressure is 120/70. Which action should the nurse take? A. Administer the dose as prescribed. B. Hold the dose and contact the healthcare provider. C. Hold the dose and recheck the blood pressure in 1 hour. D. Check the healthcare provider's prescription to clarify the dose.

A. The BP is WNL and indicates that the medication is working. (B & C) would be indicated if the BP was low (systole below 100). (D) is not required because the dose is within manufacture's recommendations.

During assessment of a client in the intensive care unit, the nurse notes that the client's breath sounds are clear upon auscultation, but jugular vein distention and muffled heart sounds are present. Which intervention should the nurse implement? A. Prepare the client for a pericardial tap. B. Administer intravenous furosemide (Lasix). C. Assist the client to cough and deep breathe. D. Instruct the client to restrict oral fluid intake.

A. The client is exhibiting symptoms of cardiac tamponade that results in reduced cardiac output. Treatment is pericardial tap. (B) is not a treatment. (C) is not priority. (D) Fluids are frequently increased but this is not as priority as (A).

Which expected outcome statement should the nurse include in a teaching plan designed to assist a client with management of an acute attack of gout? A. The client will avoid use of alcohol in managing stress. B. The client will implement a high-purine daily dietary regime. C. The client will use local heat applications for acute pain. D. The client will stop antigout medication once pain subsides.

A. The client will avoid use of alcohol in managing stress. Gout, an error in purine metabolism or excretion that results in urate crystal disposition usually in a joint of the great toe, causes acute joint inflammation and severe pain. Alcohol consumption should be avoided because it increases production or uric acid and prevents excretion

A client with cirrhosis states that his disease was cause by a blood transfusion. What information should the nurse obtain first to provide effective client teaching? A. The year the blood transfusion was received B. The amount of alcohol the client drinks C. How long the client has had cirrhosis D. The client's normal coping mechanisms

A. The nurse should first verify the clients explanation (A) since it may be accurate due to prior to 1990 blood was not screened for Hep C and hep C can cause cirrhosis. Not all cirrhosis is caused is caused by alcoholism (B) (C & D) provide useful but less relevant information.

9. Before beginning a transfusion of RBCs, which of the following actions by the nurse would be of highest priority to avoid an error during this procedure? A. Check the identifying information on the unit of blood against the patient's ID bracelet. B. Select new primary IV tubing primed with lactated Ringer's solution to use for the transfusion. C. Add the blood transfusion as a secondary line to the existing IV and used the IV controller to maintain correct flow. D. Remain with the patient for 60 minutes after beginning the transfusion to watch for signs of a transfusion reaction. The patient's identifying information (name, date of birth, medical record number) on the identification bracelet should exactly match the information on the blood bank tag that has been placed on the unit of blood. If any information does not match, the transfusions should not be hung because of possible error and risk to the patient.

A. The patient's identifying information (name, date of birth, medical record number) on the identification bracelet should exactly match the information on the blood bank tag that has been placed on the unit of blood. If any information does not match, the transfusions should not be hung because of possible error and risk to the patient.

Before beginning a transfusion of RBCs, which of the following actions by the nurse would be of highest priority to avoid an error during this procedure? A. Check the identifying information on the unit of blood against the patient's ID bracelet. B. Select new primary IV tubing primed with lactated Ringer's solution to use for the transfusion. C. Add the blood transfusion as a secondary line to the existing IV and used the IV controller to maintain correct flow. D. Remain with the patient for 60 minutes after beginning the transfusion to watch for signs of a transfusion reaction. The patient's identifying information (name, date of birth, medical record number) on the identification bracelet should exactly match the information on the blood bank tag that has been placed on the unit of blood. If any information does not match, the transfusions should not be hung because of possible error and risk to the patient.

A. The patient's identifying information (name, date of birth, medical record number) on the identification bracelet should exactly match the information on the blood bank tag that has been placed on the unit of blood. If any information does not match, the transfusions should not be hung because of possible error and risk to the patient.

The nurse is responding to telephone messages at a psychiatric day clinic. Which client situation requires immediate intervention by the nurse? A. The wife of a client with post-traumatic stress syndrome reports that her husband is threatening to kill her. B. A client with depression who is crying and tells the nurse that he has has suicidal thoughts C. A young adult diagnosed with a somatoform disorder reports having a severe headache that has become unbearable D. An adult heroin abuser who reports the onset of withdrawal and requests a refill for a prescription for methadone

A. The wife of a client with post-traumatic stress syndrome reports that her husband is threatening to kill her.

19. The nurse is caring for a postoperative patient with sudden onset of respiratory distress. The physician orders a STAT ventilation-perfusion scan. Which of the following explanations should the nurse provide to the patient about the procedure? A. This test involves injection of a radioisotope to outline the blood vessels in the lungs, followed by inhalation of a radioisotope gas. B. This test will use special technology to examine cross sections of the chest with use of a contrast dye. C. This test will use magnetic fields to produce images of the lungs and chest. D. This test involves injecting contrast dye into a blood vessel to outline the blood vessels of the lungs.

A. This test involves injection of a radioisotope to outline the blood vessels in the lungs, followed by inhalation of a radioisotope gas.A ventilation-perfusion scan has two parts. In the perfusion portion, a radioisotope is injected into the blood and the pulmonary vasculature is outlined. In the ventilation part, the patient inhales a radioactive gas that outlines the alveoli.

A client is placed on a mechanical ventilator following a cerebral hemorrhage, and vecuronium bromide (Norcuron) 0.04 mg/kg every 12 hours IV is prescribed. What is the priority nursing diagnosis for this client? A. Impaired communication related to paralysis of skeletal muscles. B. Hight risk or infection related to increased ICP. C. Potential for injury related to impaired lung expansion. D. Social isolation related to inability to communicate.

A. To increase the client's tolerance of the endotracheal intubation and/or mechanical ventilation, a skeletal-muscle relaxant such as vecuronium is usually prescribed. (A) is a serious outcome because the client cannot communicate his/her needs. (D) is not as much of a priority. (B) infection is not related to ICP. (C) is incorrect because the ventilator will ensure that the lungs are expanded.

A patient with acute exacerbation of COPD needs to receive precise amounts of oxygen. Which of the following types of equipment should the nurse prepare to use? A. Venturi mask B. Partial non-rebreather mask C. Oxygen tent D. Nasal cannula

A. Venturi mask The Venturi mask delivers precise concentrations of oxygen and should be selected whenever this is a priority concern. The other methods are less precise in terms of amount of oxygen delivered.

A nurse is working on a respiratory care unit where many of the patients are affected by asthma. Which of the following actions by the nurse would most likely increase respiratory difficulty for the patients? A. Wearing perfume to work B. Encouraging patients to ambulate daily C. Allowing the patients to eat green leafy vegetables D. Withholding antibiotic therapy until cultures are obtained

A. Wearing perfume to work People with asthma should avoid extrinsic allergens and irritants (e.g., dust, pollen, smoke, certain foods, colognes and perfumes, certain types of medications) because their airways become inflamed, producing shortness of breath, chest tightness, and wheezing. Many green leafy vegetables are rich in vitamins, minerals, and proteins, which incorporate healthy lifestyle patterns into the patients' daily living routines. Routine exercise is a part of a prudent lifestyle, and for patients with asthma the physical and psychosocial effects of ambulation can incorporate feelings of well-being, strength, and enhancement of physical endurance. Antibiotic therapy is always initiated after cultures are obtained so that the sensitivity to the organism can be readily identified.

The nurse is caring for a patient with an acute exacerbation of asthma. Following initial treatment, which of the following findings indicates to the nurse that the patient's respiratory status is improving? A. Wheezing becomes louder B. Vesicular breath sounds decrease C. Aerosol bronchodilators stimulate coughing D. The cough remains nonproductive

A. Wheezing becomes louder The primary problem during an exacerbation of asthma is narrowing of the airway and subsequent diminished air exchange. As the airways begin to dilate, wheezing gets louder because of better air exchange.

When planning care for a client with acute pancreatitis, which nursing intervention has the highest priority? A. Withhold food and fluid intake B. Initiate IV fluid replacement C. Administer antiemetics as needed D. Evaluate intake and output ratio

A. Withhold food and fluid intake

In making client care assignments, which client is best to assign to the practical nurse (PN) working on the unit with a nurse? A. an immobile client receiving low molecular weight heparin Q12H B. a client who is receiving a continuous infusion of heparin and gets out of bed BID C. A client how is being titrated off a heparin infusion and started on PO warfarin D. An ambulatory client receiving warfarin with an INR of 5 seconds.

A. an immobile client receiving low molecular weight heparin Q12H

a males client was transferred yesterday from the Emergency department to the telemetry unit because he had ST depression and resolved chest pain. When his EKG monitor alarms for ventricular tachycardia (VT), what action should the nurse take first? A. determine the clients responsiveness and respirations B. bring the crash cart to the room to defibrillate the client C. immediately initiate chest compressions D. notify the emergency response team

A. determine the clients responsiveness and respirations

28. To find the infection site associated with acute lymphangitis, the nurse should look _____ to the inflammation. A. distal B. anterior C. proximal D. contralateral

A. distal The nurse should assess distal to swelling to locate the initial site of infection. Examining proximal, contralateral, or anterior to the inflammation does not describe swelling associated with infection.

To find the infection site associated with acute lymphangitis, the nurse should look _____ to the inflammation. A. distal B. anterior C. proximal D. contralateral

A. distal The nurse should assess distal to swelling to locate the initial site of infection. Examining proximal, contralateral, or anterior to the inflammation does not describe swelling associated with infection.

12. A nurse is providing care to an adult female patient and observes that the Hb laboratory analysis result is 9 g/dl. Based on this finding, the nurse should expect to observe A. dyspnea. B. bradycardia. C. warm, dry skin. D. activity tolerance without complaint of fatigue.

A. dyspnea.Hb levels are used to determine the severity of anemia. Patients with moderate anemia (Hb 6 to 10 g/dL) may suffer from dyspnea, palpitations, diaphoresis with exertion, and chronic fatigue. Patients who are anemic usually have cool skin related to compensatory mechanism of mild vasoconstriction. Patients who are anemic experience tachycardia because of increased demands placed on the heart to meet overall metabolic requirements. Activity tolerance without complaint is not correct because patients with anemic conditions fatigue readily.

A nurse is providing care to an adult female patient and observes that the Hb laboratory analysis result is 9 g/dl. Based on this finding, the nurse should expect to observe A. dyspnea. B. bradycardia. C. warm, dry skin. D. activity tolerance without complaint of fatigue.

A. dyspnea.Hb levels are used to determine the severity of anemia. Patients with moderate anemia (Hb 6 to 10 g/dL) may suffer from dyspnea, palpitations, diaphoresis with exertion, and chronic fatigue. Patients who are anemic usually have cool skin related to compensatory mechanism of mild vasoconstriction. Patients who are anemic experience tachycardia because of increased demands placed on the heart to meet overall metabolic requirements. Activity tolerance without complaint is not correct because patients with anemic conditions fatigue readily.

35. A patient is admitted to the hospital with fever, chills, a productive cough with rusty sputum, and pleuritic chest pain. Pneumococcal pneumonia is suspected. An appropriate nursing diagnosis for the patient based on the patient's manifestations is A. hyperthermia related to acute infectious process. B. chronic pain related to ineffective pain management. C. risk for injury related to disorientation and confusion. D. ineffective airway clearance related to retained secretions.

A. hyperthermia related to acute infectious process. The patient with pneumococcal pneumonia is acutely ill with fever and the systemic manifestations of fever, such as chills, thirst, headache, and malaise. Interventions that monitor temperature and aid in lowering body temperature are appropriate. Ineffective airway clearance would be manifested by adventitious breath sounds and difficulty producing secretions. Disorientation and confusion are not noted in this patient and are not typical unless the patient is very hypoxemic. Pleuritic pain is an acute pain that is due to inflammation of the pleura.

A patient is admitted to the hospital with fever, chills, a productive cough with rusty sputum, and pleuritic chest pain. Pneumococcal pneumonia is suspected. An appropriate nursing diagnosis for the patient based on the patient's manifestations is A. hyperthermia related to acute infectious process. B. chronic pain related to ineffective pain management. C. risk for injury related to disorientation and confusion. D. ineffective airway clearance related to retained secretions.

A. hyperthermia related to acute infectious process. The patient with pneumococcal pneumonia is acutely ill with fever and the systemic manifestations of fever, such as chills, thirst, headache, and malaise. Interventions that monitor temperature and aid in lowering body temperature are appropriate. Ineffective airway clearance would be manifested by adventitious breath sounds and difficulty producing secretions. Disorientation and confusion are not noted in this patient and are not typical unless the patient is very hypoxemic. Pleuritic pain is an acute pain that is due to inflammation of the pleura.

A male client who has never smoked but has had COPD for the past 5 years is now being assessed for cancer of the lung. The nurse knows that he is most likely to develop which type of lung cancer? A. Adenocarcinoma B. Oat-cell carcinoma C. Malignant melanoma D. Squamous-cell carcinoma

A. is the only lung cancer not related to cigarette smoking related to lung scarring and fibrosis from preexisting pulmonary diseases such as TB and COPD. (B& D) are related to smoking. (C) is a skin cancer

16. To ensure the correct amount of oxygen delivery for a patient receiving 35% oxygen via a Venturi mask, it is most important that the nurse A. keep the air-entrainment ports clean and unobstructed. B. apply an adaptor to increase humidification of the oxygen. C. drain moisture condensation from the oxygen tubing every hour. D. keep the flow rate high enough to keep the bag from collapsing during inspiration.

A. keep the air-entrainment ports clean and unobstructed. Oxygen is delivered to a small jet in the center of a wide-based cone. Air is entrained (pulled through) openings in the cone as oxygen flows through the small jet. The degree of restriction or narrowness of the jet determines the amount of entrainment and the dilution of pure oxygen with room air and thus the concentration of oxygen. Although applying an adaptor can increase the humidification with the Venturi mask, it is not the best answer, because an open port is essential to proper functioning. Draining moisture condensation from the oxygen tubing is performed as often as needed, not on an hourly schedule. A plastic face mask with a reservoir bag needs to have sufficient flow rate to keep the bag inflated.

To ensure the correct amount of oxygen delivery for a patient receiving 35% oxygen via a Venturi mask, it is most important that the nurse A. keep the air-entrainment ports clean and unobstructed. B. apply an adaptor to increase humidification of the oxygen. C. drain moisture condensation from the oxygen tubing every hour. D. keep the flow rate high enough to keep the bag from collapsing during inspiration.

A. keep the air-entrainment ports clean and unobstructed. Oxygen is delivered to a small jet in the center of a wide-based cone. Air is entrained (pulled through) openings in the cone as oxygen flows through the small jet. The degree of restriction or narrowness of the jet determines the amount of entrainment and the dilution of pure oxygen with room air and thus the concentration of oxygen. Although applying an adaptor can increase the humidification with the Venturi mask, it is not the best answer, because an open port is essential to proper functioning. Draining moisture condensation from the oxygen tubing is performed as often as needed, not on an hourly schedule. A plastic face mask with a reservoir bag needs to have sufficient flow rate to keep the bag inflated.

In reviewing the preoperative laboratory findings of an adult male client who is scheduled for colon resection in the morning, the nurse notes that the client has a hemoglobin of 9 grams/dl. After verifying the accuracy of this finding with the laboratory, what action should the nurse take? A. notify the surgeon of the laboratory findings B. collect another specimen for analysis C. confirm the availability of compatible units of blood D. administer a bolus of normal saline preoperatively

A. notify the surgeon of the laboratory findings

20. Using light pressure with the index and middle fingers, the nurse cannot palpate any of the patient's superficial lymph nodes. The nurse A. records this finding as normal. B. should reassess the lymph nodes using deeper pressure. C. asks the patient about any history of any radiation therapy. D. notifies the health care provider that x-rays of the nodes will be necessary.

A. records this finding as normal. Superficial lymph nodes are evaluated by light palpation, but they are not normally palpable. It may be normal to find small (<1.0 cm), mobile, firm, nontender nodes. Deep lymph nodes are detected radiographically.

Using light pressure with the index and middle fingers, the nurse cannot palpate any of the patient's superficial lymph nodes. The nurse A. records this finding as normal. B. should reassess the lymph nodes using deeper pressure. C. asks the patient about any history of any radiation therapy. D. notifies the health care provider that x-rays of the nodes will be necessary.

A. records this finding as normal. Superficial lymph nodes are evaluated by light palpation, but they are not normally palpable. It may be normal to find small (<1.0 cm), mobile, firm, nontender nodes. Deep lymph nodes are detected radiographically.

17. While caring for a patient with respiratory disease, a nurse observes that the oxygen saturation drops from 94% to 85% when the patient ambulates. The nurse should determine that A. supplemental oxygen should be used when the patient exercises. B. ABG determinations should be done to verify the oxygen saturation reading. C. this finding is a normal response to activity and that the patient should continue to be monitored. D. the oximetry probe should be moved from the finger to the earlobe for an accurate oxygen saturation measurement during activity.

A. supplemental oxygen should be used when the patient exercises.An oxygen saturation lower than 90% indicates inadequate oxygenation. If the drop is related to activity of some type, supplemental oxygen is indicated.

While caring for a patient with respiratory disease, a nurse observes that the oxygen saturation drops from 94% to 85% when the patient ambulates. The nurse should determine that A. supplemental oxygen should be used when the patient exercises. B. ABG determinations should be done to verify the oxygen saturation reading. C. this finding is a normal response to activity and that the patient should continue to be monitored. D. the oximetry probe should be moved from the finger to the earlobe for an accurate oxygen saturation measurement during activity.

A. supplemental oxygen should be used when the patient exercises.An oxygen saturation lower than 90% indicates inadequate oxygenation. If the drop is related to activity of some type, supplemental oxygen is indicated.

The nurse notices clear nasal drainage in a patient newly admitted with facial trauma, including a nasal fracture. The nurse should: A. test the drainage for the presence of glucose. B. suction the nose to maintain airway clearance. C. document the findings and continue monitoring. D. apply a drip pad and reassure the patient this is normal.

A. test the drainage for the presence of glucose. Clear nasal drainage suggests leakage of cerebrospinal fluid (CSF). The drainage should be tested for the presence of glucose, which would indicate the presence of CSF.

43. Select all that apply. Which of the following is included in a comprehensive respiratory assessment? A. Pulse oximetry B. Chest auscultation C. Apical radial pulse D. Nail-bed assessment E. Evaluation of respiratory effort F. Rate and character of respirations

ABDEF The total assessment of the respiratory system includes pulse oximetry; auscultation; skin and nail-bed assessment for the detection of cyanosis; and rate, character, and degree of effort of respirations. The apical radial pulse is a cardiac assessment.

42. Select all that apply. Which of the following nursing actions can help clear tracheobronchial secretions in a patient with cystic fibrosis? A. Postural drainage B. Suppressing the cough C. Ensuring adequate hydration D. Administering mucolytic aerosols E. Encouraging the patient to lie flat F. Administering water-soluble vitamins

ACD Postural drainage, adequate hydration, and administration of mucolytic aerosols all encourage coughing and the clearing of secretions. A patient with cystic fibrosis will be more comfortable sitting upright.

41. Select all that apply. During initial assessment, a nurse should record which of the following manifestations of respiratory distress? A. Tachypnea B. Nasal flaring C. Thready pulse D. Panting or grunting E. Use of intercostal muscles F. An inspiratory-to-expiratory ratio of 1:2

AD Manifestations of respiratory distress include tachypnea, grunting and panting on respiration, central cyanosis, use of accessory muscles, and flaring nares.

Client admitted with complaints of severe lower abdominal pain, cramping, and diarrhea is diagnosed as having diverticulosis. what are the nutritional needs of this client throughout recovery?

Acute phase: in NPO graduating to liquids. Recovery phase: no fiber or foods that irritate the bowl Maintenance phase: high fiber diet with bulk forming laxatives to prevent pooling of foods in the pouches where they can become inflamed. avoidance of small poorly digested food such as popcorn, nuts, seeds

Life-threatening medical emergency form of Addison's Disease

Addison's Crisis

Hyperpigmentation at sun-exposed areas, pressure points, on joints, and palmar creases usually indicates

Addison's Disease

Metabolic acidosis: cause

Addition of large amounts of fixed acids to body fluids Lactic acidiosis keto acidosis phosphates and sulfates acid indigestion adrenal insufficiency

Decorticate posturing would look something like this:

Adduction of arms at shoulders Flexion of arms on chest with wrists flexed Hands fisted

Mechanical causes of intestinal obstruction

Adhesions hernia (strangulates the gut) Volvulus (twisting of the gut) Intussusception (telescoping of the gut within itself) Tumors

Abdominal Pain Intermittent Tenesmus Exacerbations of Severe Diarrhea

All symptoms exhibited with ulcerative colitis

Difficulty initiating stream Weak stream Urgency Dribbling Nocturia Intermittancy

All symptoms of BPH

Cell Membrane

Allows materials to pass into and out of the cell.

This lab would be elevated with acute pancreatitis

Amylase

Hyponatremia: signs and symptoms

Anorexia nausea / vomiting weakness Lethargy confusion seizures Na < 135

Crohn's Disease

Anywhere in terminal ileum Autoimmune Strictures, Fissures, Fistulas (UTI first sign of fistula) Weight Loss

Severe pain at McBurney's point and nausea would indicate

Appendicitis

In assessing cancer risk, the nurse identifies which woman as being at greatest risk of developing breast cancer? A) A 35-year-old multipara who never breastfed. B) A 50-year-old whose mother had unilateral breast cancer. C) A 55-year-old whose mother-in-law had bilateral breast cancer. D) A 20-year-old whose menarche occurred at age 9.

B) A 50-year-old whose mother had unilateral breast cancer. The most predictive risk factors for development of breast cancer are over 40 years of age and a positive family history (occurrence in the immediate family, i.e., mother or sister). Other risk factors include nulliparity, no history of breastfeeding, early menarche and late menopause. Although all of the women described have one of the risk factors for developing breast cancer, (B) has the greater risk over (A, C, and D).

A female client requests information about using the calendar method of contraception. Which assessment is most important for the nurse to obtain? A) Amount of weight gain or weight loss during the previous year. B) An accurate menstrual cycle diary for the past 6 to 12 months. C) Skin pigmentation and hair texture for evidence of hormonal changes. D) Previous birth-control methods and beliefs about the calendar method.

B) An accurate menstrual cycle diary for the past 6 to 12 months. The fertile period, which occurs 2 weeks prior to the onset of menses, is determined using an accurate record of the number of days of the menstrual cycles for the past 6 months, so it is most important to emphasize to the client that accuracy and compliancy of a menstrual diary (B) is the basis of the calendar method. (A and C) may be partially related to hormonal fluctuations but are not indicators for using the calendar method. (D) may demonstrate client understanding and compliancy but is not the most important aspect.

The nurse is taking a history of a newly diagnosed Type 2 diabetic who is beginning treatment. Which subjective information is most important for the nurse to note? A) A history of obesity. B) An allergy to sulfa drugs. C) Cessation of smoking three years ago. D) Numbness in the soles of the feet.

B) An allergy to sulfa drugs. An allergy to sulfa drugs may make the client unable to use some of the most common antihyperglycemic agents (sulfonylureas). The nurse needs to highlight this allergy for the healthcare provider. (A) is common and warrants counseling, but does not have the importance of (B). (C) does increase the risk for vascular disease, but it is not as important to the treatment regimen as (B). Diabetic neuropathy, as indicated by (D), is common with diabetics, but when the serum glucose is decreased, new onset numbness can possibly improve.

During suctioning, a client with an uncuffed tracheostomy tube begins to cough violently and dislodges the tracheostomy tube. Which action should the nurse implement first? A) Notify the healthcare provider for reinsertion. B) Attempt to reinsert the tracheostomy tube. C) Position the client in a lateral position with the neck extended. D) Ventilate client's tracheostomy stoma with a manual bag-mask.

B) Attempt to reinsert the tracheostomy tube. The nurse should attempt to reinsert the tracheostomy tube (B) by using a hemostat to open the tracheostomy or by grasping the retention sutures (if present) to spread the opening in insert a replacement tube (with its obturator) into the stoma. Once in place, the obturator should immediately be removed. (A, C, and D) place the client at risk of airway obstruction.

A 58-year-old client who has been post-menopausal for five years is concerned about the risk for osteoporosis because her mother has the condition. Which information should the nurse offer? A) Osteoporosis is a progressive genetic disease with no effective treatment. B) Calcium loss from bones can be slowed by increasing calcium intake and exercise. C) Estrogen replacement therapy should be started to prevent the progression osteoporosis. D) Low-dose corticosteroid treatment effectively halts the course of osteoporosis.

B) Calcium loss from bones can be slowed by increasing calcium intake and exercise. Post-menopausal females are at risk for osteoporosis due to the cessation of estrogen secretion, but a regimen including calcium, vitamin D, and weight-bearing exercise can prevent further bone loss (B). Osteoporosis can be managed with conservative therapy, such as bone metabolism regulators and estrogen replacement therapy (ERT) to improve bone density, but it is not a genetic disease (A). Although ERT is effective in managing osteoporosis, an increased risk for cancer and heart disease should be considered for individual clients. Corticosteroid therapy promotes bone resorption and is counterproductive in maintaining or increasing bone density (D).

The nurse notes that the only ECG for a 55-year-old male client scheduled for surgery in two hours is dated two years ago. The client reports that he has a history of "heart trouble," but has no problems at present. Hospital protocol requires that those over 50 years of age have a recent ECG prior to surgery. What nursing action is best for the nurse to implement? A) Ask the client what he means by "heart trouble." B) Call for an ECG to be performed immediately. C) Notify surgery that the ECG is over two years old. D) Notify the client's surgeon immediately.

B) Call for an ECG to be performed immediately. Clients over the age of 40 and/or with a history of cardiovascular disease, should receive ECG evaluation prior to surgery, generally 24 hours to two weeks before. (B) should be implemented to ensure that the client's current cardiovascular status is stable. Additional data might be valuable (A), but since time is limited, the priority is to obtain the needed ECG. Documentation of vital signs is important, but does not replace the need for the ECG (C). The surgeon only needs to be notified if the ECG cannot be completed, or if there is a significant problem (D).

The nurse is caring for a client with a continuous feeding through a percutaneous endoscopic gastrostomy (PEG) tube. Which intervention should the nurse include in the plan of care? A) Flush the tube with 50 ml of water q 8 hours. B) Check for tube placement and residual volume q4 hours. C) Obtain a daily x- ray to verify tube placement. D) Position on left side with head of bed elevated 45 degrees.

B) Check for tube placement and residual volume q4 hours. Tube placement and residual volume should be checked before each feeding (B). Tube placement is checked by aspiration of stomach contents and measurement of pH. It is important to check for residual volume because gastric emptying is often delayed during illness. There is an increased risk for aspiration of the feeding with increased residual volume. (A, C, and D) are not correct procedures to follow.

The nurse is caring for a client with a stroke resulting in right-sided paresis and aphasia. The client attempts to use the left hand for feeding and other self-care activities. The spouse becomes frustrated and insists on doing everything for the client. Based on this data, which nursing diagnosis should the nurse document for this client? A) Situational low self-esteem related to functional impairment and change in role function. B) Disabled family coping related to dissonant coping style of significant person. C) Interrupted family processes related to shift in health status of family member. D) Risk for ineffective therapeutic regimen management related to complexity of care.

B) Disabled family coping related to dissonant coping style of significant person. A stroke affects the whole family and in this case the spouse probably thinks that she is helping and needs to feel that she is contributing to the client's care. Her help is noted as being incongruent with attempts of self-care by the client thereby disabling family coping (B). The scenario does not discuss the client's self-esteem (A), interrupted family processes (C) or the risk for ineffective therapeutic regimen (D).

A 49-year-old female client arrives at the clinic for an annual exam and asks the nurse why she becomes excessively diaphoretic and feels warm during nighttime. What is the nurse's best response? A) Explain the effect of the follicle-stimulating and luteinizing hormones. B) Discuss perimenopause and related comfort measures. C) Assess lung fields and for a cough productive of blood-tinged mucous. D) Ask if a fever above 101º F has occurred in the last 24 hours.

B) Discuss perimenopause and related comfort measures. The perimenopausal period begins about 10 years before menopause with the cessation of menstruation at the average ages of 52 to 54. Lower estrogen levels causes FSH and LH secretion in bursts (surges), which triggers vasomotor instability, night sweats, and hot flashes, so discussions about the perimenopausal body's changes, comfort measures (B), and treatment options should be provided. In-depth pathophysiology of the symptoms (A) may only confuse the client. There is no indication that the client has tuberculosis and an infection, so (C and D) are not indicated.

Which intervention should the nurse implement for a female client diagnosed with pelvic relaxation disorder? A) Describe proper administration of vaginal suppositories and cream. B) Encourage the client to perform Kegel exercises 10 times daily. C) Explain the importance of using condoms when having sexual intercourse. D) Discuss the importance of keeping a diary of daily temperature and menstrual cycle events.

B) Encourage the client to perform Kegel exercises 10 times daily. Pelvic relaxation disorders are structural disorders resulting from weakening support tissues of the pelvis. (B) helps strengthen the surrounding muscles. Medication will not help correct a cystocele, rectocele, or uterine prolapse (A). (C) will help prevent sexually transmitted diseases. (D) is used to identify fertile times during the woman's menstrual cycle.

A male client receives a local anesthetic during surgery. During the post-operative assessment, the nurse notices the client is slurring his speech. Which action should the nurse take? A) Determine the client is anxious and allow him to sleep. B) Evaluate his blood pressure, pulse, and respiratory status. C) Review the client's pre-operative history for alcohol abuse. D) Continue to monitor the client for reactivity to anesthesia.

B) Evaluate his blood pressure, pulse, and respiratory status. Slurred speech in the post-operative client who received a local anesthetic is an atypical finding and may indicate neurological deficits that require further assessment, so obtaining the client's vital signs (B) will provide information about possible cardiovascular complications, such as stroke. The client's anxiety (A), a history of alcohol abuse (D), or local anesthesia (D) are unrelated to the client's sudden onset of slurred speech.

Which symptoms should the nurse expect a client to exhibit who is known to have a pheochromocytoma? A) Numbness, tingling, and cramps in the extremities. B) Headache, diaphoresis, and palpitations. C) Cyanosis, fever, and classic signs of shock. D) Nausea, vomiting, and muscular weakness.

B) Headache, diaphoresis, and palpitations. (B) is the typical triad of symptoms of tumors of the adrenal medulla (symptoms depend on the relative proportions of epinephrine and norepinephrine secretion). (A) lists the signs of latent tetany, exhibited by clients diagnosed with hypoparathyroidism. (C) lists the signs of an Addisonian (adrenal) crisis. (D) lists the signs of hyperparathyroidism.

A 32-year-old female client complains of severe abdominal pain each month before her menstrual period, painful intercourse, and painful defecation. Which additional history should the nurse obtain that is consistent with the client's complaints? A) Frequent urinary tract infections. B) Inability to get pregnant. C) Premenstrual syndrome. D) Chronic use of laxatives.

B) Inability to get pregnant. Dysmenorrhea, dyspareunia, and difficulty or painful defecation are common symptoms of endometriosis, which is the abnormal displacement of endometrial tissue in the dependent areas of the pelvic peritoneum. A history of infertility (B) is another common finding associated with endometriosis. Although (A, C, and D) are common, nonspecific gynecological complaints, the most common complaints of the client with endometriosis are pain and infertility.

The nurse is providing dietary instructions to a 68-year-old client who is at high risk for development of coronary heart disease (CHD). Which information should the nurse include? A) Limit dietary selection of cholesterol to 300 mg per day. B) Increase intake of soluble fiber to 10 to 25 grams per day. C) Decrease plant stanols and sterols to less than 2 grams/day. D) Ensure saturated fat is less than 30% of total caloric intake.

B) Increase intake of soluble fiber to 10 to 25 grams per day. To reduce risk factors associated with coronary heart disease, the daily intake of soluble fiber (B) should be increased to between 10 and 25 gm. Cholesterol intake (A) should be limited to 180 mg/day or less. Intake of plant stanols and sterols is recommended at 2 g/day (C). Saturated fat (D) intake should be limited to 7% of total daily calories.

Which reaction should the nurse identify in a client who is responding to stimulation of the sympathetic nervous system? A) Pupil constriction. B) Increased heart rate. C) Bronchial constriction. D) Decreased blood pressure.

B) Increased heart rate. Any stressor that is perceived as threatening to homeostasis acts to stimulate the sympathetic nervous system and manifests as a flight-or-fight response, which includes an increase in heart rate (B). (A, C, and D) are responses of the parasympathetic nervous system.

A client who has heart failure is admitted with a serum potassium level of 2.9 mEq/L. Which action is most important for the nurse to implement? A) Give 20 mEq of potassium chloride. B) Initiate continuous cardiac monitoring. C) Arrange a consultation with the dietician. D) Teach about the side effects of diuretics.

B) Initiate continuous cardiac monitoring. Hypokalemia (normal 3.5 to 5 mEq/L) causes changes in myocardial irritability and ECG waveform, so it is most important for the nurse to initiate continuous cardiac monitoring (B) to identify ventricular ectopy or other life-threatening dysrhythmias. Potassium chloride (A) should be given after cardiac monitoring is initiated so that the effects of potassium replacement on the cardiac rhythm can be monitored. (C and D) should be implemented when the client is stable.

A 20-year-old female client calls the nurse to report a lump she found in her breast. Which response is the best for the nurse to provide? A) Check it again in one month, and if it is still there schedule an appointment. B) Most lumps are benign, but it is always best to come in for an examination. C) Try not to worry too much about it, because usually, most lumps are benign. D) If you are in your menstrual period it is not a good time to check for lumps.

B) Most lumps are benign, but it is always best to come in for an examination. (B) provides the best response because it addresses the client's anxiety most effectively and encourages prompt and immediate action for a potential problem. (A) postpones treatment if the lump is malignant, and does not relieve the client's anxiety. (C and D) provide false reassurance and do not help relieve anxiety.

A client with a 16-year history of diabetes mellitus is having renal function tests because of recent fatigue, weakness, elevated blood urea nitrogen, and serum creatinine levels. Which finding should the nurse conclude as an early symptom of renal insufficiency? A) Dyspnea. B) Nocturia. C) Confusion. D) Stomatitis.

B) Nocturia As the glomerular filtration rate decreases in early renal insufficiency, metabolic waste products, including urea, creatinine, and other substances, such phenols, hormones, electrolytes, accumulate in the blood. In the early stage of renal insufficiency, polyuria results from the inability of the kidneys to concentrate urine and contribute to nocturia (B). (A, C, and D) are more common in the later stages of renal failure.

Which client should the nurse recognize as most likely to experience sleep apnea? A) Middle-aged female who takes a diuretic nightly. B) Obese older male client with a short, thick neck. C) Adolescent female with a history of tonsillectomy. D) School-aged male with a history of hyperactivity disorder.

B) Obese older male client with a short, thick neck. Sleep apnea is characterized by lack of respirations for 10 seconds or more during sleep and is due to the loss of pharyngeal tone which allows the pharynx to collapse during inspiration and obstructs air flow through the nose and mouth. With obstructive sleep apnea, the client is often obese or has a short, thick neck as in (B). (A, C, and D) are not typically prone to sleep apnea.

Which intervention should the nurse plan to implement when caring for a client who has just undergone a right above-the-knee amputation? A) Maintain the residual limb on three pillows at all times. B) Place a large tourniquet at the client's bedside. C) Apply constant, direct pressure to the residual limb. D) Do not allow the client to lie in the prone position.

B) Place a large tourniquet at the client's bedside. A large tourniquet should be placed in plain sight at the client's bedside (B). If severe bleeding occurs, the tourniquet should be readily available and applied to the residual limb to control hemorrhage. The residual limb should not be placed on a pillow (A) because a flexion contracture of the hip may result. (C) should be avoided because it may compromise wound healing. (D) should be encouraged to stretch the flexor muscles and to prevent flexion contracture of the hip.

During CPR, when attempting to ventilate a client's lungs, the nurse notes that the chest is not moving. What action should the nurse take first? A) Use a laryngoscope to check for a foreign body lodged in the esophagus. B) Reposition the head to validate that the head is in the proper position to open the airway. C) Turn the client to the side and administer three back blows. D) Perform a finger sweep of the mouth to remove any vomitus.

B) Reposition the head to validate that the head is in the proper position to open the airway. The most frequent cause of inadequate aeration of the client's lungs during CPR is improper positioning of the head resulting in occlusion of the airway (B). A foreign body can occlude the airway, but this is not common unless choking preceded the cardiac emergency, and (A, C and D) should not be the nurse's first action.

The nurse is planning care for a client who has a right hemispheric stroke. Which nursing diagnosis should the nurse include in the plan of care? A) Impaired physical mobility related to right-sided hemiplegia. B) Risk for injury related to denial of deficits and impulsiveness. C) Impaired verbal communication related to speech-language deficits. D) Ineffective coping related to depression and distress about disability.

B) Risk for injury related to denial of deficits and impulsiveness. With right-brain damage, a client experience difficulty in judgment and spatial perception and is more likely to be impulsive and move quickly, which placing the client at risk for falls (B). Although clients with right and left hemisphere damage may experience impaired physical mobility, the client with right brain damage will manifest physical impairments on the contralateral side of the body, not the same side (A). The client with a left-brain injury may manifest right-sided hemiplegia with speech or language deficits (C). A client with left-brain damage is more likely to be aware of the deficits and experience grief related to physical impairment and depression (D).

The nurse is assessing a client's laboratory values following administration of chemotherapy. Which lab value leads the nurse to suspect that the client is experiencing tumor lysis syndrome (TLS)? A) Serum PTT of 10 seconds. B) Serum calcium of 5 mg/dl. C) Oxygen saturation of 90%. D) Hemoglobin of 10 g/dl.

B) Serum calcium of 5 mg/dl. TLS results in hyperkalemia, hypocalcemia, hyperuricemia, and hyperphosphatemia. A serum calcium level of 5 (B), which is low, is an indicator of possible tumor lysis syndrome. (A, C, and D) are not particularly related to TLS.

Physical examination of a comatose client reveals decorticate posturing. Which statement is accurate regarding this client's status based upon this finding? A) A cerebral infectious process is causing the posturing. B) Severe dysfunction of the cerebral cortex has occurred. C) There is a probable dysfunction of the midbrain. D) The client is exhibiting signs of a brain tumor.

B) Severe dysfunction of the cerebral cortex has occurred. Decorticate posturing (adduction of arms at shoulders, flexion of arms on chest with wrists flexed and hands fisted and extension and adduction of extremities) is seen with severe dysfunction of the cerebral cortex (B). (A) is characteristic of meningitis. (C) is characterized by decerebrate posturing (rigid extension and pronation of arms and legs). A client with (D) may exhibit decorticate posturing, depending on the position of the tumor and the condition of the client.

The nurse working on a telemetry unit finds a client unconscious and in pulseless ventricular tachycardia (VT). The client has an implanted automatic defibrillator. What action should the nurse implement? A) Prepare the client for transcutaneous pacemaker. B) Shock the client with 200 joules per hospital policy. C) Use a magnet to deactivate the implanted pacemaker. D) Observe the monitor until the onset of ventricular fibrillation.

B) Shock the client with 200 joules per hospital policy. The client must be externally shocked (B) to restore an effective cardiac rhythm. The automatic defibrillator is obviously malfunctioning. (A) will not be effective during ventricular tachycardia, since it is used for asystole. Since the defibrillator is not functioning, (C) is not warranted. The client should be treated immediately to restore cardiac output (D).

The nurse is assessing a client who has a history of Parkinson's disease for the past 5 years. What symptoms should this client most likely exhibit? A) Loss of short-term memory, facial tics and grimaces, and constant writhing movements. B) Shuffling gait, masklike facial expression, and tremors of the head. C) Extreme muscular weakness, easy fatigability, and ptosis. D) Numbness of the extremities, loss of balance, and visual disturbances.

B) Shuffling gait, masklike facial expression, and tremors of the head. (B) are common clinical features of Parkinsonism. (A) are symptoms of chorea, (C) of myasthenia gravis, and (D) of multiple sclerosis.

Which description of symptoms is characteristic of a client diagnosed with trigeminal neuralgia (tic douloureux)? A) Tinnitus, vertigo, and hearing difficulties. B) Sudden, stabbing, severe pain over the lip and chin. C) Facial weakness and paralysis. D) Difficulty in chewing, talking, and swallowing.

B) Sudden, stabbing, severe pain over the lip and chin. Trigeminal neuralgia is characterized by paroxysms of pain, similar to an electric shock, in the area innervated by one or more branches of the trigeminal nerve (5th cranial) (B). (A) would be characteristic of Méniére's disease (8th cranial nerve). (C) would be characteristic of Bell's palsy (7th cranial nerve). (D) would be characteristic of disorders of the hypoglossal cranial nerve (12th).

A 58-year-old client, who has no health problems, asks the nurse about the Pneumovax vaccine. The nurse's response to the client should be based on which information? A) The vaccine is given annually before the flu season to those over 50 years of age. B) The immunization is administered once to older adults or persons with a history of chronic illness. C) The vaccine is for all ages and is given primarily to those persons traveling overseas to areas of infection. D) The vaccine will prevent the occurrence of pneumococcal pneumonia for up to five years.

B) The immunization is administered once to older adults or persons with a history of chronic illness. It is usually recommended that persons over 65 years of age and those with a history of chronic illness receive the vaccine once in a lifetime (B). (Some resources recommend obtaining the vaccine at 50 years of age.) The influenza vaccine is given once a year, not the Pneumovax (A). Although the vaccine might be given to a person traveling overseas, that is not the main rationale for administering the vaccine (C). It is usually given once in a lifetime (D), but with immunosuppressed clients or clients with a history of pneumonia re-vaccination is sometimes required.

Healthcare workers must protect themselves against becoming infected with HIV. The Center for Disease Control has issued guidelines for healthcare workers in relation to protection from HIV. These guidelines include which recommendation? A) Place HIV positive clients in strict isolation and limit visitors. B) Wear gloves when coming in contact with the blood or body fluids of any client. C) Conduct mandatory HIV testing of those who work with AIDS clients. D) Freeze HIV blood specimens at -70° F to kill the virus.

B) Wear gloves when coming in contact with the blood or body fluids of any client. The CDC guidelines recommend that healthcare workers use gloves when coming in contact with blood or body fluids from ANY client (B) since HIV is infectious before the client becomes aware of symptoms. (A) is not recommended, nor is it necessary. (C) is very controversial, difficult to enforce, and is not recommended by CDC. (D) does not guarantee to kill the virus. Additionally, the purpose of the blood specimen will determine how it is stored and handled.

A client is admitted to the hospital with a medical diagnosis of pneumococcal pneumonia. The nurse knows that the prognosis for gram-negative pneumonias (such as E. coli, Klebsiella, Pseudomonas, and Proteus) is very poor because A) they occur in the lower lobe alveoli which are more sensitive to infection. B) gram-negative organisms are more resistant to antibiotic therapy. C) they occur in healthy young adults who have recently been debilitated by an upper respiratory infection. D) gram-negative pneumonias usually affect infants and small children.

B) gram-negative organisms are more resistant to antibiotic therapy. The gram-negative organisms are resistant to drug therapy (B) which makes recovery very difficult. Gram-negative pneumonias affect all lobes of the lung (A). The mean age for contracting this type of pneumonia is 50 years (C and D), and it usually strikes debilitated persons such as alcoholics, diabetics, and those with chronic lung diseases.

The nurse is planning the care for a client who is admitted with the syndrome of inappropriate antidiuretic hormone secretion (SIADH). Which interventions should the nurse include in this client's plan of care? (Select all that apply.) A. Salt-free diet B. Quiet environment C. Deep tendon reflex assessments D. Neurologic checks E. Daily weights F. Unrestricted intake of free water

B, C, D, E. SIADH results in water retention and dilutional hyponatremia, which causes neurologic change when serum sodium levels are less than 115 mEq/L. The nurse should maintain a quiet environment (B) to prevent overstimulation that can lead to periods of disorientation, assess deep tendon reflexes (C) and neurologic checks (D) to monitor for neurologic deterioration. Daily weights (E) should be monitored to assess for fluid overload: 1 kg weight gain equals 1 L of fluid retention, which further dilutes serum sodium levels. (A and F) contribute to dilutional hyponatremia.

53. A patient is being discharged from the emergency department after being treated for epistaxis. In teaching the family first aid measures in the event the epistaxis would recur, which of the following measures would the nurse suggest? (Select all that apply.) A. Tilt patients head backwards B. Apply ice compresses to the nose C. Pinch the entire soft lower portion of the nose D. Partially insert a small gauze pad into the bleeding nostril

B,C,D First aid measures to control epistaxis includes placing the patient in a sitting position, leaning forward. Tilting the head back does not stop the bleeding, but rather allows the blood to enter the nasopharynx, which could result in aspiration or nausea/vomiting from swallowing blood. All of the other options are appropriate first aid treatment of epistaxis.

A patient is being discharged from the emergency department after being treated for epistaxis. In teaching the family first aid measures in the event the epistaxis would recur, which of the following measures would the nurse suggest? (Select all that apply.) A. Tilt patients head backwards B. Apply ice compresses to the nose C. Pinch the entire soft lower portion of the nose D. Partially insert a small gauze pad into the bleeding nostril

B,C,D First aid measures to control epistaxis includes placing the patient in a sitting position, leaning forward. Tilting the head back does not stop the bleeding, but rather allows the blood to enter the nasopharynx, which could result in aspiration or nausea/vomiting from swallowing blood. All of the other options are appropriate first aid treatment of epistaxis.

50. The nurse is teaching a patient how to self-administer ipratropium (Atrovent) via a metered dose inhaler. Which of the following instructions given by the nurse is most appropriate to help the patient learn proper inhalation technique? A. "Avoid shaking the inhaler before use." B. "Breathe out slowly before positioning the inhaler." C. "After taking a puff, hold the breath for 30 seconds before exhaling." D. "Using a spacer should be avoided for this type of medication."

B. "Breathe out slowly before positioning the inhaler." It is important to breathe out slowly before positioning the inhaler. This allows the patient to take a deeper breath while inhaling the medication thus enhancing the effectiveness of the dose.

The nurse is teaching a patient how to self-administer ipratropium (Atrovent) via a metered dose inhaler. Which of the following instructions given by the nurse is most appropriate to help the patient learn proper inhalation technique? A. "Avoid shaking the inhaler before use." B. "Breathe out slowly before positioning the inhaler." C. "After taking a puff, hold the breath for 30 seconds before exhaling." D. "Using a spacer should be avoided for this type of medication."

B. "Breathe out slowly before positioning the inhaler." It is important to breathe out slowly before positioning the inhaler. This allows the patient to take a deeper breath while inhaling the medication thus enhancing the effectiveness of the dose.

the healthcare provider is working with a client who was recently diagnosed with asthma. Which statement by the client indicates to the nurse that further teaching is needed? A. "It is normal for my heart rate to increase slightly when I use my albuterol inhaler." B. "I should always use my beclomethasone inhaler first, then follow it with my albuterol inhaler." C. "I should wait several minutes between puffs of my inhalers." D. "My albuterol can be used between regular doses if I experience difficulty breathing."

B. "I should always use my beclomethasone inhaler first, then follow it with my albuterol inhaler."

47. A patient has been receiving oxygen per nasal cannula while hospitalized for COPD. The patient asks the nurse whether oxygen use will be needed at home. Which of the following would be the most appropriate response by the nurse? A. "Long-term home oxygen therapy should be used to prevent respiratory failure." B. "Oxygen will be needed when your oxygen saturation drops to 88% and you have symptoms of hypoxia. C. "Long-term home oxygen therapy should be used to prevent heart problems related to emphysema." D. "Oxygen will not be needed until or unless you are in the terminal stages of this disease."

B. "Oxygen will be needed when your oxygen saturation drops to 88% and you have symptoms of hypoxia.Long-term oxygen therapy in the home should be considered when the oxygen saturation is 88% or less and the patient has signs of tissue hypoxia, such as cor pulmonale, erythrocytosis, or impaired mental status.

A patient has been receiving oxygen per nasal cannula while hospitalized for COPD. The patient asks the nurse whether oxygen use will be needed at home. Which of the following would be the most appropriate response by the nurse? A. "Long-term home oxygen therapy should be used to prevent respiratory failure." B. "Oxygen will be needed when your oxygen saturation drops to 88% and you have symptoms of hypoxia. C. "Long-term home oxygen therapy should be used to prevent heart problems related to emphysema." D. "Oxygen will not be needed until or unless you are in the terminal stages of this disease."

B. "Oxygen will be needed when your oxygen saturation drops to 88% and you have symptoms of hypoxia.Long-term oxygen therapy in the home should be considered when the oxygen saturation is 88% or less and the patient has signs of tissue hypoxia, such as cor pulmonale, erythrocytosis, or impaired mental status.

9. If a health care provider is planning to transfuse a patient with a unit of packed red blood cells, which of the following solutions should the health care provider hang with the transfusion? A. 5% dextrose in water B. 0.9% sodium chloride C. 5% dextrose in 0.9% sodium chloride D. 5% dextrose in lactated Ringer's solution

B. 0.9% sodium chloride The only solution appropriate for administration with whole blood or blood products is 0.9% sodium chloride. The other options are not appropriate for use with blood products.

If a health care provider is planning to transfuse a patient with a unit of packed red blood cells, which of the following solutions should the health care provider hang with the transfusion? A. 5% dextrose in water B. 0.9% sodium chloride C. 5% dextrose in 0.9% sodium chloride D. 5% dextrose in lactated Ringer's solution

B. 0.9% sodium chloride The only solution appropriate for administration with whole blood or blood products is 0.9% sodium chloride. The other options are not appropriate for use with blood products.

7. The nurse notes a physician's order written at 10:00 AM for 2 units of packed red blood cells to be administered to a patient who is anemic secondary to chronic blood loss. If the transfusion is picked up at 11:30, the nurse should plan to hang the unit no later than which of the following times? A. 11:45 AM B. 12:00 noon C. 12:30 PM D. 3:30 PM

B. 12:00 noon The nurse must hang the unit of packed red blood cells within 30 minutes of signing them out from the blood bank

The nurse notes a physician's order written at 10:00 AM for 2 units of packed red blood cells to be administered to a patient who is anemic secondary to chronic blood loss. If the transfusion is picked up at 11:30, the nurse should plan to hang the unit no later than which of the following times? A. 11:45 AM B. 12:00 noon C. 12:30 PM D. 3:30 PM

B. 12:00 noon The nurse must hang the unit of packed red blood cells within 30 minutes of signing them out from the blood bank

40. The patient has an order for albuterol 5 mg via nebulizer. Available is a solution containing 2 mg/ml. How many milliliters should the nurse use to prepare the patient's dose? A. 0.2 B. 2.5 C. 3.75 D. 5.0

B. 2.5

The patient has an order for albuterol 5 mg via nebulizer. Available is a solution containing 2 mg/ml. How many milliliters should the nurse use to prepare the patient's dose? A. 0.2 B. 2.5 C. 3.75 D. 5.0

B. 2.5

The nurse is conducting health assessments. Which assessment finding increases a 56-year-old woman's risk for developing osteoporosis? A. Body mass index (BMI) of 31 B. 20 pack-year history of cigarette smoking C. Birth control pill usage until age 45 D. Diabetes mellitus in family history

B. 20 pack-year history of cigarette smoking

2. The nurse is caring for a patient with COPD and pneumonia who has an order for arterial blood gases to be drawn. Which of the following is the minimum length of time the nurse should plan to hold pressure on the puncture site? A. 2 minutes B. 5 minutes C. 10 minutes D. 15 minutes

B. 5 minutes Following obtaining an arterial blood gas, the nurse should hold pressure on the puncture site for 5 minutes by the clock to be sure that bleeding has stopped. An artery is an elastic vessel under higher pressure than veins, and significant blood loss or hematoma formation could occur if the time is insufficient.

The nurse is caring for a patient with COPD and pneumonia who has an order for arterial blood gases to be drawn. Which of the following is the minimum length of time the nurse should plan to hold pressure on the puncture site? A. 2 minutes B. 5 minutes C. 10 minutes D. 15 minutes

B. 5 minutes Following obtaining an arterial blood gas, the nurse should hold pressure on the puncture site for 5 minutes by the clock to be sure that bleeding has stopped. An artery is an elastic vessel under higher pressure than veins, and significant blood loss or hematoma formation could occur if the time is insufficient.

Which client situation requires the most immediate intervention by the nurse? A. One day after surgery, a client complains of unpleasant flatulence B. A bedfast client experiences an episode of urinary incontinence C. An ambulatory client reports the onset of stress incontinence D. A client receiving regular dose of an opioid develops constipation

B. A bedfast client experiences an episode of urinary incontinence

Which client situation requires the most immediate intervention by the nurse? A. One day day after surgery, a client complains of unpleasant flatulence. B. A bedfast client experiences an episode of urinary incontinence. C. An ambulatory client reports the onset of stress incontinence. D. A client receiving regular dosage of an opioid develops constipation

B. A bedfast client experiences an episode of urinary incontinence.

A newly graduated and licensed registered nurse (RN) is in the second day of orientation to the hospital unit. The education director tells the charge nurse that the new graduate should be assigned to care for one client. Which client is best for the nurse to assign to this new graduate? A. A client returning from surgery following a bowel resection B. A client who is 4-days post myocardial infraction C. A 6-day postoperative client who is receiving a blood transfusion D. A client with AIDS who is in the final stages of dying

B. A client who is 4-days post myocardial infraction

After reviewing the Braden Scale findings of residents at a long-term facility, the charge nurse should to tell the unlicensed assistive personnel (UAP) to prioritize skin care for which client? A. An older adult who is unable to communicate elimination needs B. A older man who sheets are damp each time he turned C. A woman with osteoporosis who is unable to bear weight D. A poorly nourished client who requires liquid supplements

B. A older man who sheets are damp each time he turned

A 7-year-old child admitted to the hospital with acute glomerulonephritis (AGN). When obtaining the nursing history, which finding should the nurse expect to obtain? A. A recent DPT immunization B. A recent strep throat infection C. High blood cholesterol levels on routine screening D. Increased thirst and urination

B. A recent strep throat infection

A female resident of a long-term care facility is being admitted to the medical department. The client has a fractured humerus and methicillin-resistant staphylococcus aureus (MRSA). Which room should the charge nurse assign this client? A. A private room, and institute protective environment measures B. A semi-private room with a another client who also has MRSA C. A private isolation room with a vented negative airflow system D. A semi-private room with a client who has hepatitis B (HBV)

B. A semi-private room with a another client who also has MRSA

The nurse is interviewing a client who is taking interferon-alfa-2a (Roferon-A) and ribavirin (Virazole) combination therapy for hepatitis C. The client reports experiencing overwhelming feelings of depression. What action should the nurse implement first? A. Recommend mental health counseling. B. Review the medications actions and interactions. C. Assess for the client's daily activity level. D. Provide information regarding a support group.

B. Alpha-interferon and ribavirin combination therapy can cause severe depression. (A, B, C) may be implemented after physiological aspect of the situation are assessed.

4. The nurse is caring for a patient who is to receive a transfusion of two units of packed red blood cells. After obtaining the first unit from the blood bank, the nurse would ask which of the following health team members in the nurses' station to assist in checking the unit before administration? A. Unit secretary B. Another registered nurse C. A physician's assistant D. A phlebotomist

B. Another registered nurseBefore hanging a transfusion, the registered nurse must check the unit with another RN or with a licensed practical (vocational) nurse, depending on agency policy.

The nurse is caring for a patient who is to receive a transfusion of two units of packed red blood cells. After obtaining the first unit from the blood bank, the nurse would ask which of the following health team members in the nurses' station to assist in checking the unit before administration? A. Unit secretary B. Another registered nurse C. A physician's assistant D. A phlebotomist

B. Another registered nurseBefore hanging a transfusion, the registered nurse must check the unit with another RN or with a licensed practical (vocational) nurse, depending on agency policy.

The nurse who is performing blood sugar and cholesterol screenings at a community health fair determines that a female client's blood sugar is 59 mg/dl at 10:00 a.m. Which nursing intervention is most important for the nurse to implement? A. Check the client's cholesterol B. Ask the client how she's feeling C. Take the client's blood pressure D. Encourage the client to rest

B. Ask the client how she's feeling

While the nurse is conducting a daily assessment of an older woman who resides in a long-term facility, the client begins to cry and tell the nurse that her family has stopped calling and visiting. What action should the nurse take first? A. Determine the client's orientation to time and space B. Ask the client when a family member last visited her C. Review the client's record regarding social interactions D. Reassure the client of her family's love for her

B. Ask the client when a family member last visited her

An elderly client is experiencing disturbed sleep patterns. Which interventions should the nurse implement to help the client attain maximal sleep function? (select all that apply) A. Discuss need for adult day services B. Assess daytime somnolence C. Recommend evaluation of home safety D. Review medication side effect profiles E. Refer to social worker for elder care

B. Assess daytime somnolence D. Review medication side effect profiles

During the transfer of a client who had major abdominal surgery this morning, the postanesthesia care unit (PACU) nurse reports that the client, who is awake and responsive, continues to report pain and nausea after receiving morphine 2 mg IV and ondansetron 4 mg IV 45 minutes ago. Which elements of SBAR communication are missing from the report given by the PACU nurse? A. Situation B. Background C. Assessment D. Recommendation E. Rationales

B. Background C. Assessment D. Recommendation

A 43-year-old homeless, malnourished female client with a history of alcoholism is transferred to the ICU. She is placed on telemetry, and the rhythm strip shown is obtained. The nurse palpates a heart rate of 160 beats/min, and the client's blood pressure is 90/54. Based on these finding, which IV medication should the nurse administer? A. Amiodarone (Cordarone) B. Magnesium sulfate C. Lidocaine (Xylocaine) D. Procainamide (Pronestyl)

B. Because the client has chronic alcoholism, she is likely to have hypomagnesium. (B) is the recommended drug for torsades de pointes (AHA, 2005), which is a form of polymorphic ventricular tachycardia (VT), usually associated with a prolonged QT interval that occurs with hypomagnesemia. (A and D) increase the QT interval, which can cause the torsades to worsen. (C) is the antiarrhythmic of choice in most cases of drug-induced monomorphic VT, not torsades.

An older male client diagnosed with end-stage chronic obstructive pulmonary disease (COPD) is on strict bedrest, and asks the nurse, "Why can't I get out of bed? What response is best for the nurse to provide? A. A high-Fowler's position promotes lung expansion B. Bed rest decreases your body's need for oxygen C. You are so weak that you are at risk for falling D. hospital policy requires that you are assisted to get up

B. Bed rest decreases your body's need for oxygen

A client with chronic asthma is admitted to postanesthesia complaining of pain at level 8 of 10, with a BP of 124/78, pulse of 88 beats/min, and respirations of 20 breaths/min. The postanesthesia recovery prescription is, "Morphine 2 to 4 mg IV push while in recovery for pain level over 5." What intervention should the nurse implement? A. Give the medication as prescribed to decrease the client's pain. B. Call the anesthesia provider for a different medication for pain. C. Use nonpharmacologic techniques before giving the medication. D. Reassess pain level in 30 Minutes and medicate if it remains elevated.

B. Call for a different medication because morphine and meperidine (Demerol) have histamine-releasing narcotics and should be avoided when a client has asthma. (A) puts the client at risk for asthma attack. (C & D) disregard the clients prescription and pain relief.

A nurse is preparing to establish oxygen therapy for a patient with COPD, and the physician's prescription reads "oxygen per nasal cannula at 5 L per minute." Which of the following actions should the nurse take? A. Administer the oxygen as prescribed. B. Call the physician and question the correct flow rate of the oxygen. C. Establish the oxygen as prescribed and obtain an ABG. D. Change the delivery device from a nasal cannula to a simple oxygen mask.

B. Call the physician and question the correct flow rate of the oxygen. The nurse should call the physician immediately and question the flow rate for delivery of the oxygen before implementation. Oxygen is used cautiously in patients with COPD because of longstanding hypoxemia serving as the respiratory drive mechanism. If high levels of oxygen are administered, the respiratory drive can be obliterated. Changing the device to a simple oxygen mask may alter the oxygen concentration being delivered to the patient and will further enhance the obliteration of the patient's respiratory drive. Obtaining an ABG sample is not a priority at this time, and the action does not address the validity of the prescribed oxygen dosing for the patient.

The nurse is reviewing the routine medications taken by a client with chronic angle closure glaucoma. Which medication prescription should the nurse question? A. An antianginal with a therapeutic effect of vasodilation. B. An anticholinergic with a side effect of pupillary dilation. C. An antihistamine with a side effect of sedation. D. A corticosteroid with a side effect of hyperglycemia.

B. Clients with angle closure glaucoma should not take medications that dilate the pupil (B) because this can precipitate acute and severely increased intraocular pressure. (A, C, D) do not cause increased intraocular pressure, which is the primary concern.

The new mother tells the nurse that she does not want her newborn to receive any immunizations. It is the hospital's policy to routinely administer immunizations to all newborns. What intervention should the nurse implement? A. Advise the mother to sign out of the hospital AMA if wishing to refuse the immunizations B. Document that the mother has refused immunization permission for the newborn C. Administer the immunizations after first explaining the hospital policy to the mother D. Report the immunization status of the infant to the office of child protective services

B. Document that the mother has refused immunization permission for the newborn

The nurse observes that a client with COPD is exhibiting fingernail clubbing. What action should the nurse take? A. Administer a PRN dose of albuterol (Proventil) via inhaler. B. Document the assessment finding in the nurses' notes C. Assist the client to a tripod position in the bed. D. Increase the client's oxygen from 1 to 2 liters per minute.

B. Document the assessment finding in the nurses' notes

20. If a patient states, "It's hard for me to breathe and I feel short-winded all the time," what is the most appropriate terminology to be applied in documenting this assessment by a nurse? A. Apnea B. Dyspnea C. Tachypnea D. Respiratory fatigue

B. Dyspnea Dyspnea is a subjective description reflective of the patient's statement indicating difficulty in breathing. Apnea refers to absence of breath or breathing. Tachypnea refers to an increased rate of breathing, usually greater than 20 breaths per minute. Respiratory fatigue is subjective and usually refers to the patient exhibiting signs and symptoms associated with a comprehensive respiratory assessment including laborious breathing, use of accessory muscles, and slowing of respirations.

If a patient states, "It's hard for me to breathe and I feel short-winded all the time," what is the most appropriate terminology to be applied in documenting this assessment by a nurse? A. Apnea B. Dyspnea C. Tachypnea D. Respiratory fatigue

B. Dyspnea Dyspnea is a subjective description reflective of the patient's statement indicating difficulty in breathing. Apnea refers to absence of breath or breathing. Tachypnea refers to an increased rate of breathing, usually greater than 20 breaths per minute. Respiratory fatigue is subjective and usually refers to the patient exhibiting signs and symptoms associated with a comprehensive respiratory assessment including laborious breathing, use of accessory muscles, and slowing of respirations.

The nurse is giving medication to an older client who has a percutaneous esophageal gastrostomy (PEG) tube in place. Which medication drug form should the nurse question? A. Digoxin (Lanoxin) tablet B. Enteric-coated aspirin (Ecotrin) C. Furosidemide (Lasix) tablet D. Megestrol (Megace) suspension

B. Enteric-coated aspirin (Ecotrin)

A client in the third trimester of pregnancy reports that she feels some "lumpy places" in her breasts and that her nipples sometimes leak yellowish fluid. She has an appointment with her healthcare provider in two weeks. What action should the nurse take? A. Reschedule the client's appointment for the following day B. Explain that this normal secretion can be assessed at the next visit C. Tell the client to begin nipple stimulation to prepare for breast feeding D. Recommend that the client start wearing a supportive brassiere

B. Explain that this normal secretion can be assessed at the next visit

Two months after treatment with radioactive iodine (I131) (RAI) for hyperthyroidism, a male client reports the onset of extreme fatigue, depression, and hair loss. What action should the nurse take? A. Confirm that the client should continue to take his anti-thyroid medication B. Explain to the client that thyroid replacement hormones may be needed. C. Advise the client to increase his caloric intake and eat high protein foods D. Assure the client that these are time-limited side effects of the treatment

B. Explain to the client that thyroid replacement hormones may be needed.

A male client with chronic alcohol use is admitted with signs of early cirrhosis. Which nursing action should the nurse delegate to the unlicensed assistive personnel (UAP)? A. Assist client with preferred meal selections B. Gather oral hygiene products for client's use C. Evaluate client for asterixis D. Assess the client for ascites

B. Gather oral hygiene products for client's use

During a well-baby, 6-month visit, a mother tells the nurse that her infant has had fewer ear infections than her 10-year-old daughter. The nurse should explain that which vaccine is likely to have made the difference in the siblings incidence of otitis media? A. Pneumococcal vaccine B. Hemophilus Influenza type B (H1B) vaccine C. Palrvizumab vaccine for RSV D. Varicella Virus Vaccine Live

B. Hemophilus Influenza type B (H1B) vaccine

A client diagnosed with chronic kidney disease (CDK) 2 years ago is regularly treated at a community hemodialysis facility. In assessing the client before his scheduled dialysis treatment, which electrolyte imbalance should the nurse anticipate? A. Hypophosphatemia B. Hypocalcemia C. Hyponatremia D. Hypokalemia

B. Hypocalcemia develops in CKD due to chronic hyperphosphatemia not (A). (C & D) incorrect you would find hypernatremia and hyperkalemia

An adult male is admitted to the psychiatric unit from the emergency department because he is in the manic stage of bipolar disorder. He has lost 10 pounds in the last two weeks and has not bathed in a week because he has been "trying to start a new business" and "too busy to eat." He is alert and oriented to time, place, and person, but not situation. Which nursing problem has the greatest priority? A. Hygiene self-care deficit B. Imbalanced nutrition C. Disturbed sleep pattern D. Self neglect

B. Imbalanced nutrition

As a means of relieving a client's pain associated with osteoarthritis, the nurse plans to provide local rest. To implement this intervention, which action should the nurse take? A. Elevate the affected joint with an icepack on it B. Immobilize the affected joint with a splint C. Administer a prescribed local topical salicylate D. Maintain bedrest with bathroom privileges

B. Immobilize the affected joint with a splint

9. Which of the following nursing interventions is of the highest priority in helping a patient expectorate thick secretions related to pneumonia? A. Humidify the oxygen as able B. Increase fluid intake to 3L/day if tolerated. C. Administer cough suppressant q4hr. D. Teach patient to splint the affected area.

B. Increase fluid intake to 3L/day if tolerated. Although several interventions may help the patient expectorate mucus, the highest priority should be on increasing fluid intake, which will liquefy the secretions so that the patient can expectorate them more easily. Humidifying the oxygen is also helpful, but is not the primary intervention. Teaching the patient to splint the affected area may also be helpful, but does not liquefy the secretions so that they can be removed.

Which of the following nursing interventions is of the highest priority in helping a patient expectorate thick secretions related to pneumonia? A. Humidify the oxygen as able B. Increase fluid intake to 3L/day if tolerated. C. Administer cough suppressant q4hr. D. Teach patient to splint the affected area.

B. Increase fluid intake to 3L/day if tolerated. Although several interventions may help the patient expectorate mucus, the highest priority should be on increasing fluid intake, which will liquefy the secretions so that the patient can expectorate them more easily. Humidifying the oxygen is also helpful, but is not the primary intervention. Teaching the patient to splint the affected area may also be helpful, but does not liquefy the secretions so that they can be removed.

A 25-year-old client was admitted yesterday after a motor vehicle collision. Neurodiagnostic studies showed a basal skull fracture in the middle fossa. Assessment on admission revealed both halo and Battle signs. Which new symptom indicates that the client is likely to be experiencing a common life-threatening complication associated with basal skull fracture? A. Bilateral jugular vein distention. B. Oral temperature of 102 degrees F. C. Intermittent focal motor seizures. D. Intractable pain in the cervical region.

B. Increased temp indicates meningitis. (C & D) these symptoms may be exhibited but are not life threatening. (A) JVD is not a typical complication of basal skull fractures.

During a home visit, the nurse determines that a male client is experiencing symptoms that should be controlled by his prescribed medication. The client states that he forgot when he was supposed to take his medication. What is the priority nursing problem when the nurse develops the plan of care for the client? A. Self neglect related to loss of cognitive function B. Ineffective health maintenance related to lack of knowledge C. Situational low self-esteem related to symptoms of illness D. Noncompliance related to lifestyle change

B. Ineffective health maintenance related to lack of knowledge

Which condition should the nurse anticipate as a potential problem in a female client with a neurogenic bladder? A. Stress incontinence. B. Infection. C. Painless, gross hematuria. D. Peritonitis.

B. Infection is the major complication resulting from stasis of urine and subsequent catheterization. (A) is the involuntary loss of urine through an intact urethra as a result of suddenly increased pressure. (C) is the most common symptom of bladder cancer. (D) is the most common and serious complication of peritoneal dialysis.

In planning strategies to reduce a client's risk for complications following orthopedic surgery, the nurse recognizes which pathology as the underlying cause of osteomyelitis? A. Metastatic process B. Infectious process C. Autoimmune disorder D. Inflammatory disorder

B. Infectious process Osteomyelitis is serve infection of the bone, bone marrow, and surrounding tissue

A female college student is admitted to the Emergency Department following indigestion of alcohol and pain medication. A nasogastric tube and subclavian line are placed. The nurse auscultates audible breath sounds on the right side, faint sounds on the left side, and chest involvement that occurs only on the right side of the thorax. Which procedure should the nurse prepare for first? A. removal of the subclavian line and preparation for jugular insertion B. Insertion of 16g needle at the 4th intercostal space midclavicular line C. Placement of an endotracheal tube and mechanical ventilation D. Retraction of the nasogastric tube by a length of 2 cm

B. Insertion of 16g needle at the 4th intercostal space midclavicular line Based on the location of the audible breath sounds and the client's chest movements, the nurse should suspect that the subclavian line has pierced the client's left lung, causing a pneumothorax. Therefore the treatment of choice is B.

An adult male who returned from a vacation in Mexico three weeks ago calls the clinic complaining of abdominal pain, weight loss, and diarrhea. What action should the nurse take? A. Encourage the client to go to the emergency room B. Instruct the client to bring in a stool sample C. Ask the client if he is experiencing dyspnea D. Tell the client to eat toast and Gatorade sports drink

B. Instruct the client to bring in a stool sample

A male client with Addison's disease tells the nurse that he is taking hydrocortisone in a divided daily dose. He reports increasing fatigue and weakness. What action should the nurse take? A. Advise the client to skip the next scheduled dose of hydrocortisone B. Interview the client about any sources of increased stress in his life C. Instruct the client to limit his intake of oral fluids, especially at night D. Encourage the client to increase daily exercise and physical activity

B. Interview the client about any sources of increased stress in his life

When assigning clients on a medical-surgical floor to a RN and a LPN, it is best for the charge nurse to assign which client to the LPN? A. A child with bacterial meningitis with recent seizures. B. An older adult client with pneumonia and viral meningitis. C. A female client in isolation wiht meningococcal meningitis. D. A male client 1 day post-op after drainage of a brain abscess.

B. Is the most stable. A, C, D have an increased risk for elevated ICP.

A 58-year-old client, who has no health problems, asks the nurse about taking the pneumococcal vaccine (Pneumovax). Which statement give by the nurse would offer the client accurate information about this vaccine? A. "The vaccine is given annually before the flue season to those over 50 years of age." B. "The immunization is administered once to older adults or persons with a history of chronic illness." C. "The vaccine is for all ages and is given primarily to those person traveling overseas to infected areas." D. "The vaccine will prevent the occurrence of pneumococcal pneumonia for up to 5 years."

B. It is usually recommended that persons over 65 years of age and those with a history of chronic illness should receive the vaccine once in a lifetime. (A) the influenza vaccine is given annually. (C) travel is not the main rationale for the vaccine. (D) The vaccine is usually given once in a lifetime.

2. Which of the following is a factor significant in the development of anemia in men? A. Condom use B. Large hemorrhoids C. A diet high in cholesterol D. Smoking one pack of cigarettes daily

B. Large hemorrhoids Gastrointestinal (GI) tract bleeding is a common etiologic factor in men and may result from peptic ulcers, hiatal hernia, gastritis, cancer, hemorrhoids, diverticula, ulcerative colitis, or salicylate poisoning.

Which of the following is a factor significant in the development of anemia in men? A. Condom use B. Large hemorrhoids C. A diet high in cholesterol D. Smoking one pack of cigarettes daily

B. Large hemorrhoids Gastrointestinal (GI) tract bleeding is a common etiologic factor in men and may result from peptic ulcers, hiatal hernia, gastritis, cancer, hemorrhoids, diverticula, ulcerative colitis, or salicylate poisoning.

A client with Alzheimer's Disease falls in the bathroom. The nurse notifies the charge nurse and completes a fall follow-up assessment. What assessment finding warrants immediate intervention by the nurse? A. Urinary incontinence B. Left forearm hematoma C. Disorientation to surroundings D. Dislodged intravenous site

B. Left forearm hematoma

What intervention should the nurse implement to prevent edema and promote healing of a client's incision resulting from an above-the-knee amputation? A. Keep the residual limb in a dependent position B. Maintain the residual limb in a compression dressing C. Inspect the incision hourly for the first 24 hours postoperatively D. Elevate the affected limb on two pillows at all times

B. Maintain the residual limb in a compression dressing

A client who had an emergency appendectomy is being mechanically ventilated, and soft wrist restraints are in place to prevent self extubation. Which outcome is most important for the nurse to include in the client's plan of care? A. Understands pain management scale B. Maintains effective breathing patterns C. Absence of ventilator associated pneumonia D. No injuries related to soft restraints occur

B. Maintains effective breathing patterns

In assessing a pressure ulcer on a client's hip, which action should the nurse include? A. Determine the degree of elasticity surrounding the lesion B. Photograph the lesion with a ruler placed next to the lesion C. Stage the depth of the ulcer using the Braden numeric scale D. Use a gloved finger to palpate for tunneling around the lesion

B. Photograph the lesion with a ruler placed next to the lesion

While performing a skin inspection for a female adult client, the nurse observes a rash that is well circumscribed, has silvery scales and plaques, and is located on the elbows and knees. These assessment findings are likely to indicate which condition? A. Tinea corporis B. Psoriasis C. Herpes Zoster D. Drug reaction

B. Psoriasis

17. The nurse is caring for a 73-year-old patient who underwent a left total knee arthroplasty. On the third postoperative day, the patient complains of shortness of breath, slight chest pain, and that "something is wrong." Temperature is 98.4o F, blood pressure 130/88, respirations 36, and oxygen saturation 91% on room air. Which of the following should the nurse first suspect as the etiology of this episode? A. Septic embolus from the knee joint B. Pulmonary embolus from deep vein thrombosis C. New onset of angina pectoris D. Pleural effusion related to positioning in the operating room

B. Pulmonary embolus from deep vein thrombosis The patient presents the classic symptoms of pulmonary embolus: acute onset of symptoms, tachypnea, shortness of breath, and chest pain.

The nurse is caring for a 73-year-old patient who underwent a left total knee arthroplasty. On the third postoperative day, the patient complains of shortness of breath, slight chest pain, and that "something is wrong." Temperature is 98.4o F, blood pressure 130/88, respirations 36, and oxygen saturation 91% on room air. Which of the following should the nurse first suspect as the etiology of this episode? A. Septic embolus from the knee joint B. Pulmonary embolus from deep vein thrombosis C. New onset of angina pectoris D. Pleural effusion related to positioning in the operating room

B. Pulmonary embolus from deep vein thrombosis The patient presents the classic symptoms of pulmonary embolus: acute onset of symptoms, tachypnea, shortness of breath, and chest pain.

The nurse is caring for six clients on a medical-surgical unit. Which interventions should the nurse delegate to the unlicensed assistive personnel (UAP)? (select all that apply) A. Assess daily weights for trends B. Record vital signs every four hours C. Assist with ambulation as prescribed D. Provide oral care after meals E. Monitor for signs of dehydration

B. Record vital signs every four hours C. Assist with ambulation as prescribed D. Provide oral care after meals

An older female client with dementia is transferred from a long term care unit to an acute care unit. The client's children express concern that their mother's confusion is worsening. How should the nurse respond? A. "It is to be expected that older people will experience progressive confusion." B. "Confusion in an older person often follows relocation to new surroundings." C. "The dementia is progressing rapidly, but we will do everything we can to keep your mother safe." D. "The acute care staff is not as experienced as the long-term care staff at dealing with dementia."

B. Relocation often results in confusion among older clients and is stressful to clients of all ages. (A) is an inaccurate stereotype. (C) is most likely false there are many factors that cause increased temporary confusion. (D) may be true but does not offer the family a sense of security about the care.

The nurse is managing clients who are mechanically ventilated. The client with which assessment finding requires the most immediate intervention by the nurse? A. Audible voice when client is trying to communicate B. Restrained and restless with a low volume alarm C. High pressure alarm when client is coughing D. Diminished breath sounds in the right posterior base

B. Restrained and restless with a low volume alarm

A male client presents to the clinic with large draining ulcers on his lower legs that are characteristic of Kapok's sarcoma lesions. He is accompanied by two family members. What actions should the nurse take? A. Ask the family members to wear gloves when touching the client B. Send family to the waiting area while the client's history is taken C. Obtain a blood sample to determine of the client is HIV positive D. Complete a head to toe assessment to identify other signs of HIV

B. Send family to the waiting area while the client's history is taken

37. The nurse is scheduled to give a dose of salmeterol by metered dose inhaler (MDI). The nurse would administer the right drug by selecting the inhaler with which of the following trade names? A. Vanceril B. Serevent C. AeroBid D. Atrovent

B. Serevent The trade or brand name for salmeterol, an adrenergic bronchodilator, is Serevent.

The nurse is scheduled to give a dose of salmeterol by metered dose inhaler (MDI). The nurse would administer the right drug by selecting the inhaler with which of the following trade names? A. Vanceril B. Serevent C. AeroBid D. Atrovent

B. Serevent The trade or brand name for salmeterol, an adrenergic bronchodilator, is Serevent.

An older male client comes to the geriatric screening clinic complaining of pain in his left calf. The nurse notices a reddened area on the calf of his right leg that is warm to touch and the nurse suspects that the client may have thrombophlebitis. Which addition assessment is most important for the nurse to perform? A. Measure calf circumference. B. Auscultate the client's breath sounds. C. Observe for ecchymosis and petechiae. D. Obtain the client's blood pressure.

B. Since the client may have a pulmonary embolus secondary to the thrombophlebitis. A. Would support the nurses assessment. C. Least helpful since bruising is not associated with thrombophlebitis. D. Less important then auscultation.

A newborn is apnea for 20 seconds. What action should the nurse implement? A. Place oxygen cannula near the nares B. Stimulate by gently rubbing infant's trunk C. Begin cardiopulmonary resuscitation D. Suction the infant's nano-oropharynx

B. Stimulate by gently rubbing infant's trunk

26. The nurse is assigned to care for a patient in the emergency department admitted with an exacerbation of asthma. The patient has received a β-adrenergic bronchodilator and supplemental oxygen. If the patient's condition does not improve, the nurse should anticipate which of the following is likely to be the next step in treatment? A. Pulmonary function testing B. Systemic corticosteroids C. Biofeedback therapy D. Intravenous fluids

B. Systemic corticosteroids Systemic corticosteroids speed the resolution of asthma exacerbations and are indicated if the initial response to the β-adrenergic bronchodilator is insufficient.

The nurse is assigned to care for a patient in the emergency department admitted with an exacerbation of asthma. The patient has received a β-adrenergic bronchodilator and supplemental oxygen. If the patient's condition does not improve, the nurse should anticipate which of the following is likely to be the next step in treatment? A. Pulmonary function testing B. Systemic corticosteroids C. Biofeedback therapy D. Intravenous fluids

B. Systemic corticosteroids Systemic corticosteroids speed the resolution of asthma exacerbations and are indicated if the initial response to the β-adrenergic bronchodilator is insufficient.

A male client with chronic renal failure (CRF) is admitted to the intensive care unit after missing his last three appointments at the dialysis center. His arterial blood gas (ABG) results are: pH 7.32; PaCO2 32 mmHG; HCO3 18 mEq/L. Which assessment finding should the nurse expect this client to exhibit? A. Diaphoresis B. Tachypnea C. Hypotension D. Bradycardia

B. Tachypnea

An unlicensed assistive personnel (UAP) is teamed with a nurse who is caring for four clients. Client A is admitted to the medical unit for heart failure (HF) Client B has just returned from surgery Client C, with chronic obstructive pulmonary disease (COPD), needs new oxygen tubing Client D is waiting to be discharged Which activity should the nurse delegate to the UAP as having the highest priority? A. Obtain the daily weight for the client with HF B. Take vital signs of the client who just returned form surgery C. Gather equipment for oxygen tubing change for the client with COPD D. Assist client into the wheelchair for discharge

B. Take vital signs of the client who just returned form surgery

A male client returns to the mental health clinic for assistance with his anxiety reaction that is manifested by a rapid heartbeat, sweating, shaking, and nausea while driving over the bay bridge. What action in the treatment plan should the nurse implement? A.Tell client to drive over the bridge until fear is manageable B. Teach client to listen to music or audio books while driving C. Encourage client to have spouse drive in stressful places D. Recommend that the client avoid driving over the bridge

B. Teach client to listen to music or audio books while driving

The nurse assesses a postoperative client. Oxygen is being administered at 2 L/min and a saline lock is in place. Assessment shows cool, pale, moist skin. The client is very restless and has scant urine in the urinary drainage bag. What intervention should the nurse implement first. A. Measure urine specific gravity. B. Obtain IV fluids for infusion protocol. C. Prepare for insertion of a central venous catheter. D. Auscultate the client's breath sounds.

B. The client is at risk for hypovolemic shock and is exhibiting early signs. Start IV to restore tissue perfusion. (A, C, D) are all important but less of a priority.

A client with a prescription for "do not resuscitate" (DNR) begins to manifest signs of impending death. After notifying the family of the client's status, what priority action should the nurse implement? A. The healthcare provider should be notified of the client's status B. The client's need for pain medication should be determined C. The chaplain should be requested to come to the client's bedside D. The client's signs of impending death should be documented

B. The client's need for pain medication should be determined

A female client with a nasogastric tube attached to low suction states that she is nauseated. The nurse assesses that there has been no drainage through the nasogastric tube in the last 2 hours. Which action should the nurse take first? A. Irrigate the nasogastric tube with sterile normal saline. B. Reposition the client on her side. C. Advance the nasogastric tube 5 cm. D. Administer an intravenous antiemetic as prescribed.

B. The priority is to determined if the tube is functioning correctly, which would relieve the client's nausea. The least invasive intervention is to reposition the client (B), should be attempted first, followed by (A & C) if these are unsuccessful then (D).

A 55-year-old male client is admitted to the coronary care unit having suffered an acute myocardial infarction (MI). Within 24 hours of the occurrence, the nurse can expect to find which systemic sign? A. Elevated serum amylase level B. Elevated CM-MB level C. Prolonged prothrombin time (PT) D. Elevated serum BUN and creatinine

B. Tissue damage in the myocardium causes the release of cardiac enzymes into the blood system. An elevated CM-MB is a recognized indicator of an MI. It peaks 12 - 24 hours and returns to normal within 48 - 78 hours. (A) would indicate pancreatitis or a gastric disorder. (D) Although an elevated BUN might be related to an acute MI it is usually associated with dehydration, high protein intake or gastrointestinal bleeding and creatine levels indicate renal damage. (C) Indicates effective anticoagulation therapy.

A client slips and falls while getting out of bed and the charge nurse instructs the nurse who is caring for the client to complete an incident report. What is the main purpose in having the nurse complete the incident report? A. To ensure that the nurse caring for the client takes responsibility for the incident. B. To provide computer documentation of the incident as a basis for further investigation C. To protect the nurse and the hospital against charges of malpractice D. To reprimand the nurse for not providing safe care for the client

B. To provide computer documentation of the incident as a basis for further investigation

The nurse is explaining the need to reduce salt intake to a client with primary hypertension. What explanation should the nurse provide? A. High salt can damage the lining of the blood vessels B. Too much salt can cause the kidneys to retain fluid C. Excessive salt can cause blood vessels to contract D. Salt can cause inflammation inside the blood vessels

B. Too much salt can cause the kidneys to retain fluid

Which description of symptoms is characteristic of a client with diagnosed with trigeminal neuralgia (tic douloureux)? A. Tinnitus, vertigo, and hearing difficulties. B. Sudden, stabbing, severe pain over the lip and chin. C. Unilateral facial weakness and paralysis. D. Difficulty in talking, chewing, and swallowing.

B. Trigeminal neuralgia is characterized by paroxysms of pain, similar to an electric shock, in the area innervated by one or more branches of the trigeminal nerve. A. Characteristic of Meniere's C. Characteristic of Bell palsey D. Characteristic of disorders of the hypoglossal (12th cranial nerve)

A client with dementia who is cared for at home by her husband becomes increasingly confused in the evening. Her husband reports to the home health care nurse that his wife often believes that she is waiting for the oil to be changed in her car and insists on leaving. Which recommendation should the nurse provide to this husband? A. Remind his wife that she is just a little confused B. Try to interest his wife in a different activity C. Give his wife a PRN dose of haloperidol (Haldol) D. Show his wife her car that is still in the garage

B. Try to interest his wife in a different activity

The nurse plans to collect a 24-hour urine specimen for a creatine clearance test. Which instruction should the nurse provide to the adult male client? A. Urinate immediately into a urinal, and the lab will collect the specimen every 6 hours, for the next 24 hours B. Urinate at a specified time, discard this urine, and collect all subsequent urine during the next 24 hours C. For the next 24 hours, notify nurse when the bladder is full, and the nurse will collect catheterized specimens D. Cleanse around the meatus, discard first portion of voiding, and collect the rest in a sterile bottle

B. Urinate at a specified time, discard this urine, and collect all subsequent urine during the next 24 hours

The nurse observes ventricular fibrillation on telemetry and upon entering the clients bathroom finds the client unconscious on the floor. What intervention should the nurse implement first? A. Administer an antidysrhythmic medication. B. Start cardiopulmonary resuscitation. C. Defibrillate the client at 200 joules. D. Assess the client's pulse oximetry.

B. Ventricular fibrillation is a life-threatening dysrhythmia and CPR should be started immediately. A & C are appropriate but B is the priority. D does not address the seriousness of the situation.

48. Before discharge, the nurse discusses activity levels with a 61-year-old patient with COPD and pneumonia. Which of the following exercise goals is most appropriate once the patient is fully recovered from this episode of illness? A. Slightly increase activity over the current level. B. Walk for 20 minutes a day, keeping the pulse rate less than 130 beats per minute. C. Limit exercise to activities of daily living to conserve energy. D. Swim for 10 min/day, gradually increasing to 30 min/day.

B. Walk for 20 minutes a day, keeping the pulse rate less than 130 beats per minute. The patient will benefit from mild aerobic exercise that does not stress the cardiorespiratory system. The patient should be encouraged to walk for 20 min/day, keeping the pulse rate less than 75% to 80% of maximum heart rate (220 minus patient's age).

Before discharge, the nurse discusses activity levels with a 61-year-old patient with COPD and pneumonia. Which of the following exercise goals is most appropriate once the patient is fully recovered from this episode of illness? A. Slightly increase activity over the current level. B. Walk for 20 minutes a day, keeping the pulse rate less than 130 beats per minute. C. Limit exercise to activities of daily living to conserve energy. D. Swim for 10 min/day, gradually increasing to 30 min/day.

B. Walk for 20 minutes a day, keeping the pulse rate less than 130 beats per minute. The patient will benefit from mild aerobic exercise that does not stress the cardiorespiratory system. The patient should be encouraged to walk for 20 min/day, keeping the pulse rate less than 75% to 80% of maximum heart rate (220 minus patient's age).

Debilitating anginal pain can be decreased in some clients by the administration of beta-blocking agents such as nadolol (Corgard). Which client requires the nurse to use extreme caution when administering Corgard? A. A 56-year-old air traffic controller who had bypass surgery 2 years ago. B. A 47-year-old kindergarten teacher diagnosed with asthma 40 years ago C. A 52-year-old unemployed stock broker who refuses treatment for alcoholism D. A 60-year-old retired librarian who takes a diuretic daily for hypertension.

B. asthma must be carefully monitored because beta blockers because it can induce cardiogenic shock and reduce bronchodilation efforts. (A & D) this medication is indicated and (C) it is not contraindicated.

13. If a patient with an uncuffed tracheostomy tube coughs violently during suctioning and dislodges the tracheostomy tube, a nurse should first A. call the physician. B. attempt to reinsert the tracheostomy tube. C. position the patient in a lateral position with the neck extended. D. cover the stoma with a sterile dressing and ventilate the patient with a manual bag-mask until the physician arrives.

B. attempt to reinsert the tracheostomy tube.Retention sutures may be grasped (if present) and the tracheostomy opening spread, or a hemostat may be used to spread the opening. The obturator is inserted into the replacement tube (one size smaller than the original tube), lubricated with saline solution, and inserted into the stoma at a 45-degree angle to the neck. If the attempt is successful, the obturator tube should immediately be removed.

If a patient with an uncuffed tracheostomy tube coughs violently during suctioning and dislodges the tracheostomy tube, a nurse should first A. call the physician. B. attempt to reinsert the tracheostomy tube. C. position the patient in a lateral position with the neck extended. D. cover the stoma with a sterile dressing and ventilate the patient with a manual bag-mask until the physician arrives.

B. attempt to reinsert the tracheostomy tube.Retention sutures may be grasped (if present) and the tracheostomy opening spread, or a hemostat may be used to spread the opening. The obturator is inserted into the replacement tube (one size smaller than the original tube), lubricated with saline solution, and inserted into the stoma at a 45-degree angle to the neck. If the attempt is successful, the obturator tube should immediately be removed.

The nurse prepares to suction a client using nasotracheal suctioning. Which ongoing assessments should the nurse plan to complete while performing the procedure? A. amount of wall suction B. color of the lips C. breathing pattern D. breath sounds E. appearance of secretions

B. color of the lips C. breathing pattern E. appearance of secretions

1. A nurse is reviewing the hematologic test results for a patient in whom the hematocrit (Hct) is reported at a reading of 30%. Based on this result, the nurse should interpret that the patient A. is susceptible to bleeding disorders. B. has fewer red blood cells than normal. C. is experiencing an inflammatory response. D. is experiencing an acute hemolytic crisis.

B. has fewer red blood cells than normal.The Hct is the measure of the volume of red blood cells in whole blood expressed as a percentage. This test is useful in the diagnosis of anemia, polycythemia, and abnormal hydration states. Patients who are susceptible to bleeding disorders likely will have a low platelet count. The inflammatory response may best be evaluated by examination of results that include the white blood cell count with differential analysis. Acute hemolytic crisis develops in patients receiving blood components in which incompatibility occurs or in patients with bleeding disorders or conditions that promote cellular damage, such as damage associated with shock.

A nurse is reviewing the hematologic test results for a patient in whom the hematocrit (Hct) is reported at a reading of 30%. Based on this result, the nurse should interpret that the patient A. is susceptible to bleeding disorders. B. has fewer red blood cells than normal. C. is experiencing an inflammatory response. D. is experiencing an acute hemolytic crisis.

B. has fewer red blood cells than normal.The Hct is the measure of the volume of red blood cells in whole blood expressed as a percentage. This test is useful in the diagnosis of anemia, polycythemia, and abnormal hydration states. Patients who are susceptible to bleeding disorders likely will have a low platelet count. The inflammatory response may best be evaluated by examination of results that include the white blood cell count with differential analysis. Acute hemolytic crisis develops in patients receiving blood components in which incompatibility occurs or in patients with bleeding disorders or conditions that promote cellular damage, such as damage associated with shock.

Nursing interventions for the patient with aplastic anemia are directed toward the prevention of the complications of A. fatigue and dyspnea. B. hemorrhage and infection. C. thromboemboli and gangrene. D. cardiac arrhythmias and heart failure.

B. hemorrhage and infection. Hemorrhage from thrombocytopenia and infection from neutropenia are the greatest risks for the patient with aplastic anemia. The patient will experience fatigue from anemia, but bleeding and infection are the major causes of death in aplastic anemia.

A family member was taught to suction a client's tracheostomy prior to the client's discharge from the hospital. Which observation by the nurse indicates that the family member is capable of correctly performing the suctioning technique? A. Turns on the continuous wall suction to -190 mm Hg B. Inserts the catheter until resistance or coughing occurs C. Withdraws the catheter while maintaining suctioning D. Re-clears the tracheostomy after suctioning the mouth

B. indicates correct technique for performing suctioning. Suction pressure should be between -80 and -120 (A). The catheter should be withdrawn 1-2 cm at a time with intermittent suction (C). (D) introduces pathogens.

3. In preparing the preoperative teaching plan for a patient who is to undergo a total laryngectomy, a nurse should give highest priority to the A. tracheostomy being in place for 2 to 3 days. B. patient's not being able to speak normally again. C. insertion of a gastrostomy feeding tube during surgery. D. patient's not being able to perform deep-breathing exercises.

B. patient's not being able to speak normally again. Patients who have a total laryngectomy have a permanent tracheostomy and will need to learn how to speak using alternative methods, such as an artificial larynx. The tracheostomy will be permanent to allow normal breathing patterns and air exchange. After surgery, the patient's nutrition is supplemented with enteral feedings, and when the patient can swallow secretions, oral feedings can begin. Deep-breathing exercises should be performed with the patient at least every 2 hours to prevent further pulmonary complications.

In preparing the preoperative teaching plan for a patient who is to undergo a total laryngectomy, a nurse should give highest priority to the A. tracheostomy being in place for 2 to 3 days. B. patient's not being able to speak normally again. C. insertion of a gastrostomy feeding tube during surgery. D. patient's not being able to perform deep-breathing exercises.

B. patient's not being able to speak normally again. Patients who have a total laryngectomy have a permanent tracheostomy and will need to learn how to speak using alternative methods, such as an artificial larynx. The tracheostomy will be permanent to allow normal breathing patterns and air exchange. After surgery, the patient's nutrition is supplemented with enteral feedings, and when the patient can swallow secretions, oral feedings can begin. Deep-breathing exercises should be performed with the patient at least every 2 hours to prevent further pulmonary complications.

19. A patient with hemophilia is hospitalized with acute knee pain and swelling. An appropriate nursing intervention for the patient includes A. wrapping the knee with an elastic bandage. B. placing the patient on bed rest and applying ice to the joint. C. gently performing range-of-motion (ROM) exercises to the knee to prevent adhesions. D. administering nonsteroidal anti-inflammatory drugs (NSAIDs) as needed for pain.

B. placing the patient on bed rest and applying ice to the joint. During an acute bleeding episode in a joint, it is important to totally rest the involved joint and slow bleeding with application of ice. Drugs that decrease platelet aggregation, such as aspirin or NSAIDs, should not be used for pain. As soon as bleeding stops, mobilization of the affected area is encouraged with range-of-motion (ROM) exercises and physical therapy.

A patient with hemophilia is hospitalized with acute knee pain and swelling. An appropriate nursing intervention for the patient includes A. wrapping the knee with an elastic bandage. B. placing the patient on bed rest and applying ice to the joint. C. gently performing range-of-motion (ROM) exercises to the knee to prevent adhesions. D. administering nonsteroidal anti-inflammatory drugs (NSAIDs) as needed for pain.

B. placing the patient on bed rest and applying ice to the joint. During an acute bleeding episode in a joint, it is important to totally rest the involved joint and slow bleeding with application of ice. Drugs that decrease platelet aggregation, such as aspirin or NSAIDs, should not be used for pain. As soon as bleeding stops, mobilization of the affected area is encouraged with range-of-motion (ROM) exercises and physical therapy.

36. The resurgence in TB resulting from the emergence of multidrug-resistant strains of Mycobacterium tuberculosis is primarily the result of A. a lack of effective means to diagnose TB. B. poor compliance with drug therapy in patients with TB. C. the increased population of immunosuppressed individuals with AIDS. D. indiscriminate use of antitubercular drugs in treatment of other infections.

B. poor compliance with drug therapy in patients with TB. Drug-resistant strains of TB have developed because TB patients' compliance to drug therapy has been poor and there has been general decreased vigilance in monitoring and follow-up of TB treatment. Antitubercular drugs are almost exclusively used for TB infections. TB can be effectively diagnosed with sputum cultures. The incidence of TB is at epidemic proportions in patients with HIV, but this does not account for drug-resistant strains of TB.

The resurgence in TB resulting from the emergence of multidrug-resistant strains of Mycobacterium tuberculosis is primarily the result of A. a lack of effective means to diagnose TB. B. poor compliance with drug therapy in patients with TB. C. the increased population of immunosuppressed individuals with AIDS. D. indiscriminate use of antitubercular drugs in treatment of other infections.

B. poor compliance with drug therapy in patients with TB. Drug-resistant strains of TB have developed because TB patients' compliance to drug therapy has been poor and there has been general decreased vigilance in monitoring and follow-up of TB treatment. Antitubercular drugs are almost exclusively used for TB infections. TB can be effectively diagnosed with sputum cultures. The incidence of TB is at epidemic proportions in patients with HIV, but this does not account for drug-resistant strains of TB.

2. A patient admitted to the emergency department with tension pneumothorax and mediastinal shift following an automobile crash is most likely to exhibit A. bradycardia. B. severe hypotension. C. mediastinal flutter. D. a sucking chest wound.

B. severe hypotension. Mediastinal shift may cause compression of the lung in the direction of the shift and compression, traction, torsion, or kinking of the great vessels. Blood return to the heart is dangerously impaired and causes a subsequent decrease in cardiac output and blood pressure. Tachycardia is a clinical manifestation of tension pneumothorax. An uncovered opened pneumothorax is associated with a sucking chest wound and mediastinal flutter.

A patient admitted to the emergency department with tension pneumothorax and mediastinal shift following an automobile crash is most likely to exhibit A. bradycardia. B. severe hypotension. C. mediastinal flutter. D. a sucking chest wound.

B. severe hypotension. Mediastinal shift may cause compression of the lung in the direction of the shift and compression, traction, torsion, or kinking of the great vessels. Blood return to the heart is dangerously impaired and causes a subsequent decrease in cardiac output and blood pressure. Tachycardia is a clinical manifestation of tension pneumothorax. An uncovered opened pneumothorax is associated with a sucking chest wound and mediastinal flutter.

25. A 75-year-old obese patient who is snoring loudly and having periods of apnea several times each night is most likely experiencing A. narcolepsy. B. sleep apnea. C. sleep deprivation. D. paroxysmal nocturnal dyspnea.

B. sleep apnea. Sleep apnea is most common in obese patients. Typical symptoms include snoring and periods of apnea. Narcolepsy is when a patient falls asleep unexpectedly. Sleep deprivation could result from sleep apnea. Paroxysmal nocturnal dyspnea occurs when a patient has shortness of breath during the night.

A 75-year-old obese patient who is snoring loudly and having periods of apnea several times each night is most likely experiencing A. narcolepsy. B. sleep apnea. C. sleep deprivation. D. paroxysmal nocturnal dyspnea.

B. sleep apnea. Sleep apnea is most common in obese patients. Typical symptoms include snoring and periods of apnea. Narcolepsy is when a patient falls asleep unexpectedly. Sleep deprivation could result from sleep apnea. Paroxysmal nocturnal dyspnea occurs when a patient has shortness of breath during the night.

A client who recently underwent a tracheostomy is being prepared for discharge to home. Which instructions is most important for the nurse to include in the discharge plan? A. explain how to use communication tools B. teach tracheal suctioning techniques C. encourage self-care and independence D. demonstrate how to clean tracheostomy site

B. teach tracheal suctioning techniques

40. Select all that apply. Atelectasis can be caused by A. long-term smoking. B. inadequate surfactant. C. localized airway obstruction. D. an increase in lung expansion. E. an increase in elastic recoil.

BCE The collapse of lung tissue has several causes, including reduced lung expansion, localized airway obstruction, inadequate surfactant, and an increase in elastic recoil. Smoking, although harmful, does not in itself cause atelectasis.

Select all that apply. Atelectasis can be caused by A. long-term smoking. B. inadequate surfactant. C. localized airway obstruction. D. an increase in lung expansion. E. an increase in elastic recoil.

BCE The collapse of lung tissue has several causes, including reduced lung expansion, localized airway obstruction, inadequate surfactant, and an increase in elastic recoil. Smoking, although harmful, does not in itself cause atelectasis.

21. Select all that apply. Which of the following are significant risk factors for leukemia? A. Being a longtime smoker B. Employment in an oil refinery C. History of hemophilia in parent D. Having Down syndrome E. Having a twin brother with leukemia F. Treatment with an alkylating agent = 3 years ago

BDEF Exposure to chemical agents, treatment with alkylating cancer drugs, leukemia in a sibling, and the patient's having Down syndrome are all risk factors for leukemia.

Select all that apply. Which of the following are significant risk factors for leukemia? A. Being a longtime smoker B. Employment in an oil refinery C. History of hemophilia in parent D. Having Down syndrome E. Having a twin brother with leukemia F. Treatment with an alkylating agent = 3 years ago

BDEF Exposure to chemical agents, treatment with alkylating cancer drugs, leukemia in a sibling, and the patient's having Down syndrome are all risk factors for leukemia.

An infection of the pia mater and arachnoid layers of the meninges,and the CSF inthe subarachnoid space

Bacterial Meningitis

Diagnostic study for diverticular diseases

Barium enema or colonoscopy positive for diverticular disease. Barium not used during active phase of illness. Obstruction, Ileus or perforation confirmed by abdominal radiograph.

Initial care for DVT would be

Bedrest Elevate Extremity

A Cushing Syndrome post-op patient is on

Bedrest until BP stabilizes

Resuscitative/emergent stage: 48-72 hrs

Begins at time of injury and concludes with the restoration of capillary permeability. Characterized by fluid shift from intravascular to interstitial and shock.dnsjd Candid Expect to administer large volumes of fluid. Fluid replacement formulas calculated from time of injury.

Treatment of bladder spasms:

Belladonna and Opium Suppositories (Used to treat moderate to severe pain caused by muscle spasms in the tubes that connect the kidneys to the bladder)

Assess all male clients older than 50 years for symptoms of ______ ___________ __________. Occurs in 80% of men older than 80 years

Benign Prostatic Hypertrophy

Fibroadenoma

Benign, mobile Occurs btwn 15-40 years old Small, painless Round, well-delineated Solid, firm, rubbery Size not affected by menstruation Pregnancy stimulates dramatic growth

-cocci

Berry shaped bacteria.

Occurs often because of the presence of clots in the catheter

Bladder Spasms

After a thyroidectomy, the nurse should assess for

Bleeding Irregular Breathing Neck Swelling Frequent Swallowing Sensations of fullness at incision Choking Blood on dressings anteriorly AND posteriorly

Ischemia

Blood is held back from an area.

Insufficient oxygenation occurs with chronic BRONCHITIS and leads to generalized cyanosis and often right-sided heart failure (cor pulmonale)

Blue Bloater

Type of breath sounds that are heard over areas of density and consolidation.

Bronchial Breath Sounds

Involuntary fixation of the hips when the neck is flexed toward the chest

Brudzinski's Signs

_______ can be heard over the thyroid gland in hyperthyroidism.

Bruits

Ascites

Buildup of fluid in the abdomen. Paracentesis may be needed.

Diverticulosis

Bulging pouches in the G.I. wall which push the mucosal lining through the surrounding muscle

Severity is determined by

Burn depth

Which finding should the nurse identify as most significant for a client diagnosed with polycystic kidney disease (PKD)? A) Hematuria. B) 2 pounds weight gain. C) 3+ bacteria in urine. D) Steady, dull flank pain.

C) 3+ bacteria in urine. Urinary tract infections (UTI) for a client with PKD require prompt antibiotic therapy to prevent renal damage and scarring which may cause further progression of the disease, so bacteria in the urine (C) is the most significant finding at this time. (A) is an expected finding from the rupture of the cysts. (B) does not provide a time frame to determine if the weight gain is a significant fluid fluctuation, which is determined within a 24-hour time frame. Although kidney pain can also be abrupt, episodic, and colicky related to bleeding into the cysts, (D) is more likely an early symptom in PKD.

After the fourth dose of gentamicin sulfate (Garamycin) IV, the nurse plans to draw blood samples to determine peak and trough levels. When are the best times to draw these samples? A) 15 minutes before and 15 minutes after the next dose. B) One hour before and one hour after the next dose. C) 5 minutes before and 30 minutes after the next dose. D) 30 minutes before and 30 minutes after the next dose.

C) 5 minutes before and 30 minutes after the next dose. Peak drug serum levels are achieved 30 minutes after IV administration of aminoglycosides. The best time to draw a trough is the closest time to the next administration (C). (A, B, and D) are not as good a time to draw the trough as (C). (B and D) are not the best times to draw the peak of an aminoglycoside that has been administered IV.

A client taking a thiazide diuretic for the past six months has a serum potassium level of 3. The nurse anticipates which change in prescription for the client? A) The dosage of the diuretic will be decreased. B) The diuretic will be discontinued. C) A potassium supplement will be prescribed. D) The dosage of the diuretic will be increased.

C) A potassium supplement will be prescribed. This client's potassium level is too low (normal is 3.5 to 5). Taking a thiazide diuretic often results in a loss of potassium, so a potassium supplement needs to be prescribed to restore a normal serum potassium level (C). (A, B, and D) are not recommended actions for restoring a normal serum potassium level.

A female client taking oral contraceptives reports to the nurse that she is experiencing calf pain. What action should the nurse implement? A) Determine if the client has also experienced breast tenderness and weight gain. B) Encourage the client to begin a regular, daily program of walking and exercise. C) Advise the client to notify the healthcare provider for immediate medical attention. D) Tell the client to stop taking the medication for a week to see if symptoms subside.

C) Advise the client to notify the healthcare provider for immediate medical attention. Calf pain is indicative of thrombophlebitis, a serious, life-threatening complication associated with the use of oral contraceptives which requires further assessment and possibly immediate medical intervention (C). (A) are symptoms of oral contraceptive use, but are of less immediacy than (C). (B) may cause an embolism if thrombophlebitis is present. By not seeking immediate attention, (D) is potentially dangerous to the client.

Based on the analysis of the client's atrial fibrillation, the nurse should prepare the client for which treatment protocol? A) Diuretic therapy. B) Pacemaker implantation. C) Anticoagulation therapy. D) Cardiac catheterization.

C) Anticoagulation therapy. The client is experiencing atrial fibrillation, and the nurse should prepare the client for anticoagulation therapy (C) which should be prescribed before rhythm control therapies to prevent cardioembolic events which result from blood pooling in the fibrillating atria. (A, B, and D) are not indicated.

The nurse is teaching a client with maple syrup urine disease (MSUD), an autosomal recessive disorder, about the inheritance pattern. Which information should the nurse provide? A) This recessive disorder is carried only on the X chromosome. B) Occurrences mainly affect males and heterozygous females. C) Both genes of a pair must be abnormal for the disorder to occur. D) One copy of the abnormal gene is required for this disorder.

C) Both genes of a pair must be abnormal for the disorder to occur. Maple syrup urine disease (MSUD) is a type of autosomal recessive inheritance disorder in which both genes of a pair must be abnormal for the disorder to be expressed (C). MSUD is not an x-linked (A and B) dominant or recessive disorder or an autosomal dominant inheritance disorder. Both genes of a pair, not (D), must be present.

Which healthcare practice is most important for the nurse to teach a postmenopausal client? A) Wear layers of clothes if experiencing hot flashes. B) Use a water-soluble lubricant for vaginal dryness. C) Consume adequate foods rich in calcium. D) Participate in stimulating mental exercises.

C) Consume adequate foods rich in calcium. Bone density loss associated with osteoporosis increases at a more rapid rate when estrogen levels begin to fall, so the most important healthcare practice during menopause is ensuring an adequate calcium (C) intake to help maintain bone density and prevent osteoporosis. Although practices such as (A and B) may reduce some of the discomforts for a postmenopausal female, calcium intake is more important than comfort measures. Although social and mental exercises stimulate thought, there is no scientific evidence that mental exercises (D) prevent dementia or common forgetfulness associated with reduced hormonal levels.

A client is admitted to the medical intensive care unit with a diagnosis of myocardial infarction. The client's history indicates the infarction occurred ten hours ago. Which laboratory test result should the nurse expect this client to exhibit? A) Elevated LDH. B) Elevated serum amylase. C) Elevated CK-MB. D) Elevated hematocrit.

C) Elevated CK-MB. The cardiac isoenzyme CK-MB (C) is the most sensitive and most reliable indicator of myocardial damage of all the cardiac enzymes. It peaks within 12 to 20 hours after myocardial infarction (MI). (A) is a cardiac enzyme that peaks around 48 hours after an MI. (B) is expected with acute pancreatitis. (D) would be expected in a client with a fluid volume deficit, which is not a typical finding in MI.

Which postmenopausal client's complaint should the nurse refer to the healthcare provider? A) Breasts feel lumpy when palpated. B) History of white nipple discharge. C) Episodes of vaginal bleeding. D) Excessive diaphoresis occurs at night.

C) Episodes of vaginal bleeding. Postmenopausal vaginal bleeding (C) may be an indication of endometrial cancer, which should be reported to the healthcare provider. Compared to a new-onset of a single lump, breasts that feel lumpy (A) overall may be a normal variant or a finding consistent with nonmalignant fibrocystic disease. Up to 80% of women experience (B), depending on sexual stimulation or hormonal levels, and is no longer recommended as a reportable symptom when discovered during breast self-exam (BSE). The client may need further teaching concerning (D), a disturbing symptom, but it is not as important as (C).

While working in the emergency room, the nurse is exposed to a client with active tuberculosis. When should the nurse plan to obtain a tuberculin skin test? A) Immediately after the exposure. B) Within one week of the exposure. C) Four to six weeks after the exposure. D) Three months after the exposure.

C) Four to six weeks after the exposure. A tuberculin skin test is effective 4 to 6 weeks after an exposure (C), so the individual with a known exposure should wait 4 to 6 weeks before having a tuberculin skin test.

The nurse is completing an admission interview and assessment on a client with a history of Parkinson's disease. Which question should provide information relevant to the client's plan of care? A) Have you ever experienced any paralysis of your arms or legs? B) Have you ever sustained a severe head injury? C) Have you ever been 'frozen' in one spot, unable to move? D) Do you have headaches, especially ones with throbbing pain?

C) Have you ever been 'frozen' in one spot, unable to move? Clients with Parkinson's disease frequently experience difficulty in initiating, maintaining, and performing motor activities. They may even experience being rooted to the spot and unable to move (C). Parkinson's disease does not cause (A). Parkinson's disease is not usually associated with (B), nor does it typically cause (D).

The nurse knows that lab values sometimes vary for the older client. Which data should the nurse expect to find when reviewing laboratory values of an 80-year-old male? A) Increased WBC, decreased RBC. B) Increased serum bilirubin, slightly increased liver enzymes. C) Increased protein in the urine, slightly increased serum glucose levels. D) Decreased serum sodium, an increased urine specific gravity.

C) Increased protein in the urine, slightly increased serum glucose levels. In older adults, the protein found in urine slightly rises probably as a result of kidney changes or subclinical urinary tract infections. The serum glucose increases slightly due to changes in the kidney. The specific gravity declines by age 80 from 1.032 to 1.024.

The nurse is working with a 71-year-old obese client with bilateral osteoarthritis (OA) of the hips. What recommendation should the nurse make that is most beneficial in protecting the client's joints? A) Increase the amount of calcium intake in the diet. B) Apply alternating heat and cold therapies. C) Initiate a weight-reduction diet to achieve a healthy body weight. D) Use a walker for ambulation to lessen weight-bearing on the hips.

C) Initiate a weight-reduction diet to achieve a healthy body weight. Achieving a healthy weight (C) is critical to protect the joints of clients with OA. Increasing the amount of calcium in the client's diet (A) will not protect hip joints from the effects of OA. Thermal therapies may lessen pain and stiffness from OA but are not protective of the joints (B). Assistive devices such as a walker may be beneficial to help avoid falls and assist in ambulation but are not protective against OA's effects (D).

During an interview with a client planning elective surgery, the client asks the nurse, "What is the advantage of having a preferred provider organization insurance plan?" Which response is best for the nurse to provide? A) Long-term relationships with healthcare providers are more likely. B) There are fewer healthcare providers to choose from than in an HMO plan. C) Insurance coverage of employees is less expensive to employers. D) An individual can become a member of a PPO without belonging to a group.

C) Insurance coverage of employees is less expensive to employers. The financial advantage of (C) is the feature of a PPO that is most relevant to the average consumer. The nurse must have knowledge about PPOs, which provide discounted rates to large employers who provide insurance coverage for their employees. In return, the insurance company receives a large pool of clients for their facilities. (A, B, and D) are not accurate representations of the PPO.

The nurse is planning care to prevent complication for a client with multiple myeloma. Which intervention is most important for the nurse to include? A) Safety precautions during activity. B) Assess for changes in size of lymph nodes. C) Maintain a fluid intake of 3 to 4 L per day. D) Administer narcotic analgesic around the clock.

C) Maintain a fluid intake of 3 to 4 L per day. Multiple myeloma is a malignancy of plasma cells that infiltrate bone causing demineralization and hypercalcemia, so maintaining a urinary output of 1.5 to 2 L per day requires an intake of 3 to 4 L (C) to promote excretion of serum calcium. Although the client is at risk for pathologic fractures due to diffuse osteoporosis, mobilization and weight bearing (A) should be encouraged to promote bone reabsorption of circulating calcium, which can cause renal complications. (B) is a component of ongoing assessment. Chronic pain management (D) should be included in the plan of care, but prevention of complications related to hypercalcemia is most important.

During lung assessment, the nurse places a stethoscope on a client's chest and instructs him/her to say "99" each time the chest is touched with the stethoscope. What should be the correct interpretation if the nurse hears the spoken words "99" very clearly through the stethoscope? A) This is a normal auscultatory finding. B) May indicate pneumothorax. C) May indicate pneumonia. D) May indicate severe emphysema.

C) May indicate pneumonia. This test (whispered pectoriloquy) demonstrates hyperresonance and helps determine the clarity with which spoken words are heard upon auscultation. Normally, the spoken word is not well transmitted through lung tissue, and is heard as a muffled or unclear transmission of the spoken word. Increased clarity of a spoken word is indicative of some sort of consolidation process (e.g., tumor, pneumonia) (C), and is not a normal finding (A). When lung tissue is filled with more air than normal, the voice sounds are absent or very diminished (e.g., pneumothorax, severe emphysema) (B and D).

The nurse is assessing a client who smokes cigarettes and has been diagnosed with emphysema. Which finding should the nurse expect this client to exhibit? A) A decreased total lung capacity. B) Normal arterial blood gases. C) Normal skin coloring. D) An absence of sputum.

C) Normal skin coloring. The differentiation between the "pink puffer" and the "blue bloater" is a well-known method of differentiating clients exhibiting symptoms of emphysema (normal color but puffing respirations) from those exhibiting symptoms of chronic bronchitis (edematous, cyanotic, shallow respirations) (C). Total lung capacity is increased in emphysema since these clients have hyperinflated lungs (A). Arterial blood gases are typically abnormal (B). (D) is indicative of bronchitis, while clients with emphysema usually have copious amounts of thick, white sputum.

A 67-year-old woman who lives alone is admitted after tripping on a rug in her home and fractures her hip. Which predisposing factor probably led to the fracture in the proximal end of her femur? A) Failing eyesight resulting in an unsafe environment. B) Renal osteodystrophy resulting from chronic renal failure. C) Osteoporosis resulting from hormonal changes. D) Cardiovascular changes resulting in small strokes which impair mental acuity.

C) Osteoporosis resulting from hormonal changes. The most common cause of a fractured hip in elderly women is osteoporosis, resulting from reduced calcium in the bones as a result of hormonal changes in later life (C). (A) may or may not have contributed to the accident, but it had nothing to do with the hip being involved. (B) is not a common condition of the elderly; it is common in chronic renal failure. (D) may occur in some people, but does not affect the fragility of the bones as osteoporosis does.

An elderly male client comes to the geriatric screening clinic complaining of pain in his left calf. The nurse notices a reddened area on the calf of his right leg which is warm to the touch and suspects it might be thrombophlebitis. Which type of pain should further confirm this suspicion? A) Pain in the calf awakening him from a sound sleep. B) Calf pain on exertion which stops when standing in one place. C) Pain in the calf upon exertion which is relieved by rest and elevating the extremity. D) Pain upon arising in the morning which is relieved after some stretching and exercise.

C) Pain in the calf upon exertion which is relieved by rest and elevating the extremity. Thrombophlebitis pain is relieved by rest and elevation of the extremity (C). It typically occurs with exercise at the site of the thrombus, and is aggravated by placing the extremity in a dependent position, such as standing in one place (B). (A and D) describe pain that is not common with thrombophlebitis.

In assessing a client diagnosed with primary hyperaldosteronism, the nurse expects the laboratory test results to indicate a decreased serum level of which substance? A) Sodium. B) Antidiuretic hormone. C) Potassium. D) Glucose.

C) Potassium. Clients with primary aldosteronism exhibit a profound decline in the serum levels of potassium (C) (hypokalemia)--hypertension is the most prominent and universal sign. (A) is normal or elevated, depending on the amount of water reabsorbed with the sodium. (B) is decreased with diabetes insipidus. (D) is not affected by primary aldosteronism.

A client who was in a motor vehicle collision was admitted to the hospital and the right knee was placed in skeletal traction. The nurse has documented this nursing diagnosis in the client's medical record: "Potential for impairment of skin integrity related to immobility from traction." Which nursing intervention is indicated based on this diagnosis statement? A) Release the traction q4h to provide skin care. B) Turn the client for back care while suspending traction. C) Provide back and skin care while maintaining the traction. D) Give back care after the client is released from traction.

C) Provide back and skin care while maintaining the traction. (C) indicates that back care is performed while traction is left intact, which is the correct intervention for maintaining skin integrity. Maintaining skin integrity and providing back care is difficult when a client is in traction, but it cannot be delayed until the client is removed from traction (D). The nurse should never release the traction (A and B).

A client with multiple sclerosis has experienced an exacerbation of symptoms, including paresthesias, diplopia, and nystagmus. Which instruction should the nurse provide? A) Stay out of direct sunlight. B) Restrict intake of high protein foods. C) Schedule extra rest periods. D) Go to the emergency room immediately.

C) Schedule extra rest periods. Exacerbations of the symptoms of MS occur most commonly as the result of fatigue and stress. Extra rest periods should be scheduled (C) to reduce the symptoms. (A, B, and D) are not necessary.

When preparing a client who has had a total laryngectomy for discharge, which instruction is most important for the nurse to include in the discharge teaching? A) Recommend that the client carry suction equipment at all times. B) Instruct the client to have writing materials with him at all times. C) Tell the client to carry a medic alert card stating that he is a total neck breather. D) Tell the client not to travel alone.

C) Tell the client to carry a medic alert card stating that he is a total neck breather. It is imperative that total neck breathers carry a medic alert notice (C) so that if they have a cardiac arrest, mouth-to-neck breathing can be done. Mouth-to-mouth resuscitation will not help them. They do not need to carry (A) nor refrain from (D). There are many alternative means of communication for clients who have had a laryngectomy; depending on (B) is probably the least effective. How do you know he can read and write?

A client with early breast cancer receives the results of a breast biopsy and asks the nurse to explain the meaning of staging and the type of receptors found on the cancer cells. Which explanation should the nurse provide? A) Lymph node involvement is not significant. B) Small tumors are aggressive and indicate poor prognosis. C) The tumor's estrogen receptor guides treatment options. D) Stage I indicates metastasis.

C) The tumor's estrogen receptor guides treatment options. Treatment decisions (C) and prediction of prognosis are related to the tumor's receptor status, such as estrogen and progesterone receptor status which commonly are well-differentiated, have a lower chance of recurrence, and are receptive to hormonal therapy. Tumor staging designates tumor size and spread of breast cancer cells into axillary lymph nodes, which is one of the most important prognostic factors in early-stage breast cancer, not (A). Larger tumors are more likely to indicate poor prognosis, not (B). Stage I indicates the cancer is localized and has not spread systemically (D).

What discharge instruction is most important for a client after a kidney transplant? A) Weigh weekly. B) Report symptoms of secondary Candidiasis. C) Use daily reminders to take immunosuppressants. D) Stop cigarette smoking.

C) Use daily reminders to take immunosuppressants. After renal transplantation, acute rejection is a risk for several months, so immunosuppressive therapy, such as corticosteroids and azathioprine (Imuran), is essential in preventing rejection, so the priority instruction includes measures, such as daily reminders (C), to ensure the client takes these medications regularly. Daily weights, not weekly (A), provides a better indicator of weight gain related to rejection. Although fungal infections related to the immunosuppression should be reported (B), it is more important to ensure medication compliance to prevent rejection. Although smoking (D) increases the risk of atherosclerotic vascular disease which is common in clients with an organ transplant, (C) remains the priority.

The nurse is planning care for a client with newly diagnosed diabetes mellitus that requires insulin. Which assessment should the nurse identify before beginning the teaching session? A) Present knowledge related to the skill of injection. B) Intelligence and developmental level of the client. C) Willingness of the client to learn the injection sites. D) Financial resources available for the equipment.

C) Willingness of the client to learn the injection sites. If a client is incapable or does not want to learn, it is unlikely that learning will occur, so motivation is the first factor the nurse should assess before teaching (C). To determine learning needs, the nurse should assess (A), but this is not the most important factor for the nurse to assess. (B and D) are factors to consider, but not as vital as (C).

Which information about mammograms is most important to provide a post-menopausal female client? A) Breast self-examinations are not needed if annual mammograms are obtained. B) Radiation exposure is minimized by shielding the abdomen with a lead-lined apron. C) Yearly mammograms should be done regardless of previous normal x-rays. D) Women at high risk should have annual routine and ultrasound mammograms.

C) Yearly mammograms should be done regardless of previous normal x-rays. The current breast screening recommendation is a yearly mammogram after age 40 (C). Breast self-exam (A) continues to be a priority recommendation for all women because a small lump (or tumor) is often first felt by a woman before a mammogram is obtained. The radiation exposure from a mammogram is low, so (B) is not normally provided. The frequency of using routine and ultrasound mammograms (D) in women with high-risk variables, such as a history of breast cancer, the presence of BRC1 and BRC2 genes, or 2 first-degree relatives with breast cancer, should be recommended and followed closely by the healthcare provider.

The nurse should be correct in withholding a dose of digoxin in a client with congestive heart failure without specific instruction from the healthcare provider if the client's A) serum digoxin level is 1.5. B) blood pressure is 104/68. C) serum potassium level is 3. D) apical pulse is 68/min.

C) serum potassium level is 3. Hypokalemia (C) can precipitate digitalis toxicity in persons receiving digoxin which will increase the chance of dangerous dysrhythmias (normal potassium level is 3.5 to 5.5 mEq/L). The therapeutic range for digoxin is 0.8 to 2 ng/ml (toxic levels= >2 ng/ml); (A) is within this range. (B) would not warrant the nurse withholding the digoxin. The nurse should withhold the digoxin if the apical pulse is less than 60/min (D).

A client has taken steroids for 12 years to help manage chronic obstructive pulmonary disease (COPD). When making a home visit, which nursing function is of greatest importance to this client? Assess the client's A) pulse rate, both apically and radially. B) blood pressure, both standing and sitting. C) temperature. D) skin color and turgor.

C) temperature. It is very important to check the client's temperature (C). Infection is the most common factor precipitating respiratory distress. Clients with COPD who are on maintenance doses of corticosteroids are particularly predisposed to infection. (A and B) are important data for baseline and ongoing assessment, but they are not as important as temperature measurement for this client who is taking steroids. Assessment of skin color and turgor is less important (D).

39. Select all that apply. Which of the following are clinical manifestations of tension pneumothorax? A. Midline trachea B. Severe hypertension C. Progressive cyanosis D. A loud bruit on affected side E. Asymmetrical chest wall movement F. Subcutaneous emphysema in the neck

C,E, F The indicators of tension pneumothorax are asymmetrical chest wall movement, severe hypotension, subcutaneous emphysema in the neck and upper chest, and progressive cyanosis.

51. Which of the following statements made by a patient with COPD indicates a need for further education regarding the use of an ipratropium inhaler? A. "I should rinse my mouth following the two puffs to get rid of the bad taste." B. "I should wait at least 1 to 2 minutes between each puff of the inhaler." C. "If my breathing gets worse, I should keep taking extra puffs of the inhaler until I can breathe more easily." D. "Because this medication is not fast-acting, I cannot use it in an emergency if my breathing gets worse.

C. "If my breathing gets worse, I should keep taking extra puffs of the inhaler until I can breathe more easily." The patient should not take extra puffs of the inhaler at will to make breathing easier. Excessive treatment could trigger paradoxical bronchospasm, which would worsen the patient's respiratory status.

Which of the following statements made by a patient with COPD indicates a need for further education regarding the use of an ipratropium inhaler? A. "I should rinse my mouth following the two puffs to get rid of the bad taste." B. "I should wait at least 1 to 2 minutes between each puff of the inhaler." C. "If my breathing gets worse, I should keep taking extra puffs of the inhaler until I can breathe more easily." D. "Because this medication is not fast-acting, I cannot use it in an emergency if my breathing gets worse.

C. "If my breathing gets worse, I should keep taking extra puffs of the inhaler until I can breathe more easily." The patient should not take extra puffs of the inhaler at will to make breathing easier. Excessive treatment could trigger paradoxical bronchospasm, which would worsen the patient's respiratory status.

19. Which of the following statements made by a nurse would indicate proper teaching principles regarding feeding and tracheostomies? A. "Follow each spoon of food consumed with a drink of fluid." B. "Thin your foods to a liquid consistency whenever possible." C. "Tilt your chin forward toward the chest when swallowing your food." D. "Make sure your cuff is overinflated before eating if you have swallowing problems."

C. "Tilt your chin forward toward the chest when swallowing your food." A nurse should instruct a patient to tilt the chin toward the chest, which will close the glottis and allow food to enter the normal passageway. Ideally, foods should be of a thick consistency to enable effective swallowing and reduce the risk of aspiration. Overinflation of the cuff causes swallowing difficulties. Fluids should be consumed in small amounts after swallowing to prevent the risk of aspiration.

Which of the following statements made by a nurse would indicate proper teaching principles regarding feeding and tracheostomies? A. "Follow each spoon of food consumed with a drink of fluid." B. "Thin your foods to a liquid consistency whenever possible." C. "Tilt your chin forward toward the chest when swallowing your food." D. "Make sure your cuff is overinflated before eating if you have swallowing problems."

C. "Tilt your chin forward toward the chest when swallowing your food." A nurse should instruct a patient to tilt the chin toward the chest, which will close the glottis and allow food to enter the normal passageway. Ideally, foods should be of a thick consistency to enable effective swallowing and reduce the risk of aspiration. Overinflation of the cuff causes swallowing difficulties. Fluids should be consumed in small amounts after swallowing to prevent the risk of aspiration.

A 58-year-old client with chronic kidney disease (CKD) is receiving aluminum hydroxide (Amphojel). He tells the nurse that since he does not have indigestion there is no need for him to take the antacid with his meals. Which response is best for the nurse to provide? A. "CKD stresses your body to over-secrete gastric juices, and antacids help neutralize them." B. "OK, I will let your healthcare provider know that you do not need the antacid." C. "Your serum phosphate levels are up, and aluminum antacids prevent absorption of phosphates in foods." D. "I will hold the antacids for now, and if you get indigestion, I will bring it back."

C. "Your serum phosphate levels are up, and aluminum antacids prevent absorption of phosphates in foods."

6. When preparing to administer an ordered blood transfusion, the nurse selects which of the following intravenous solutions to use when priming the blood tubing? A. 5% dextrose in water B. Lactated Ringer's C. 0.9% sodium chloride D. 0.45% sodium chloride

C. 0.9% sodium chloride The blood set should be primed before the transfusion with 0.9% sodium chloride, also known as normal saline. It is also used to flush the blood tubing after the infusion is complete to ensure the patient receives blood that is left in the tubing when the bag is empty.

When preparing to administer an ordered blood transfusion, the nurse selects which of the following intravenous solutions to use when priming the blood tubing? A. 5% dextrose in water B. Lactated Ringer's C. 0.9% sodium chloride D. 0.45% sodium chloride

C. 0.9% sodium chloride The blood set should be primed before the transfusion with 0.9% sodium chloride, also known as normal saline. It is also used to flush the blood tubing after the infusion is complete to ensure the patient receives blood that is left in the tubing when the bag is empty.

The home health nurse is assessing a male client being treated for Parkinson disease with levodopa-carbidopa (Sinemet). The nurse observes that he does not demonstrate any apparent emotions when speaking and rarely blinks. Which intervention should the nurse implement? A. Perform a complete cranial nerve assessment. B. Instruct the client that he may be experiencing medication toxicity. C. Document the presence of these assessment findings. D. Advise the client to seek immediate medical evaluation.

C. A mask-like expression and infrequent blinking are common clinical features of Parkinsonism. The nurse should document the findings. (A & D) are not necessary. Signs of toxicity (B) are dyskinesia, hallucinations, and psychosis.

A central venous catheter has been inserted via a jugular vein and a radiography has confirmed placement of the catheter. A prescription has been received for stat medication but IV fluids have not yet been started. What action should the nurse take prior to administering the prescribed medication? A. Assess for signs of jugular vein distention. B. Obtain the needed intravenous solution. C. Administer a bolus of normal saline solution. D. Flush the line with heparinized saline.

C. A medication can be administered central line without IV fluids, flush with normal saline to remove heparin that may counteract with the medication. (B) is used following the medication and a second saline bolus. (A) will not impact the the med administration and is not a priority. (B) Administration of the stat medication is more of a priority than (B).

What symptom is characteristic of ureteral colic in the clients diagnosed with renal calculi? A. Symptoms of irritation associated with urinary tract infection B. Intense, deep ache in the costovertebral region C. Acute, excruciating, wave-like pain radiating to the genitalia D. Chills, fever, and dysuria

C. Acute, excruciating, wave-like pain radiating to the genitalia

A female nurse who took drugs from the unit for personal use was temporarily released from duty. After completion of mandatory counseling, the nurse has asked administration to allow her to return to work. When the nurse administrator approaches the charge nurse with the impaired nurse's request, what action is best for the charge nurse to take? A.Since treatment is completed, assign the nurse to routine RN responsibilities B. Ask to meet with the impaired nurse's therapist before allowing her back on the unit C. All the impaired nurse to return to work and monitor medication administration D. Meet with the staff to assess their feelings about the impaired nurse's return to the unit

C. All the impaired nurse to return to work and monitor medication administration

A female nurse who took drugs from the unit for personal use was temporary released from duty. After completion of mandatory counseling, the nurse has asked administration to allow her to return to work. When the nurse administrator approaches the charge nurse with the impaired nurse's request, what action is best for the charge nurse to take? A. Since treatment is completed, assign the nurse to routine RN responsibilities B. Ask to meet with the impaired nurse's therapist before allowing her back on the unit C. Allow the impaired nurse to return to work and monitor medication administration D. Meet with staff to assess the feelings about the impaired nurse's return to work.

C. Allow the impaired nurse to return to work and monitor medication administration

The nurse is obtaining a blood sample via venipuncture from a preschool-aged child. Which intervention should the nurse implement? A. Explain in very simple terms why the blood is needed B. Encourage the child to talk about this experience C. Apply a large colored band-aid to the puncture site D. Place the labeled specimen in a paper cup for transport

C. Apply a large colored band-aid to the puncture site

About 85 victims of a train derailment are brought to the Emergency Department of a small rural hospital. An older male with extensive crush injuries to his lower extremities and pelvis has a blood pressure of 42/28, a thready pulse of 120 beats/minute, and a respiratory rate of 10 breaths/minute with periods of apnea. Using the disaster triage system, which action should the nurse take? A. Obtain the crash cart and defibrillator B. Transport to radiology department C. Assign a black triage color D. Initiate a large bore IV infusion

C. Assign a black triage color

An adult, Muslim client with ulcerative colitis was admitted to the post-surgical unit earlier today following a bowel resection with temporary colostomy. Which intervention is most important for the nurse to implement? A. Teach the client how to perform stoma care B. Allow family members to visit whenever they wish C. Assign a care provider of the same gender D. Evaluate the client's current nutritional status

C. Assign a care provider of the same gender

During a clinic visit, a male client with heart failure (HF) reports that he has gained 4 pounds (1.8 kg) in the last 3 days. Which action should the nurse implement? A. Recommend controlled portions at mealtimes B. Assess for bilateral pitting pedal edema C. Auscultate all lung fields for fine crackles D. Encourage a reduced intake of table salts

C. Auscultate all lung fields for fine crackles

A male client with multiple myeloma is admitted with pneumonia and pancytopenia. The nurse reviews the complete blood cell count findings and identifies a platelet count of 20,000 cells/mm^3. Which intervention should the nurse include in the client's plan of care? A. Pace activities between planned rest periods. B. Monitor intake and output C. Avoid intramuscular injections D. Limit exposure to visitors with respiratory infections

C. Avoid intramuscular injections

A client taking tamoxifen citrate following a lumpectomy reports several problems to the nurse. It is most important for the nurse to follow-up on which reported problem? A. Erratic menstrual periods B. Bone pain C. Calf tenderness D. Anorexia and nausea

C. Calf tenderness

The nurse is assessing the normal development of a 9-year-old male infant. Which information should the nurse obtain from the mother? A. Has the child started to walk? B. Is the baby able to lift his head when prone? C. Can the child sit alone? D. Does the baby roll from abdomen to back?

C. Can the child sit alone?

A 12-lead electrocardiogram (ECG) indicated a ST elevation in leads V1 to V4, for a client who reports having chest pain. The healthcare provider prescribes tissue plasminogen activator (t-PA). Prior to initiating the infusion, which intervention is most important for the nurse to implement? A. Place the ECG findings in the client's record B. Obtain a signed informed consent C. Complete pre-infusion checklist D. Insert two large bore IV sites

C. Complete pre-infusion checklist

An adult female client is admitted to the psychiatric unit because of a complex hand washing ritual she preforms daily that takes two hours or longer to complete. She worries about staying clean and refuses to sit on any of the chairs in the day area. This client's hand washing is an example of which clinical behavior? A. Obsession B. Addiction C. Compulsion D. Phobia

C. Compulsion

The preeclamptic client who delivered 24 hours ago remains in the labor and delivery recovery room. She continues to receive magnesium sulfate at 3 grams per hour. Her total input is limited to 125 ml per hour, and her urinary output for the last hour was 850 ml. What intervention should the nurse implement? A. Discontinue the magnesium sulfate immediately B. Decrease the client's IV rate to 50 ml per hour C. Continue with the plan of care for this client D. Change the client's diet to NPO status

C. Continue with the plan of care for this client

A client exposed to tuberculosis is scheduled to begin prophylactic treatment with isoniazid. Which information is most important for the nurse to note before administering the initial dose? A. Conversion of the client's PPD test from negative to positive B. Length of time of the exposure to tuberculosis C. Current diagnosis of hepatitis B D. History of intravenous drug abuse

C. Current diagnosis of hepatitis B

Which assessment finding has the highest priority when planning nursing care for a client with peptic ulcer disease (PUD)? A. Epigastric pain after eating B. Dizziness when rising form a sitting position C. Dark tarry liquid stool D. Weight loss of 10 pounds in the past month

C. Dark tarry liquid stool

The nurse is triaging victims of a tornado at an emergency shelter. An adult woman who has been wandering and crying comes to the nurse. What action should the nurse take? A. Check the client's temperature, blood sugar, and urine output B. Transport the client for laboratory and electrocardiogram (EKG) C. Delegate care of the crying client to an unlicensed assistant D. Send the client to the shelter's nutrient center to obtain water and food

C. Delegate care of the crying client to an unlicensed assistant

A client's serum potassium test level is 6 mEq/L. The laboratory indicates, "Specimen is hemolyzed." What action should the nurse take? A. Encourage the client to eat a banana B. Notify the healthcare provider of the laboratory finding C. Draw a new blood specimen D. Obtain a prescription for sodium polystyrene sulfonate (Kayexalate)

C. Draw a new blood specimen

43. The nurse evaluates that nursing interventions to promote airway clearance in a patient admitted with COPD are successful based on which of the following findings? A. Absence of dyspnea B. Improved mental status C. Effective and productive coughing D. PaO2 within normal range for the patient

C. Effective and productive coughing The issue of the question is airway clearance, which is most directly evaluated as successful if the patient can engage in effective and productive coughing.

The nurse evaluates that nursing interventions to promote airway clearance in a patient admitted with COPD are successful based on which of the following findings? A. Absence of dyspnea B. Improved mental status C. Effective and productive coughing D. PaO2 within normal range for the patient

C. Effective and productive coughing The issue of the question is airway clearance, which is most directly evaluated as successful if the patient can engage in effective and productive coughing.

A male client tells the nurse that he is concerned that he may have a stomach ulcer, because he is experiencing heartburn and a dull gnawing pain that is relieved when he eats. What is the best response by the nurse? A. Advice the client the needs to seek immediate medical evaluation and treatment of these symptoms. B. Assure the client that the symptoms may on reflect reflux, since ulcer pain is not relieved with food. C. Encourage the client to obtain a complete a physical exam since the symptoms are consistent with an ulcer D. Instruct the client that these mild symptoms can generally be resolved with changes in the diet.

C. Encourage the client to obtain a complete a physical exam since the symptoms are consistent with an ulcer

The nurse initiates a one-to-one relationship with a 35-year-old depressed female client who was recently admitted to the psychiatric facility. Which nursing action is most effective in promoting the development of a therapeutic relationship? A. At your first meeting clearly define the unit's rules and policies B. Obtain client data from her family to use at the first meeting with the client C. Ensure that scheduled appointments begin according to the schedule D. Re-direct all conversations to discussions about feelings of low self-esteem

C. Ensure that scheduled appointments begin according to the schedule

A newly hired male unlicensed assistive personnel (UAP) is assigned to a home healthcare team along with two experienced UAP's. Which intervention should the home health nurse implement to ensure adequate care for all clients? A. Assign the newly hired UAP to clients who require the least complex level of care B. Ask the most experienced UAP on the team to partner with the newly hired UAP C. Evaluate the newly hired UAP's level of competency by observing him deliver care D. Review the UAP's skills checklist and experience wit the person who hired him

C. Evaluate the newly hired UAP's level of competency by observing him deliver care

During the admission interview, the nurse learns that a newly admitted adult client has a six month history of reoccurring somatic pain. Which problem is most important for the nurse to further explore with the client? A. Nausea and vomiting B. Episodes of injury related to falls C. Feelings of depression D. Periods of anxiety and restlessness

C. Feelings of depression

46. Nursing assessment findings of jugular vein distention and pedal edema would be indicative of which of the following complications of emphysema? A. Acute respiratory failure B. Pulmonary edema caused by left-sided heart failure C. Fluid volume excess secondary to cor pulmonale D. Secondary respiratory infection

C. Fluid volume excess secondary to cor pulmonale Cor pulmonale is a right-sided heart failure caused by resistance to right ventricular outflow due to lung disease. With failure of the right ventricle, the blood emptying into the right atrium and ventricle would be slowed, leading to jugular venous distention and pedal edema.

Nursing assessment findings of jugular vein distention and pedal edema would be indicative of which of the following complications of emphysema? A. Acute respiratory failure B. Pulmonary edema caused by left-sided heart failure C. Fluid volume excess secondary to cor pulmonale D. Secondary respiratory infection

C. Fluid volume excess secondary to cor pulmonale Cor pulmonale is a right-sided heart failure caused by resistance to right ventricular outflow due to lung disease. With failure of the right ventricle, the blood emptying into the right atrium and ventricle would be slowed, leading to jugular venous distention and pedal edema.

Immediately after an elective cardioversion for rapid supraventricular tachycardia (SVT), a male client who was premedicated with hydromorphone (Dilaudid) and midazolam (Versed) is difficult to arouse. His vital signs are: oxygen saturation 94% while receiving oxygen at 2 L/minute per nasal cannula, heart rate 78 beats/minute, respirations 6 breaths/minute, and blood pressure 102/70. Which intervention should the nurse implement? A. Increase oxygen to 4 L/minute B. Prepare for another cardioversion C. Give IV naloxone (Narcan) D. Infuse normal saline IV bolus

C. Give IV naloxone (Narcan)

During the change of shift report, the charge nurse reviews the infusions being received by the clients on the oncology unit. The client receiving which infusion should be seen first?

C. Has the highest risk for respiratory depression and therefor should be seen first. (A) Risk of hypotension. (B) Lowest risk. (D) Risk of nephrotoxicity and phlebitis.

23. Which of the following positions is most appropriate for the nurse to place a patient experiencing an asthma exacerbation? A. Supine B. Lithotomy C. High-Fowler's D. Reverse Trendelenburg

C. High-Fowler'sThe patient experiencing an asthma attack should be placed in high-Fowler's position to allow for optimal chest expansion and enlist the aid of gravity during inspiration.

Which of the following positions is most appropriate for the nurse to place a patient experiencing an asthma exacerbation? A. Supine B. Lithotomy C. High-Fowler's D. Reverse Trendelenburg

C. High-Fowler'sThe patient experiencing an asthma attack should be placed in high-Fowler's position to allow for optimal chest expansion and enlist the aid of gravity during inspiration.

The nurse know that normal lab values expected for an adult may vary in an older client. Which data would the nurse expect to find when reviewing laboratory values of an 80-year-old man who is in good overall health. A. Complet blood count reveals increased WBC and decreased RBC counts. B. Chemistries reveal an increased serum bilirubin with slightly increased liver enzymes. C. Urinalysis reveals slight protein in the urine and bacteriuria with pyuria. D. Serum electrolytes reveal a decreased sodium level with an increased potassium level.

C. In older adults the protein found in urine is slightly risen as a result of kidney changes or subclinical UTIs and the client frequently experiences asymptomatic bacteriuria and pyuria as a result of incomplete bladder emptying. (A, B, D) are not normal findings.

8. Which of the following clinical manifestations would the nurse expect to find during assessment of a patient admitted with pneumococcal pneumonia? A. Hyperresonance on percussion B. Fine crackles in all lobes on auscultation C. Increased vocal fremitus on palpation D. Vesicular breath sounds in all lobes

C. Increased vocal fremitus on palpation. A typical physical examination finding for a patient with pneumonia is increased vocal fremitus on palpation. Other signs of pulmonary consolidation include dullness to percussion, bronchial breath sounds, and crackles in the affected area.

Which of the following clinical manifestations would the nurse expect to find during assessment of a patient admitted with pneumococcal pneumonia? A. Hyperresonance on percussion B. Fine crackles in all lobes on auscultation C. Increased vocal fremitus on palpation D. Vesicular breath sounds in all lobes

C. Increased vocal fremitus on palpation. A typical physical examination finding for a patient with pneumonia is increased vocal fremitus on palpation. Other signs of pulmonary consolidation include dullness to percussion, bronchial breath sounds, and crackles in the affected area.

A community health nurse is concerned about the incidence of asthma among preschool aged children in a metropolitan area. Which interventions reflects a primary prevention strategy the nurse might initiate to combat the chronic illness? A. Partner with a major pharmaceutical company to provide nebulizers at a reduced cost to inner-city children B. Refer parents of preschoolers with asthma to a support group sponsored by the American Lung Association C. Inform the city council of the need to strengthen the city's air pollution ordinances D. Offer free asthma screening to children at a health fair sponsored by a local hospital

C. Inform the city council of the need to strengthen the city's air pollution ordinances

Which intervention regarding immediate postoperative care should the nurse plan to include in the preoperative teaching of a client scheduled for an incisional rotator cuff repair? A. Ice will be applied to the incision prior to exercising to help decrease pain B. A cast will be used to hold the joint securely in place until it is healed C. It will be necessary to wear a sling to keep the joint still D. A special machine will be used to keep the joint moving

C. It will be necessary to wear a sling to keep the joint still

28. While teaching a patient with asthma about the appropriate use of a peak flow meter, the nurse instructs the patient to do which of the following? A. Use the flow meter each morning after taking medications to evaluate their effectiveness. B. Empty the lungs and then inhale quickly through the mouthpiece to measure how fast air can be inhaled. C. Keep a record of the peak flow meter numbers if symptoms of asthma are getting worse. D. Increase the doses of the long-term control medication if the peak flow numbers decrease.

C. Keep a record of the peak flow meter numbers if symptoms of asthma are getting worse. It is important to keep track of peak flow readings daily and when the patient's symptoms are getting worse. The patient should have specific directions as to when to call the physician based on personal peak flow numbers. Peak flow is measured by exhaling into the meters and should be assessed before and after medications to evaluate their effectiveness.

While teaching a patient with asthma about the appropriate use of a peak flow meter, the nurse instructs the patient to do which of the following? A. Use the flow meter each morning after taking medications to evaluate their effectiveness. B. Empty the lungs and then inhale quickly through the mouthpiece to measure how fast air can be inhaled. C. Keep a record of the peak flow meter numbers if symptoms of asthma are getting worse. D. Increase the doses of the long-term control medication if the peak flow numbers decrease.

C. Keep a record of the peak flow meter numbers if symptoms of asthma are getting worse. It is important to keep track of peak flow readings daily and when the patient's symptoms are getting worse. The patient should have specific directions as to when to call the physician based on personal peak flow numbers. Peak flow is measured by exhaling into the meters and should be assessed before and after medications to evaluate their effectiveness.

The nurse is assessing a 4-year-old with eczema. Her skin is dry and scaly, and the mother reports that she frequently scratches her skin to the point of causing bleeding. Which guideline is indicated for care for this child? A. Apply baby lotion to her skin twice daily B. Allow her to wear only 100% cotton clothing C. Keep her nails trimmed short D. Bathe her daily with bath oil

C. Keep her nails trimmed short

An infant has a medical diagnosis of tracheosophageal fistula (TEF). Which nursing intervention is indicated for this infant prior to surgical repair? A. Administer isotonic enemas as prescribed B. Evaluate the infant's tolerance for small volume of formula C. Maintain suction equipment available at all times D. Prepare the child for a barium enema to correct the condition

C. Maintain suction equipment available at all times

6. What type of anemia is associated with folate deficiency? A. Microcytic B. Pernicious C. Megaloblastic D. Iron deficiency

C. Megaloblastic Megaloblastic anemia is nutritional anemia; large immature red blood cells with a decreased oxygen-carrying capacity can occur as a result of impaired DNA synthesis. Folic acid is used in the synthesis of DNA and helps convert B12 to coenzyme form. Folic acid is needed for growth and development of red blood cells. Microcytic anemia is anemia with abnormally small erythrocytes (red blood cells) in Hb. This anemia is associated with vitamin B6 (pyridoxine) deficiency. Pernicious anemia is caused by a deficiency of vitamin B12. Iron-deficiency anemia results from loss of blood or deficient intake of iron foods or disease states in which the body does not absorb or utilize iron as it should.

What type of anemia is associated with folate deficiency? A. Microcytic B. Pernicious C. Megaloblastic D. Iron deficiency

C. Megaloblastic Megaloblastic anemia is nutritional anemia; large immature red blood cells with a decreased oxygen-carrying capacity can occur as a result of impaired DNA synthesis. Folic acid is used in the synthesis of DNA and helps convert B12 to coenzyme form. Folic acid is needed for growth and development of red blood cells. Microcytic anemia is anemia with abnormally small erythrocytes (red blood cells) in Hb. This anemia is associated with vitamin B6 (pyridoxine) deficiency. Pernicious anemia is caused by a deficiency of vitamin B12. Iron-deficiency anemia results from loss of blood or deficient intake of iron foods or disease states in which the body does not absorb or utilize iron as it should.

The nurse identifies which recent event as placing a client at high risk for cardiogenic shock? A. Gunshot wounds to the chest and abdomen B. Traumatic amputation of the leg at the groin C. Myocardial infraction in the right ventricle D. Multiple bee stings around the head and neck

C. Myocardial infraction in the right ventricle

20. During assessment of a 45-year-old patient with asthma, the nurse notes wheezing and dyspnea. The nurse interprets that these symptoms are related to which of the following pathophysiologic changes? A. Laryngospasm B. Overdistention of the alveoli C. Narrowing of the airway D. Pulmonary edema

C. Narrowing of the airwayNarrowing of the airway leads to reduced airflow, making it difficult for the patient to breathe and producing the characteristic wheezing.

During assessment of a 45-year-old patient with asthma, the nurse notes wheezing and dyspnea. The nurse interprets that these symptoms are related to which of the following pathophysiologic changes? A. Laryngospasm B. Overdistention of the alveoli C. Narrowing of the airway D. Pulmonary edema

C. Narrowing of the airwayNarrowing of the airway leads to reduced airflow, making it difficult for the patient to breathe and producing the characteristic wheezing.

16. While ambulating a patient with metastatic lung cancer, the nurse observes a drop in oxygen saturation from 93% to 86%. Which of the following nursing interventions is most appropriate based upon these findings? A. Continue with ambulation as this is a normal response to activity. B. Move the oximetry probe from the finger to the earlobe for more accurate monitoring during activity. C. Obtain a physician's order for supplemental oxygen to be used during ambulation and other activity. D. Obtain a physician's order for arterial blood gas determinations to verify the oxygen saturation.

C. Obtain a physician's order for supplemental oxygen to be used during ambulation and other activity. An oxygen saturation level that drops below 90% with activity indicates that the patient is not tolerating the exercise and needs to have supplemental oxygen applied.

While ambulating a patient with metastatic lung cancer, the nurse observes a drop in oxygen saturation from 93% to 86%. Which of the following nursing interventions is most appropriate based upon these findings? A. Continue with ambulation as this is a normal response to activity. B. Move the oximetry probe from the finger to the earlobe for more accurate monitoring during activity. C. Obtain a physician's order for supplemental oxygen to be used during ambulation and other activity. D. Obtain a physician's order for arterial blood gas determinations to verify the oxygen saturation.

C. Obtain a physician's order for supplemental oxygen to be used during ambulation and other activity. An oxygen saturation level that drops below 90% with activity indicates that the patient is not tolerating the exercise and needs to have supplemental oxygen applied.

11. When assessing a patient's respiratory status, which of the following nonrespiratory data are most important for the nurse to obtain? A. Height and weight B. Neck circumference C. Occupation and hobbies D. Usual daily fluid intake

C. Occupation and hobbiesMany respiratory problems occur as a result of chronic exposure to inhalation irritants. Common occupational sources of inhalation irritants include mines, granaries, farms, lawn care companies, paint, plastics and rubber manufacture, and building remodeling. Hobbies associated with inhalation irritants include woodworking, metal finishing, furniture refinishing, painting, and ceramics. Daily fluids, height, and weight are more related to respiratory problems secondary to cardiac issues.

When assessing a patient's respiratory status, which of the following nonrespiratory data are most important for the nurse to obtain? A. Height and weight B. Neck circumference C. Occupation and hobbies D. Usual daily fluid intake

C. Occupation and hobbiesMany respiratory problems occur as a result of chronic exposure to inhalation irritants. Common occupational sources of inhalation irritants include mines, granaries, farms, lawn care companies, paint, plastics and rubber manufacture, and building remodeling. Hobbies associated with inhalation irritants include woodworking, metal finishing, furniture refinishing, painting, and ceramics. Daily fluids, height, and weight are more related to respiratory problems secondary to cardiac issues.

A client arrives in the emergency department (ED) with slurred speech and right-sided weakness. Which information is most important? A. Family history of stroke B. Changes in vision C. Onset of symptoms D. Severity of headache

C. Onset of symptoms

30. The nurse determines that a patient is experiencing common adverse effects from the inhaled corticosteroid beclomethasone (Beclovent) after noting which of the following? A. Adrenocortical dysfunction and hyperglycemia B. Elevation of blood glucose and calcium levels C. Oropharyngeal candidiasis and hoarseness D. Hypertension and pulmonary edema

C. Oropharyngeal candidiasis and hoarseness Oropharyngeal candidiasis and hoarseness are common adverse effects from the use of inhaled corticosteroids because the medication can lead to overgrowth of organisms and local irritation if the patient does not rinse the mouth following each dose.

The nurse is completing an admission inter for a client with Parkinson disease. Which question will provide addition information about manifestations the client is likely to experience? A. "Have you ever experienced and paralysis of your arms or legs?" B. " Do you have frequent blackout spells?" C. "Have you ever been 'frozen' in one spot, unable to move?" D. "Do you have headaches, especially ones with throbbing pain?"

C. Parkinson clients frequently experience difficulty in initiating, maintaining, and performing motor activities. They may even experience being rooted, unable to move. (A, B, D) Does not typically occur in Parkinson.

15. When reviewing the results of a 83-year-old patient's blood tests, which of the following findings would be of most concern to the nurse? A. Platelets of 150,000/µl B. Serum iron of 50 mcg/dl C. Partial thromboplastin time (PTT) of 60 seconds D. Erythrocyte sedimentation rate (ESR) of 35 mm in 1 hour

C. Partial thromboplastin time (PTT) of 60 seconds In aging, the partial thromboplastin time (PTT) is normally decreased, so an abnormally high PTT of 60 seconds is an indication that bleeding could readily occur. Platelets are unaffected by aging, and 150,000 is a normal count. Serum iron levels are decreased and the erythrocyte sedimentation rate (ESR) is significantly increased with aging, as are reflected in these values.

When reviewing the results of a 83-year-old patient's blood tests, which of the following findings would be of most concern to the nurse? A. Platelets of 150,000/µl B. Serum iron of 50 mcg/dl C. Partial thromboplastin time (PTT) of 60 seconds D. Erythrocyte sedimentation rate (ESR) of 35 mm in 1 hour

C. Partial thromboplastin time (PTT) of 60 seconds In aging, the partial thromboplastin time (PTT) is normally decreased, so an abnormally high PTT of 60 seconds is an indication that bleeding could readily occur. Platelets are unaffected by aging, and 150,000 is a normal count. Serum iron levels are decreased and the erythrocyte sedimentation rate (ESR) is significantly increased with aging, as are reflected in these values.

30. Which of the following conditions or factors in a 64-year-old patient diagnosed with head and neck cancer most likely contributed to this health problem? A. Patient's hobby is oil painting. B. Patient's father also had head and neck cancer. C. Patient uses chewing tobacco and drinks beer daily. D. Patient quit school at age 16 and has worked in a butcher shop for more than 40 years.

C. Patient uses chewing tobacco and drinks beer daily. Many environmental risk factors contribute to the development of head and neck cancer, although the actual cause is unknown. There does not appear to be a genetic predisposition to this type of cancer. The two most important risk factors are tobacco and alcohol use, especially in combination. Other risk factors include chewing tobacco, pipe smoking, marijuana use, voice abuse, chronic laryngitis, exposure to industrial chemicals or hardwood dust, and poor oral hygiene.

Which of the following conditions or factors in a 64-year-old patient diagnosed with head and neck cancer most likely contributed to this health problem? A. Patient's hobby is oil painting. B. Patient's father also had head and neck cancer. C. Patient uses chewing tobacco and drinks beer daily. D. Patient quit school at age 16 and has worked in a butcher shop for more than 40 years.

C. Patient uses chewing tobacco and drinks beer daily. Many environmental risk factors contribute to the development of head and neck cancer, although the actual cause is unknown. There does not appear to be a genetic predisposition to this type of cancer. The two most important risk factors are tobacco and alcohol use, especially in combination. Other risk factors include chewing tobacco, pipe smoking, marijuana use, voice abuse, chronic laryngitis, exposure to industrial chemicals or hardwood dust, and poor oral hygiene.

14. A 71-year-old patient is admitted with acute respiratory distress related to cor pulmonale. Which of the following nursing interventions is most appropriate during admission of this patient? A. Delay any physical assessment of the patient and review with the family the patient's history of respiratory problems. B. Perform a comprehensive health history with the patient to review prior respiratory problems. C. Perform a physical assessment of the respiratory system and ask specific questions related to this episode of respiratory distress. D. Complete a full physical examination to determine the effect of the respiratory distress on other body functions.

C. Perform a physical assessment of the respiratory system and ask specific questions related to this episode of respiratory distress.Because the patient is having respiratory difficulty, the nurse should ask specific questions about this episode and perform a physical assessment of this system. Further history taking and physical examination of other body systems can proceed once the patient's acute respiratory distress is being managed.

A 71-year-old patient is admitted with acute respiratory distress related to cor pulmonale. Which of the following nursing interventions is most appropriate during admission of this patient? A. Delay any physical assessment of the patient and review with the family the patient's history of respiratory problems. B. Perform a comprehensive health history with the patient to review prior respiratory problems. C. Perform a physical assessment of the respiratory system and ask specific questions related to this episode of respiratory distress. D. Complete a full physical examination to determine the effect of the respiratory distress on other body functions.

C. Perform a physical assessment of the respiratory system and ask specific questions related to this episode of respiratory distress.Because the patient is having respiratory difficulty, the nurse should ask specific questions about this episode and perform a physical assessment of this system. Further history taking and physical examination of other body systems can proceed once the patient's acute respiratory distress is being managed.

Following a devastating hurricane, a client is admitted for dehydration as the result of vomiting and diarrhea that occurred after ingesting contaminated water. The client expresses feelings of fear and anger about the destruction of homes, the loss of property due to the storm, and the looting that occurred following the storm. According to Maslow's hierarchy of need, what priority need should be addressed first? A. Seld-actualization B. Love and belonging C. Physiological needs D. Safety and security

C. Physiological needs

Before placing a client's dentures in the sink for cleansing, what action should the nurse take? A. Fill the sink with half-strength peroxide solution B. Loosen dried secretions with a toothbrush C. Place a washcloth on the bottom of the sink D. Hold the dentures in a stream of running water

C. Place a washcloth on the bottom of the sink

10. During discharge teaching for a 65-year-old patient with emphysema and pneumonia, which of the following vaccines should the nurse recommend the patient receive? A. S. aureus B. H. influenzae C. Pneumococcal D. Bacille Calmette-Guérin (BCG)

C. Pneumococcal The pneumococcal vaccine is important for patients with a history of heart or lung disease, recovering from a severe illness, age 65 or over, or living in a long-term care facility.

During discharge teaching for a 65-year-old patient with emphysema and pneumonia, which of the following vaccines should the nurse recommend the patient receive? A. S. aureus B. H. influenzae C. Pneumococcal D. Bacille Calmette-Guérin (BCG)

C. Pneumococcal The pneumococcal vaccine is important for patients with a history of heart or lung disease, recovering from a severe illness, age 65 or over, or living in a long-term care facility.

When checking a third-grader's height and weight, the school notes that these measurements have not changed in the last year. The child is currently taking daily vitamins, albuterol, and methylphenidate for attention deficit hyperactivity disorder (ADHD). Which intervention should the nurse implement? A. Report the finding to the parents B. Document findings in the child's school life C. Refer child to the family healthcare provider D. Encourage child to get more sleep

C. Refer child to the family healthcare provider

A female client, newly diagnosed with breast cancer, is scheduled for a mastectomy next week. During the preoperative assessment, she complains that her husband has become withdrawn and complains about her irritability, and frequently crying. How should the nurse respond? A. Explain that a positive attitude helps reduce preoperative complications B. Encourage the spouse to be more supportive at this difficult time C. Refer the couple to a counselor to help them with coping strategies D. Inquire if the couple has met with a minister to discuss their feelings

C. Refer the couple to a counselor to help them with coping strategies

What action should the nurse implement to reduce a client's risk for nosocomial infection? A. Apply a face shield or googles before irrigating an infected wound B. Wear sterile gloves to administer an intravenous medication through a saline loc C. Replace continuous tube feeding bag and tubing at least daily D. Obtain a prescription to irrigate a urinary catheter with sterile saline daily

C. Replace continuous tube feeding bag and tubing at least daily

A client receives a prescription for an intramuscular pain medication. The nurse uses the Z-track method to administer the injection Which rationale supports the nurse's use of this method? A. Prevents injury to the underlying bones, nerves, and blood vessels B. Minimize client's discomfort a the injection site C. Seals needle track to avoid medication leakage through the tissue D. Ensures medication reaches the intramuscular site

C. Seals needle track to avoid medication leakage through the tissue

A client receives a prescription for an intramuscular pain medication. The nurse uses the Z-track method to administer the injection. Which rationale supports the nurse's use of this method? A. Prevents injury to the underlying bones, nerves, and blood vessels B Minimizes client's discomfort at the injection site C. Seals needle track to avoid medication leakage through the tissue D. Ensures medication reaches the intramuscular site

C. Seals needle track to avoid medication leakage through the tissue

The nurse is caring for a critically ill client with cirrhosis of the liver who has a nasogastric tube draining bright red blood. The nurse notes that the client's serum hemoglobin and hematocrit are decreased. What additional change in lab data should the nurse expect? A. Increased serum albumin B. Decreased serum creatinine C. Decreased serum ammonia D. Increased liver function tests

C. The breakdown of glutamine in the intestine and the increased activity of colonic bacteria from the digestion of proteins increases the ammonia levels in the clients with advanced liver disease, so removal of blood, a protein source, from the intestines results in reduced ammonia. (A, B, D) will not be significantly impacted by the removal of blood.

What is the correct procedure for performing an ophthalmoscopic examination on a client's right eye? A. Instruct the client to look at the examiner's nose and not move his/her eyes during the exam. B. Set ophthalmoscope on the plus 2 to 3 lens and hold it in front of the examiner's right eye. C. From a distance of 8 to 12 inches and slightly to the side, shine the light into the client's pupil. D. For optimum visualization, keep the ophthalmoscope at least 3 inches for the client's eye

C. The client should focus on a distant object in order to promote pupil dilation. The ophthalmoscope should be set on the 0 lens to begin (creates no correction) and should be held in front of the examiner's left eye when examining the client's right eye and kept 1" from the client's eye for optimum visualization. (A, B, D) are incorrect procedures.

During report, the nurse learns that a client with tumor lysis syndrome is receiving an IV infusion containing insulin. Which assessment should the nurse complete first? A. Review the client's history for diabetes mellitus. B. Observe the extremity distal to the IV site. C. Monitor the client's serum potassium and blood glucose. D. Evaluate the client's oxygen saturation and breath sounds.

C. The client with tumor lysis syndrome may experience hyperkalemia, therefor it is important to monitor serum potassium and blood glucose levels. (A, B, D) are not as priority.

The community mental health nurse is planning to visit four clients with schizophrenia. Which client should the nurse see first? A. The young woman who believes she is to blame for her divorce B. The client with a history of substance abuse who is living in a halfway house C. The father who took his children from school because aliens were after them D. The client who needs to be evaluated for medication compliance.

C. The father who took his children from school because aliens were after them

12. If a nurse is assessing a patient whose recent blood gas determination indicated a pH of 7.32 and respirations are measured at 32 breaths/min, which of the following is the most appropriate nursing assessment? A. The rapid breathing is causing the low pH. B. The nurse should sedate the patient to slow down respirations. C. The rapid breathing is an attempt to compensate for the low pH. D. The nurse should give the patient a paper bag to breathe into to correct the low pH.

C. The rapid breathing is an attempt to compensate for the low pH. The respiratory system influences pH (acidity) through control of carbon dioxide exhalation. Thus, rapid breathing increases the pH. Breathing into a paper bag aids a patient who is hyperventilating; in respiratory alkalosis, it aids in lowering the pH. The use of sedation can cause respiratory depression and hypoventilation, resulting in an even lower pH.

If a nurse is assessing a patient whose recent blood gas determination indicated a pH of 7.32 and respirations are measured at 32 breaths/min, which of the following is the most appropriate nursing assessment? A. The rapid breathing is causing the low pH. B. The nurse should sedate the patient to slow down respirations. C. The rapid breathing is an attempt to compensate for the low pH. D. The nurse should give the patient a paper bag to breathe into to correct the low pH.

C. The rapid breathing is an attempt to compensate for the low pH. The respiratory system influences pH (acidity) through control of carbon dioxide exhalation. Thus, rapid breathing increases the pH. Breathing into a paper bag aids a patient who is hyperventilating; in respiratory alkalosis, it aids in lowering the pH. The use of sedation can cause respiratory depression and hypoventilation, resulting in an even lower pH.

The nurse observes a newly-employed unlicensed assistive personnel (UAP) taking an elderly client's blood pressure. The nurse says "You need to start over. The blood pressure reading you obtained was falsely high for this client." What is the most likely explanation for the erroneous reading? A. The UAP was standing above the sphygmomanometer while taking the reading B. The sphygmomanometer was two feet away from the UAP during the procedure C. The size of the cuff used was too small for this adult client's arm D. The client's arm was elevated above the level of the heart

C. The size of the cuff used was too small for this adult client's arm

A client who is at 36 weeks gestation is admitted with severe preeclampsia. After a 6 gram loading dose of magnesium sulfate is administered, an intravenous infusion of magnesium sulfate at a rate of 2 grams/hour is initiated. Which assessment finding warrants immediate intervention by the nurse? A. Blood pressure 162/94 B. Complaint of headache C. Urine output 20 ml/hour D. Nausea and vomiting

C. Urine output 20 ml/hour urinary output of less than 30 ml/hour indicates that the kidneys are being affected by the high level of magnesium, which is excreted through kidneys.

The nurse is preparing a client for a scheduled cesarean section. In which order should the nurse perform these actions? A. Prepping the site with iodophor (Betadine) B. Inserting an indwelling catheter C. Verifying the consent form is signed D. Performing a time-out procedure

C. Verifying the consent form is signed B. Inserting an indwelling catheter A. Prepping the site with iodophor (Betadine) D. Performing a time-out procedure

33. A patient with an acute pharyngitis is seen at the clinic with fever and severe throat pain that affects swallowing. On inspection the throat is reddened and edematous with patchy yellow exudates. The nurse anticipates that collaborative management will include A. treatment with antibiotics. B. treatment with antifungal agents. C. a throat culture or rapid strep antigen test. D. treatment with medication only if the pharyngitis does not resolve in 3 to 4 days.

C. a throat culture or rapid strep antigen test. Although inadequately treated β-hemolytic streptococcal infections may lead to rheumatic heart disease or glomerulonephritis, antibiotic treatment is not recommended until strep infections are definitely diagnosed with culture or antigen tests. The manifestations of viral and bacterial infections are similar, and appearance is not diagnostic except when candidiasis is present.

A patient with an acute pharyngitis is seen at the clinic with fever and severe throat pain that affects swallowing. On inspection the throat is reddened and edematous with patchy yellow exudates. The nurse anticipates that collaborative management will include A. treatment with antibiotics. B. treatment with antifungal agents. C. a throat culture or rapid strep antigen test. D. treatment with medication only if the pharyngitis does not resolve in 3 to 4 days.

C. a throat culture or rapid strep antigen test. Although inadequately treated β-hemolytic streptococcal infections may lead to rheumatic heart disease or glomerulonephritis, antibiotic treatment is not recommended until strep infections are definitely diagnosed with culture or antigen tests. The manifestations of viral and bacterial infections are similar, and appearance is not diagnostic except when candidiasis is present.

A client in the intensive care unit is being mechanically ventilated , has an indwelling urinary catheter in place, and is exhibiting signs of restlessness. Which action should the nurse take first? A. administer PRN dose of lorazepam B. auscultate bilateral breath sounds C. check urinary catheter for obstruction D. review the heart rhythm on cardiac monitor

C. check urinary catheter for obstruction

32. An excess of carbon dioxide in the blood causes an increased respiratory rate and volume because CO2 A. displaces oxygen on hemoglobin, leading to a decreased PaO2. B. causes an increase in the amount of hydrogen ions available in the body. C. combines with water to form carbonic acid, lowering the pH of cerebrospinal fluid. D. directly stimulates chemoreceptors in the medulla to increase respiratory rate and volume.

C. combines with water to form carbonic acid, lowering the pH of cerebrospinal fluid. A combination of excess CO2 and H2O results in carbonic acid, which lowers the pH of the cerebrospinal fluid and stimulates an increase in the respiratory rate. Peripheral chemoreceptors in the carotid and aortic bodies also respond to increases in PaCO2 to stimulate the respiratory center. Excess CO2 does not increase the amount of hydrogen ions available in the body but does combine with the hydrogen of water to form an acid.

An excess of carbon dioxide in the blood causes an increased respiratory rate and volume because CO2 A. displaces oxygen on hemoglobin, leading to a decreased PaO2. B. causes an increase in the amount of hydrogen ions available in the body. C. combines with water to form carbonic acid, lowering the pH of cerebrospinal fluid. D. directly stimulates chemoreceptors in the medulla to increase respiratory rate and volume.

C. combines with water to form carbonic acid, lowering the pH of cerebrospinal fluid. A combination of excess CO2 and H2O results in carbonic acid, which lowers the pH of the cerebrospinal fluid and stimulates an increase in the respiratory rate. Peripheral chemoreceptors in the carotid and aortic bodies also respond to increases in PaCO2 to stimulate the respiratory center. Excess CO2 does not increase the amount of hydrogen ions available in the body but does combine with the hydrogen of water to form an acid.

34. Following a supraglottic laryngectomy, the patient is taught how to use the supraglottic swallow to minimize the risk of aspiration. In teaching the patient about this technique, the nurse instructs the patient to A. perform Valsalva maneuver immediately after swallowing. B. breathe between each Valsalva maneuver and cough sequence. C. cough after swallowing to remove food from the top of the vocal cords. D. practice swallowing thin, watery fluids before attempting to swallow solid foods.

C. cough after swallowing to remove food from the top of the vocal cords. A supraglottic laryngectomy involves removal of the epiglottis and false vocal cords, and the removal of the epiglottis allows food to enter the trachea. Supraglottic swallowing requires performance of the Valsalva maneuver before placing food in the mouth and swallowing. The patient then coughs to remove food from the top of the vocal cords, swallows again, and then breathes after the food has been removed from the vocal cords.

Following a supraglottic laryngectomy, the patient is taught how to use the supraglottic swallow to minimize the risk of aspiration. In teaching the patient about this technique, the nurse instructs the patient to A. perform Valsalva maneuver immediately after swallowing. B. breathe between each Valsalva maneuver and cough sequence. C. cough after swallowing to remove food from the top of the vocal cords. D. practice swallowing thin, watery fluids before attempting to swallow solid foods.

C. cough after swallowing to remove food from the top of the vocal cords. A supraglottic laryngectomy involves removal of the epiglottis and false vocal cords, and the removal of the epiglottis allows food to enter the trachea. Supraglottic swallowing requires performance of the Valsalva maneuver before placing food in the mouth and swallowing. The patient then coughs to remove food from the top of the vocal cords, swallows again, and then breathes after the food has been removed from the vocal cords.

11. If a patient has pernicious anemia, the nurse should provide information regarding A. frequent bouts of dyspnea. B. risks relative to dehydration. C. deficiency of intrinsic factor. D. lack of any effective treatment for this condition.

C. deficiency of intrinsic factor. Pernicious anemia is a type of anemia caused by failure of absorption of vitamin B12 (cobalamin). The most common cause is lack of intrinsic factor, a glucoprotein produced by the parietal cells of the gastric lining.

If a patient has pernicious anemia, the nurse should provide information regarding A. frequent bouts of dyspnea. B. risks relative to dehydration. C. deficiency of intrinsic factor. D. lack of any effective treatment for this condition.

C. deficiency of intrinsic factor. Pernicious anemia is a type of anemia caused by failure of absorption of vitamin B12 (cobalamin). The most common cause is lack of intrinsic factor, a glucoprotein produced by the parietal cells of the gastric lining.

The nurse presenting information about fetal development to a group of parents who are attending a Lamaze birthing class. When discussing cephalocaudal fetal development, which information should the nurse provide? A. A set order in fetal development is expected B. growth normally occurs within one organ at a time C. development progresses from head to rump D. organ formation is directed from brain development

C. development progresses from head to rump fetal systems are developed in pre-determined order, best described in the direction of head to rump

18. A nurse establishes the presence of a tension pneumothorax when assessment findings reveal a(n) A. absence of lung sounds on the affected side. B. inability to auscultate tracheal breath sounds. C. deviation of the trachea toward the side opposite the pneumothorax. D. shift of the point of maximal impulse (PMI) to the left, with bounding pulses.

C. deviation of the trachea toward the side opposite the pneumothorax. Tension pneumothorax is caused by rapid accumulation of air in the pleural space, causing severely high intrapleural pressure. This results in collapse of the lung, and the mediastinum shifts toward the unaffected side, which is subsequently compressed.

A nurse establishes the presence of a tension pneumothorax when assessment findings reveal a(n) A. absence of lung sounds on the affected side. B. inability to auscultate tracheal breath sounds. C. deviation of the trachea toward the side opposite the pneumothorax. D. shift of the point of maximal impulse (PMI) to the left, with bounding pulses.

C. deviation of the trachea toward the side opposite the pneumothorax. Tension pneumothorax is caused by rapid accumulation of air in the pleural space, causing severely high intrapleural pressure. This results in collapse of the lung, and the mediastinum shifts toward the unaffected side, which is subsequently compressed.

37. The chronic inflammation of the bronchi characteristic of chronic obstructive pulmonary disease (COPD) results in A. collapse of small bronchioles on expiration. B. permanent, abnormal dilation of the bronchi. C. hyperplasia of mucus-secreting cells and bronchial edema. D. destruction of the elastic and muscular structures of the bronchial wall.

C. hyperplasia of mucus-secreting cells and bronchial edema. Chronic bronchitis is characterized by chronic inflammation of the bronchial lining, with edema and increased mucus production. Collapse of small bronchioles on expiration is common in emphysema, and abnormal dilation of the bronchi because of destruction of the elastic and muscular structures is characteristic of bronchiectasis.

The chronic inflammation of the bronchi characteristic of chronic obstructive pulmonary disease (COPD) results in A. collapse of small bronchioles on expiration. B. permanent, abnormal dilation of the bronchi. C. hyperplasia of mucus-secreting cells and bronchial edema. D. destruction of the elastic and muscular structures of the bronchial wall.

C. hyperplasia of mucus-secreting cells and bronchial edema. Chronic bronchitis is characterized by chronic inflammation of the bronchial lining, with edema and increased mucus production. Collapse of small bronchioles on expiration is common in emphysema, and abnormal dilation of the bronchi because of destruction of the elastic and muscular structures is characteristic of bronchiectasis.

8. A person complains of fatigue and malaise and has a slight temperature elevation for 2 days before symptoms of influenza (fever, chest congestion, and productive cough) become noticeable. During the time immediately before the illness is diagnosed, the patient A. could avoid contracting the disease if treatment is begun with antibiotics. B. is unable to spread the disease because it is still in the incubation period. C. is in the prodromal stage and is highly contagious and able to spread the disease. D. has a nosocomial infection, which affects approximately two million individuals a year.

C. is in the prodromal stage and is highly contagious and able to spread the disease. The prodromal stage is a short period of time (hours to several days) immediately preceding the onset of an illness during which the patient is very contagious. Antibiotics are not effective against viral illnesses. The incubation period is the time from entry of the organism to the onset of symptoms and, in some viral illnesses, may be contagious. Nosocomial infections are those acquired in a hospital, and this scenario does not suggest the source of the infection.

A person complains of fatigue and malaise and has a slight temperature elevation for 2 days before symptoms of influenza (fever, chest congestion, and productive cough) become noticeable. During the time immediately before the illness is diagnosed, the patient A. could avoid contracting the disease if treatment is begun with antibiotics. B. is unable to spread the disease because it is still in the incubation period. C. is in the prodromal stage and is highly contagious and able to spread the disease. D. has a nosocomial infection, which affects approximately two million individuals a year.

C. is in the prodromal stage and is highly contagious and able to spread the disease. The prodromal stage is a short period of time (hours to several days) immediately preceding the onset of an illness during which the patient is very contagious. Antibiotics are not effective against viral illnesses. The incubation period is the time from entry of the organism to the onset of symptoms and, in some viral illnesses, may be contagious. Nosocomial infections are those acquired in a hospital, and this scenario does not suggest the source of the infection.

7. During care of a patient with multiple myeloma, an important nursing intervention is A. limiting activity to prevent pathologic fractures. B. assessing for changes in size and characteristics of lymph nodes. C. maintaining a fluid intake of 3 to 4 L/day to dilute calcium load. D. administering narcotic analgesics continuously to control bone pain.

C. maintaining a fluid intake of 3 to 4 L/day to dilute calcium load. Adequate hydration must be maintained to minimize problems from hypercalcemia. The goal of a urinary output of 1.5 to 2 L/day requires an intake of 3 to 4 L/day.

During care of a patient with multiple myeloma, an important nursing intervention is A. limiting activity to prevent pathologic fractures. B. assessing for changes in size and characteristics of lymph nodes. C. maintaining a fluid intake of 3 to 4 L/day to dilute calcium load. D. administering narcotic analgesics continuously to control bone pain.

C. maintaining a fluid intake of 3 to 4 L/day to dilute calcium load. Adequate hydration must be maintained to minimize problems from hypercalcemia. The goal of a urinary output of 1.5 to 2 L/day requires an intake of 3 to 4 L/day.

A client who was splashed with a chemical has both eyes covered with bandages. When assisting the client with eating, which intervention should the nurse instruct the unlicensed assistive personal (UAP) to implement? A. feed the client the entire meal B. provide with only finer food C. orient the client to location of food on the plate D. ask family member to visit during meal time to assist with feeding

C. orient the client to location of food on the plate

5. Absorption of vitamin B12 may be decreased in older adults because of decreased A. intestinal motility. B. production of bile by the liver. C. production of intrinsic factor by the stomach. D. synthesis of cobalamin (vitamin B12) by intestinal bacteria.

C. production of intrinsic factor by the stomach. Older persons are at risk for deficiency of cobalamin (pernicious anemia) because of a naturally occurring reduction of the intrinsic factor by the stomach mucosa. Absorption of cobalamin relies on intrinsic factor. Both must be present for absorption. Megaloblastic anemia is related to folate dysfunction. Intestinal motility (peristalsis) is the motion that moves food down the GI tract. The rhythmic contractions of muscles cause wave-like motions. Lack of peristalsis is called "paralytic ileus." Bile is produced in the liver, is stored and concentrated in the gallbladder, and is released into the duodenum when fat is eaten. Bile emulsifies fats and prepares them for enzyme digestion in order for the nutrient to be absorbed into lymph and eventually into blood vessels to the liver. Vitamin K (the blood-clotting vitamin) is synthesized by intestinal bacteria.

Absorption of vitamin B12 may be decreased in older adults because of decreased A. intestinal motility. B. production of bile by the liver. C. production of intrinsic factor by the stomach. D. synthesis of cobalamin (vitamin B12) by intestinal bacteria.

C. production of intrinsic factor by the stomach. Older persons are at risk for deficiency of cobalamin (pernicious anemia) because of a naturally occurring reduction of the intrinsic factor by the stomach mucosa. Absorption of cobalamin relies on intrinsic factor. Both must be present for absorption. Megaloblastic anemia is related to folate dysfunction. Intestinal motility (peristalsis) is the motion that moves food down the GI tract. The rhythmic contractions of muscles cause wave-like motions. Lack of peristalsis is called "paralytic ileus." Bile is produced in the liver, is stored and concentrated in the gallbladder, and is released into the duodenum when fat is eaten. Bile emulsifies fats and prepares them for enzyme digestion in order for the nutrient to be absorbed into lymph and eventually into blood vessels to the liver. Vitamin K (the blood-clotting vitamin) is synthesized by intestinal bacteria.

9. In older adults, infection after exposure to respiratory illness is most likely to A. result in similar rates of infection as in the younger adult. B. be easily prevented with the use of antibiotics after being exposed. C. result in serious lower respiratory infection related to weakened respiratory muscles and fewer cilia. D. be less serious because the older adult has less contact with younger children who are most likely to carry serious infections.

C. result in serious lower respiratory infection related to weakened respiratory muscles and fewer cilia. Changes in the older adult respiratory system make older adults more susceptible to infections that can be very serious and life threatening. Use of antibiotics to "prevent" lung infections is not recommended and is ineffective for viral infections.

In older adults, infection after exposure to respiratory illness is most likely to A. result in similar rates of infection as in the younger adult. B. be easily prevented with the use of antibiotics after being exposed. C. result in serious lower respiratory infection related to weakened respiratory muscles and fewer cilia. D. be less serious because the older adult has less contact with younger children who are most likely to carry serious infections.

C. result in serious lower respiratory infection related to weakened respiratory muscles and fewer cilia. Changes in the older adult respiratory system make older adults more susceptible to infections that can be very serious and life threatening. Use of antibiotics to "prevent" lung infections is not recommended and is ineffective for viral infections.

38. In teaching the patient with COPD about the need for physical exercise, the nurse informs the patient that A. all patients with COPD should be able to increase walking gradually up to 20 min/day. B. a bronchodilator inhaler should be used to relieve exercise-induced dyspnea immediately after exercise. C. shortness of breath is expected during exercise but should return to baseline within 5 minutes after the exercise. D. monitoring the heart rate before and after exercise is the best way to determine how much exercise can be tolerated.

C. shortness of breath is expected during exercise but should return to baseline within 5 minutes after the exercise.Shortness of breath usually increases during exercise, but the activity is not being overdone if breathing returns to baseline within 5 minutes after stopping. Bronchodilators can be administered 10 minutes before exercise but should not be administered for at least 5 minutes after activity to allow recovery. Patients are encouraged to walk 15 to 20 minutes a day with gradual increases, but actual patterns will depend on patient tolerance. Dyspnea most frequently limits exercise and is a better indication of exercise tolerance than is heart rate in the patient with COPD.

In teaching the patient with COPD about the need for physical exercise, the nurse informs the patient that A. all patients with COPD should be able to increase walking gradually up to 20 min/day. B. a bronchodilator inhaler should be used to relieve exercise-induced dyspnea immediately after exercise. C. shortness of breath is expected during exercise but should return to baseline within 5 minutes after the exercise. D. monitoring the heart rate before and after exercise is the best way to determine how much exercise can be tolerated.

C. shortness of breath is expected during exercise but should return to baseline within 5 minutes after the exercise.Shortness of breath usually increases during exercise, but the activity is not being overdone if breathing returns to baseline within 5 minutes after stopping. Bronchodilators can be administered 10 minutes before exercise but should not be administered for at least 5 minutes after activity to allow recovery. Patients are encouraged to walk 15 to 20 minutes a day with gradual increases, but actual patterns will depend on patient tolerance. Dyspnea most frequently limits exercise and is a better indication of exercise tolerance than is heart rate in the patient with COPD.

16. If a patient with blood type O Rh- is given AB Rh- blood, the nurse would expect A. the patient's Rh factor to react with the RBCs of the donor blood. B. no adverse reaction because the patient has no antibodies against the donor blood. C. the anti-A and anti-B antibodies in the patient's blood to hemolyze the donor blood. D. the anti-A and anti-B antibodies in the donor blood to hemolyze the patient's blood.

C. the anti-A and anti-B antibodies in the patient's blood to hemolyze the donor blood. A patient with O Rh+ blood has no A or B antigens on the red cell but does have anti-A and anti-B antibodies in the blood and has an Rh antigen. AB Rh- blood has both A and B antigens on the red cell but no Rh antigen and no anti-A or anti-B antibodies. If the AB Rh- blood is given to the patient with O Rh+ blood, the antibodies in the patient's blood will react with the antigens in the donor blood, causing hemolysis of the donor cells. There will be no Rh reaction because the donor blood has no Rh antigen.

If a patient with blood type O Rh- is given AB Rh- blood, the nurse would expect A. the patient's Rh factor to react with the RBCs of the donor blood. B. no adverse reaction because the patient has no antibodies against the donor blood. C. the anti-A and anti-B antibodies in the patient's blood to hemolyze the donor blood. D. the anti-A and anti-B antibodies in the donor blood to hemolyze the patient's blood.

C. the anti-A and anti-B antibodies in the patient's blood to hemolyze the donor blood. A patient with O Rh+ blood has no A or B antigens on the red cell but does have anti-A and anti-B antibodies in the blood and has an Rh antigen. AB Rh- blood has both A and B antigens on the red cell but no Rh antigen and no anti-A or anti-B antibodies. If the AB Rh- blood is given to the patient with O Rh+ blood, the antibodies in the patient's blood will react with the antigens in the donor blood, causing hemolysis of the donor cells. There will be no Rh reaction because the donor blood has no Rh antigen.

Upon entering the room of a patient who has just returned from surgery for total laryngectomy and radical neck dissection, a nurse should recognize a need for intervention when finding A. a gastrostomy tube that is clamped. B. the patient coughing blood-tinged secretions from the tracheostomy. C. the patient positioned in a lateral position with the head of the bed flat. D. 200 ml of serosanguineous drainage in the patient's portable drainage device.

C. the patient positioned in a lateral position with the head of the bed flat. After total laryngectomy and radical neck dissection, a patient should be placed in a semi-Fowler's position to decrease edema and limit tension on the suture line.

Continuous Bladder Irrigation

CBI

A patient comes to the ER and the doctor discovers the patient is having an MI...what lab would the nurse see elevated?

CKMB

Blue Bloater

COPD

This lab would indicate possible tumor lysis syndrome being present

Calcium of 5

Characteristics of type 1 DM

Can become hyper glycemic and ketosis prone relatively easily. Can develop DKA from under management of glucose

Diagnostic testing for Crohn's disease

Capsule endoscopy has shown greater sensitivity than radiography

Delivery of synchronized electrical shocks to the myocardium

Cardioversion

What am I describing here: Loss of short-term memory Facial tics & grimaces Constant writhing movements

Chorea

Causes of cirrhosis

Chronic alcohol ingestion viral hepatitis exposure to heptotoxins infections congenital abnormalities chronic severe right-sided heart failure

Signs and symptoms of chronic pancreatitis

Continuous burning or gnawing abdominal pain recurrent attacks of severe upper abdominal and back pain ascites diarrhea, steatorrhea weight loss Jaundice signs and symptoms of diabetes mellitus

This type of catheter is used to restore urinary drainage in patients that are suspected to have BPH

Coudé

Characteristic of adolescence

Creates a new definition of self and roles with others

Inflammatory bowel diseases

Crohn's disease and ulcerative colitis

When the adrenal gland is working in overdrive, it causes

Cushing Syndrome

The most common form of mastalgia (breast pain)

Cyclic

A client reports unprotected sexual intercourse one week ago and is worried about HIV exposure. An initial HIV antibody screen (ELISA) is obtained. The nurse teaches the client that seroconversion to HIV positive relies on antibody production by B lymphocytes after exposure to the virus. When should the nurse recommend the client return for repeat blood testing? A) 6 to 18 months. B) 1 to 12 months. C) 1 to 18 weeks. D) 6 to 12 weeks.

D) 6 to 12 weeks. Although the HIV antigen is detectable approximately 2 weeks after exposure, seroconversion to HIV positive may take up to 6 to 12 weeks (D) after exposure, so the client should return to repeat the serum screen for the presence of HIV antibodies during that time frame. (A) will delay treatment if the client tests positive. (B and C) may provide inaccurate results because the time frame maybe too early to reevaluate the client.

A client who is HIV positive asks the nurse, "How will I know when I have AIDS?" Which response is best for the nurse to provide? A) Diagnosis of AIDS is made when you have 2 positive ELISA test results. B) Diagnosis is made when both the ELISA and the Western Blot tests are positive. C) I can tell that you are afraid of being diagnosed with AIDS. Would you like for me to call your minister? D) AIDS is diagnosed when a specific opportunistic infection is found in an otherwise healthy individual.

D) AIDS is diagnosed when a specific opportunistic infection is found in an otherwise healthy individual. AIDS is diagnosed when one of several processes defined by the CDC is present in an individual who is not otherwise immunosuppressed (D) (PCP, candidacies, crytpococcus, cryptosporidiosis, Kaposi's sarcoma, CNS lymphomas). (A and B) identify the presence of HIV, indicating a high probability that in time the individual will develop AIDS, but do not necessarily denote the presence of AIDS. (C) is telling the client how he/she feels (afraid) and is dismissing the situation to the minister. This client is asking a question and specific medical information needs to be provided.

A client with diabetes mellitus is experiencing polyphagia. Which outcome statement is the priority for this client? A) Fluid and electrolyte balance. B) Prevention of water toxicity. C) Reduced glucose in the urine. D) Adequate cellular nourishment.

D) Adequate cellular nourishment. Diabetes mellitus Type 1 is characterized by hyperglycemia that precipitates glucosuria and polyuria (frequent urination), polydipsia (excessive thirst), and polyphagia (excessive hunger). Polyphagia is a consequence of cellular malnourishment when insulin deficiency prevents utilization of glucose for energy, so the outcome statement should include stabilization of adequate cellular nutrition (D). (A, B, and C) relate to subsequent osmolar fluid shifts related to glucosuria, polyuria, and polydipsia.

Two days postoperative, a male client reports aching pain in his left leg. The nurse assesses redness and warmth on the lower left calf. What intervention should be most helpful to this client? A) Apply sequential compression devices (SCDs) bilaterally. B) Assess for a positive Homan's sign in each leg. C) Pad all bony prominences on the affected leg. D) Advise the client to remain in bed with the leg elevated.

D) Advise the client to remain in bed with the leg elevated. The client is exhibiting symptoms of deep vein thrombosis (DVT), a complication of immobility. The initial care includes bedrest and elevation of the extremity (D). SCDs are used to prevent thrombophlebitis, not for treatment, when a clot might be dislodged (A). Once a client has thrombophlebitis, (B) is contraindicated because of the possibility of dislodging a clot. (C) is indicated to prevent pressure ulcers, but is not a therapeutic action for thrombophlebitis.

The nurse is assisting a client out of bed for the first time after surgery. What action should the nurse do first? A) Place a chair at a right angle to the bedside. B) Encourage deep breathing prior to standing. C) Help the client to sit and dangle legs on the side of the bed. D) Allow the client to sit with the bed in a high Fowler's position.

D) Allow the client to sit with the bed in a high Fowler's position. The first step is to raise the head of the bed to a high Fowler's position (D), which allow venous return to compensate from lying flat and vasodilating effects of perioperative drugs. (A, B, and C) are implemented after (D).

A client taking furosemide (Lasix), reports difficulty sleeping. What question is important for the nurse to ask the client? A) What dose of medication are you taking? B) Are you eating foods rich in potassium? C) Have you lost weight recently? D) At what time do you take your medication?

D) At what time do you take your medication? The nurse needs to first determine at what time of day the client takes the Lasix (D). Because of the diuretic effect of Lasix, clients should take the medication in the morning to prevent nocturia. The actual dose of medication (A) is of less importance than the time taken. (B) is not related to the insomnia. (C) is valuable information about the effect of the diuretic, but is not likely to be related to insomnia.

What types of medications should the nurse expect to administer to a client during an acute respiratory distress episode? A) Vasodilators and hormones. B) Analgesics and sedatives. C) Anticoagulants and expectorants. D) Bronchodilators and steroids.

D) Bronchodilators and steroids. Besides supplemental oxygen, the ARDS client needs medications to widen air passages, increase air space, and reduce alveolar membrane inflammation, i.e., bronchodilators and steroids (D). (A) would not help the condition. (B) would further depress the client and compromise the ability to breathe. Anticoagulants would be contraindicated since clotting of the blood is not yet a problem, and expectorants are not appropriate for this critically ill client (C).

A female client is brought to the clinic by her daughter for a flu shot. She has lost significant weight since the last visit. She has poor personal hygiene and inadequate clothing for the weather. The client states that she lives alone and denies problems or concerns. What action should the nurse implement? A) Notify social services immediately of suspected elderly abuse. B) Discuss the need for mental health counseling with the daughter. C) Explain to the client that she needs to take better care of herself. D) Collect further data to determine whether self-neglect is occurring.

D) Collect further data to determine whether self-neglect is occurring. Changes in weight and hygiene may be indicators of self-neglect or neglect by family members. Further assessment is needed (D) before notifying social services (A) or discussing a need for counseling (B). Until further information is obtained, explanations about the client's needs are premature (C).

An elderly client is admitted with a diagnosis of bacterial pneumonia. The nurse's assessment of the client is most likely to reveal which sign/symptom? A) Leukocytosis and febrile. B) Polycythemia and crackles. C) Pharyngitis and sputum production. D) Confusion and tachycardia.

D) Confusion and tachycardia. The onset of pneumonia in the elderly may be signaled by general deterioration, confusion, increased heart rate or increased respiratory rate (D). (A, B, and C) are often absent in the elderly with bacterial pneumonia.

The nurse assesses a client with advanced cirrhosis of the liver for signs of hepatic encephalopathy. Which finding should the nurse consider an indication of progressive hepatic encephalopathy? A) An increase in abdominal girth. B) Hypertension and a bounding pulse. C) Decreased bowel sounds. D) Difficulty in handwriting.

D) Difficulty in handwriting. A daily record in handwriting may provide evidence of progression or reversal of hepatic encephalopathy leading to coma (D). (A) is a sign of ascites. (B) are not seen with hepatic encephalopathy. (C) does not indicate an increase in serum ammonia level which is the primary cause of hepatic encephalopathy.

A client experiencing uncontrolled atrial fibrillation is admitted to the telemetry unit. What initial medication should the nurse anticipate administering to the client? A) Xylocaine (Lidocaine). B) Procainamide (Pronestyl). C) Phenytoin (Dilantin). D) Digoxin (Lanoxin).

D) Digoxin (Lanoxin). Digoxin (Lanoxin) (D) is administered for uncontrolled, symptomatic atrial fibrillation resulting in a decreased cardiac output. Digoxin slows the rate of conduction by prolonging the refractory period of the AV node, thus slowing the ventricular response, decreasing the heart rate, and effecting cardiac output. (A, B, and C) are not indicated in the initial treatment of uncontrolled atrial fibrillation.

The nurse is teaching a female client who uses a contraceptive diaphragm about reducing the risk for toxic shock syndrome (TSS). Which information should the nurse include? (Select all that apply.) A) Remove the diaphragm immediately after intercourse. B) Wash the diaphragm with an alcohol solution. C) Use the diaphragm to prevent conception during the menstrual cycle. D) Do not leave the diaphragm in place longer than 8 hours after intercourse. E) Contact a healthcare provider a sudden onset of fever grater than 101º F appears. F) Replace the old diaphragm every 3 months.

D) Do not leave the diaphragm in place longer than 8 hours after intercourse. E) Contact a healthcare provider a sudden onset of fever grater than 101º F appears. Correct selections are (D and E). The diaphragm needs to remain against the cervix for 6 to 8 hours to prevent pregnancy but should not remain for longer than 8 hours (D) to avoid the risk of TSS. If a sudden fever occurs, the client should notify the healthcare provider (E). (A) increases the risk of pregnancy, and (B) can reduce the integrity of the barrier contraceptive but neither prevents the risk of TSS. The diaphragm should not be used during menses (C) because it obstructs the menstrual flow and is not indicated because conception does not occur during this time. (F) is not necessary.

When providing discharge teaching for a client with osteoporosis, the nurse should reinforce which home care activity? A) A diet low in phosphates. B) Skin inspection for bruising. C) Exercise regimen, including swimming. D) Elimination of hazards to home safety.

D) Elimination of hazards to home safety. Discussion about fall prevention strategies is imperative for the discharged client with osteoporosis so that advice about safety measures can be given (D). A low phosphorus diet is not recommended in the treatment of osteoporosis (A). Bruising (B) is not a related symptom to osteoporosis. Weight-bearing exercise is most beneficial for clients with osteoporosis. Swimming (C) is not a weight-bearing exercise.

A client with a completed ischemic stroke has a blood pressure of 180/90 mm Hg. Which action should the nurse implement? A) Position the head of the bed (HOB) flat. B) Withhold intravenous fluids. C) Administer a bolus of IV fluids. D) Give an antihypertensive medications.

D) Give an antihypertensive medications. Most ischemic strokes occur during sleep when baseline blood pressure declines or blood viscosity increases due to minimal fluid intake. Completed strokes usually produce neurologic deficits within an hour, the client's current elevated blood pressure requires antihypertensive medication (D). Positioning the HOB flat (A) decreases venous drainage and contributes to cerebral edema post stroke. Increased blood viscosity during sleep may be related to reduced fluids, so (B) is not indicated. Increasing the vascular fluid volume increases the blood pressure, so (C) is not indicated.

The nurse is receiving report from surgery about a client with a penrose drain who is to be admitted to the postoperative unit. Before choosing a room for this client, which information is most important for the nurse to obtain? A) If suctioning will be needed for drainage of the wound. B) If the family would prefer a private or semi-private room. C) If the client also has a Hemovac® in place. D) If the client's wound is infected.

D) If the client's wound is infected. Penrose drains provide a sinus tract or opening and are often used to provide drainage of an abscess. The fact that the client has a penrose drain should alert the nurse to the possibility that the client is infected. To avoid contamination of another postoperative client, it is most important to place an infected client in a private room (D). A penrose drain does not require (A). Although (B) is information that should be considered, it does not have the priority of (D). (C) is used to drain fluid from a dead space and is not important in choosing a room.

A client has a staging procedure for cancer of the breast and ask the nurse which type of breast cancer has the poorest prognosis. Which information should the nurse offer the client? A) Stage II. B) Invasive infiltrating ductal carcinoma. C) T1N0M0. D) Inflammatory with peau d'orange.

D) Inflammatory with peau d'orange. Inflammatory breast cancer, which has a thickened appearance like an orange peel (peau d'orange), is the most aggressive form of breast malignancies (D). Staging classifies cancer by the extension or spread of the disease, and (A) indicates limited local spread. (B) indicates cancer cells have spread from the ducts into the surrounding breast tissue only. TNM classification is used to indicate the extent of the disease process according to tumor size, regional spread lymph nodes involvement, and metastasis, and (C) indicates early cancer with small in situ involvement, no lymph node involvement, and no distant metastases.

Despite several eye surgeries, a 78-year-old client who lives alone has persistent vision problems. The visiting nurse is discussing painting the house with the client. The nurse suggests that the edge of the steps should be painted which color? A) Black. B) White. C) Light green. D) Medium yellow.

D) Medium yellow. Yellow is the easiest for a person with failing vision to see (D). (A) will be almost impossible to see at night because the shadows of the steps will be too difficult to determine, and would pose a safety hazard. (B) is very hard to see with a glare from the sun and it could hurt the eyes in the daytime to look at them. (C) is a pastel color and is difficult for elderly clients to see.

How should the nurse position the electrodes for modified chest lead one (MCL I) telemetry monitoring? A) Positive polarity right shoulder, negative polarity left shoulder, ground left chest nipple line. B) Positive polarity left shoulder, negative polarity right chest nipple line, ground left chest nipple line. C) Positive polarity right chest nipple line, negative polarity left chest nipple line, ground left shoulder. D) Negative polarity left shoulder, positive polarity right chest nipple line, ground left chest nipple line.

D) Negative polarity left shoulder, positive polarity right chest nipple line, ground left chest nipple line. In MCL I monitoring, the positive electrode is placed on the client's mid-chest to the right of the sternum, and the negative electrode is placed on the upper left part of the chest (D). The ground may be placed anywhere, but is usually placed on the lower left portion of the chest. (A, B, and C) describe incorrect placement of electrodes for telemetry monitoring.

What instruction should the nurse give a client who is diagnosed with fibrocystic changes of the breast? A) Observe cyst size fluctuations as a sign of malignancy. B) Use estrogen supplements to reduce breast discomfort. C) Notify the healthcare provider if whitish nipple discharge occurs. D) Perform a breast self-exam (BSE) procedure monthly.

D) Perform a breast self-exam (BSE) procedure monthly. Fibrocystic changes in the breast are related to excess fibrous tissue, proliferation of mammary ducts and cyst formation that cause edema and nerve irritation. These changes obscure typical diagnostic tests, such as mammography, due to an increased breast density. Women with fibrocystic breasts should be instructed to carefully perform monthly BSE (D) and consider changes in any previous "lumpiness." Fibrocystic disease does not increase the risk of breast cancer (A). Cyst size fluctuates with the menstrual cycle, and typically lessens after menopause, and responds with a heightened sensitivity to circulating estrogen (B), which is not indicated. Nipple discharge associated with fibrocystic breasts is often milky or watery-milky and is an expected finding (C).

During a health fair, a 72-year-old male client tells the nurse that he is experiencing shortness of breath. Auscultation reveals crackles and wheezing in both lungs. Suspecting that the client might have chronic bronchitis, which classic symptom should the nurse expect this client to have? A) Racing pulse with exertion. B) Clubbing of the fingers. C) An increased chest diameter. D) Productive cough with grayish-white sputum.

D) Productive cough with grayish-white sputum. Chronic bronchitis, one of the diseases comprising the diagnosis of COPD, is characterized by a productive cough with grayish-white sputum (D), which usually occurs in the morning and is often ignored by smokers. (A) is not related to chronic bronchitis; however, it is indicative of other problems such as ventricular tachycardia and should be explored. (B and C) are symptoms of emphysema and are not consistent with the other symptoms. (C) is usually referred to as a "barrel chest."

The healthcare provider prescribes aluminum and magnesium hydroxide (Maalox), 1 tablet PO PRN, for a client with chronic renal failure who is complaining of indigestion. What intervention should the nurse implement? A) Administer 30 minutes before eating. B) Evaluate the effectiveness 1 hour after administration. C) Instruct the client to swallow the tablet whole. D) Question the healthcare provider's prescription.

D) Question the healthcare provider's prescription. Magnesium agents are not usually used for clients with renal failure due to the risk of hypermagnesemia, so this prescription should be questioned by the nurse (D). (A, B, and C) are not recommended nursing actions for the administration of aluminum and magnesium hydroxide (Maalox).

A client with gastroesophageal reflux disease (GERD) has been experiencing severe reflux during sleep. Which recommendation by the nurse is most effective to assist the client? A) Losing weight. B) Decreasing caffeine intake. C) Avoiding large meals. D) Raising the head of the bed on blocks.

D) Raising the head of the bed on blocks. Raising the head of the bed on blocks (D) (reverse Trendelenburg position) to reduce reflux and subsequent aspiration is the most effective recommendation for a client experiencing severe gastroesophageal reflux during sleep. (A, B and C) may be effective recommendations but raising the head of the bed is more effective for relief during sleep.

A client with cirrhosis develops increasing pedal edema and ascites. What dietary modification is most important for the nurse to teach this client? A) Avoid high carbohydrate foods. B) Decrease intake of fat soluble vitamins. C) Decrease caloric intake. D) Restrict salt and fluid intake.

D) Restrict salt and fluid intake. Salt and fluid restrictions are the first dietary modifications for a client who is retaining fluid as manifested by edema and ascites (D). (A, B, and C) will not impact fluid retention.

In preparing a discharge plan for a 22-year-old male client diagnosed with Buerger's disease (thromboangiitis obliterans), which referral is most important? A) Genetic counseling. B) Twelve-step recovery program. C) Clinical nutritionist. D) Smoking cessation program.

D) Smoking cessation program. Buerger's disease is strongly related to smoking. The most effective means of controlling symptoms and disease progression is through smoking cessation (D). The cause of Buerger's disease is unknown; a genetic predisposition is possible, but (A) will not be of value. The client with Buerger's disease does not need referral to a 12-step program any more than the general population (B). Diet is not a significant factor in the disease, and general healthy diet guidelines can be provided by the nurse (C).

A 57-year-old male client is scheduled to have a stress-thallium test the following morning and is NPO after midnight. At 0130, he is agitated because he cannot eat and is demanding food. Which response is best for the nurse to provide to this client? A) I'm sorry sir, you have a prescription for nothing by mouth from midnight tonight. B) I will let you have one cracker, but that is all you can have for the rest of tonight. C) What did the healthcare provider tell you about the test you are having tomorrow? D) The test you are having tomorrow requires that you have nothing by mouth tonight.

D) The test you are having tomorrow requires that you have nothing by mouth tonight. (D) is the most therapeutic statement because the nurse is responding to the client's question. (A) is not an explanation and the nurse should teach the client why eating is prohibited after midnight, rather than enforcing this requirement without an explanation for it. (B) may result in an inaccurate test result, or may cause the test to be cancelled, which could also delay diagnosis and treatment. (C) defers the responsibility for answering the client's question to the healthcare provider, when the nurse could address the situation through client teaching.

The nurse is planning to initiate a socialization group for older residents of a long-term facility. Which information is most useful to the nurse when planning activities for the group? A) The length of time each group member has resided at the nursing home. B) A brief description of each resident's family life. C) The age of each group member. D) The usual activity patterns of each member of the group.

D) The usual activity patterns of each member of the group. An older person's level of activity (D) is a determining factor in adjustment to aging as described by the Activity Theory of Aging. All information described in the options might be useful to the nurse, but the most useful information initially would be an assessment of each individual's adjustment to the aging process.

Dysrhythmias are a concern for any client. However, the presence of a dysrhythmia is more serious in an elderly person because A) elderly persons usually live alone and cannot summon help when symptoms appear. B) elderly persons are more likely to eat high-fat diets which make them susceptible to heart disease. C) cardiac symptoms, such as confusion, are more difficult to recognize in the elderly. D) elderly persons are intolerant of decreased cardiac output which may result in dizziness and falls.

D) elderly persons are intolerant of decreased cardiac output which may result in dizziness and falls. Cardiac output is decreased with aging (D). Because of loss of contractility and elasticity, blood flow is decreased and tachycardia is poorly tolerated. Therefore, if an elderly person experiences dysrhythmia (tachycardia or bradycardia), further compromising their cardiac output, they are more likely to experience syncope, falls, transient ischemic attacks, and possibly dementia. Most elderly persons do not eat high-fat diets (B) and most are not confused (C). Although many elderly persons do live alone, inability to summon help (A) cannot be assumed.

A 77-year-old female client is admitted to the hospital. She is confused, has no appetite, is nauseated and vomiting, and is complaining of a headache. Her pulse rate is 43 beats per minute. Which question is a priority for the nurse to ask this client or her family on admission? "Does the client A) have her own teeth or dentures?" B) take aspirin and if so, how much?" C) take nitroglycerin?" D) take digitalis?"

D) take digitalis?" Elderly persons are particularly susceptible to digitalis intoxication (D) which manifests itself in such symptoms as anorexia, nausea, vomiting, diarrhea, headache, and fatigue. Although it is important to obtain a complete medication history (B and C), the symptoms described are classic for digitalis toxicity, and assessment of this problem should be made promptly. (A) is irrelevant.

11. The nurse evaluates that discharge teaching for a patient hospitalized with pneumonia has been most effective when the patient states which of the following measures to prevent a relapse? A. "I will increase my food intake to 2400 calories a day to keep my immune system well." B. "I must use home oxygen therapy for 3 months and then will have a chest x-ray to reevaluate." C. "I will seek immediate medical treatment for any upper respiratory infections." D. "I should continue to do deep-breathing and coughing exercises for at least 6 weeks."

D. "I should continue to do deep-breathing and coughing exercises for at least 6 weeks." It is important for the patient to continue with coughing and deep breathing exercises for 6 to 8 weeks until all of the infection has cleared from the lungs. A patient should seek medical treatment for upper respiratory infections that persist for more than 7 days. Increased fluid intake, not caloric intake, is required to liquefy secretions. Home O2 is not a requirement unless the patient's oxygenation saturation is below normal.

The nurse evaluates that discharge teaching for a patient hospitalized with pneumonia has been most effective when the patient states which of the following measures to prevent a relapse? A. "I will increase my food intake to 2400 calories a day to keep my immune system well." B. "I must use home oxygen therapy for 3 months and then will have a chest x-ray to reevaluate." C. "I will seek immediate medical treatment for any upper respiratory infections." D. "I should continue to do deep-breathing and coughing exercises for at least 6 weeks."

D. "I should continue to do deep-breathing and coughing exercises for at least 6 weeks." It is important for the patient to continue with coughing and deep breathing exercises for 6 to 8 weeks until all of the infection has cleared from the lungs. A patient should seek medical treatment for upper respiratory infections that persist for more than 7 days. Increased fluid intake, not caloric intake, is required to liquefy secretions. Home O2 is not a requirement unless the patient's oxygenation saturation is below normal.

17. The nurse evaluates that teaching for the patient with iron deficiency anemia has been effective when the patient states A. "I will need to take the iron supplements the rest of my life." B. "I will increase my dietary intake of milk and milk products." C. "I should increase my activity to increase my aerobic capacity." D. "I should take the iron for several months after my blood is normal."

D. "I should take the iron for several months after my blood is normal." To replace the body's iron stores, iron supplements should be continued for 2 to 3 months after the Hb level returns to normal, but if the cause of the iron deficiency is corrected, the supplements do not need to be taken for a lifetime. Milk and milk products are poor sources of dietary iron. Activity should be gradually increased as Hb levels return to normal because aerobic capacity can be increased when adequate Hb is available.

The nurse evaluates that teaching for the patient with iron deficiency anemia has been effective when the patient states A. "I will need to take the iron supplements the rest of my life." B. "I will increase my dietary intake of milk and milk products." C. "I should increase my activity to increase my aerobic capacity." D. "I should take the iron for several months after my blood is normal."

D. "I should take the iron for several months after my blood is normal." To replace the body's iron stores, iron supplements should be continued for 2 to 3 months after the Hb level returns to normal, but if the cause of the iron deficiency is corrected, the supplements do not need to be taken for a lifetime. Milk and milk products are poor sources of dietary iron. Activity should be gradually increased as Hb levels return to normal because aerobic capacity can be increased when adequate Hb is available.

31. The nurse determines that the patient understood medication instructions about the use of a spacer device when taking inhaled medications after hearing the patient state which of the following as the primary benefit? A. "Now I will not need to breathe in as deeply when taking the inhaler medications." B. "This device will make it so much easier and faster to take my inhaled medications." C. "I will pay less for medication because it will last longer." D. "More of the medication will get down into my lungs to help my breathing."

D. "More of the medication will get down into my lungs to help my breathing." A spacer assists more medication to reach the lungs, with less being deposited in the mouth and the back of the throat.

The nurse determines that the patient understood medication instructions about the use of a spacer device when taking inhaled medications after hearing the patient state which of the following as the primary benefit? A. "Now I will not need to breathe in as deeply when taking the inhaler medications." B. "This device will make it so much easier and faster to take my inhaled medications." C. "I will pay less for medication because it will last longer." D. "More of the medication will get down into my lungs to help my breathing."

D. "More of the medication will get down into my lungs to help my breathing." A spacer assists more medication to reach the lungs, with less being deposited in the mouth and the back of the throat.

4. The nurse is teaching a patient who is to undergo bone marrow aspiration. Which of the following statements made by the nurse would indicate correct instruction regarding the site for the aspiration procedure? A. "The health care provider will perform the aspiration by needle to the femur." B. "The health care provider will perform the aspiration by needle to the scapula." C. "The health care provider will perform the aspiration by needle to the antecubital fossa." D. "The health care provider will perform the aspiration by needle to the posterior iliac crest."

D. "The health care provider will perform the aspiration by needle to the posterior iliac crest." Bone marrow samples are commonly taken from the posterior iliac crest or, as an alternative, the sternum may be aspirated. These sites provide relative ease in accessing the bone marrow via the biopsy needle. The antecubital fossa, femur, and scapula do not allow access to bone marrow while also providing reduced risk of harm to the patient.

The nurse is teaching a patient who is to undergo bone marrow aspiration. Which of the following statements made by the nurse would indicate correct instruction regarding the site for the aspiration procedure? A. "The health care provider will perform the aspiration by needle to the femur." B. "The health care provider will perform the aspiration by needle to the scapula." C. "The health care provider will perform the aspiration by needle to the antecubital fossa." D. "The health care provider will perform the aspiration by needle to the posterior iliac crest."

D. "The health care provider will perform the aspiration by needle to the posterior iliac crest." Bone marrow samples are commonly taken from the posterior iliac crest or, as an alternative, the sternum may be aspirated. These sites provide relative ease in accessing the bone marrow via the biopsy needle. The antecubital fossa, femur, and scapula do not allow access to bone marrow while also providing reduced risk of harm to the patient.

A 60-year-old male client with cancer of the liver has been in a hepatic coma for the past 24 hours. On admission, the client signed a release of information to his family. His oldest son arrives from out of town and asks the nurse how his father is doing. Which response is best for the nurse to provide? A. "I know you are concerned about your father. Would you like to talk about your feelings?" B. The healthcare provider will be here this afternoon and can explain your father's condition to you and the rest of the family." C. "Your father's condition is extremely critical. Would you like me to call the hospital chaplain to talk with you?" D. "Your father has given no response for 24 hours. His condition is extremely critical at this time."

D. "Your father has given no response for 24 hours. His condition is extremely critical at this time."

5. Before starting a transfusion of packed red blood cells for an anemic patient, the nurse would arrange for a peer to monitor his or her other assigned patients for how many minutes when the nurse begins the transfusion? A. 60 B. 5 C. 30 D. 15

D. 15 As part of standard procedure, the nurse remains with the patient for the first 15 minutes after hanging a blood transfusion. Patients who are likely to have a transfusion reaction will more often exhibit signs within the first 15 minutes that the blood is infusing.

Before starting a transfusion of packed red blood cells for an anemic patient, the nurse would arrange for a peer to monitor his or her other assigned patients for how many minutes when the nurse begins the transfusion? A. 60 B. 5 C. 30 D. 15

D. 15 As part of standard procedure, the nurse remains with the patient for the first 15 minutes after hanging a blood transfusion. Patients who are likely to have a transfusion reaction will more often exhibit signs within the first 15 minutes that the blood is infusing.

42. The patient has an order for albuterol 5 mg via nebulizer. Available is a solution containing 1 mg/ml. How many milliliters should the nurse use to prepare the patient's dose? A. 0.2 B. 2.5 C. 3.75 D. 5.0

D. 5.0

The patient has an order for albuterol 5 mg via nebulizer. Available is a solution containing 1 mg/ml. How many milliliters should the nurse use to prepare the patient's dose? A. 0.2 B. 2.5 C. 3.75 D. 5.0

D. 5.0

1. The arterial blood gas (ABG) readings that indicate compensated respiratory acidosis are a PaCO2 of A. 30 mm Hg and bicarbonate level of 24 mEq/L. B. 30 mm Hg and bicarbonate level of 30 mEq/L. C. 50 mm Hg and bicarbonate level of 20 mEq/L. D. 50 mm Hg and bicarbonate level of 30 mEq/L.

D. 50 mm Hg and bicarbonate level of 30 mEq/L. If compensation is present, carbon dioxide and bicarbonate are abnormal (or nearly so) in opposite directions (e.g., one is acidotic and the other alkalotic).

The arterial blood gas (ABG) readings that indicate compensated respiratory acidosis are a PaCO2 of A. 30 mm Hg and bicarbonate level of 24 mEq/L. B. 30 mm Hg and bicarbonate level of 30 mEq/L. C. 50 mm Hg and bicarbonate level of 20 mEq/L. D. 50 mm Hg and bicarbonate level of 30 mEq/L.

D. 50 mm Hg and bicarbonate level of 30 mEq/L. If compensation is present, carbon dioxide and bicarbonate are abnormal (or nearly so) in opposite directions (e.g., one is acidotic and the other alkalotic).

The nurse know that a client taking diuretics must be assessed for the development of hypokalemia, and that hypokalemia will create changes in the client's normal ECG tracing. Which ECG change would be an expected finding in the client with hypokalemia? A. Tall, spiked T waves B. A prolonged QT interval C. A widening QRS complex D. Presence of a U wave

D. A U wave is a positive deflection following the T wave and is often present with hypokalemia. A, B, C indicate hyperkalemia.

Assessment of the fetal heart rate is an important finding when caring for a laboring client. Deceleration in fetal heart rate that are of most concern occur at what time during the contraction cycle? A. Before a contraction B. During a contraction C, Between contractions D. After a contraction

D. After a contraction

Which client should the charge nurse on the oncology unit assign to an RN, rather than a practical nurse (PN) A. A young adult client who is experiencing fatigue while undergoing a series of external beam radiation treatments for stage 1 cancer. B. A middle-aged male client who has just undergone an excision biopsy and has been told that his tumor appears to be benign C. An adult client in remission after a series of chemotherapy treatments who is receiving intramuscular iron injections for anemia D. An elderly female client with cancer and her children who are trying to decide whether to change to palliative care measures or continue disease control

D. An elderly female client with cancer and her children who are trying to decide whether to change to palliative care measures or continue disease control

After administering a proton pump inhibitor (PPI), which action should the nurse take to evaluate the effectiveness of the medication? A. Auscultate for bowel sounds in all quadrants B. Monitor the client's serum electrolyte levels C. Measure the client's fluid intake and output D. Ask the client about gastrointestinal pain

D. Ask the client about gastrointestinal pain

A relative comes into the emergency department asking for information about a female adult client who was admitted in stable condition following a motor vehicle collision. What action should the triage nurse take? A. Determine what is happening with the client, then provide the relative with her current status B. Inform the relative that legally no information can be provided to him or her C. Ask the relative to wait in the waiting area until the healthcare provider can see him or her D. Ask the client if she would like to talk with the relative, then bring the relative to the bedside

D. Ask the client if she would like to talk with the relative, then bring the relative to the bedside

The nurse plans to administer 5,000 units of heparin, an anticoagulant. Which procedure should the nurse implement when administering this drug? A. Massage injection site after administration to ensure that the solution is dissolved B. Prior to injecting the solution, check for bleeding by aspirating the plunger C. Administer IM injections into the fatty portion of the upper arm D. Assess all needle insertion sites daily for hematoma and signs of inflammation

D. Assess all needle insertion sites daily for hematoma and signs of inflammation

The nurse plans to administer 5,000 units of heparin, an anticoagulant. Which procedure should the nurse implement when administering this drug? A. Massage injection site after administration to ensure that the solution is dissolved. B. Prior to injecting the solution, check for bleeding by aspirating the plunger C. Administer IM injections into the fatty portion of the upper arm. D. Assess all needle insertion sites daily for hematoma and signs of inflammation

D. Assess all needle insertion sites daily for hematoma and signs of inflammation

The nurse plans to administer 5,00o units of heparin, an anticoagulant. Which procedure should the nurse implement when administering this drug? A. Massage injection site after administration to ensure that the solution is dissolved B. Prior to injecting the solution, check for bleeding by aspirating the plunger C. Administer IM injections into the fatty portion of the upper arm D. Assess all needle insertion sites daily for hematoma and signs of inflammation

D. Assess all needle insertion sites daily for hematoma and signs of inflammation

The nurse who works in labor and delivery is reassigned to the cardiac care unit for the because of a low census in labor and delivery. Which assignment is best for the charge nurse to give this nurse? A. Transfer a client to another hit B. Monitor the central telemetry C. Perform the admission of a new client D. Assis cardiac nurse with their assignments

D. Assis cardiac nurse with their assignments

27. A patient with COPD is receiving oxygen at 2 L/min. While in the supine position for a bath, the patient complains of shortness of breath. What is the most appropriate first nursing action? A. Increase the flow of oxygen. B. Perform tracheal suctioning. C. Report this to the physician. D. Assist the patient to Fowler's position.

D. Assist the patient to Fowler's position. Breathing is easier in Fowler's position because it permits greater expansion of the chest cavity. If repositioning does not improve the situation, then oxygenation and physician reporting might be appropriate. The patient would not benefit from tracheal suctioning.

A patient with COPD is receiving oxygen at 2 L/min. While in the supine position for a bath, the patient complains of shortness of breath. What is the most appropriate first nursing action? A. Increase the flow of oxygen. B. Perform tracheal suctioning. C. Report this to the physician. D. Assist the patient to Fowler's position.

D. Assist the patient to Fowler's position. Breathing is easier in Fowler's position because it permits greater expansion of the chest cavity. If repositioning does not improve the situation, then oxygenation and physician reporting might be appropriate. The patient would not benefit from tracheal suctioning.

The nurse is scheduled to give a dose of ipratropium bromide by metered dose inhaler. The nurse would administer the right drug by selecting the inhaler with which of the following trade names? A. Vanceril B. Pulmicort C. AeroBid D. Atrovent

D. Atrovent The trade or brand name for ipratropium bromide, an anticholinergic medication, is Atrovent.

A 3-year-old boy is brought to the emergency department after the mother found the child in the back yard holding a piece of a toy in his hand and in respiratory distress. The child is dusky with a loud, inspiratory stridor and weak attempts to cough. Which actions should the nurse implement? A. Obtain a pulse oximetry reading and arterial blood gases B. Request a start chest x-ray and prepare medications for an asthmatic episode C. Determine if the child ingested a toxic substance and if vomiting occurred D. Auscultate all pulmonary lung fields and attempt a Heimlich maneuver

D. Auscultate all pulmonary lung fields and attempt a Heimlich maneuver

A client who has had an open cholecystectomy two weeks ago comes to the emergency department with complaints of nausea, abdominal distention, and pain. Which assessment should the nurse implement? A. Obtain a hemoccult of the client's stool B. Palpate the liver and spleen C. Perform a digital rectal exam D. Auscultate all quadrants of the abdomen

D. Auscultate all quadrants of the abdomen

7. Which of the following physical assessment findings in a patient with pneumonia best supports the nursing diagnosis of ineffective airway clearance? A. Oxygen saturation of 85% B. Respiratory rate of 28 C. Presence of greenish sputum D. Basilar crackles

D. Basilar crackles The presence of adventitious breath sounds indicates that there is accumulation of secretions in the lower airways. This would be consistent with a nursing diagnosis of ineffective airway clearance because the patient is retaining secretions.

Which of the following physical assessment findings in a patient with pneumonia best supports the nursing diagnosis of ineffective airway clearance? A. Oxygen saturation of 85% B. Respiratory rate of 28 C. Presence of greenish sputum D. Basilar crackles

D. Basilar crackles The presence of adventitious breath sounds indicates that there is accumulation of secretions in the lower airways. This would be consistent with a nursing diagnosis of ineffective airway clearance because the patient is retaining secretions.

The nurse administers an isotonic intravenous solution to a client in septic shock. Which parameter is most important for the nurse to monitor to determine if this treatment is effective? A. White blood cell count (WBC) B. Body temperature C. Hemoglobin and hematocrit D. Blood pressure

D. Blood pressure

After routine physical examination, the healthcare provider admits a woman with a history of Systemic Lupus Erythematosis (SLE) to the hospital because she has 3+ pitting ankle edema and blood in her urine. Which assessment finding warrants immediate intervention by the nurse? A. Dark, rust-colored urine B. Urine output 300 ml/hour C. Joint and muscle aches D. Blood pressure 170/91

D. Blood pressure 170/91

22. Respiratory acidosis is at highest risk in a patient with A. hypokalemia. B. pulmonary fibrosis. C. salicylate overdose. D. COPD.

D. COPD. Chronic respiratory acidosis is most commonly caused by COPD. Pulmonary fibrosis, hypokalemia, and salicylate overdose do not predispose a patient to respiratory acidosis. Hypokalemia can lead to cardiac dysrhythmias. Salicylate overdose results in central nervous system changes, and pulmonary fibrosis can result in respiratory arrest.

Respiratory acidosis is at highest risk in a patient with A. hypokalemia. B. pulmonary fibrosis. C. salicylate overdose. D. COPD.

D. COPD. Chronic respiratory acidosis is most commonly caused by COPD. Pulmonary fibrosis, hypokalemia, and salicylate overdose do not predispose a patient to respiratory acidosis. Hypokalemia can lead to cardiac dysrhythmias. Salicylate overdose results in central nervous system changes, and pulmonary fibrosis can result in respiratory arrest.

The home health nurse visits a client with heart failure (HF). Assessment findings include: temperature 97.6F, pulse 116 beats/minute, respiratory rate 36 breaths/minute, blood pressure 140/70, pulse oximeter 86% on 2 L/min of oxygen, and crackles are heard throughout the lung fields. Which intervention has the highest priority? A. Start an IV of normal saline (NS) at 100 ml/hr B. Assess for edema and weigh client C. Obtain a prescription for an increased oxygen rate D. Call 911 and prepare the client for transport

D. Call 911 and prepare the client for transport

What is the goal when planning nursing care for a client with edema and leg discoloration secondary to chronic venous insufficiency? A. Adequate oxygenation will be restored B. Client will manifest normal urine output C. Client will demonstrate improved fluid-balance D. Client's skin integrity will remain intact

D. Client's skin integrity will remain intact

The charge nurse in the Labor and Delivery Unit makes assignments for a nurse and unlicensed assistive personnel (UAP). A client in labor is admitted with contractions occurring every 3 to 5 minutes. Which task should be assigned to the UAP? A. Teach patterned breathing B. Apply external fetal monitor C. Measure the fundal height D. Collet a urine specimen

D. Collet a urine specimen

At bedtime, an unlicensed assistive personnel (UAP) is positioning a client with obstructive sleep apnea syndrome (OSAS). The UAP elevates the head of the bed and encourages the client to turn on side. In supervising the UAP, what action should the nurse take? A. After leaving the room, discuss correct positioning with the UAP B. Reposition the client in a supine position with the feet elevated pillows C. Remind the UAP to pad the side rails to reduce risk for injury D. Confirm that the UAP has placed the call bell within reach of the client.

D. Confirm that the UAP has placed the call bell within reach of the client.

A client who is receiving an ACE inhibitor for hypertension calls the clinic and reports the recent onset of a cough to the nurse. What action should the nurse implement? A. Advise the client to come to the clinic immediately for further assessment. B. Instruct the client to discontinue use of the drug, and make an appointment at the clinic. C. Suggest that the client lear to accept the cough as a side effect to a necessary prescription. D. Encourage the client to keep taking the drug until seen by the HCP.

D. Cough is a common s/e of ACE inhibitors and is not an indication to discontinue the medication. (A) immediate evaluation is not needed. (B) an antihypertensive should not be stopped abruptly. (C) is demeaning since the cough may be disruptive to the client and other medications may produce results without the s/e.

A female client with severe renal impairment is receiving enoxaparin (Lovenox) 30 mg SUBQ BID. Which laboratory value due to enoxaparin should the nurse report to the healthcare provider? A. Calcium 9 mg/dl (9 mmol/L SI) B. Hemoglobin 12 grams/dl (120 mmol/L SI) C. Partial thromboplastin time (PTT) 30 seconds D. Creatinine clearance 25 ml/minute

D. Creatinine clearance 25 ml/minute

The nurse is assessing the emotional status of a client with Parkinson's disease. Which client finding is most helpful in planning goals to meet the client's emotional needs? A. Stares straight ahead without blinking B. Face does not convey any emotion C. Uses a monotone when speaking D. Cries frequently during the interview

D. Cries frequently during the interview

The healthcare provider prescribes Isosorbibide (Isordil) 40 mg every 8 hours for a male client with acute angina pectoris. Which finding should the nurse report to the healthcare provider prior to administering Isordil? A. Serum cholesterol of 200 mg/dl B. Hemoglobin of 14.8 grams/dl C. Takes enteric coated aspirin daily D. Currently takes sildenafil (Viagra)

D. Currently takes sildenafil (Viagra)

A terminally ill client on a palliative care unit has an advanced directive stipulating comfort measure only. The client has not taken oral fluids in the last 36 hours and is not receiving intravenous fluids. The clients blood pressure is 64/38 and urinary output is 50 ml for the last 12 hours. What is the priority nursing intervention? A. Assess for the presence of dependent edema B. Prepare to initiate intravenous fluids C. Gently massage the client's bladder D. Determine the client's level of discomfort

D. Determine the client's level of discomfort

The nurse-manager of a pediatric units needs to assign a room for a 6-month-old diagnosed with respiratory syncytial virus (RSV). Which room assignment is the best for the nurse-manager to make? A. Private room furtherest form the nurses station B. Double room with a 6-month-old on droplet precautions C. Private room with negative air pressure D. Double room with a 4-month-old who has RSV

D. Double room with a 4-month-old who has RSV

The father brings his pre-school-aged son to the rural urgent care clinic because the child fell from a horse earlier today. Which finding indicates to the nurse that further assessment is required for possible abuse or neglect? A. A dislocated shoulder and fractured wrist on X-ray B. Bruises, abrasions, and restricted movement in right shoulder, elbow, and wrist C. Father's presence during the child's assessment and physical examination D. Dry, peeling skin, ridged nails, and significantly underweight

D. Dry, peeling skin, ridged nails, and significantly underweight

The nurse should instruct the parents of an 11-year-old with Type 1 diabetes mellitus to carefully watch their child for symptoms of diabetic ketoacidosis at risk for becoming ketoacidotic? A. While adjusting the amount of the insulin dosage B. When changing to a new brand of insulin C. After skipping two or more meals consecutively D. During the course of an acute illness

D. During the course of an acute illness

The parents of a 6-year-old recently diagnosed with Duchenne muscular dystrophy tell the nurse that their child wants to continue attending swimming classes. How should the nurse respond? A. Explain that their child is too young to understand risks associated with swimming B. Provide a list of alternative activities that are less likely to cause the child to experience fatigue C. Suggest that the child can be encouraged to participate in a team sport to encourage socialization D. Encourage the parents to allow the child to continue attending swimming lessons with supervision

D. Encourage the parents to allow the child to continue attending swimming lessons with supervision

The healthcare provider prescribes digoxin (Lanoxin) 0.5 mg PO daily for a client with heart failure. When the nurse scans the medication label, "digoxin 0.25 mg/tablet," using an electronic scanner, a "pop-up" window in the electronic medical record indicates "Error in dose." What action should the nurse take? A. Request the pharmacy to deliver the correct dose B. Rescan the medication label until it registers C. Notify the healthcare provider of the error in the data D. Enter the value, 2 tablets, administered

D. Enter the value, 2 tablets, administered

After receiving a prescribed dose of quinapril (Accupril), losartan (Cozaar), and clonidine (Catapres), a female client tells the nurse that she usually takes Accupril at 0800, Cozaar at 1600, and clonidine at 2200 at home. The nurse informs the charge nurse that an error was made during the morning medication administration. What action should the charge nurse implement first? A. Implement orthostatic safety precautions B. Assess the client's level of consciousness C. Increase the client's fluid intake during the day D. Evaluate the clients blood pressure

D. Evaluate the clients blood pressure

22. The nurse is assigned to care for a patient who has anxiety and an exacerbation of asthma. Which of the following is the primary reason for the nurse to carefully inspect the chest wall of this patient? A. Observe for signs of diaphoresis B. Allow time to calm the patient C. Monitor the patient for bilateral chest expansion D. Evaluate the use of intercostal muscles

D. Evaluate the use of intercostal muscles The nurse physically inspects the chest wall to evaluate the use of intercostal (accessory) muscles, which gives an indication of the degree of respiratory distress experienced by the patient.

The nurse is assigned to care for a patient who has anxiety and an exacerbation of asthma. Which of the following is the primary reason for the nurse to carefully inspect the chest wall of this patient? A. Observe for signs of diaphoresis B. Allow time to calm the patient C. Monitor the patient for bilateral chest expansion D. Evaluate the use of intercostal muscles

D. Evaluate the use of intercostal muscles The nurse physically inspects the chest wall to evaluate the use of intercostal (accessory) muscles, which gives an indication of the degree of respiratory distress experienced by the patient.

A clinical trial is recommended for a client with metastatic breast cancer, but she refuses to participate and tells her family that she does not wish to have further treatments. The client's son and daughter ask the nurse to try to convince their mother to reconsider this decision. How should the nurse respond? A. ask the client with her children present if she fully understands the decision she has made B. Discuss success of clinical trials and ask the client to consider participating for one month C. Explain to the family that they must accept their mother's decision D. Explore the client's decision to refuse treatment and offer support

D. Explore the client's decision to refuse treatment and offer support

14. Which of the following foods is high in iron? A. Citrus fruits B. Milk products C. Yellow vegetables D. Green leafy vegetables

D. Green leafy vegetables Green leafy vegetables are high in iron. Foods cooked in iron pots and foods such as liver (the richest source), oysters, lean meats, kidney beans, whole wheat bread, kale, spinach, egg yolk, turnip tops, beet greens, carrots, apricots, and raisins are also high in iron.

Which of the following foods is high in iron? A. Citrus fruits B. Milk products C. Yellow vegetables D. Green leafy vegetables

D. Green leafy vegetables Green leafy vegetables are high in iron. Foods cooked in iron pots and foods such as liver (the richest source), oysters, lean meats, kidney beans, whole wheat bread, kale, spinach, egg yolk, turnip tops, beet greens, carrots, apricots, and raisins are also high in iron.

41. When planning patient teaching about emphysema, the nurse understands that the symptoms of emphysema are caused by which of the following? A. Hypertrophy and hyperplasia of goblet cells in the bronchi B. Collapse and hypoventilation of the terminal respiratory unit C. An overproduction of the antiprotease alpha1-antitrypsin D. Hyperinflation of alveoli and destruction of alveolar walls

D. Hyperinflation of alveoli and destruction of alveolar walls In emphysema, there are structural changes that include hyperinflation of alveoli, destruction of alveolar walls, destruction of alveolar capillary walls, narrowing of small airways, and loss of lung elasticity.

When planning patient teaching about emphysema, the nurse understands that the symptoms of emphysema are caused by which of the following? A. Hypertrophy and hyperplasia of goblet cells in the bronchi B. Collapse and hypoventilation of the terminal respiratory unit C. An overproduction of the antiprotease alpha1-antitrypsin D. Hyperinflation of alveoli and destruction of alveolar walls

D. Hyperinflation of alveoli and destruction of alveolar walls In emphysema, there are structural changes that include hyperinflation of alveoli, destruction of alveolar walls, destruction of alveolar capillary walls, narrowing of small airways, and loss of lung elasticity.

A young woman with multiple sclerosis just received several immunizations in preparation for moving into a college dormitory. Two days later, she reports to the nurse that she is experiencing increasing fatigue and visual problems. What teaching should the nurse provide? A. Plans to move into the dormitory need to be postponed for at least a semester B. These early signs of an infection may require medical treatment with antibiotics C. These are common side effects of the vaccines and will resolve in a few days D. Immunizations can trigger a relapse of the disease, so get plenty of extra rest

D. Immunizations can trigger a relapse of the disease, so get plenty of extra rest

What is the primary purpose for initiating nursing interventions that promote good nutrition, rest and exercise, and stress reduction for clients diagnosed with an HIV infection? A. Increase ability to carry out activities of daily living B. Promote a feeling of general well-being C. Prevent spread of infection to others D. Improve function of the immune system

D. Improve function of the immune system

Which is the primary purpose for initiating nursing interventions that promote good nutrition, rest and exercise, and stress reduction for clients diagnosed with an HIV infection? A. Increase ability to carry out activities of daily living B. Promote a feeling of general well-being C. Prevent spread of infection to others D. Improve function of the immune system

D. Improve function of the immune system

A client is admitted with syncopal episodes related to a third degree heart block. After the placement of a transcutaneous pacemaker, the nurse observes several episodes of the pacemaker's failure to sense. What action should the nurse take? A. Turn off the pacemaker B. Adjust the miliamperes (mA) C. Increase the pacemaker rate D. Increase the sensitively

D. Increase the sensitively

5. A nurse is performing assessment for a patient diagnosed with chronic obstructive pulmonary disease (COPD). Which of the following findings should the nurse expect to observe? A. Nonproductive cough B. Prolonged inspiration C. Vesicular breath sounds D. Increased anterior-posterior chest diameter

D. Increased anterior-posterior chest diameter An increased anterior-posterior diameter is a compensatory mechanism experienced by patients with COPD and is caused by air-trapping. Patients with COPD have a productive cough, often expectorating copious amounts of sputum. Because of air-trapping, patients with COPD experience a prolonged expiration because the rate of gas on exhalation takes longer to escape. Chest auscultation for patients with COPD often reveals wheezing, crackles, and other adventitious breath sounds.

A nurse is performing assessment for a patient diagnosed with chronic obstructive pulmonary disease (COPD). Which of the following findings should the nurse expect to observe? A. Nonproductive cough B. Prolonged inspiration C. Vesicular breath sounds D. Increased anterior-posterior chest diameter

D. Increased anterior-posterior chest diameter An increased anterior-posterior diameter is a compensatory mechanism experienced by patients with COPD and is caused by air-trapping. Patients with COPD have a productive cough, often expectorating copious amounts of sputum. Because of air-trapping, patients with COPD experience a prolonged expiration because the rate of gas on exhalation takes longer to escape. Chest auscultation for patients with COPD often reveals wheezing, crackles, and other adventitious breath sounds.

2. When assessing lab values on a patient admitted with septicemia, the nurse would expect to find: A. Increased platelets B. Decreased red blood cells C. Decreased erythrocyte sedimentation rate (ESR) D. Increased bands in the WBC differential (shift to the left)

D. Increased bands in the WBC differential (shift to the left) When infections are severe, such as in septicemia, more granulocytes are released from the bone marrow as a compensatory mechanism. To meet the increased demand, many young, immature polymorphonuclear neutrophils (bands) are released into circulation. WBCs are usually reported in order of maturity, with the less mature forms on the left side of a written report. Hence, the term "shift to the left" is used to denote an increase in the number of bands.

When assessing lab values on a patient admitted with septicemia, the nurse would expect to find: A. Increased platelets B. Decreased red blood cells C. Decreased erythrocyte sedimentation rate (ESR) D. Increased bands in the WBC differential (shift to the left)

D. Increased bands in the WBC differential (shift to the left) When infections are severe, such as in septicemia, more granulocytes are released from the bone marrow as a compensatory mechanism. To meet the increased demand, many young, immature polymorphonuclear neutrophils (bands) are released into circulation. WBCs are usually reported in order of maturity, with the less mature forms on the left side of a written report. Hence, the term "shift to the left" is used to denote an increase in the number of bands.

10. The blood bank notifies the nurse that the two units of blood ordered for an anemic patient are ready for pick up. The nurse should take which of the following actions to prevent an adverse effect during this procedure? A. Immediately pick up both units of blood from the blood bank. B. Regulate the flow rate so that each unit takes at least 4 hours to transfuse. C. Set up the Y-tubing of the blood set with dextrose in water as the flush solution. D. Infuse the blood slowly for the first 15 minutes of the transfusion.

D. Infuse the blood slowly for the first 15 minutes of the transfusion. Because a transfusion reaction is more likely to occur at the beginning of a transfusion, the nurse should initially infuse the blood at a rate no faster than 2 ml/min and remain with the patient for the first 15 minutes after hanging a unit of blood.

The blood bank notifies the nurse that the two units of blood ordered for an anemic patient are ready for pick up. The nurse should take which of the following actions to prevent an adverse effect during this procedure? A. Immediately pick up both units of blood from the blood bank. B. Regulate the flow rate so that each unit takes at least 4 hours to transfuse. C. Set up the Y-tubing of the blood set with dextrose in water as the flush solution. D. Infuse the blood slowly for the first 15 minutes of the transfusion.

D. Infuse the blood slowly for the first 15 minutes of the transfusion. Because a transfusion reaction is more likely to occur at the beginning of a transfusion, the nurse should initially infuse the blood at a rate no faster than 2 ml/min and remain with the patient for the first 15 minutes after hanging a unit of blood.

After placement of a left subclavian central venous catheter (CVC), the nurse receives report of the X-ray findings that indicate the CVC tip is in the client's superior vena cava. Which action should the nurse implement? A. Notify the healthcare provider of the need to reposition the catheter B. Remove the catheter and apply direct pressure for 5 minutes C. Secure the catheter using aseptic techniques D. Initiate intravenous fluids as prescribed

D. Initiate intravenous fluids as prescribed

Two unlicensed assistive personal (UAP) are arguing loudly in the hallway of the extended care facility about who will shower a male resident who defecated in his bed. What action is best for charge nurse to take? A. Shower the client with the help of a practical nurse B. Reassign the clients care to another staff member C. Document the conflict in the employee personnel files D. Instruct both UAP to shower the client immediately

D. Instruct both UAP to shower the client immediately

A client who is experiencing panic attacks receives a prescription for the benzodiazepine alprazolam (Xanax). Which instructions should the nurse provide the client? A. Explain that it may take up to two weeks before the anxiety starts to get better B. Instruct the client to notify the healthcare provider if tremors of the tongue occur C. Discuss the importance of obtaining monthly blood work to assess for toxicity D. Instruct the client on safety issues this medication causes drowsiness

D. Instruct the client on safety issues this medication causes drowsiness

The nurse is preparing to conduct discharge teaching for a client who had an anaphylactic reaction following administration of ampicillin (Omnipen-N). What instruction is essential for the nurse to provide this client prior to discharge? A. Inform the client that it is essential to take all of the prescribed ampicillin B. Teach the client how to self-administer epinephrine in case a reaction occurs again C. Tell the client to take medication with food to decrease the possibility of future reactions D. Instruct the client to wear a medic-alert bracelet so penicillin will not be given again

D. Instruct the client to wear a medic-alert bracelet so penicillin will not be given again

38. The nurse is evaluating whether a patient understands how to safely determine whether a metered dose inhaler is empty. The nurse interprets that the patient understands this important information to prevent medication underdosing when the patient describes which method to check the inhaler? A. Place it in water to see if it floats. B. Shake the canister while holding it next to the ear. C. Check the indicator line on the side of the canister. D. Keep track of the number of inhalations used.

D. Keep track of the number of inhalations used. It is no longer appropriate to see if a canister floats in water or not as research has demonstrated this is not accurate. The best method to determine when to replace an inhaler is by knowing the maximum puffs available per MDI and then replacing when those inhalations have been used.

The nurse is evaluating whether a patient understands how to safely determine whether a metered dose inhaler is empty. The nurse interprets that the patient understands this important information to prevent medication underdosing when the patient describes which method to check the inhaler? A. Place it in water to see if it floats. B. Shake the canister while holding it next to the ear. C. Check the indicator line on the side of the canister. D. Keep track of the number of inhalations used.

D. Keep track of the number of inhalations used. It is no longer appropriate to see if a canister floats in water or not as research has demonstrated this is not accurate. The best method to determine when to replace an inhaler is by knowing the maximum puffs available per MDI and then replacing when those inhalations have been used.

A female who was admitted for alcohol detoxification is nauseated and describes feeling like roaches are crawling all over her. She is tremulous, and her blood pressure is 146/92; her pulse rate is 94 beats/minute; and her temperature is 100.8F. Which PRN medication should the nurse administer first? A. Ondansetron (Zofran) B. Acetaminophen (Tylenol) C. Ramiprl (Altace) D. Lorazepam (Ativan)

D. Lorazepam (Ativan)

The nurse plans to administer 1 teaspoon of a liquid medication to a toddler. What is the most accurate way to administer the medication? A. Give medication using a medication dropper B. Administer from a clean teaspoon C. Use a medicine cup to measure the dose D. Measure the medication in an oral syringe

D. Measure the medication in an oral syringe

After receiving lactulose, a client with hepatic encephalopathy has several loose stools. What action should the nurse implement? A. Send a stool specimen to the lab B. Measure abdominal girth C. Encourage increased fiber in diet D. Monitor mental status

D. Monitor mental status Lactulose is administered to clients with hepatic encephalopathy to lower serum ammonia levels, which should improve the client's mental status

15. When planning appropriate nursing interventions for a patient with metastatic lung cancer and a 60-pack-year history of cigarette smoking, the nurse recognizes that the smoking has most likely decreased the patient's underlying respiratory defenses because of impairment of which of the following? A. Reflex bronchoconstriction B. Ability to filter particles from the air C. Cough reflex D. Mucociliary clearance

D. Mucociliary clearance Smoking decreases the ciliary action in the tracheobronchial tree, resulting in impaired clearance of respiratory secretions, chronic cough, and frequent respiratory infections.

When planning appropriate nursing interventions for a patient with metastatic lung cancer and a 60-pack-year history of cigarette smoking, the nurse recognizes that the smoking has most likely decreased the patient's underlying respiratory defenses because of impairment of which of the following? A. Reflex bronchoconstriction B. Ability to filter particles from the air C. Cough reflex D. Mucociliary clearance

D. Mucociliary clearance Smoking decreases the ciliary action in the tracheobronchial tree, resulting in impaired clearance of respiratory secretions, chronic cough, and frequent respiratory infections.

13. If a patient with arthritis develops iron-deficiency anemia, a nurse should ask about the patient's use of A. alcoholic beverages. B. stool softeners and laxatives. C. caffeinated foods and beverages. D. NSAIDs.

D. NSAIDs. NSAIDs decrease the level of vitamin C, which aids in the absorption of iron. These drugs also compete with folate and vitamin K and may cause gastritis. Excessive alcoholic beverage consumption can cause stomach irritation; alcohol would not be directly related to iron-deficiency anemia unless bleeding ulcers or gastritis were to occur. NSAID consumption, not stool softeners and laxative use, would be suspect for iron-deficiency anemia. Caffeinated foods and beverages can cause gastric irritation and discomfort but are not associated with iron-deficiency anemia.

If a patient with arthritis develops iron-deficiency anemia, a nurse should ask about the patient's use of A. alcoholic beverages. B. stool softeners and laxatives. C. caffeinated foods and beverages. D. NSAIDs.

D. NSAIDs. NSAIDs decrease the level of vitamin C, which aids in the absorption of iron. These drugs also compete with folate and vitamin K and may cause gastritis. Excessive alcoholic beverage consumption can cause stomach irritation; alcohol would not be directly related to iron-deficiency anemia unless bleeding ulcers or gastritis were to occur. NSAID consumption, not stool softeners and laxative use, would be suspect for iron-deficiency anemia. Caffeinated foods and beverages can cause gastric irritation and discomfort but are not associated with iron-deficiency anemia.

The nurse is assessing a client who presents with jaundice. Which assessment finding is the most significant indication that further follow up is needed? A. Urine specific gravity of 1.03 with a urine output of 500 ml in 8 hours B. Frothy, tea-colored urine C. Clay-colored stools and complaints of pruritus D. Serum amylase and lipase levels that are twice their normal levels

D. Obstructive cholelithiasis and alcoholism are the two major causes of pancreatitis, and an elevated serum amylase and lipase indicate pancreatic injury. (A) is a normal finding. (B & C) are expected findings for jaundice.

44. When caring for a patient with COPD, the nurse identifies a nursing diagnosis of imbalanced nutrition less than body requirements after noting a weight loss of 30 lb. Which of the following would be an appropriate intervention to add to the plan of care for this patient? A. Teach the patient to use frozen meals at home that can be microwaved. B. Provide a high-calorie, high-carbohydrate, nonirritating, frequent feeding diet. C. Order fruits and fruit juices to be offered between meals. D. Order a high-calorie, high-protein diet with six small meals a day.

D. Order a high-calorie, high-protein diet with six small meals a day.Because the patient with COPD needs to use greater energy to breathe, there is often decreased oral intake because of dyspnea. A full stomach also impairs the ability of the diaphragm to descend during inspiration, interfering with the work of breathing. Finally, the metabolism of a high carbohydrate diet yields large amounts of CO2, which may lead to acidosis in patients with pulmonary disease. For these reasons, the patient with emphysema should take in a high-calorie, high-protein diet, eating six small meals per day.

When caring for a patient with COPD, the nurse identifies a nursing diagnosis of imbalanced nutrition less than body requirements after noting a weight loss of 30 lb. Which of the following would be an appropriate intervention to add to the plan of care for this patient? A. Teach the patient to use frozen meals at home that can be microwaved. B. Provide a high-calorie, high-carbohydrate, nonirritating, frequent feeding diet. C. Order fruits and fruit juices to be offered between meals. D. Order a high-calorie, high-protein diet with six small meals a day.

D. Order a high-calorie, high-protein diet with six small meals a day.Because the patient with COPD needs to use greater energy to breathe, there is often decreased oral intake because of dyspnea. A full stomach also impairs the ability of the diaphragm to descend during inspiration, interfering with the work of breathing. Finally, the metabolism of a high carbohydrate diet yields large amounts of CO2, which may lead to acidosis in patients with pulmonary disease. For these reasons, the patient with emphysema should take in a high-calorie, high-protein diet, eating six small meals per day.

13. Which of the following nursing interventions is most appropriate to enhance oxygenation in a patient with unilateral malignant lung disease? A. Positioning patient on right side. B. Maintaining adequate fluid intake C. Performing postural drainage every 4 hours D. Positioning patient with "good lung down"

D. Positioning patient with "good lung down" Therapeutic positioning identifies the best position for the patient assuring stable oxygenation status. Research indicates that positioning the patient with the unaffected lung (good lung) dependent best promotes oxygenation in patients with unilateral lung disease. For bilateral lung disease, the right lung down has best ventilation and perfusion. Increasing fluid intake and performing postural drainage will facilitate airway clearance, but positioning is most appropriate to enhance oxygenation.

Which of the following nursing interventions is most appropriate to enhance oxygenation in a patient with unilateral malignant lung disease? A. Positioning patient on right side. B. Maintaining adequate fluid intake C. Performing postural drainage every 4 hours D. Positioning patient with "good lung down"

D. Positioning patient with "good lung down" Therapeutic positioning identifies the best position for the patient assuring stable oxygenation status. Research indicates that positioning the patient with the unaffected lung (good lung) dependent best promotes oxygenation in patients with unilateral lung disease. For bilateral lung disease, the right lung down has best ventilation and perfusion. Increasing fluid intake and performing postural drainage will facilitate airway clearance, but positioning is most appropriate to enhance oxygenation.

33. The nurse determines that the patient is not experiencing adverse effects of albuterol (Proventil) after noting which of the following patient vital signs? A. Oxygen saturation 96% B. Respiratory rate of 18 C. Temperature of 98.4° F D. Pulse rate of 76

D. Pulse rate of 76 Albuterol is a β2-agonist that can sometimes cause adverse cardiovascular effects. These would include tachycardia and angina. A pulse rate of 76 indicates that the patient did not experience tachycardia as an adverse effect.

The nurse determines that the patient is not experiencing adverse effects of albuterol (Proventil) after noting which of the following patient vital signs? A. Oxygen saturation 96% B. Respiratory rate of 18 C. Temperature of 98.4° F D. Pulse rate of 76

D. Pulse rate of 76 Albuterol is a β2-agonist that can sometimes cause adverse cardiovascular effects. These would include tachycardia and angina. A pulse rate of 76 indicates that the patient did not experience tachycardia as an adverse effect.

An 18-year-old female client is admitted to the unit after ingesting an overdose of Phenobarbital (Luminal). She is unresponsive and ABG results are: pH 7.18, PaCO2 60 mmHg, and HCO3 26 mEq/L. Which interpretation of the client's ABG results by the nurse is accurate? A. Metabolic acidosis, uncompensated B. Respiratory acidosis, compensated C. Metabolic alkalosis, compensated D. Respiratory acidosis, uncompensated

D. Respiratory acidosis, uncompensated

The nurse plans to help an 18-year-old developmentally disabled female client ambulate on the first postoperative day. When the nurse tells her it is time to get out of bed, the client becomes angry and yells at the nurse. "Get out of here! I'll get up when I'm ready." Which response should the nurse provide? A. "Your healthcare provider has prescribed ambulation on the first postoperative day." B. "You must ambulate to avoid serious complications that are much more painful." C. "I know how you feel; you're angry about having to do this, but it is required." D. "I'll be back in 30 minutes to help you get out of bed and walk around the room."

D. Returning in 30 minutes provides a cooling off period, is firm, direct, nonthreatening, and avoids argument with the client. B is threatening. C. assumes what the client is feeling. A. avoids the nurse's responsibility to ambulate the client.

When preparing to start change-of-shift report, the charge nurse observes an unlicensed assistive personnel (UAP) walking in the hallway with a urine specimen that is not covered. After telling the UAP to cover the specimen, what intervention should the charge nurse implement? A. Ask the nurse educator to review infection control policies with all UAPs. B. Gather input from other staff about the quality of the UAP's performance. C. Direct the UAP to collect all specimens using universal precautions D. Review infection control guidelines with the UAP at the next opportunity

D. Review infection control guidelines with the UAP at the next opportunity

Beginning with the vena cavae, what is the normal sequence of blood circulation through the heart? A. Mitral valve B. Left atrium C. Pulmonary semilunar valve D. Right atrium E. Tricuspid valve F. Aortic semilunar valve G. Left ventricle H. Right ventricle

D. Right atrium E. Tricuspid valve H. Right ventricle C. Pulmonary semilunar valve B. Left atrium A. Mitral valve G. Left ventricle F. Aortic semilunar valve

36. The nurse is assisting a patient to learn self-administration of beclomethasone two puffs inhalation q6hr. The nurse explains that the best way to prevent oral infection while taking this medication is to do which of the following as part of the self-administration techniques? A. Chew a hard candy before the first puff of medication. B. Ask for a breath mint following the second puff of medication. C. Rinse the mouth with water before each puff of medication. D. Rinse the mouth with water following the second puff of medication.

D. Rinse the mouth with water following the second puff of medication. The patient should rinse the mouth with water following the second puff of medication to reduce the risk of fungal overgrowth and oral infection.

The nurse is assisting a patient to learn self-administration of beclomethasone two puffs inhalation q6hr. The nurse explains that the best way to prevent oral infection while taking this medication is to do which of the following as part of the self-administration techniques? A. Chew a hard candy before the first puff of medication. B. Ask for a breath mint following the second puff of medication. C. Rinse the mouth with water before each puff of medication. D. Rinse the mouth with water following the second puff of medication.

D. Rinse the mouth with water following the second puff of medication. The patient should rinse the mouth with water following the second puff of medication to reduce the risk of fungal overgrowth and oral infection.

The nurse is assessing a 75-year-old male client for symptoms of hyperglycemia. Which symptom of hyperglycemia is an older adult most likely to exhibit? A. Polyuria B. Polydipsia C. Weight loss D. Infection

D. S/Sx of hyperglycemia in older adults may include fatigue, infection, and neuropathy (such as sensory changes). (A, B, C) are classic symptoms and may be absent in the older adult.

The unit manager of an acute care unit evaluates the time management skills of the nursing staff and determines that one staff nurse is consistently behind in meeting the needs of assigned clients. What action should the unit manager take? A. Plan to reassign some of the clients to another nurse the nest day B. Determine if the nurse is having personal problems that affect work. C. Request that the nursing supervisor meet with the nurse today D. Schedule a private meeting with the staff nurse as soon as possible

D. Schedule a private meeting with the staff nurse as soon as possible

An adolescent male is transferred from the medical unit to the mental health unit because his condition is stable after taking an assortment of prescription drugs. Based on the admission interview, the nurse determines that the client is still having suicidal ideations. What intervention is most important for the nurse to implement? A. Try to determine what life event precipitated the suicide attempt B. Place the client in lock-up until the psychiatrist releases him C. Reassure the client that he is in a safe place D. Search the client's belongings for potential weapons

D. Search the client's belongings for potential weapons

A client with alcohol-related liver disease is admitted to the unit. Which prescription should the nurse question as possibly inappropriate for the client? A. Vitamin K1 (AquaMEPHYTON) 5 mg IM daily B. High-calorie, low-sodium diet C. Fluid restriction to 1500 ml/day D. Pentobarbital (Nembutal sodium) 50 mg at bedtime for rest

D. Sedatives such as Nembutal are contraindicated for clients with liver damage and can have dangerous consequences. (A) is often prescribed since normal clotting mechanism is damaged. (B) is needed to restore energy. (C) Fluids are restricted to decrease ascites which often accompanies cirrhosis, particularly in later stages of the disease.

A female client who had a total thyroidectomy several weeks ago is admitted with myxedema coma. Which finding indicates that the client has been noncompliant with her postoperative treatment plan? A. Systolic blood pressure consistently greater than 160 B. Suppressed levels of thyroid stimulating hormone (TSH) C. Telemetry reveals atrial fibrillation D. Serum T3 and T4 levels below normal

D. Serum T3 and T4 levels below normal

18. In the case of pulmonary embolus from deep vein thrombosis, which of the following actions should the nurse take first? A. Notify the physician. B. Administer a nitroglycerin tablet sublingually. C. Conduct a thorough assessment of the chest pain. D. Sit the patient up in bed as tolerated and apply oxygen.

D. Sit the patient up in bed as tolerated and apply oxygen.The patient's clinical picture is consistent with pulmonary embolus, and the first action the nurse takes should be to assist the patient. For this reason, the nurse should sit the patient up as tolerated and apply oxygen before notifying the physician.

In the case of pulmonary embolus from deep vein thrombosis, which of the following actions should the nurse take first? A. Notify the physician. B. Administer a nitroglycerin tablet sublingually. C. Conduct a thorough assessment of the chest pain. D. Sit the patient up in bed as tolerated and apply oxygen.

D. Sit the patient up in bed as tolerated and apply oxygen.The patient's clinical picture is consistent with pulmonary embolus, and the first action the nurse takes should be to assist the patient. For this reason, the nurse should sit the patient up as tolerated and apply oxygen before notifying the physician.

12. After admitting a patient to the medical unit with a diagnosis of pneumonia, the nurse will verify that which of the following physician orders have been completed before administering a dose of cefotetan (Cefotan) to the patient? A. Serum laboratory studies ordered for AM B. Pulmonary function evaluation C. Orthostatic blood pressures D. Sputum culture and sensitivity

D. Sputum culture and sensitivityThe nurse should ensure that the sputum for culture and sensitivity was sent to the laboratory before administering the cefotetan. It is important that the organisms are correctly identified (by the culture) before their numbers are affected by the antibiotic; the test will also determine whether the proper antibiotic has been ordered (sensitivity testing). Although antibiotic administration should not be unduly delayed while waiting for the patient to expectorate sputum, all of the other options will not be affected by the administration of antibiotics.

After admitting a patient to the medical unit with a diagnosis of pneumonia, the nurse will verify that which of the following physician orders have been completed before administering a dose of cefotetan (Cefotan) to the patient? A. Serum laboratory studies ordered for AM B. Pulmonary function evaluation C. Orthostatic blood pressures D. Sputum culture and sensitivity

D. Sputum culture and sensitivityThe nurse should ensure that the sputum for culture and sensitivity was sent to the laboratory before administering the cefotetan. It is important that the organisms are correctly identified (by the culture) before their numbers are affected by the antibiotic; the test will also determine whether the proper antibiotic has been ordered (sensitivity testing). Although antibiotic administration should not be unduly delayed while waiting for the patient to expectorate sputum, all of the other options will not be affected by the administration of antibiotics.

A female client receives a prescription for alendronate sodium (Fosamx) to treat her newly diagnosed osteoporosis. What instruction should the nurse include in the client's teaching plan? A. Eat within 30 minutes of taking the medication B. Ingest an antacid 30 minutes prior to taking the tablet C. Consume a light snack with the medication D. Take on an empty stomach with a full glass of water

D. Take on an empty stomach with a full glass of water

An unlicensed assistive personnel (UAP) informs the nurse who is giving medications that a female client is crying. The client was just informed that she has a malignant tumor. What action should the nurse implement first? A. Provide the client with a PRN antianxiety medication and allow privacy for her to grieve B. instruct the UAP to notify the client's spiritual advisor of her need for counseling C. Ask another nurse to finish giving medications and attend to the client immediately D. Tell the client that the nurse will be back to talk to her after medications are given

D. Tell the client that the nurse will be back to talk to her after medications are given

An emergency department nurse is giving discharge instructions to the wife of a young adult male client who sustained a concussion after a fall. The nurse should provide the wife with what instruction as part of the discharge teaching plan? A. Encourage the wife to bring her husband back to the emergency department if he experiences headaches within the next 24 hours B. Teach the wife how to complete a Glasgow Scale (GCS) to do at home C. Provide written instructions on determining pupil constriction D. Tell the wife to bring her husband back to the emergency department if he has projectile vomiting or an unsteady gait

D. Tell the wife to bring her husband back to the emergency department if he has projectile vomiting or an unsteady gait

A nurse is caring for an elderly client who recently attempted suicide with a overdose of sedatives. Which conclusion regarding this client's achievement of normal development is accurate? A. Suicide attempts that occur in the elderly population are most likely due to declining physical health B. The client was unsuccessful in resolving Trust vs. Mistrust issues, resulting in anger turned inward C. Role confusion often occurs in the elderly due to various levels of dementia, which explains suicidal tenderness D. The adult who is unfulfilled at an advanced age fails to achieve ego integrity, and instead experiences despair

D. The adult who is unfulfilled at an advanced age fails to achieve ego integrity, and instead experiences despair

A client with a large pleural effusion undergoes a thoracentesis. Following the procedure, which observation warrants immediate intervention by the nurse? A. The client complains of pain at the insertion site B. The client's chest x-ray indicates decreased pleural effusion C. The client's arterial blood gases are pH 7.35; PaO2 85; PaCO2 35; HCO3 26 D. The client has asymmetrical chest wall expansion

D. The client has asymmetrical chest wall expansion

The nurse in the outpatient department is caring for a client who had a right femoral cardiac catheterization two hours ago. What assessment finding requires immediate intervention? A. The client's right foot is warm to touch B. The client's blood pressure is 110/70 and pulse 88 C. The client's pulse oximeter reading is 98% D. The client wants assistance walking to the bathroom

D. The client wants assistance walking to the bathroom

The nurse assesses a patient with shortness of breath for evidence of long-standing hypoxemia by inspecting: A. Chest excursion B. Spinal curvatures C. The respiratory pattern D. The fingernail and its base

D. The fingernail and its base Clubbing, a sign of long-standing hypoxemia, is evidenced by an increase in the angle between the base of the nail and the fingernail to 180 degrees or more, usually accompanied by an increase in the depth, bulk, and sponginess of the end of the finger.

Twelve hours after chest tube insertion for hemothorax, the nurse notes that the client's drainage has decreased from 50 ml/hr to 5 ml/hr. What is the best inital action for the nurse to take? A. Document this expected decrease in drainage. B. Clamp the chest tube while assessing for air leaks. C. Milk the tube to remove any excessive blood clot build up. D. Assess for kinks or dependent loops in the tubing.

D. The least invasive action should be performed to assess the decrease in drainage. (A) is completed after assessing for and problems causing the decreased drainage. (B) is no longer protocol because the increased pressure may be harmful for the client. (C) is an appropriate nursing action after the tube has been assessed for kinks or dependent loops.

An older client is admitted with a diagnosis of bacterial pneumonia. The nurse's assessment of the client will most likely reveal which S/SX? A. Leukocytosis and febrile. B. Polycythemia and crackles. C. Pharyngitis and sputum production. D. Confusion and tachycardia.

D. The onset of pneumonia is the older may be signaled by general deterioration, confusion, increased heart rate or increased respiratory rate. (A, B, C) are often absent in the older with bacterial pneumonia.

A frail, elderly female with rheumatoid arthritis (RA) complains to the nurse that the weight of the sheets on her legs hurts all the time. Which action should the nurse implement? A. Soak her hands in warm water when resting B. Administer an analgesic at the hour of sleep C. Provide a soft blanket for covering the client D. Use a bed cradle to keep linens off her legs

D. Use a bed cradle to keep linens off her legs

After a 92-year-old client fractured a hip trying to get out of bed, a nurse is accused of failing to notify the healthcare provider that the client was disoriented. In determining whether the nurse is guilty, a jury would consider which standard? A. What the nurse was taught in school about similar client care situations B. What an experienced lawyer would advise to be done in a similar situation C. What a well-educated healthcare consumer would expect in the same situation D. What a reasonable and prudent nurse would have done in the same situation

D. What a reasonable and prudent nurse would have done in the same situation

The nurse is completing a neurological assessment. What observation indicates an abnormal pupil response? A. When shinning the light into the eye, the pupil contracts briskly B. The optic disc appears edematous and engorged C. As the nurse's finger is brought in closer to the eye, the pupils contract D. When shining the light into the right eye, the left pupil does not constrict

D. When shining the light into the right eye, the left pupil does not constrict

25. The nurse identifies the nursing diagnosis of activity intolerance for a patient with asthma. The nurse assesses for which of the following etiologic factor for this nursing diagnosis in patients with asthma? A. Anxiety and restlessness B. Effects of medications C. Fear of suffocation D. Work of breathing

D. Work of breathingWhen the patient does not have sufficient gas exchange to engage in activity, the etiologic factor is often the work of breathing. When patients with asthma do not have effective respirations, they use all available energy to breathe and have little left over for purposeful activity.

The nurse identifies the nursing diagnosis of activity intolerance for a patient with asthma. The nurse assesses for which of the following etiologic factor for this nursing diagnosis in patients with asthma? A. Anxiety and restlessness B. Effects of medications C. Fear of suffocation D. Work of breathing

D. Work of breathingWhen the patient does not have sufficient gas exchange to engage in activity, the etiologic factor is often the work of breathing. When patients with asthma do not have effective respirations, they use all available energy to breathe and have little left over for purposeful activity.

A client presents at the clinic with blepharitis. What instruction should the nurse provide for home care? A. use bilateral eye patches while sleeping to prevent injury to eyes B. wear sunglasses when out of doors to prevent photophobia C. apply cool moist compresses for 20 minutes followed by warm moist compresses D. apply warm moist compresses then gently scrub eyelids with dilute baby shampoo

D. apply warm moist compresses then gently scrub eyelids with dilute baby shampoo

21. To prevent atelectasis in an 82-year-old patient with a hip fracture, a nurse should A. supply oxygen. B. suction the upper airway. C. ambulate the patient frequently. D. assist the patient with aggressive coughing and deep breathing.

D. assist the patient with aggressive coughing and deep breathing. Decreased mobility after surgery in older adults creates the possibility of fluid buildup and retention in lung tissue. One of the primary goals of nursing intervention is to prevent atelectasis in a high-risk patient. Aggressive coughing and deep breathing can prevent atelectasis in the postoperative patient.

To prevent atelectasis in an 82-year-old patient with a hip fracture, a nurse should A. supply oxygen. B. suction the upper airway. C. ambulate the patient frequently. D. assist the patient with aggressive coughing and deep breathing.

D. assist the patient with aggressive coughing and deep breathing. Decreased mobility after surgery in older adults creates the possibility of fluid buildup and retention in lung tissue. One of the primary goals of nursing intervention is to prevent atelectasis in a high-risk patient. Aggressive coughing and deep breathing can prevent atelectasis in the postoperative patient.

4. After a posterior nasal pack is inserted by a physician, the patient is very anxious and states, "I don't feel like I'm breathing right." The immediate intervention the nurse should initiate is to A. monitor ABGs. B. reassure the patient that this is normal discomfort. C. cut the pack strings and pull the packing out with a hemostat. D. direct a flashlight into the patient's mouth and inspect the oral cavity.

D. direct a flashlight into the patient's mouth and inspect the oral cavity. The nurse should inspect the oral cavity for the presence of blood, soft palate necrosis, and proper placement of the posterior plug. If the posterior plug is visible, the physician should be notified for readjustment of the packing. Reassurance, cutting the strings, and ABGs are not top priority interventions. The nurse needs further data before intervening.

After a posterior nasal pack is inserted by a physician, the patient is very anxious and states, "I don't feel like I'm breathing right." The immediate intervention the nurse should initiate is to A. monitor ABGs. B. reassure the patient that this is normal discomfort. C. cut the pack strings and pull the packing out with a hemostat. D. direct a flashlight into the patient's mouth and inspect the oral cavity.

D. direct a flashlight into the patient's mouth and inspect the oral cavity. The nurse should inspect the oral cavity for the presence of blood, soft palate necrosis, and proper placement of the posterior plug. If the posterior plug is visible, the physician should be notified for readjustment of the packing. Reassurance, cutting the strings, and ABGs are not top priority interventions. The nurse needs further data before intervening.

10. Anticoagulant therapy is used in the treatment of thromboembolic disease because anticoagulants can A. dissolve the thrombi. B. decrease blood viscosity. C. prevent absorption of vitamin K. D. inhibit the synthesis of clotting factors.

D. inhibit the synthesis of clotting factors. Anticoagulant therapy is based on the premise that the initiation or extension of thrombi can be prevented by inhibiting the synthesis of clotting factors or by accelerating their inactivation. The anticoagulants heparin and warfarin do not induce thrombolysis but effectively prevent clot extension.

Anticoagulant therapy is used in the treatment of thromboembolic disease because anticoagulants can A. dissolve the thrombi. B. decrease blood viscosity. C. prevent absorption of vitamin K. D. inhibit the synthesis of clotting factors.

D. inhibit the synthesis of clotting factors. Anticoagulant therapy is based on the premise that the initiation or extension of thrombi can be prevented by inhibiting the synthesis of clotting factors or by accelerating their inactivation. The anticoagulants heparin and warfarin do not induce thrombolysis but effectively prevent clot extension.

The practical nurse (PN) reports to the charge nurse that the unlicensed assistive personal (UAP) did not adhere to the agency's fall prevention protocols when caring for a client at risk for falls. What action should the charge nurse implement? A. encourage the PN to complete an adverse occurrence report B. instruct the PN to supervise the UAP more close the next day C. plan to assign the UAP to more stable clients the next day D. meet with the UAP to discuss the observations made by the PN

D. meet with the UAP to discuss the observations made by the PN

An adult is brought to the emergency department by ambulance following a motorcycle accident. He was not wearing a helmet and presents with periorbital bruising and bloody drainage from both ears. Which assessment finding warrants immediate intervention by the nurse? A. rebound abdominal tenderness B. diminished bilateral breath sounds C. rub pain with deep inspiration D. nausea with projectile vomiting

D. nausea with projectile vomiting

31. A patient's ABGs include a PaO2 of 88 mm Hg and a PaCO2 of 38 mm Hg and mixed venous blood gases include a PvO2 of 40 mm Hg and PvCO2 of 46 mm Hg. These findings indicate that the patient has A. impaired cardiac output. B. unstable hemodynamics. C. inadequate delivery of oxygen to the tissues. D. normal capillary oxygen-carbon dioxide exchange.

D. normal capillary oxygen-carbon dioxide exchange. Normal venous blood gas values reflect the normal uptake of oxygen from arterial blood and the release of carbon dioxide from cells into the blood, resulting in a much lower PaO2 and an increased PaCO2. The pH is also decreased in mixed venous blood gases because of the higher PvCO2. Normal mixed venous blood gases also have much lower PvO2 and SvO2 than arterial blood bases. Mixed venous blood gases are used when patients are hemodynamically unstable to evaluate the amount of oxygen delivered to the tissue and the amount of oxygen consumed by the tissues.

A patient's ABGs include a PaO2 of 88 mm Hg and a PaCO2 of 38 mm Hg and mixed venous blood gases include a PvO2 of 40 mm Hg and PvCO2 of 46 mm Hg. These findings indicate that the patient has A. impaired cardiac output. B. unstable hemodynamics. C. inadequate delivery of oxygen to the tissues. D. normal capillary oxygen-carbon dioxide exchange.

D. normal capillary oxygen-carbon dioxide exchange. Normal venous blood gas values reflect the normal uptake of oxygen from arterial blood and the release of carbon dioxide from cells into the blood, resulting in a much lower PaO2 and an increased PaCO2. The pH is also decreased in mixed venous blood gases because of the higher PvCO2. Normal mixed venous blood gases also have much lower PvO2 and SvO2 than arterial blood bases. Mixed venous blood gases are used when patients are hemodynamically unstable to evaluate the amount of oxygen delivered to the tissue and the amount of oxygen consumed by the tissues.

7. The most appropriate position to assist a patient with chronic obstructive pulmonary disease (COPD) who is having difficulty breathing would be a A. high Fowler's position without a pillow behind the head. B. semi-Fowler's position with a single pillow behind the head. C. right side-lying position with the head of the bed at 45 degrees' elevation. D. sitting upright and forward position with arms supported on an over-the-bed table.

D. sitting upright and forward position with arms supported on an over-the-bed table.Sitting upright and leaning forward with arms supported on an over-the-bed table would be of most help to this patient, because it allows for expansion of the thoracic cage in all four directions (front, back, and two sides).

The most appropriate position to assist a patient with chronic obstructive pulmonary disease (COPD) who is having difficulty breathing would be a A. high Fowler's position without a pillow behind the head. B. semi-Fowler's position with a single pillow behind the head. C. right side-lying position with the head of the bed at 45 degrees' elevation. D. sitting upright and forward position with arms supported on an over-the-bed table.

D. sitting upright and forward position with arms supported on an over-the-bed table.Sitting upright and leaning forward with arms supported on an over-the-bed table would be of most help to this patient, because it allows for expansion of the thoracic cage in all four directions (front, back, and two sides).

15. When administering oxygen to a patient with COPD with the potential for carbon dioxide narcosis, the nurse should A. never administer oxygen at a rate of more than 2 L/min. B. monitor the patient's use of oxygen to detect oxygen dependency. C. monitor the patient for symptoms of oxygen toxicity, such as paresthesias. D. use ABGs as a guide to determine what FIO2 level meets the patient's needs.

D. use ABGs as a guide to determine what FIO2 level meets the patient's needs. It is critical to start oxygen at low flow rates and then use ABGs as a guide to determine what FIO2 level is sufficient and can be tolerated.

When administering oxygen to a patient with COPD with the potential for carbon dioxide narcosis, the nurse should A. never administer oxygen at a rate of more than 2 L/min. B. monitor the patient's use of oxygen to detect oxygen dependency. C. monitor the patient for symptoms of oxygen toxicity, such as paresthesias. D. use ABGs as a guide to determine what FIO2 level meets the patient's needs.

D. use ABGs as a guide to determine what FIO2 level meets the patient's needs. It is critical to start oxygen at low flow rates and then use ABGs as a guide to determine what FIO2 level is sufficient and can be tolerated.

Diagnostic findings in the diuretic phase

Decreased fluid volume decrease potassium further decrease in sodium low urine specific gravity <1.020

2nd° burn

Deep partial thickness destruction of epidermis and upper layers of dermis, injury to deeper portions of the dermis, painful, appears red or white, weeps fluid, blisters present, hair follicles intact, very edematous, blanching followed by capillary refill, heals without surgical intervention

How does the nurse prevent hypoxia during suctioning?

Deliver 100% O2 (Hyperinflating) before andafter each endotracheal suctioning

Opiate drugs

Depress gastric motility given with caution. Assess for abdominal distention, pain, rigidity. S/S of shock: increased heart rate decrease BP

Hypernatremia: causes

Diabetes insipidus heatstroke hyperventilation renal failure Cushing syndrome

Signs and symptoms of ulcerative colitis

Diarrhea abdominal pain intermittent tenesmus rectal bleeding liquid stools: may pass 10 to 20 liquid stools per day weakness fatigue

Hypocalcemia: signs and symptoms

Diarrhea numbness tingling of extremities positive trousseau sign Ca <8.5

Hypokalemia: causes

Diarrhea vomiting gastric suction hyperaldosteronism bulimia Cushing syndrome

Diagnostic test for colorectal cancer

Digital examination, flexible fiber optic sigmoidoscopy with biopsy colonoscopy Barium enema

American Cancer Society recommendations for early detection

Digital rectal exam every year after 40. stool blood test every year after 50. colonoscopy examination every 10 years after the age of 50.

Nursing plan for chronic pancreatitis

Dilaudid/fentanyl/morphine as needed. Bland, low fat diet. Admin pancreatic enzymes: pancreatin or pancrelipase with meals.

HESI HINT

Diverticulosis is the presence of patches in the wall of the intestine. diverticulitis is an inflammation of the diverticula (pouches)

Bowel obstruction: mechanical

Due to disorders outside the bowl caused by disorders within the bowl or by blockage of the lumen in the intestine

Symptoms of Acute Prostatitis

Dysuria Frequency Urgency CLOUDY URINE Retention caused by Swelling Post-Ejaculatory Pain

Hyperkalemia

ECG=tall spiked T wave, prolonged QT intervial, widening QRS complex are all signs of hyperkalemia; tumor lysis syndrome

How to determine presence and location of peptic ulcer's

EGD barium swallow gastric analysis indicating increased levels of stomach acid

Transitioning through young adulthood is characterized by

Establishing independence Completing education Beginning a career Coming self-supporting

The most common cause of secondary HTN in women is use of what type of drug:

Estrogen-Containing Oral Contraceptives

Cushing syndrome

Excess adrenocorticoid activity. Also caused by adrenal, pituitary or hypothalamus tumors

This is a normal disturbing symptom of menopause and postmenopause:

Excessive diaphoresis

Ways to diagnose DM

Fasting plasma glucose greater than 126 HBA1c greater than 6.5% random blood glucose greater than 200 OGTT greater than 200

Manifestations of Addison's Disease

Fatigue Weight Loss Postural Hypotension Salt Craving High Potassium Hyperpigmentation

Hypokalemia: signs and symptoms

Fatigue anorexia nausea muscle weakness dysrhythmias K <3.5

Signs and symptoms of Addison disease

Fatigue weight-loss anorexia postural hypotension hypoglycemia hyponatremia hyperkalemia hyperpigmentation loss of body hair hypovolemia

Can cause dehydration because of excessive fluid loss due to diaphoresis

Fever

Increases metabolism and the demand for oxygen

Fever; Increased Temperature

Location of pain to determine location of stone

Flank pain means the stone is in the kidney or upper ureter. Pain radiating to the abdomen stone is likely to be in the ureter or bladder

Eye _______ can develop with myopia; however if, acute in onset, they may indicate retinal detachment

Floaters

diagnostic test for hernia and gerd

Fluoroscopy barium swallow Gastroscopy

Cholelithiasis

Formation or presence of stones in the gallbladder

3rd° burn

Full thickness and deep full thickness involves total destruction of dermis and epidermis, skin cannot regenerate, require skin grafting, underlying tissue may be involved, wound appears dry and leathery as eschar develops, painless

Long acting insulin

Glargine (Lantus) Detemir (Levemir)

This finding would support the suspected diagnosis of a left-sided brain attack

Global Aphasia

What labs are imperative to monitor if a patient is receiving an albumin infusion?

H&H

Reflects the amount of circulating RBCs in periphery

HBG

The ELISA and Western Blot tests are used to identify the presence of

HIV

If a patient's K+ level is low, the nurse would do what with a prescription for Digitalis?

HOLD IT

The presence of ________ can indicated narrow-angle glaucoma

Halos

Hypothyroidism

Hashimoto, myxedema. Hypofunction of the thyroid gland resulting in insufficiency of thyroid hormone

Typical triad of symptoms of tumors on the adrenal medulla

Headache Diaphoresis Palpitations

Signs and symptoms of hypoglycemia

Headache nausea sweating Tremors lethargy hunger confusion slurred speech tingling around mouth

Dietary interventions with burn clients

High calorie, high protein, high carb, no free water, tube feeding at night, weigh daily

Diabetes mellitus

High levels of glucose resulting from defects in insulin secretion, insulin action or both

Diet requirements for Addison's disease

High sodium low potassium high carb

Diet for colorectal cancer

High-fiber low-fat foods Cruciferous vegetables (broccoli, cauliflower, cabbage, kale)

Corticosteroids

Hydrocortisone Prednisone Dexamethasone Medrol

Diabetes mellitus is characterized by

Hyper glycemia

Hyperosmolar hyperglycemia nonketotic syndrome characteristics

Hyperglycemia greater than 600, plasma hyperosmolarity, dehydration, changed mental status, absent ketone bodies

Lab data for Cushing's

Hyperglycemia hypernatremia hypokalemia increased plasma cortisol Decreased eosinophils and lymphocytes

Assess for ________ for patients taking potassium-sparing diuretics (spironolactone)

Hyperkalemia

Hypercalcemia: causes

Hyperparathyroidism prolonged immobilization excess calcium supplementation

Addison disease

Hypo function of the adrenal cortex. Sudden withdrawal from corticosteroids. characterized by lack of cortisol, aldosterone.

Respiratory acidosis: cause

Hypo ventilation COPD pulmonary disease obesity sleep apnea

Cold and clammy, need some candy!

Hypoglycemia

The most frequent side effect associated with thiazide and loop diuretics is

Hypokalemia

__________ can precipitate digitalis toxicity

Hypokalemia

Signs and symptoms of myxedema coma

Hypotension Hypothermia Hyponatremia Hypoglycemia Respiratory failure

The symptoms of this condition generally result from the slowing down of body systems

Hypothyroidism

may experience personality changes, weight gain, hoarseness, hair loss, anemia, SOB on exertion, low exercise tolerance

Hypothyroidism

What should be considered when T-wave inversion is present?

Hypoxia

Focus of PID treatment

Identify the cause of the pelvic pain

Diagnostic findings in oliguric phase

Increased BUN and creatinine increased potassium decrease sodium decreased pH fluid overload high urine specific gravity >1.020

Possibly expected lab values of an elderly person would include:

Increased protein in urine Slightly increased BS

This vaccine is usually administered once a year

Influenza

When a patient is receiving an albumin infusion, these measure should be taken:

Infuse within 4 hours Use large gauge catheter Monitor H&H Assess for increased bleeding afterwards

-graph

Instrument to record.

-scope

Instrument to visually examine.

Viscera

Internal organs.

If you are caring for a patient taking Lasix, you will probably notice the patient is also taking something like,

K-Dur, a potassium supplement (Because Lasix is potassium-wasting thiazide diuretic)

How should a patient be instructed to help identify the cause of pelvic pain causing PID?

Keep a journal of symptoms and onset factors

Topical substances used to break down hardened skin and shed the top layer of dead skin to treat warts, calluses, corns, acne, and psoriasis

Keratolytics

Pain in the neck when the thigh is flexed on the abdomen and the leg extended at the knee

Kernig's Sign

PPI's

Lansolrazole (Prevacid) Pantoprazole (protonix) Omeprazole (Prilosec)

Ways to fix PID pelvic pain issues

Laparoscopic Surgery Hysterectomy (last resort)

This is needed to definitively diagnose endometriosis

Laparoscopy

Signs and symptoms of diverticular diseases

Left lower quadrant pain Increased flatus rectal bleeding

Location of gastric ulcers

Lesser curvature of the stomach

Thyroid storm

Life threatening symptoms include fever, tachycardia, agitation, anxiety and hypertension. Maintain an airway.

Cause of chronic pancreatitis

Long term alcohol use

Thrombocytopenia

Low platelets (below 150,000) Bleeding is a huge risk

Diet for Crohn's disease

Low residue Low fat High protein high calorie diet with no dairy products

Diet for ulcerative colitis

Low residue low-fat high-protein high calorie no dairy products

Diet for cirrhosis

Low sodium low potassium low fat high carb

Diet for chronic renal failure

Low-protein low sodium low potassium low phosphate

Diet for Cushing's

Low-sodium, encourage consumption of foods that contain vitamin D and calcium

Action of alpha glucose inhibitors

Lowers blood glucose by blunting sugar levels after meals

Action of sulfonylureas

Lowers blood sugar by stimulating the release of insulin by the beta cells of the pancreas and causes tissues to take up and store glucose more easily

Action of biguanides

Lowers serum glucose levels by inhibiting hepatic glucose production and increasing sensitivity of peripheral tissue to insulin

RBC's

M: 4.7 - 6.1 F: 4.2 - 5.4

Hematocrit:

M: 42 - 52 F: 37 - 47

Nursing plan for acute pancreatitis

Maintain NPO status Maintain NG suction Dilaudid/fentanyl as needed Antacids, histamine H2 receptor blocking drugs, anticholingerics, PPI Monitor for hypocalcemia

Sarcoma

Malignant tumor of flesh tissue.

Breast Cancer

Malignant, Stationary Hard, irregular shape Poorly delineated Nipple discharge (not always) Nipple retraction Dimpling, Infiltration, Induration Peau d'orange

If a patient develops lymphedema, these are indicated as interventions:

Massage technique to mobilize fluids Compression bandage Elevate arm level with heart

Signs and symptoms of Cushing's

Moon face truncal obesity Buffalo hump abdominal striae thinning of skin hyperpigmentation Hirsutism in females bruises easily hypertension

Manifestations of Cushing Syndrome

Moon-face, Buffalo Hump Truncal/Android Obesity Purplish Striae Thin, Fragile Skin Florid Cheeks, Acne Poor Wound Healing LE Edema Hyperglycemia

Stoma care

More distal the stoma is the greater the chance of continence. The lower the stomas location is in the G.I. tract the more solid the stool.

Hypercalcemia: signs and symptoms

Muscle weakness constipation anorexia polyuria polydipsia dysrhythmias Ca >10.5

Diaphragm

Muscular wall separating the abdominal and thoracic cavities.

What am I describing here: Extreme muscular weakness Easy fatigability Ptosis

Myasthenia Gravis

The extreme state of hypothyroidism

Myxedema

It may be required for a SBO patient to receive an

NG Tube for decompression

For patients with perforation, these nursing interventions are very important:

NG Tube to drain blood LR and albumin Blood Replacement CVP Monitoring Urinary Catheter Heart Monitoring

A client admitted with complaints of constipation, thready stools and rectal bleeding over the past few months is diagnosed with a rectal mass. what are the nursing priorities for this client?

NPO NG tube IV fluids surgical preparations of bowel foods and fluids are restricted for 8 to 10 hours before surgery If patient has bowel obstruction, bowel cleansing is contraindicated. Oral erythromycin and neomycin are given. All clients who require surgery for obstruction undergo NG intubation and suction before surgery.

How does an MCL I get placed on the body?

Negative polarity on L shoulder Positive polarity on R chest nipple line Ground on left chest nipple line

How to Avoid Lymphedema

No BP readings on that arm No sticks, scrapes, bites, etc. on arm No dependent position of arm

When a cancer patient is at risk for bleeding, due to low platelet count, the following measures/precautions should be taken:

No plucking of body hair No straight-edge razors Avoid aspirin Restrict weight-lifting/vigorous exercise Don't bend head lower than waist Use soft-bristle toothbrush Avoid Sub-Q injections Absolutely no IM injections

Signs and symptoms of BPH

Nocturia hesitancy terminal dribbling decrease in size & force of stream increased frequency of voiding with decrease amount

Pancreatitis

Nonbacterial inflammation of the pancreas

Isotonic IV solution

Normal saline Lactated ringers 5% dextrose in water

Fibrocystic Changes

Not usually discreet Moveable and soft

The less predictive, but notable risk factors for breast cancer risk are:

Nulliparity Never breastfed before Early menarche Late menopause

Location of duodenal ulcers

Occur in the duodenum. Most common location of peptic ulcer disease

Rehabilitation phase

Occurs from Major wound closure to return to optimal level of physical and psychosocial adjustment. Characterized by grafting and rehabilitation specific to the client.

Acute phase: 48-72 hrs after injury

Occurs from beginning of diuresis to near completion of wound closure. Characterized by fluid shift from interstitial to intravascular. Focus on infection control wound care & closure, pain management, nutritional support, physical therapy

Hyperkalemia: causes

Oliguria acidosis renal failure Addison's disease

Three phases of acute renal failure or

Oliguric Diuretic Recovery

Drugs for thyroid storm

PTU (propylthiouracil) Tapazole (methimazole)

Otalgia

Pain in the ear.

Thrombophlebitis is characterized by this type of pain:

Pain upon exertion, which is relieved by rest and elevating the extremity

Bowel obstruction: nonmechanical

Paralytic ileus which does not involve any actual physical obstruction but results from inability of bowel itself to function

Hepatitis C: route of infection

Parenteral Sexual contact

Hepatitis B: route of infection

Parenteral, oral, fecal, direct contact, sexual contact

Exacerbations of MS

Paresthesias Diplopia Nystagmus

What am I describing here: Shuffling Gait Mask-like Facial Expression Tremors of Head

Parkinson's Disease

levodopa (Sinemet)

Parkinsons Disease lessen tremors increases amount of levodopa to CNS (dopamine to the brain) s/s toxicity=dyskinesia, hallucinations, psychosis

Mitochondria

Part of a cell where catabolism primarily occurs.

Endoplasmic Reticulum

Part of the cell where formation of proteins occurs.

Intestinal obstruction

Partial or complete blockage of intestinal flow that occurs mostly in the small intestines

Urinary tract obstruction

Partial or complete blockage of the flow of urine at any point in the urinary system

Long acting: peak

Peak less

Histologist

Person who studies tissues.

Adipose

Pertaining to fat.

Catecholamine-producing tumor that arises in the adrenal medulla

Pheochromocytoma

Originated most commonly in the adrenal medulla and result in excessive secretion of catecholamines, resulting in life-threatening high BP

Pheochromocytomas

Karyotype

Picture of nuclear structures arranged in numerical order.

Barrel chest is indicative of EMPHYSEMA and is caused by use of accessory muscles to breathe. The person works harder to breathe, but the amount of O2 taken in is adequate to oxygenate the tissues

Pink Puffer

This vaccine is administered ONCE to older adults, usually over 65, with a history of chronic illness

Pneumovax

HESI hint

Potassium affects the heart. Limit high potassium foods and salt substitutes. Limit fluid and sodium intake in acute renal failure pts

Three major types of acute renal failure

Prerenal-interference with renal perfusion. Intrarenal-damage to renal parenchyma Postrenal-obstruction in the urinary track anywhere from the tubules to the urethral meatus

Chronic pancreatitis

Progressive destructive disease that causes permanent dysfunction

Chronic renal failure: end stage renal disease

Progressive, irreversible damage to the nephrons and glomeruli, resulting in uremia

-ptosis

Prolapse

THE GFR is most often used as an indicator of the level of ___________ consumption

Protein

Nursing plans for acute phase

Provide infection control, no live plants or flowers in room, perform ROM, prevent contractures, provide fluid therapy, provide adequate nutrition ( up to 5000 calories per day)

Myxedema manifests as:

Puffiness, facial edema Mask-like affect Mental sluggishness, subnormal temp, hypotension, hypoventilation

A tornado warning alarm has been activated at the local hospital. Which action should the charge nurse working on a surgical unit implement first? A. Instruct the nursing staff to close all window blinds and curtains in clients' rooms. B. Move clients and visitors into the hallways and close all doors to clients' rooms. C. Visually confirm the location of the tornado by checking the windows on the unit. D. Assist all visitors with evacuation down the stairs in a calm and orderly manner.

Rationale: Rational: B In the event of a tornado, all persons should be moved into the hallways, away from windows, to prevent flying debris from causing injury (B). Although (A) may help decrease the amount of flying debris, it is not safe to leave clients in rooms with closed blinds; (B) is a higher priority at this time. Hospital staff should stay away from windows to avoid injury and should focus on client evacuation into hallways rather than (C). (D) is not the first action that should be taken.

A client is diagnosed with an acute small bowel obstruction. Which assessment finding requires the most immediate intervention by the nurse? A. Fever of 102° F B. Blood pressure of 150/90 mm Hg C. Abdominal cramping D. Dry mucous membranes

Rationale: Rationale: A A sudden increase in temperature is an indicator of peritonitis. The nurse should notify the health care provider immediately (A). (B, C, and D) are also findings that require intervention by the nurse, but are of less priority than (A). (B) may indicate a hypertensive condition but is not as acute a condition as peritonitis. (C) is an expected finding in clients with small bowel obstruction and may require medication. (D) indicates probable fluid volume deficit, which requires fluid volume replacement.

Which content about self-care should the nurse include in the teaching plan of a female client who has genital herpes? (Select all that apply.) A. Encourage annual physical and Pap smear. B. Take antiviral medication as prescribed. C. Use condoms to avoid transmission to others. D. Warm sitz baths may relieve itching. E. Use Nystatin suppositories to control itching. F. Use a douche with weak vinegar solution to decrease itching.

Rationale: The nurse should include (A, B, C, and D) in the teaching plan of a female client with genital herpes. (E) is specific for Candida infections, and (F) is used to treat Trichomonas.

An 81-year-old male client has emphysema. He lives at home with his cat and manages self-care with no difficulty. When making a home visit, the nurse notices that this client's tongue is somewhat cracked and his eyeballs appear sunken into his head. Which nursing intervention is indicated? A. Help the client determine ways to increase his fluid intake. B. Obtain an appointment for the client to have an eye examination. C. Instruct the client to use oxygen at night and increase the humidification. D. Schedule the client for tests to determine his sensitivity to cat hair.

Rationale: A Clients with COPD should ingest 3 L of fluids daily but may experience a fluid deficit because of shortness of breath. The nurse should suggest creative methods to increase the intake of fluids (A), such as having fruit juices in disposable containers readily available. (B) is not indicated. Humidified oxygen will not effectively treat the client's fluid deficit, and there is no indication that the client needs supplemental oxygen at night (C). These symptoms are not indicative of (D) and may unnecessarily upset the client, who depends on his pet for socialization.

During the shift report, the charge nurse informs a nurse that she has been assigned to another unit for the day. The nurse begins to sigh deeply and tosses about her belongings as she prepares to leave, making it known that she is very unhappy about being floated to the other unit. What is the best immediate action for the charge nurse to take? A. Continue with the shift report and talk to the nurse about the incident at a later time. B. Ask the nurse to call the house supervisor to see if she must be reassigned. C. Stop the shift report and remind the nurse that all staff are floated equally. D. Inform the nurse that her behavior is disruptive to the rest of the staff.

Rationale: A Continuing with the shift report (A) is the best immediate action because it allows the nurse who was floated some cooling off time. At a later time (after the nurse has cooled off) the charge nurse should discuss the conduct of the nurse in private. (B) encourages the nurse to shirk the float assignment. (C) is disruptive. Reprimanding the nurse in front of the staff would increase the nurse's hostility, so the nurse should be counseled in private (D).

Based on the clinical manifestations of Cushing's syndrome, which nursing intervention would be appropriate for a client who is newly diagnosed with Cushing's syndrome? A. Monitor blood glucose levels daily. B. Increase intake of fluids high in potassium. C. Encourage adequate rest between activities. D. Offer the client a sodium-enriched menu.

Rationale: A Cushing's syndrome results from a hypersecretion of glucocorticoids in the adrenal cortex. Clients with Cushing's syndrome often develop diabetes mellitus. Monitoring of serum glucose levels (A) assesses for increased blood glucose levels so that treatment can begin early. A common finding in Cushing's syndrome is generalized edema. Although potassium is needed, it is generally obtained from food intake, not by offering potassium-enhanced fluids (B). Fatigue is usually not an overwhelming factor in Cushing's syndrome, so an emphasis on the need for rest (C) is not indicated A low-calorie, low-carbohydrate, low-sodium diet is not recommended (D).

A 63-year-old client with type 2 diabetes mellitus is admitted for treatment of an ulcer on the heel of the left foot that has not healed with wound care. The nurse observes that the entire left foot is darker in color than the right foot. Which additional symptom should the nurse expect to find? A. Pedal pulses will be weak or absent in the left foot. B. The client will state that the left foot is usually warm. C. Flexion and extension of the left foot will be limited. D. Capillary refill of the client's left toes will be brisk.

Rationale: A Symptoms associated with decreased blood supply are weak or absent pedal and tibial pulses (A). The client with diabetes experiences vascular scarring as a result of atherosclerotic changes in the peripheral vessels. This results in compromised perfusion to the dependent extremities, which further delays wound healing in the affected foot. Although flexion and extension may be limited (C), depending on the degree of damage, this is not always the case. (B and D) are signs of adequate perfusion of the foot, which would not be expected in this client.

The nurse is observing an unlicensed assistive personnel (UAP) performing morning care for a bedridden client with Huntington's disease. Which care measure is most important for the nurse to supervise? A. Oral care B. Bathing C. Foot care D. Catheter care

Rationale: A The client with Huntington's disease experiences problems with motor skills such as swallowing and is at high risk for aspiration, so the highest priority for the nurse to observe is the UAP's ability to perform oral care safely (A). (B, C, and D) do not necessarily require registered nurse (RN) supervision because they do not ordinarily pose life-threatening consequences.

A client with hypertension has been receiving ramipril (Altace), 5 mg PO, daily for 2 weeks and is scheduled to receive a dose at 0900. At 0830, the client's blood pressure is 120/70 mm Hg. Which action should the nurse take? A. Administer the prescribed dose at the scheduled time. B. Hold the dose and contact the health care provider. C. Hold the dose and recheck the blood pressure in 1 hour. D. Check the health care provider's prescription to clarify dose.

Rationale: A The client's blood pressure is within normal limits, indicating that the ramipril, an antihypertensive, is having the desired effect and should be administered (A). (B and C) would be appropriate if the client's blood pressure was excessively low (<100 mm Hg systolic) or if the client were exhibiting signs of hypotension such as dizziness. This prescribed dose is within the normal dosage range, as defined by the manufacturer; therefore, (D) is not necessary.

The nurse is preparing a teaching plan for a group of healthy adults. Which individual is most likely to maintain optimum health? A. A teacher whose blood glucose levels average 126 mg/dL daily with oral antidiabetic drugs B. An accountant whose blood pressure averages 140/96 mm Hg and who says he does not have time to exercise C. A stock broker whose total serum cholesterol level dropped to 290 mg/dL with diet modifications D. A recovering IV heroin user who contracted hepatitis more than 10 years ago

Rationale: A The diabetic teacher (A) has assumed responsibility for self-care, so among those listed, is the most likely to maintain optimum health. (B) has expressed a lack of interest in health promotion. (C) continues to demonstrate a high-risk cholesterol level despite a reported attempt at dietary modifications. Previous IV drug use and a history of hepatitis (D) make this individual a health risk despite the fact that the individual is in recovery.

A client diagnosed with angina pectoris complains of chest pain while ambulating in the hallway. Which action should the nurse implement first? A. Support the client to a sitting position. B. Ask the client to walk slowly back to the room. C. Administer a sublingual nitroglycerin tablet. D. Provide oxygen via nasal cannula.

Rationale: A The nurse should safely assist the client to a resting position (A) and then perform (C and D). The client must cease all activity immediately, which will decrease the oxygen requirement of the myocardial muscle. After these interventions are implemented, the client can be escorted back to the room via wheelchair or stretcher (B).

In caring for a client with acute diverticulitis, which assessment data warrants immediate nursing intervention? A. The client has a rigid hard abdomen and elevated WBC. B. The client has left lower quadrant pain and an elevated temperature. C. The client is refusing to eat any of the meal and is complaining of nausea. D. The client has not had a bowel movement in 2 days and has a soft abdomen.

Rationale: A* A hard rigid abdomen and elevated WBC is indicative of peritonitis (A), which is a medical emergency and should be reported to the health care provider immediately. (B and C) are expected clinical manifestations of diverticulitis. (D) does not warrant immediate intervention.

The nurse is administering a nystatin suspension (Mycostatin) for stomatitis. Which instruction will the nurse provide to the client when administering this medication? A. "Hold the medication in your mouth for a few minutes before swallowing it." B. "Do not drink or eat milk products for 1 hour prior to taking this medication." C. "Dilute the medication with juice to reduce the unpleasant taste and odor." D. "Take the medication before meals to promote increased absorption."

Rationale: A* Mycostatin is prescribed for fungal infections of the mouth. The client should swish the medication in the mouth for 2 minutes and then swallow (A). (B) does not affect administration of this medication. The medication should not be diluted because this will reduce its effectiveness (C). (D) is not necessary.

During assessment of a client in the intensive care unit, the nurse notes that the client's breath sounds are clear on auscultation, but jugular vein distention and muffled heart sounds are present. Which intervention should the nurse implement? A. Prepare the client for a pericardial tap. B. Administer intravenous furosemide (Lasix). C. Assist the client to cough and breathe deeply. D. Instruct the client to restrict the oral fluid intake.

Rationale: A* The client is exhibiting symptoms of cardiac tamponade, a collection of fluid in the pericardial sac that results in a reduction in cardiac output, which is a potentially fatal complication of pericarditis. Treatment for tamponade is a pericardial tap (A). Lasix IV is not indicated for treatment of pericarditis (B). Because the client's breath sounds are clear, (C) is not a priority. Fluids are frequently increased (D) in the initial treatment of tamponade to compensate for the decrease in cardiac output, but this is not the same priority as (A).

An emaciated homeless client presents to the emergency department complaining of a productive cough, with blood-tinged sputum and night sweats. Which action is most important for the emergency department triage nurse to implement for this client? A. Initiate airborne infection precautions. B. Place a surgical mask on the client. C. Don an isolation gown and latex gloves. D. Start protective (reverse) isolation precautions.

Rationale: A* This client is exhibiting classic symptoms of tuberculosis (TB), and the client is from a high-risk population for TB. Therefore, airborne infection precautions (A), which are indicated for TB, should be used with this client. (B) is used with droplet precautions. There is no evidence that (C or D) would be warranted at this time.

The nurse is caring for a client with a chest tube to water seal drainage that was inserted 10 days ago because of a ruptured bullae and pneumothorax. Which finding should the nurse report to the health care provider before the chest tube is removed? A. Tidaling of water in water seal chamber B. Bilateral muffled breath sounds at bases C. Temperature of 101° F D. Absence of chest tube drainage for 2 days

Rationale: A* Tidaling (rising and falling of water with respirations) in the water seal chamber should be reported to the health care provider before the chest tube is removed (A) to rule out an unresolved pneumothorax or persistent air leak, which is characteristic of a ruptured bullae caused by abnormally wide changes in negative intrathoracic pressure. (B) may indicate hypoventilation from chest tube discomfort and usually improves when the chest tube is removed. (C) usually indicates an infection, which may not be related to the chest tube. (D) is an expected finding.

A client is placed on a mechanical ventilator following a cerebral hemorrhage, and vecuronium bromide (Norcuron), 0.04 mg/kg every 12 hours IV, is prescribed. What is the priority nursing diagnosis for this client? A. Impaired communication related to paralysis of skeletal muscles B. High risk for infection related to increased intracranial pressure C. Potential for injury related to impaired lung expansion D. Social isolation related to inability to communicate

Rationale: A* To increase the client's tolerance of endotracheal intubation and/or mechanical ventilation, a skeletal muscle relaxant such as vecuronium is usually prescribed. (A) is a serious outcome because the client cannot communicate his or her needs. Although this client might also experience (D), it is not a priority when compared with (A). Infection is not related to increased intracranial pressure (B). The respirator will ensure that the lungs are expanded, so (C) is incorrect.

The nurse is conducting an osteoporosis screening clinic at a health fair. What information should the nurse provide to individuals who are at risk for osteoporosis? (Select all that apply.) A. Encourage alcohol and smoking cessation. B. Suggest supplementing diet with vitamin E. C. Promote regular weight-bearing exercises. D. Implement a home safety plan to prevent falls. E. Propose a regular sleep pattern of 8 hours nightly.

Rationale: A,C,D (A, C, and D) are factors that decrease the risk for developing osteoporosis. Vitamin D and calcium are important supplements to aid in the decrease of bone loss (B). Regular sleep patterns are important to overall health but are not identified with a decreasing risk for osteoporosis (E).

A family member was taught to suction a client's tracheostomy prior to the client's discharge from the hospital. Which observation by the nurse indicates that the family member is capable of correctly performing the suctioning technique? A. Turns on the continuous wall suction to −190 mm Hg B. Inserts the catheter until resistance or coughing occurs C. Withdraws the catheter while maintaining suctioning D. Reclears the tracheostomy after suctioning the mouth

Rationale: B (B) indicates correct technique for performing suctioning. Suction pressure should be between −80 and −120 mm Hg, not −190 mm Hg (A). The catheter should be withdrawn 1 to 2 cm at a time with intermittent, not continuous, suction (C). (D) introduces pathogens unnecessarily into the tracheobronchial tree.

A hospitalized client is receiving nasogastric tube feedings via a small-bore tube and a continuous pump infusion. He begins to cough and produces a moderate amount of white sputum. Which action should the nurse take first? A. Auscultate the client's breath sounds. B. Turn off the continuous feeding pump. C. Check placement of the nasogastric tube. D. Measure the amount of residual feeding.

Rationale: B A productive cough may indicate that the feeding has been aspirated. The nurse should first stop the feeding (B) to prevent further aspiration. (A, C, and D) should all be performed before restarting the tube feeding if no evidence of aspiration is present and the tube is in place.

An older male client comes to the outpatient clinic complaining of pain in his left calf. The nurse notices a reddened area on the calf of his right leg that is warm to the touch, and the nurse suspects that the client may have thrombophlebitis. Which additional assessment is most important for the nurse to perform? A. Measure the client's calf circumference. B. Auscultate the client's breath sounds. C. Observe for ecchymosis and petechiae. D. Obtain the client's blood pressure.

Rationale: B All these techniques provide useful assessment data. The most important is to auscultate the client's breath sounds (B) because the client may have a pulmonary embolus secondary to the thrombophlebitis. (A) may provide data that support the nurse's suspicion of thrombophlebitis. (C) is the least helpful assessment because bruising is not a typical finding associated with thrombophlebitis. (D) is always useful in evaluating the client's response to a problem but is of less immediate priority than breath sound auscultation.

A 43-year-old homeless, malnourished female client with a history of alcoholism is transferred to the ICU. She is placed on telemetry, and the rhythm strip shown is obtained. The nurse palpates a heart rate of 160 beats/min, and the client's blood pressure is 90/54 mm Hg. Based on these findings, which IV medication should the nurse administer? A. Amiodarone (Cordarone) B. Magnesium sulfate C. Lidocaine (Xylocaine) D. Procainamide (Pronestyl)

Rationale: B Because the client has chronic alcoholism, she is likely to have hypomagnesemia. (B) is the recommended drug for torsades de pointes, which is a form of polymorphic ventricular tachycardia (VT) usually associated with a prolonged QT interval that occurs with hypomagnesemia. (A and D) increase the QT interval, which can cause the torsades to worsen. (C) is the antiarrhythmic of choice in most cases of drug-induced monomorphic VT, not torsades.

Which instruction should the nurse teach a female client about the prevention of toxic shock syndrome? A. "Get immunization against human papillomavirus (HPV)." B. "Change your tampon frequently." C. "Empty your bladder after intercourse." D. "Obtain a yearly flu vaccination."

Rationale: B Certain strains of Staphylococcus aureus produce a toxin that can enter the bloodstream through the vaginal mucosa. Changing the tampon frequently (B) reduces the exposure to these toxins, which are the primary cause of toxic shock syndrome. (A) helps prevent cervical cancer, not toxic shock syndrome. (C) can lessen the incidence of urinary tract infection. (D) can help prevent some individuals from contracting the flu and pneumonia, but no relationship to toxic shock syndrome has been proven.

A 25-year-old client was admitted yesterday after a motor vehicle collision. Neurodiagnostic studies have shown a basal skull fracture in the middle fossa. Assessment on admission revealed both halo and Battle signs. Which new symptom indicates that the client is likely to be experiencing a common life-threatening complication associated with a basal skull fracture? A. Bilateral jugular venous distention B. Oral temperature of 102° F C. Intermittent focal motor seizures D. Intractable pain in the cervical region

Rationale: B Clients with basilar skull fractures are at high risk for infection of the brain, as indicated by an increased oral temperature (B), because the fracture leaves the meninges open to bacterial invasion. Clients may experience (C and D), but these findings do not pose as great a life-threatening risk as infection. Jugular distention (A) is not a typical complication of basal skull fractures

The nurse is interviewing a client who is taking interferon-alfa-2a (Roferon-A) and ribavirin (Virazole) combination therapy for hepatitis C. The client reports experiencing overwhelming feelings of depression. Which action should the nurse implement first? A. Recommend mental health counseling. B. Review the medication actions and interactions. C. Assess for the client's daily activity level. D. Provide information regarding a support group.

Rationale: B Interferon-alfa-2a and ribavirin combination therapy can cause severe depression (B); therefore, it is most important for the nurse to review the medication effects and report these to the health care provider. (A, C, and D) might be implemented after the physiologic aspects of the situation have been assessed.

A 58-year-old client who has no health problems asks the nurse about receiving the pneumococcal vaccine (Pneumovax). Which statement given by the nurse would offer the client accurate information about this vaccine? A. The vaccine is given annually before the flu season to those older than 50 years. B. The immunization is administered once to older adults or those at risk for illness. C. The vaccine is for all ages and is given primarily to those persons traveling overseas to areas of infection. D. The vaccine will prevent the occurrence of pneumococcal pneumonia for up to 5 years.

Rationale: B It is usually recommended that persons older than 65 years and those with a history of chronic illness should receive the vaccine once in their lifetime (B). Some recommend receiving the vaccine at 50 years of age. The influenza vaccine is given once a year, not Pneumovax (A). Although the vaccine might be given to a person traveling overseas, that is not the main rationale for administering the vaccine (C). The vaccine is usually given once in a lifetime (D), but with immunosuppressed clients or clients with a history of pneumonia, revaccination is sometimes required.

Which change in laboratory values indicates to the nurse that a client with rheumatoid arthritis may be experiencing an adverse effect of methotrexate (Mexate) therapy? A. Increase in rheumatoid factor B. Decrease in hemoglobin level C. Increase in blood glucose level D. Decrease in erythrocyte sedimentation rate (ESR; sed rate)

Rationale: B Methotrexate is an immunosuppressant. A common side effect is bone marrow depression, which would be reflected by a decrease in the hemoglobin level (B). (A) indicates disease progression but is not a side effect of the medication. (C) is not related to methotrexate. (D) indicates that inflammation associated with the disease has diminished.

Which abnormal laboratory finding indicates that a client with diabetes needs further evaluation for diabetic nephropathy? A. Hypokalemia B. Microalbuminuria C. Elevated serum lipid levels D. Ketonuria

Rationale: B Microalbuminuria (B) is the earliest sign of diabetic nephropathy and indicates the need for follow-up evaluation. Hyperkalemia, not (A), is associated with end-stage renal disease caused by diabetic nephropathy. (C) may be elevated in end-stage renal disease. (D) may signal the onset of diabetic ketoacidosis (DKA).

A nurse is assisting an 82-year-old client with ambulation and is concerned that the client may fall. Which area contains the older person's center of gravity? A. Head and neck B. Upper torso C. Bilateral arms D. Feet and legs

Rationale: B Stooped posture results in the upper torso (B) becoming the center of gravity for older persons. The center of gravity for adults is the hips. However, as a person grows older, a stooped posture is common because of changes caused by osteoporosis and normal bone degeneration. Furthermore, the knees, hips, and elbows flex. The head and neck (A) and feet and legs (D) are not the center of gravity in the older adult. Although the arms (C) comprise a part of the upper torso, they do not reflect the best and most complete answer.

The nurse assesses a postoperative client whose skin is cool, pale, and moist. The client is very restless and has scant urine output. Oxygen is being administered at 2 L/min, and a saline lock is in place. Which intervention should the nurse implement first? A. Measure the urine specific gravity. B. Obtain IV fluids for infusion per protocol. C. Prepare for insertion of a central venous catheter. D. Auscultate the client's breath sounds.

Rationale: B The client is at risk for hypovolemic shock because of the postoperative status and is exhibiting early signs of shock. A priority intervention is the initiation of IV fluids (B) to restore tissue perfusion. (A, C, and D) are all important interventions, but are of less priority than (B).

A male client has just undergone a laryngectomy and has a cuffed tracheostomy tube in place. When initiating bolus tube feedings postoperatively, when should the nurse inflate the cuff? A. Immediately after feeding B. Just prior to tube feeding C. Continuous inflation is required D. Inflation is not required

Rationale: B The cuff should be inflated before the feeding to block the trachea and prevent food from entering (B) if oral feedings are started while a cuffed tracheostomy tube is in place. It should remain inflated throughout the feeding to prevent aspiration of food into the respiratory system. (A and D) place the client at risk for aspiration. (C) places the client at risk for tracheal wall necrosis.

A female client with a nasogastric tube attached to low suction states that she is nauseated. The nurse assesses that there has been no drainage through the nasogastric tube in the last 2 hours. Which action should the nurse take first? A. Irrigate the nasogastric tube with sterile normal saline. B. Reposition the client on her side. C. Advance the nasogastric tube 5 cm. D. Administer an intravenous antiemetic as prescribed.

Rationale: B The immediate priority is to determine if the tube is functioning correctly, which would then relieve the client's nausea. The least invasive intervention, repositioning the client (B), should be attempted first, followed by (A and C), unless either of these interventions is contraindicated. If these measures are unsuccessful, the client may require (D).

When assigning clients on a medical-surgical floor to an RN and a PN, it is best for the charge nurse to assign which client to the PN? A. A young adult with bacterial meningitis with recent seizures B. An older adult client with pneumonia and viral meningitis C. A female client in isolation with meningococcal meningitis D. A male client 1 day postoperative after drainage of a brain abscess

Rationale: B The most stable client is (B). (A, C, and D) are all at high risk for increased intracranial pressure and require the expertise of the RN for assessment and management of care.

The nurse is planning care for a client with diabetes mellitus who has gangrene of the toes to the midfoot. Which goal should be included in this client's plan of care? A. Restore skin integrity. B. Prevent infection. C. Promote healing. D. Improve nutrition.

Rationale: B The prevention of infection is a priority goal for this client (B). Gangrene is the result of necrosis (tissue death). If infection develops, there is insufficient circulation to fight the infection and the infection can result in osteomyelitis or sepsis. Because tissue death has already occurred, (A and C) are unattainable goals. (D) is important but of less priority than (B).

A client with congestive heart failure and atrial fibrillation develops ventricular ectopy with a pattern of 8 ectopic beats/min. Which action should the nurse take based on this observation? A. Assess for bilateral jugular vein distention. B. Increase oxygen flow via nasal cannula. C. Administer PRN furosemide (Lasix). D. Auscultate for a pleural friction rub.

Rationale: B This client should have the oxygen flow immediately increased to promote oxygenation of the myocardium (B). Ventricular ectopy, characterized by multiple PVCs, is often caused by myocardial ischemia exacerbated by hypokalemia. The nurse would expect the client in congestive heart failure to have some degree of (A), which does not exacerbate the ectopy. (C) could create a more severe hypokalemia, which could increase the ectopy. The client is not exhibiting signs of (D).

Which description of symptoms is characteristic of a client diagnosed with trigeminal neuralgia (tic douloureux)? A. Tinnitus, vertigo, and hearing difficulties B. Sudden, stabbing, severe pain over the lip and chin C. Unilateral facial weakness and paralysis D. Difficulty in chewing, talking, and swallowing

Rationale: B Trigeminal neuralgia is characterized by paroxysms of pain, similar to an electric shock, in the area innervated by one or more branches of the trigeminal nerve (cranial V) (B). (A) would be characteristic of Ménière's syndrome (cranial nerve VIII). (C) would be characteristic of Bell's palsy (cranial nerve VII). (D) would be characteristic of disorders of the hypoglossal (cranial nerve XII).

A client on telemetry has a pattern of uncontrolled atrial fibrillation with a rapid ventricular response. Based on this finding, the nurse anticipates assisting the physician with which treatment? A. Administer lidocaine,75 mg intravenous push. B. Perform synchronized cardioversion. C. Defibrillate the client as soon as possible. D. Administer atropine, 0.4 mg intravenous push

Rationale: B With uncontrolled atrial fibrillation, the treatment of choice is synchronized cardioversion (B) to convert the cardiac rhythm back to normal sinus rhythm. (A) is a medication used for ventricular dysrhythmias. (C) is not for a client with atrial fibrillation; it is reserved for clients with life-threatening dysrhythmias, such as ventricular fibrillation and unstable ventricular tachycardia. (D) is the drug of choice in symptomatic sinus bradycardia, not atrial fibrillation.

A client on telemetry has a pattern of uncontrolled atrial fibrillation with a rapid ventricular response. Based on this finding, the nurse anticipates assisting the physician with which treatment? A. Administer lidocaine,75 mg intravenous push. B. Perform synchronized cardioversion. C. Defibrillate the client as soon as possible. D. Administer atropine, 0.4 mg intravenous push.

Rationale: B With uncontrolled atrial fibrillation, the treatment of choice is synchronized cardioversion (B) to convert the cardiac rhythm back to normal sinus rhythm. (A) is a medication used for ventricular dysrhythmias. (C) is not for a client with atrial fibrillation; it is reserved for clients with life-threatening dysrhythmias, such as ventricular fibrillation and unstable ventricular tachycardia. (D) is the drug of choice in symptomatic sinus bradycardia, not atrial fibrillation.

The nurse is assessing a male client with acute pancreatitis. Which finding requires the most immediate intervention by the nurse? A. The client's amylase level is three times higher than the normal level. B. While the nurse is taking the client's blood pressure, he has a carpal spasm. C. On a 1 to 10 scale, the client tells the nurse that his epigastric pain is at 7. D. The client states that he will continue to drink alcohol after going home.

Rationale: B* A positive Trousseau sign (B) indicates hypocalcemia and always requires further assessment and intervention, regardless of the cause (40% to 75% of those with acute pancreatitis experience hypocalcemia, which can have serious, systemic effects). A key diagnostic finding of pancreatitis is serum amylase and lipase levels that are two to five times higher than the normal value (A). Severe boring pain is an expected symptom for this diagnosis (C), but dealing with the hypocalcemia is a priority over administering an analgesic. Long-term planning and teaching (D) do not have the same immediate importance as a positive Trousseau sign

The nurse is caring for a client with a fractured right elbow. Which assessment finding has the highest priority and requires immediate intervention? A. Ecchymosis over the right elbow area B. Deep unrelenting pain in the right arm C. An edematous right elbow D. The presence of crepitus in the right elbow

Rationale: B* Compartment syndrome is a condition involving increased pressure and constriction of the nerves and vessels within an anatomic compartment, causing pain uncontrolled by opioids, and neurovascular compromise (B). (A) is an expected finding. (C) related to compartment syndrome cannot be seen, and any visible edema is an expected finding related to the injury. (D) is an expected finding.

A client diagnosed with chronic kidney disease (CKD) 2 years ago is regularly treated at a community hemodialysis facility. Before his scheduled dialysis treatment, which electrolyte imbalance should the nurse anticipate? A. Hypophosphatemia B. Hypocalcemia C. Hyponatremia D. Hypokalemia

Rationale: B* Hypocalcemia (B) develops in CKD because of chronic hyperphosphatemia, not (A). Increased phosphate levels cause the peripheral deposition of calcium and resistance to vitamin D absorption needed for calcium absorption. Prior to dialysis, the nurse would expect to find the client hypernatremic and hyperkalemic, not with (C or D).

What is the most important nursing priority for a client who has been admitted for a possible kidney stone? A. Reducing dairy products in the diet B. Straining all urine C. Measuring intake and output D. Increasing fluid intake

Rationale: B* Straining all urine (B) is the most important nursing action to take in this case. Encouraging fluid intake (D) is important for any client who may have a kidney stone, but is even more important to strain all urine. Straining urine will enable the nurse to determine when the kidney stone has been passed and may prevent the need for surgery. (C) is not the highest priority action. (A) is usually not recommended until the stone is obtained and the content of the stone is determined. Even then, dietary restrictions are controversial

The nurse notes that a client who is scheduled for surgery the next morning has an elevated blood urea nitrogen (BUN) level. Which condition is most likely to have contributed to this finding? A. Myocardial infarction 2 months ago B. Anorexia and vomiting for the past 2 days C. Recently diagnosed type 2 diabetes mellitus D. Skeletal traction for a right hip fracture

Rationale: B* The blood urea nitrogen (BUN) level indicates the effectiveness of the kidneys in filtering waste from the blood. Dehydration, which could be caused by vomiting, would cause an increased the BUN level (B). (A) would affect serum enzyme levels, not the BUN level. (C) would primarily affect the blood glucose level; renal failure that could increase the BUN level would be unlikely in a client newly diagnosed with type 2 diabetes. Effects of (D) might affect the complete blood count (CBC) but would not directly increase the BUN level.

A client with chronic asthma is admitted to the PACU complaining of pain at a level of 8 on a 1 to 10 scale, with a blood pressure of 124/78 mm Hg, pulse of 88 beats/min, and respirations of 20 breaths/min. The PACU recovery prescription is, "Morphine, 2 to 4 mg IV push, while in recovery for pain level over 5." Which intervention should the nurse implement? A. Give the medication as prescribed to decrease the client's pain. B. Call the anesthesia provider for a different medication for pain. C. Use nonpharmacologic techniques before giving the medication. D. Reassess the pain level in 30 minutes and medicate if it remains elevated.

Rationale: B* The nurse should call the provider for a different medication (B) because morphine is a histamine-releasing opioid and should be avoided when the client has asthma. (A) is unsafe because it puts the client at risk for an asthma exacerbation. Even if the drug were safe for the client, (C and D) both disregard the prescription and the client's need for pain relief in the immediate postoperative period.

The nurse teaches a client with type 2 diabetes nutritional strategies to decrease obesity. Which food item(s) chosen by the client indicates understanding of the teaching? (Select all that apply.) A. White bread B. Salmon C. Broccoli D. Whole milk E. Banana

Rationale: B,C,E (B, C, and E) provide fresh fruits, lean meats and fish, vegetables, whole grains, and low-fat dairy products. All are recommended by the American Diabetes Association (ADA) and are a part of the My Plate guidelines recommended by the U.S. Department of Agriculture (USDA). Whole milk (D) is high in fat and is not recommended by ADA. White bread is milled, a process that removes the essential nutrients. It should be avoided for weight loss and is a poor choice for the client with diabetes (A).

The home health nurse is assessing a male client being treated for Parkinson's disease with carbidopa-levodopa (Sinemet). The nurse observes that he does not demonstrate any apparent emotion when speaking and rarely blinks. Which intervention should the nurse implement? A. Perform a complete cranial nerve assessment. B. Instruct the client that he may be experiencing medication toxicity. C. Document the presence of these assessment findings. D. Advise the client to seek immediate medical evaluation.

Rationale: C A masklike expression and infrequent blinking are common clinical features of parkinsonism. The nurse should document these expected findings (C). (A and D) are not necessary. Signs of toxicity (B) of levodopa-carbidopa (Sinemet) include dyskinesia, hallucinations, and psychosis.

A client is ready for discharge following the creation of an ileostomy. Which instruction should the nurse include in discharge teaching? A. Replace the stoma appliance every day. B. Use warm tap water to irrigate the ileostomy. C. Change the bag when the seal is broken. D. Measure and record the ileostomy output.

Rationale: C A seal must be maintained to prevent leakage of irritating liquid stool onto the skin (C). (A) is excessive and can cause skin irritation and breakdown. Ileostomies produce liquid fecal drainage, so (B) is not necessary. (D) is not needed.

A postoperative client receives a Schedule II opioid analgesic for pain. Which assessment finding requires the most immediate intervention by the nurse? A. Hypoactive bowel sounds with abdominal distention B. Client reports continued pain of 8 on a 10-point scale C. Respiratory rate of 12 breaths/min, with O2 saturation of 85% D. Client reports nausea after receiving the medication

Rationale: C Administration of a Schedule II opioid analgesic can result in respiratory depression (C), which requires immediate intervention by the nurse to prevent respiratory arrest. (A, B, and D) require action by the nurse but are of less priority than (C).

During the change of shift report, the charge nurse reviews the infusions being received by clients on the oncology unit. The client receiving which infusion should be assessed first? A. Continuous IV infusion of magnesium B. One-time infusion of albumin C. Continuous epidural infusion of morphine D. Intermittent infusion of IV vancomycin

Rationale: C All four of these clients have the potential to have significant complications. The client with the morphine epidural infusion (C) is at highest risk for respiratory depression and should be assessed first. (A) can cause hypotension. The client receiving (B) is at lowest risk for serious complications. Although (D) can cause nephrotoxicity and phlebitis, these problems are not as immediately life threatening as (C).

Client census is often used to determine staffing needs. Which method of obtaining census determination for a particular unit provides the best formula for determining long-range staffing patterns? A. Midnight census B. Oncoming shift census C. Average daily census D. Hourly census

Rationale: C An average daily census (C) is determined by trend data and takes into account seasonal and daily fluctuations, so it is the best method for determining staffing needs. (A) and (B) provide data at a certain point in time and that data could change quickly. It is unrealistic to expect to obtain an hourly census (D), and such data would only provide information about a certain point in time.

The nurse is completing an admission interview for a client with Parkinson's disease. Which question will provide additional information about manifestations that the client is likely to experience? A. "Have you ever experienced any paralysis of your arms or legs?" B. "Do you have frequent blackout spells?" C. "Have you ever been frozen in one spot, unable to move?" D. "Do you have headaches, especially ones with throbbing pain?"

Rationale: C Clients with Parkinson's disease frequently experience difficulty in initiating, maintaining, and performing motor activities. They may even experience being rooted to the spot and unable to move (C). Parkinson's disease does not typically cause (A, B, or D).

During report, the nurse learns that a client with tumor lysis syndrome is receiving an IV infusion containing insulin. Which assessment should the nurse complete first? A. Review the client's history for diabetes mellitus. B. Observe the extremity distal to the IV site. C. Monitor the client's serum potassium and blood glucose levels. D. Evaluate the client's oxygen saturation and breath sounds.

Rationale: C Clients with tumor lysis syndrome may experience hyperkalemia, requiring the addition of insulin to the IV solution to reduce the serum potassium level. It is most important for the nurse to monitor the client's serum potassium and blood glucose levels to ensure that they are not at dangerous levels (C). (A, B, and D) provide valuable assessment data but are of less priority than (C)

Which data would the nurse expect to find when reviewing laboratory values of an 80-year-old man who is in good health overall? A. Complete blood count reveals increased white blood cell (WBC) and decreased red blood cell (RBC) counts. B. Chemistries reveal an increased serum bilirubin level with slightly increased liver enzyme levels. C. Urinalysis reveals slight protein in the urine and bacteriuria, with pyuria. D. Serum electrolytes reveal a decreased sodium level and increased potassium level.

Rationale: C In older adults, the protein found in urine slightly rises, probably as a result of kidney changes or subclinical urinary tract infections, and clients frequently experience asymptomatic bacteriuria and pyuria as a result of incomplete bladder emptying (C). Laboratory findings in (A, B, and D) are not considered to be normal findings in an older adult.

When educating a client after a total laryngectomy, which instruction would be most important for the nurse to include in the discharge teaching? A. Recommend that the client carry suction equipment at all times. B. Instruct the client to have writing materials with him at all times. C. Tell the client to carry a medical alert card that explains his condition. D. Caution the client not to travel outside the United States alone.

Rationale: C Neck breathers carry a medical alert card (C) that notifies health care personnel of the need to use mouth to stoma breathing in the event of a cardiac arrest in this client. Mouth to mouth resuscitation will not establish a patent airway. (A and D) are not necessary. There are many alternative means of communication for clients who have had a laryngectomy; dependence on writing messages (B) is probably the least effective.

A client in the emergency department is bleeding profusely from a gunshot wound to the abdomen. In what position should the nurse immediately place the client to promote maintenance of the client's blood pressure above a systolic pressure of 90 mm Hg? A. Place the client in a 45-degree Trendelenburg position to promote cerebral blood flow. B. Turn the client prone to place pressure on the abdominal wound to help staunch the bleeding. C. Maintain the client in a supine position to reduce diaphragmatic pressure and visualize the wound. D. Put the client on the right side to apply pressure to the liver and spleen to stop hemorrhaging.

Rationale: C Placing the client in a supine position (C) reduces diaphragmatic pressure, thereby enhancing oxygenation, and allows for visualization of the abdominal wound. (A) compromises diaphragmatic expansion and inhibits pressoreceptor activity. (B) places the client at risk of evisceration of the abdominal wound and increased bleeding. (D) will not stop internal bleeding in the liver and spleen caused by the gunshot wound.

A 55-year-old male client has been admitted to the hospital with a medical diagnosis of chronic obstructive pulmonary disease (COPD). Which risk factor is the most significant in the development of this client's COPD? A. The client's father was diagnosed with COPD in his 50s. B. A close family member contracted tuberculosis last year. C. The client smokes one to two packs of cigarettes per day. D. The client has been 40 pounds overweight for 15 years.

Rationale: C Smoking, considered to be a modifiable risk factor, is the most significant risk factor for the development of COPD (C). The exact mechanism of genetic and hereditary implications (A) for the development of COPD is still under investigation, although exposure to similar predisposing factors (e.g., smoking or inhaling second-hand smoke) may increase the likelihood of COPD incidence among family members. (B and D) do not exceed the risks associated with cigarette smoking in the development of COPD.

When a nurse assesses a client receiving total parenteral nutrition (TPN), which laboratory value is most important for the nurse to monitor regularly? A. Albumin B. Calcium C. Glucose D. Alkaline phosphatase

Rationale: C TPN solutions contain high concentrations of glucose, so the blood glucose level is often monitored as often as q6h because of the risk for hyperglycemia (C). (A) is monitored periodically because an increase in the albumin level, a serum protein, is generally a desired effect of TPN. (B) may be added to TPN solutions, but calcium imbalances are not generally a risk during TPN administration. (D) may be decreased in the client with malnutrition who receives TPN, but abnormal values, reflecting liver or bone disorders, are not a common complication of TPN administration.

What is the correct location for placement of the hands for manual chest compressions during cardiopulmonary resuscitation (CPR) on the adult client? A. Just above the xiphoid process, on the upper third of the sternum B. Below the xiphoid process, midway between the sternum and the umbilicus C. Just above the xiphoid process, on the lower third of the sternum D. Below the xiphoid process, midway between the sternum and the first rib

Rationale: C The correct placement of the hands for chest compressions in CPR is just above the notch where the ribs meet the sternum on the lower part of the sternum (C). (A) is too high. (B) would not compress the heart. (D) would likely cause damage to both structures, possibly causing a puncture of the heart, and would not render effective compressions.

A 62-year-old woman who lives alone tripped on a rug in her home and fractured her hip. Which predisposing factor most likely contributed to the fracture in the proximal end of her femur? A. Failing eyesight resulting in an unsafe environment B. Renal osteodystrophy resulting from chronic kidney disease (CKD) C. Osteoporosis resulting from declining hormone levels D. Cerebral vessel changes causing transient ischemic attacks

Rationale: C The most common cause of a fractured hip in older women is osteoporosis, resulting from reduced calcium in the bones as a result of hormonal changes in the perimenopausal years (C). (A) may or may not have contributed to the accident, but eye changes were not involved in promoting the hip fracture. (B) is not a common condition of older people but is associated with CKD. Although (D) may result in transient ischemic attacks (TIAs) or stroke, it will not result in fragility of the bones, as does osteoporosis.

The nurse includes frequent oral care in the plan of care for a client scheduled for an esophagogastrostomy for esophageal cancer. This intervention is included in the client's plan of care to address which nursing diagnosis? A. Fluid volume deficit B. Self-care deficit C. Risk for infection D. Impaired nutrition

Rationale: C The primary reason for performing frequent mouth care preoperatively is to reduce the risk of postoperative infection (C) because these clients may be regurgitating retained food particles, blood, or pus from the tumor. Meticulous oral care should be provided several times a day before surgery. Although oral care will be of benefit to the client who may also be experiencing (A, B, or D), these problems are not the primary reason for the provision of frequent oral care.

The nurse includes frequent oral care in the plan of care for a client scheduled for an esophagogastrostomy for esophageal cancer. This intervention is included in the client's plan of care to address which nursing diagnosis? A. Fluid volume deficit B. Self-care deficit C. Risk for infection D. Impaired nutrition

Rationale: C The primary reason for performing frequent mouth care preoperatively is to reduce the risk of postoperative infection (C) because these clients may be regurgitating retained food particles, blood, or pus from the tumor. Meticulous oral care should be provided several times a day before surgery. Although oral care will be of benefit to the client who may also be experiencing (A, B, or D), these problems are not the primary reason for the provision of frequent oral care.

A home health nurse knows that a 70-year-old male client who is convalescing at home following a hip replacement is at risk for developing pressure ulcers. Which physical characteristic of aging puts the client at risk? A. 16% increase in overall body fat B. Reduced melanin production C. Thinning of the skin, with loss of elasticity D. Calcium loss in the bones

Rationale: C Thin nonelastic skin (C) is an important factor in pressure formation. The proportion of body fat to lean mass increases with age (A) and might help decrease ulcer tendency. (B) causes gray hair. (D) can contribute to broken bones, but it is probably not a factor in pressure ulcer formation.

The nurse is caring for a critically ill client with cirrhosis of the liver who has a nasogastric tube draining bright red blood. The nurse notes that the client's serum hemoglobin and hematocrit levels are decreased. Which additional change in laboratory data should the nurse expect? A. Increased serum albumin level B. Decreased serum creatinine C. Decreased serum ammonia level D. Increased liver function test results

Rationale: C* The breakdown of glutamine in the intestine and the increased activity of colonic bacteria from the digestion of proteins increase ammonia levels in clients with advanced liver disease, so removal of blood, a protein source, from the intestine results in a reduced level of ammonia (C). (A, B, and D) will not be significantly affected by the removal of blood.

A 74-year-old male client is admitted to the intensive care unit (ICU) with a diagnosis of respiratory failure secondary to pneumonia. Currently, he is ventilator-dependent, with settings of tidal volume (VT) of 750 mL and an intermittent mandatory ventilation (IMV) rate of 10 breaths/min. Arterial blood gas (ABG) results are as follows: pH, 7.48; Paco2, 30 mm Hg; Pao2, 64 mm Hg; HCO3, 25 mEq/L; and Fio2, 0.80. Which intervention should the nurse implement first? A. Increase the ventilator VT to 850 mL. B. Decrease the ventilator IMV to a rate of 8 breaths/min. C. Reduce the Fio2 to 0.70 and redraw ABGs. D. Add 5 cm positive end-expiratory pressure (PEEP).

Rationale: D Adding PEEP (D) helps improve oxygenation while reducing Fio2 to a less toxic level. (A, B, and C) will not result in improved oxygenation and could cause further complications for this client, who is experiencing respiratory failure.

The nurse is preparing a 45-year-old client for discharge from a cancer center following ileostomy surgery for colon cancer. Which discharge goal should the nurse include in this client's discharge plan? A. Reduce the daily intake of animal fat to 10% of the diet within 6 weeks. B. Exhibit regular, soft-formed stool within 1 month. C. Demonstrate the irrigation procedure correctly within 1 week. D. Attend an ostomy support group within 2 weeks.

Rationale: D Attending a support group (D) will be beneficial to the client and should be encouraged because adaptation to the ostomy can be difficult. This goal is attainable and is measurable. (A) is not specifically related to ileostomy care. The client with an ileostomy will not be able to accomplish (B). (C) is not necessary.

A female client who received a nephrotoxic drug is admitted with acute renal failure and asks the nurse if she will need dialysis for the rest of her life. Which pathophysiologic consequence should the nurse explain that supports the need for temporary dialysis until acute tubular necrosis subsides? A. Azotemia B. Oliguria C. Hyperkalemia D. Nephron obstruction

Rationale: D CKD is characterized by progressive and irreversible destruction of nephrons, frequently caused by hypertension and diabetes mellitus. Nephrotoxins cause acute tubular necrosis, a reversible acute renal failure, which creates renal tubular obstruction from endothelial cells that are sloughed or become edematous. The obstruction of urine flow will resolve (D) with the return of an adequate glomerular filtration rate and, when it does, dialysis will no longer be needed. (A, B, and C) are manifestations seen in the acute and chronic forms of kidney disease.

When developing a discharge teaching plan for a client after the insertion of a permanent pacemaker, the nurse writes a goal of "The client will verbalize symptoms of pacemaker failure." Which behavior indicates that the goal has been met? A. The client demonstrates the procedures to change the rate of the pacemaker using a magnet. B. The client carries a card in his wallet stating the type and serial number of the pacemaker. C. The client tells the nurse that it is important to report redness and tenderness at the insertion site. D. The client states that changes in the pulse and feelings of dizziness are significant changes.

Rationale: D Changes in pulse rate and/or rhythm may indicate pacer failure. Feelings of dizziness may be caused by a decreased heart rate, leading to decreased cardiac output (D). The rate of a pacemaker is not changed by a client, although the client may be familiar with this procedure as explained by his health care provider (A). (B) is an important step in preparing the client for discharge but does not demonstrate knowledge of the symptoms of pacer failure. (C) indicates symptoms of possible incisional infection or irritation but do not indicate pacer failure.

A client who is receiving an angiotensin-converting enzyme (ACE) inhibitor for hypertension calls the clinic and reports the recent onset of a cough to the nurse. Which action should the nurse implement? A. Advise the client to come to the clinic immediately for further assessment. B. Instruct the client to discontinue use of the drug and to make an appointment at the clinic. C. Suggest that the client learn to accept the cough as a side effect to a necessary prescription. D. Encourage the client to keep taking the drug until seen by the health care provider.

Rationale: D Coughing is a common side effect of ACE inhibitors and is not an indication to discontinue the medication (D). Immediate evaluation is not needed (A). Antihypertensive medications should not be stopped abruptly (B) because rebound hypertension may occur. (C) is demeaning because the cough may be very disruptive to the client, and other antihypertensive medications may produce the desired effect without the adverse effect.

A central venous catheter has been inserted via a jugular vein, and a radiograph has confirmed placement of the catheter. A prescription has been received for a medication STAT, but IV fluids have not yet been started. Which action should the nurse take prior to administering the prescribed medication? A. Assess for signs of jugular venous distention. B. Obtain the needed intravenous solution. C. Flush the line with heparinized solution. D. Flush the line with normal saline.

Rationale: D Medication can be administered via a central line without additional IV fluids. The line should first be flushed with a normal saline solution (D) to ensure patency. Insufficient evidence exists on the effectiveness of flushing catheters with heparin (C). (A) will not affect the decision to administer the medication and is not a priority. Administration of the medication STAT is of greater priority than (B).

The nurse is assessing a client who presents with jaundice. Which assessment finding is most important for the nurse to follow up? A. Urine specific gravity of 1.03 B. Frothy, tea-colored urine C. Clay-colored stools D. Elevated serum amylase and lipase levels

Rationale: D Obstructive cholelithiasis and alcoholism are the two major causes of pancreatitis, and elevated serum amylase and lipase levels (D) indicate pancreatic injury. (A) is a normal finding. (B and C) are expected findings related to jaundice.

One day after a Billroth II surgery, a male client suddenly grabs his right chest and becomes pale and diaphoretic. Vital signs are assessed as blood pressure 100/80 mm Hg, pulse 110 beats/min, and respirations 36 breaths/min. Which action is most important for the nurse to take? A. Provide a paper bag for his hyperventilation. B. Administer a prescribed PRN analgesic. C. Have the client drink a glass of sweetened fruit juice. D. Apply oxygen at 2 L via nasal cannula.

Rationale: D Pulmonary embolism and pneumothorax are risks associated with major abdominal surgery. The nurse should immediately provide oxygen while performing further assessment (D). A rapid respiratory rate should not be treated as hyperventilation (A). (B) should not be administered until more ominous causes are ruled out or treated. There is no evidence that the client is hypoglycemic (C).

A client with type 2 diabetes takes metformin (Glucophage) daily. The client is scheduled for major surgery requiring general anesthesia the next day. The nurse anticipates which approach to manage the client's diabetes best while the client is NPO during the perioperative period? A. NPO except for metformin and regular snacks B. NPO except for oral antidiabetic agent C. Novolin N insulin subcutaneously twice daily D. Regular insulin subcutaneously per sliding scale

Rationale: D Regular insulin dosing based on the client's blood glucose levels (sliding scale) is the best method to achieve control of the client's blood glucose while the client is NPO and coping with the major stress of surgery (D). (A) increases the risk of vomiting and aspiration. (B and C) provide less precise control of the blood glucose level.

A client with cirrhosis develops increasing pedal edema and ascites. Which dietary modification is most important for the nurse to teach this client? A. Avoid high-carbohydrate foods. B. Decrease intake of fat-soluble vitamins. C. Decrease caloric intake. D. Restrict salt and fluid intake.

Rationale: D Salt and fluid restrictions are the first dietary modifications for a client who is retaining fluid as manifested by edema and ascites (D). (A, B, and C) will not affect fluid retention.

The nurse is assessing a 75-year-old client for symptoms of hyperglycemia. Which symptom of hyperglycemia is an older adult most likely to exhibit? A. Polyuria B. Polydipsia C. Weight loss D. Infection

Rationale: D Signs and symptoms of hyperglycemia in older adults may include fatigue, infection (D), and evidence of neuropathy (e.g., sensory changes). The nurse needs to remember that classic signs and symptoms of hyperglycemia, such as (A, B, and C) and polyphagia, may be absent in older adults.

A practical nurse (PN) tells the charge nurse in a long-term facility that she does not want to be assigned to one particular resident. She reports that the male client keeps insisting that she is his daughter and begs her to stay in his room. What is the best managerial decision? A. Notify the family that the resident will have to be discharged if his behavior does not improve. B. Notify administration of the PN's insubordination and need for counseling about her statements. C. Ask the PN what she has done to encourage the resident to believe that she is his daughter. D. Reassign the PN until the resident can be assessed more completely for reality orientation.

Rationale: D Temporary reassignment (D) is the best option until the resident can be examined and his medications reviewed. He may have worsening cerebral dysfunction from an infection or electrolyte imbalance. (A) is not the best option because the family cannot control the resident's actions. The administration may need to know about the situation, but not as a case of insubordination (B). Implying that the PN is somehow creating the situation is inappropriate until a further evaluation has been conducted (C).

A practical nurse (PN) tells the charge nurse in a long-term facility that she does not want to be assigned to one particular resident. She reports that the male client keeps insisting that she is his daughter and begs her to stay in his room. What is the best managerial decision? A. Notify the family that the resident will have to be discharged if his behavior does not improve. B. Notify administration of the PN's insubordination and need for counseling about her statements. C. Ask the PN what she has done to encourage the resident to believe that she is his daughter. D. Reassign the PN until the resident can be assessed more completely for reality orientation.

Rationale: D Temporary reassignment (D) is the best option until the resident can be examined and his medications reviewed. He may have worsening cerebral dysfunction from an infection or electrolyte imbalance. (A) is not the best option because the family cannot control the resident's actions. The administration may need to know about the situation, but not as a case of insubordination (B). Implying that the PN is somehow creating the situation is inappropriate until a further evaluation has been conducted (C).

The nurse on a medical surgical unit is receiving a client from the postanesthesia care unit (PACU) with a Penrose drain. Before choosing a room for this client, which information is most important for the nurse to obtain? A. If suctioning will be needed for drainage of the wound B. If the family would prefer a private or semiprivate room C. If the client also has a Hemovac in place D. If the client's wound is infected

Rationale: D The fact that the client has a Penrose drain should alert the nurse to the possibility that the surgical wound is infected (D). Penrose drains provide a sinus tract or opening and are often used to provide drainage of an abscess. To avoid contamination of another postoperative client, it is most important to place any client with an infected wound in a private room. A Penrose drain does not require (A). Although (B) is helpful information, it does not have the priority of (D). A Hemovac (C) is used to drain fluid from a dead space and is not a determinant for the room assignment.

The nurse notes that the client's drainage has decreased from 50 to 5 mL/hr 12 hours after chest tube insertion for hemothorax. What is the best initial action for the nurse to take? A. Document this expected decrease in drainage. B. Clamp the chest tube while assessing for air leaks. C. Milk the tube to remove any excessive blood clot buildup. D. Assess for kinks or dependent loops in the tubing.

Rationale: D The least invasive nursing action should be performed first to determine why the drainage has diminished (D). (A) is completed after assessing for any problems causing the decrease in drainage. (B) is no longer considered standard protocol because the increase in pressure may be harmful to the client. (C) is an appropriate nursing action after the tube has been assessed for kinks or dependent loops.

The nurse notes that the client's drainage has decreased from 50 to 5 mL/hr 12 hours after chest tube insertion for hemothorax. What is the best initial action for the nurse to take? A. Document this expected decrease in drainage. B. Clamp the chest tube while assessing for air leaks. C. Milk the tube to remove any excessive blood clot buildup. D. Assess for kinks or dependent loops in the tubing.

Rationale: D The least invasive nursing action should be performed first to determine why the drainage has diminished (D). (A) is completed after assessing for any problems causing the decrease in drainage. (B) is no longer considered standard protocol because the increase in pressure may be harmful to the client. (C) is an appropriate nursing action after the tube has been assessed for kinks or dependent loops.

The nurse witnesses a baseball player receive a blunt trauma to the back of the head with a softball. What assessment data should the nurse collect immediately? A. Reactivity of deep tendon reflexes, comparing upper with lower extremities B. Vital sign readings, excluding blood pressure if needed equipment is unavailable C. Memory of events that occurred before and after the blow to the head D. Ability to open the eyes spontaneously before any tactile stimuli are given

Rationale: D The level of consciousness (LOC) should be established immediately when a head injury has occurred. Spontaneous eye opening (D) is a simple measure of alertness that indicates that arousal mechanisms are intact. (A) is not the best indicator of LOC. Although (B) is important, vital signs are not the best indicators of LOC and can be evaluated after the client's LOC has been determined. (C) can be assessed after LOC has been established by assessing eye opening.

Which statement reflects the highest priority nursing diagnosis for an older client recently admitted to the hospital for a new-onset cardiac dysrhythmia? A. Diarrhea related to medication side effects B. Anxiety related to fear of recurrent anginal episodes C. Altered nutrition related to high serum lipid levels D. Risk for injury related to syncope and confusion

Rationale: D The loss of cardiac function in aging decreases cardiac output, so dysrhythmias, particularly tachycardias, are poorly tolerated. With onset of a tachycardic or bradycardic dysrhythmia, cardiac output is compromised further, placing the client at risk of syncope and falling, as well as confusion (D). (A) is of high priority but less so than maintaining client safety. Clients may experience (B) as a result of a newly diagnosed cardiac condition, but this nursing diagnosis does not have the priority of (D). (C) also does not have the priority of (D).

An older client is admitted with a diagnosis of bacterial pneumonia. Which symptom should the nurse report to the health care provider after assessing the client? A. Leukocytosis and febrile B. Polycythemia and crackles C. Pharyngitis and sputum production D. Confusion and tachycardia

Rationale: D The onset of pneumonia in the older client may be signaled by general deterioration, confusion, increased heart rate, and/or increased respiratory rate (D). (A, B, and C) are often absent in the older client with bacterial pneumonia.

The nurse is giving preoperative instructions to a 14-year-old client scheduled for surgery to correct a spinal curvature. Which statement by the client best demonstrates that learning has taken place? A. "I will read all the teaching booklets you gave me before surgery." B. "I have had surgery before, so I know what to expect afterward." C. "All the things people have told me will help me take care of my back." D. "Let me show you the method of turning I will use after surgery."

Rationale: D The outcome of learning is best demonstrated when the client not only verbalizes an understanding but can also provide a return demonstration (D). A 14-year-old client may or may not follow through with (A), and there is no measurement of learning. (B) may help the client understand the surgical process, but the type of surgery may have been very different, with differing postoperative care. In (C), the client may be saying what the nurse wants to hear without expressing any real understanding of what to do after surgery.

Which nursing action is necessary for the client with a flail chest? A. Withhold prescribed analgesic medications. B. Percuss the fractured rib area with light taps. C. Avoid implementing pulmonary suctioning. D. Encourage coughing and deep breathing.

Rationale: D Treatment of flail chest is focused on preventing atelectasis and related complications of compromised ventilation by encouraging coughing and deep breathing (D). This condition is typically diagnosed in clients with three or more rib fractures, resulting in paradoxic movement of a segment of the chest wall. (C) should not be avoided because suctioning is necessary to maintain pulmonary toilet in clients who require mechanical ventilation. (A) should not be withheld. (B) should not be applied because the fractures are clearly visible on the chest radiograph.

The nurse assesses a client who has been prescribed furosemide (Lasix) for cardiac disease. Which electrocardiographic change would be a concern for a client taking a diuretic? A. Tall, spiked T waves B. A prolonged QT interval C. A widening QRS complex D. Presence of a U wave

Rationale: D* A U wave (D) is a positive deflection following the T wave and is often present with hypokalemia (low potassium level). (A, B, and C) are all signs of hyperkalemia.

A 77-year-old female client is admitted to the hospital with confusion and anorexia of several days' duration. She has symptoms of nausea and vomiting and is currently complaining of a headache. The client's pulse rate is 43 beats/min. The nurse is most concerned about the client's history related to which medication? A. Warfarin (Coumadin) B. Ibuprofen (Motrin) C. Nitroglycerin (Nitrostat) D. Digoxin (Lanoxin)

Rationale: D* Older persons are particularly susceptible to the buildup of cardiac glycosides, such as digoxin (Lanoxin) or digitoxin (medications derived from digitalis) (D), to a toxic level in their systems. Toxicity can cause anorexia, nausea, vomiting, diarrhea, headache, and fatigue. (A, B, and C) are unlikely to result in the symptoms described.

A client with alcohol-related liver disease is admitted to the unit. Which prescription should the nurse call the health care provider about for reverification for this client? A. Vitamin K1 (AquaMEPHYTON), 5 mg IM daily B. High-calorie, low-sodium diet C. Fluid restriction to 1500 mL/day D. Pentobarbital (Nembutal sodium) at bedtime for rest

Rationale: D* Sedatives such as Nembutal (D) are contraindicated for clients with liver damage and can have dangerous consequences. (A) is often prescribed because the normal clotting mechanism is damaged. (B) is needed to help restore energy to the debilitated client. Sodium is often restricted because of edema. Fluids are restricted (C) to decrease ascites, which often accompanies cirrhosis, particularly in the later stages of the disease.

Signs and symptoms of Colorectal cancer

Rectal bleeding change in bowel habits abdominal pain weight-loss abdominal distention History of polyps family history of cancer

Signs of inhalation burn

Red or burned face, cinched facial and nasal hairs, conjunctivitis, sooty nasal mucosal or bloody sputum, hoarseness, rails or wheezing denoting smoke inhalation

Laryngectomy

Removal of the voice box.

Most common causes of secondary hypertension

Renal Disease

-oid

Resembling

GERD

Result of an incompetent lower esophageal sphincter that allows regurgitation of acidic gastric contents into the esophagus

Type 2 DM

Results from progressive secretory insulin deficit and or defect in insulin uptake

A patient is experiencing difficulty in judgment and spatial perception...making the patient more likely to be impulsive...Which side of the brain is affected here?

Right Hemisphere

For an MCL I, the positive polarity gets placed here:

Right chest nipple line

A patient with a left-brain injury may manifest

Right-sided hemiplegia Speech or language deficits

Manifestations of perforation

Rigid, board-like abdomen Shallow, rapid breathing Tachycardia Sudden, dramatic onset of pain spreading throughout abdomen

Signs and symptoms of acute pancreatitis

Severe mid epigastric pain radiating to back (LUQ) abdominal guarding nausea/vomiting elevated temperature tachycardia decreased BP elevated amylase, lipase and glucose levels

Crohn's disease

Severe, acute chronic inflammation extending throughout all layers of intestinal mucosal. Periods of remission interspersed with periods of exacerbation

Lack of respirations for 10 seconds or more during sleep, due to the loss of pharyngeal tone

Sleep Apnea

Easily transmitted over consolidated tissue

Sound Waves

Pleural Cavity

Space between the membranes around the lungs.

ST-segment depression can occur with what type of angina?

Stable Angina

TSS

Staphlococcus aures produce a toxin that can enter the blood stream through vaginal mucosa. wash hands before and change tampon frequently 4-6 hours

Use only _______ ________ for bladder irrigation after TURP

Sterile Saline

An Addison's Crisis can be triggered by

Stress Withdrawal of Therapy Adrenal Surgery Sudden destruction of pituitary gland

The # 1 cause of a _______ in hypertensive clients is noncompliance with a medication regimen

Stroke

Cardiac output is determined by:

Stroke Volume X Heart Rate

Larynx

Structure in the trachea.

Mucosal healing agent

Sucralfate (carafate)

An emergency referral is indicated when a person experiences:

Sudden Eye Pain

Urinary retention manifests as

Sudden, painful inability to urinate Distended bladder

Amniocentesis

Surgical puncture to remove fluid from the sac around the embryo.

Angioplasty

Surgical repair of blood vessel.

Signs of HYPOglycemia

Sweating Dizziness Trembling Tachycardia

Increased HR is a response to stimulation of which nervous system?

Sympathetic (SNS = fight or flight)

Pacemaker that fires only when the client's HR falls below a rate set on the generator

Synchronous or Demand Pacemaker

Two topical immunosuppressants to help treat atopic dermatitis when other drugs have not worked:

Tacrolimus and Pimecrolimus

Signs & symptoms of hypothyroidism

Thin, dry hair Thick, brittle nails constipation bradycardia Goiter PeriOrbital edema cold intolerance weight gain

Hypernatremia: signs and symptoms

Thirst hyperpyrexia dry mouth hallucinations irritability lethargy Na > 145

Pharynx

Throat.

Treatment of hyperthyroidism

Thyroid ablation, radiation, thyroidectomy, adenectomy

Burns

Tissue injury or necrosis caused by transfer of energy from a heat source to the body

If a cancer patient has extremely low platelets (<20k)

Transfuse platelets

Most common treatment

Transurethral resection of the prostate gland

Action of PPI's

Treatment of erosive esophagitis associated with GERD

Action of antacids

Treatment of peptic ulcer's. Work by neutralizing or reducing acidity of stomach contents. differences in absorption rate

Action of histamine 2 antagonists

Treatment of peptic ulcer's. prophylactic treatment for client at risk for developing ulcers.

-therapy

Treatment.

Tests for hypocalcemia

Trousseau sign Chvostek sign

Malignant myeloma

Tumor of bone marrow.

Colorectal cancer

Tumors in the colon

Food or beverages high in _________ canalso induce HTN in the patient who has pheochromocytoma

Tyramine

ECG

U wave is positive deflectionfollowing the T wave often present in hypokalemia tall spiked T wave, prolonged QT intervial, widening QRS complex are all signs of hyperkalemia

Peptic ulcer disease

Ulceration that penetrates the mucosal wall of the G.I. tract

ST-segment elevation can occur with what type of angina?

Unstable Angina, Ventricular Aneurysms, Pericarditis

Vitamin supplements for cirrhosis

Vitamin A B complex C K

__________ enhances the absorption of iron by the body.

Vitamin C

Antagonist of coumadin:

Vitamin K

cirrhosis

Vitamin K1 (AquaMephyton) high calorie, low sodium diet sodium restriction w/ edema fluids restricted to decrease ascites late stage = ascites

Corticosteroids may increase

WBCs

These tests are ran when infection is suspected

WBCs Blood Cultures

Nursing Interventions for patients with Prostatitis

Warm Sitz Bath Prostatic Massage

HESI hint

Watch for signs of hyperkalemia: dizziness weakness cardiac irregularities muscle cramps diarrhea nausea

When the pituitary gland releases too much ADH, the following occurs:

Water retention Urine output less than 20mL/hr Hyponatremia - due to dilutation

Physical findings of cirrhosis

Weakness anorexia palpable liver jaundice Fector hepaticus Asterix mental and behavioral changes bruising, erythema Dry skin spider angiomas ascites, peripheral neuropathy Palmer erythema

What does a patient need to know when living with Addison's Disease?

Wear a MEDICAL ALERT band Lifelong hormone replacement Know symptoms associated with under- or overdoses

Signs and symptoms for hyperthyroidism

Weight-loss increased appetite diarrhea heat intolerance tachycardia diaphoresis nervousness Exophthalmos T3 elevated above 220 T4 elevated above 12

Diet for diverticular diseases

Well-balanced, high fiber. Sometimes low residue, bland foods

Hepatitis

Widespread inflammation of liver cells usually caused by virus

_________ is the easiest color for a person with failing vision to see.

Yellow

Clinical manifestations of Jaundice

Yellow skin, sclera or mucous membranes(bilirubin in skin). Dark colored urine (bilirubin in urine). Clay colored stools (absence of bilirubin in stools)

asthma

a chronic inflammatory lung disease that results in airflow obstruction; characterized by recurring episodes of paroxysmal dyspnea, wheezing on expiration and/or inspiration caused by constriction of the bronchi, coughing, and viscous mucoid bronchial secretions.

pneumothorax

a collection of air or gas in the pleural space causing the lung to collapse.

pancreatic insufficiency

a condition characterized by inadequate production and secretion of pancreatic hormones or enzymes.

obstructive sleep apnea

a condition characterized by partial or complete upper airway obstruction during sleep, causing apnea and hypopnea.

chylothorax

a condition marked by lymphatic fluid in the pleural space caused by a leak in the thoracic duct.

O2 toxicity

a condition of oxygen overdosage caused by prolonged exposure to a high levels of oxygen; may inactivate pulmonary surfactant and lead to development of acute respiratory distress syndrome.

compliance

a measure of the ease of expansion of the lungs and thorax.

Laryngeal stridor is

a medical emergency indicating airway obstruction or tetany

esophageal speech

a method of swallowing air, trapping it in the esophagus, and releasing it to create sound.

tension pneumothorax

a pneumothorax with rapid accumulation of air in the pleural space causing severely high intrapleural pressures with resultant tension on the heart and great vessels.

lung abscess

a pus-containing lesion of the lung parenchyma that results in a cavity formed by necrosis of lung tissue.

chemoreceptor

a sensory nerve cell that responds to a change in the chemical composition (PaCO2 and pH) of the fluid around it.

chest physiotherapy

a series of maneuvers including percussion, vibration, and postural drainage designed to promote clearance of excessive respiratory secretions.

alpha 1-antitrypsin

a serum protein produced by the liver normally found in the lungs that inhibits proteolytic enzymes of white cells from lysing lung tissue; genetic deficiency of this protein can cause emphysema.

status asthmaticus

a severe, life-threatening asthma attack that is refractory to usual treatment and places the patient at risk for developing respiratory failure.

trigger

a substance, object, or agent that initiates or stimulates an action; in asthma, any stimuli that initiates the IgE

tracheotomy

a surgical incision into the trachea for the purpose of establishing an airway; performed below a blockage by a foreign body, tumor, or edema of the glottis.

pursed-lip breathing

a technique of exhaling against pursed lips to prolong exhalation, preventing bronchiolar collapse and air trapping; done to increase expiratory airway pressure, improve oxygenation of the blood, and help prevent early airway closure.

pulmonary embolism

a thromboembolic occlusion of the pulmonary vasculature resulting from thrombi in the venous circulation or right side of the heart and from other sources, such as amniotic fluid, air, fat, bone marrow, and foreign IV material that travel as emboli until lodging in the pulmonary vessels.

It is important to check a thyroidectomy patient for laryngeal edema and make sure to always have

a trach kit at the bedside!

This is critical for people with OA

achieving a healthy weight, to protect the joints

To prevent organ rejection after a transplant, it is most important that a client

adhere to an immunosuppressive therapy regimen

When a cancer patient receiving chemo complains of nausea, the nurse should

administer antiemetics BEFORE therapy

Nursing plans for emergent phase

admission care, monitor hydration status, monitor respiratory functioning, asses for paralytic ileus, wound care, pain management, asses for circulatory compromise, proper nutrition.

metered-dose inhaler

aerosolized drug delivered in a specific amount by activating the inhaler or by inhaling.

To prevent pregnancy, a diaphragm needs to remain

against the cervix for 6 to 8 hours after intercourse

sundowning

agitated behavior in the evening observe for tirdness at the end of the day

neuro function

altered neuro function =

Antacids

aluminum hydroxide & magnesium hydroxide

absorption atelectasis

alveolar collapse that occurs when high concentrations of oxygen are given and oxygen replaces nitrogen in the alveoli; if airway obstruction occurs, the oxygen is absorbed into the bloodstream and the alveoli collapse.

pulmonary edema

an abnormal accumulation of fluid in the alveoli and interstitial spaces of the lungs caused most commonly by heart failure; an acute, life-threatening situation in which the lung alveoli become filled with serous or serosanguineous fluid caused most commonly by heart failure.

pleural effusion

an abnormal accumulation of fluid in the intrapleural spaces of the lungs.

hyperreactivity

an abnormal condition in which responses to stimuli are exaggerated.

emphysema

an abnormal condition of the pulmonary system, characterized by overinflation and destructive changes in alveolar walls.

apnea

an absence of spontaneous respirations.

empyema

an accumulation of purulent exudates in a body cavity, especially the pleural space, as a result of bacterial infection, such as pleurisy or tuberculosis.

pneumonia

an acute inflammation of the lungs, often caused by inhaled pneumococci of the species Streptococcus pneumoniae.

cystic fibrosis

an autosomal recessive, multisystem disease characterized by altered function of the exocrine glands involving primarily the lungs, pancreas, and sweat glands.

A patient that had just undergone cancer-removing surgery in her lower nether-regions will most likely have

an indwelling catheter for 1-2 days due to temporary bladder atony

tuberculosis

an infectious disease caused by Mycobacterium tuberculosis; usually involves the lungs but also occurs in the larynx, kidneys, bones, adrenal glands, lymph nodes, and meninges and can be disseminated throughout the body.

A patient experiencing a-fib would need this type of therapy

anti-coagulation

SCDs

are a DVT preventative treatment, and should not be used if it is confirmed that a DVT exists

If a patient with a PEG tube has increased residual volume, it places him/her at risk for

aspiration

PEG tube placement is checked by

aspiration of stomach contents and measurement of pH

After a mastectomy, a patient should

be in Semi-Fowler's Elevate affected arm on pillow Flex & Extend fingers in recovery Graduate to post-op exercises Have pain managed Avoid dependent position of arm

In the Health Belief Model, the loss of a limb by a family member would

be the strongest event or cue to action that would increase perceived seriousness of the disease

nasal polyps

benign mucous membrane masses that form slowly in response to repeated inflammation of the sinus or nasal mucosa and project into the nasal cavity.

BPH

benign prostatic hyperplasia

nadolol (Corgard)

beta blocker dibilatating anginal pain bypass surgery patients use with diuretic for hypertension use extreme caution with respiratory problems (asthma) and congestive heart failure

Low platelets places a patient at risk for

bleeding

The post-op Cushing Syndrome patient is at increased risk for

bleeding and infection

Cullen's Sign indicates Pancreatitis...What does it look like?

bluish umbilical area

Autosomal recessive disorders occur only with

both genes of a pair that are both abnormal

diaphragmatic breathing

breathing with the use of the diaphragm to achieve maximum inhalation and slow respiratory rate.

Medications used for ARDS

bronchodilators and steroids

If a thyroidectomy patient elicits manifestations of hypocalcemia, it may require a

calcium injection

digitalis (Lanoxin) digoxin

cardiac glycoside can build up toxic levels s/s anexoria, nausea, vomiting, diarrhea, headache, fatigue

Anti-coagulation therapy is indicated for a-fib BEFORE rhythm control therapies to prevent

cardio-embolic events (which result from blood pooling in the fibrillating atria)

It is imperative that total-beck-breathers

carry a medic alert notice

If a female post-op patient (for lwr reproductive system) has not urinated, and it's 8 hours after the surgery,

catheterization is necessary

When a patient is prescribed Flomax, it's important to teach

change positions slowly, such as when standing up

permanent pacemaker

changes in pulse rate in rythem may indicate pacer failure dizziness may be due to decreased heart rate leading to decreased cardiac output; should carry a card in wallet with type and serial number of pacemaker; report redness and tenderness - s/s infection

If you are preforming CPR and you don't see the chest rise after giving a rescue breath, you must

check the patient's head for proper positioning

A patient with hypothyroidism may have elevated

cholesterol and triglycerides

hypomagnesemia

chronic alcholic ie HR 160 BP 90/54 give IV magnesium sulphate prolonged QT intervial

Irrigating the bladder in the post-op period ensures that

clotted blood is removed and drainage of urine is happening

A patient's K+ levels seem low. Before you give potassium supplements it would important to first

connect a patient to continuous cardiac monitoring (what's the sense in administering K+ if you don't have a measurement to guide you first?)

rhonchi

continuous rumbling, snoring, or rattling sounds from obstruction of large airways with secretions; most prominent on expiration; change often evident after coughing or suctioning.

The client with right brain damage will manifest physical impairments on the

contralateral side of the body

COPD

contributing factor=smoking

ACE inhibitor

cough os a common side effect hypertension do not stop abruptly (rebound hypertension may occur)

Nitroglycerin is used to

counteract vasoconstriction and subsequent angina

Difficulty in chewing, talking, and swallowing could indicate damage to

cranial nerve 12, hypoglossal

pleural friction rub

creaking or grating sound from roughened, inflamed surfaces of the pleura rubbing together, evident during inspiration, expiration, or both and no change with coughing; usually uncomfortable, especially on deep inspiration.

When a patient presents with BPH symptoms, a nurse can anticipate the doctor ordering a ___________ blood test to rule out renal insufficiency

creatinine

Laryngectomy

cuff should be inflated only prior to feeding

A coudé is

curved, allowing it to slip past obstructive tissue in patients with BPH

Severe dysfunction of the cerebral cortex may manifest as

decorticate posturing

Propranolol (Inderal), is a beta-adrenergic blocking agent, which would

decrease the HR

chest tube

decreased drainage =assess for kinks or dependant loops -do not clamp off

Hypoglycemia

deficiency of glucose in the bloodstream

If elevating the extremity relieves the pain associated with thrombophlebitis, then

dependent positions would cause the patient great pain

Hypoglossal (12th crainal nerve)

difficulty chewing, talking and swallowing

With Parkinson's, a client may frequently experience

difficulty initiating, maintaining, and performing motor activities Feeling rooted to one spot, unable to move

Elderly persons are intolerant of decreased CO, which may result in

dizziness and falls

dry powder inhaler

dry powdered drug delivered by inhalation.

vecuronium (Norcuron) is used

during intubation, as a skeletal-muscle relaxant/paralysis

Most ischemic strokes occur

during sleep, when baseline BP declines or blood viscosity increases due to minimal fluid intake

pulmonary hypertension

elevated pulmonary pressure resulting from an increase in pulmonary vascular resistance to blood flow through small arteries and arterioles.

adventitious sounds

extra breath sounds that are not normally heard, such as crackles, rhonchi, wheezes, and pleural friction rubs.

This may help MS exacerbations

extra rest periods

A patient comes in with a diagnosis of hyperthyroidism. When you look into the patient's eyes you notice:

eyelid retraction and infrequent blinking

Exacerbations of MS symptoms occur most commonly as a result of

fatigue and stress

Pacemaker failure could manifest as

feelings of dizziness

It's not uncommon for a patient with hyperthyroidism to experience

fine tremors of hands, tongue, and eyelids

The BP of a post-op Cushing patient often

fluctuates, which makes it very important to keep the patient on BR until it stabilizes

Post-Op female reproductive cancer patient (not breast); Within the first 8 hours of surgery, it is imporant to

frequently observe for signs of bleeding, especially on the perineal pad

olpthalmoscopic exam

from a distance of 8-12 inches and slightly to the side, shine the light in the clients pupil; client should focus on distant object to promote dialation, olpthalmoscopic set at 0 lens to begin, should be held in front of the examiners left eye when examining clients right eye and kept within 1 inch of clients eye for optimum visualation

Clear liquids move up to

full liquids, and then advance as tolerated

Thiazide diuretics, such as _____________, are _____________.

furosemide (Lasix), potassium-wasting

It may be necessary to do this while a hyperthyroidism patient sleeps

gently tape eyes shut because of eyelid retraction

These types of organisms tend to be more resistant to antibiotic therapy

gram-negative

hypercapnia

greater than normal amounts of carbon dioxide in the blood (PaCO2 > 45 mm Hg); also called hypercarbia.

This would be elevated with fluid volume deficit

hematocrit

Large amounts of frank red blood during an irrigation would indicate

hemorrhage, call physician pronto

A daily record in handwriting may provide evidence of the progression or reversal of

hepatic encephalopathy

Elevated AST and ALT labs indicate

hepatotoxicity or damage to liver cells

Tumor lysis syndrome results in

hyperkalemia hypocalcemia hyperuricemia hyperphosphatemia

tumor lysis syndrome

hyperkalemia may occur =requires insulin to reduce serum potassium = monitor serium potassium and blood glucose levels

A client with renal failure that takes Maalox is at risk for

hypermagnesemia

cor pulmonale

hypertrophy of the right side of the heart, with or without heart failure, resulting from pulmonary hypertension.

+ Chvostek and + Trousseau signs indicate

hypocalcemia

Magnesium Sulphate

hypomagnesemia reccomended for torsadesde pointes a form of polymorphic ventrical tackycardia associated with a prolonged QT intervial that occurs with hypomagnesemia

methotrexate (Mexate)

immunosuppressant can cause bone marrow depression rheumatoid arthritis lab=hemaglobin decrease =adverse side effect

The most frequent cause of inadequate aeration of a client's lungs during CPR is

improper positioning of the head

Dobutamine (Dobutrex) is a direct-acting cardiac stimulant, which would

increase the HR

Since a patient with emphysema has hyperinflated lungs, a nurse would find

increased total lung capacity

When the immune system is suppressed, the patient is at risk for

infection

neurogenic bladder

infection - from stasis of urine and subsuquent catheterization

HIV is __________, even if symptoms are not present.

infectious

pleurisy (pleuritis)

inflammation of the pleura.

A thickened area where the skin folds under the breast is an

inframammary ridge, a normal anatomical finding

You suspect a patient is having a transfusion reaction, so you stop the transfusion and then

infuse NS through separate IV tubing

To irrigate a bladder, the nurse would

instill 50mL of fluid, then withdraw with the syringe to remove clots

Brachytherapy uses ________ beams.

internal

After a patient has a surgery for BPH, the nurse may have to

irrigate the bladder

Purulent sputum

is a sign of infection

Albumin should be infused within 4 hours because

it does not contain preservatives to keep it good

CPR

just above the xiphoid process on the lower third of the sternum

A priority intervention for a stroke patient would be

keep HOB elevated

It is not incommon for the BPH-surgical patient to have a weak sphincter after surgery and after the removal of the catheter. To help patient regain strength and control, the nurse should teach

kegel exercises

Aldosteronism

lab =decreased serum level of potassium hypokalemia hypertension

Perforation is common in

large penetrating duodenal ulcers

A client with a ______-sided brain injury would be most likely to be aware of the associated deficits and experience grief related to them

left

In an MCL I, the ground gets placed on the

left chest nipple line

For an MCL I, the negative polarity gets placed here:

left shoulder

Drainage of CBI should be

light pink, without clots

Ileostomy has

liquid and won't smell

hypocapnia

low arterial carbon dioxide pressure (PaCO2 < 35 mm Hg); also called hypocarbia.

Furosemide (Lasix), a loop diuretic, would

lower BP

These are contraindicated in patients with renal failure

magnesium agents, such as Maalox

Multiple Myeloma

malignancy of plasma cells that infiltrate bone causing demineralization and hypercalcemia

Cyclic breast pain coincides with ___________ and the pain is related to

menstrual cycle, hormonal sensitivity

HGB is between 10-12 This indicates ________ anemia.

mild

Symptoms of chronic prostatitis are the same but ________ than the symptoms of acute prostatitis.

milder

HGB is between 6-10. This indicates __________ anemia.

moderate

BSEs should be performed

monthly

IV's infusions potential problems

morphine, continous epidural = respiratory depression magnesium continous infusion = hypotension vancomycin intermittent infusion = nephrotoxicity & phlebitits

osteoporosis

most common cause of fractured hip= reduced calcium in bones result of hormonal changes during perimenaopausal years

CKMB is the most sensitive and reliable indicator of

myocardial damage

gangrene

necrosis/tissue death priority prevent infection

Elevated creatinine indicates

nephron loss, reducing filtration

Early symptoms of renal insufficiencty

nocturia

normocapnia

normal arterial carbon dioxide pressure (PaCO2 35 to 45 mm Hg).

epistaxis

nosebleed

If a female reproductive-cancer-correcting post-surgical patient complains of back aches and has a decreased urinary output, the nurse needs to

notify the physician

If outflow of CBI is blocked and patency cannot be reestablished by manual irrigation, it is imperative that the nurse

notify the physician

In sleep apnea, the client usually is

obese, with a short, thick neck

chronic bronchitis

obstructive pulmonary disease characterized by excessive production of mucus and chronic inflammatory changes in the bronchi, resulting in a cough with expectoration for at least 3 months of the year for more than 2 consecutive years.

TPN total protein nutrition

only regular insulin is adm. IV return containing NPH

The most common cause of fractured hips in elderly women is

osteoporosis, due to reduced Ca in bones, related to hormonal changes

Respiratory alkalosis

pH: high PCO2: low HCO3: normal

Metabolic alkalosis

pH: high PCO2: normal HCO3: high

Respiratory acidosis

pH: low PCO2: high HCO3: normal

hypokalemia

patients on diuretics will change patients normal ECG = U wave is positive deflection following the T wave often present in hypokalemia

If outflow of CBI is blocked, the next step is to

perform manual irrigation to try to reestablish patency

The ______________ period begins about 10 years before menopause.

perimenopausal

small bowel obstruction

peritonis w/ Temperature of 102 notify HCP immediately abdominal cramping

Alopecia associated with radiation therapy is

permanent

Essentially, for an MCL I, the ground can be

placed anywhere, but is usually placed on the lower left portion of the chest

hospital-acquired pneumonia

pneumonia occurring 48 hours or longer after hospital admission and not incubating at the time of hospitalization.

Hearing the patient say 99 very clearly through the stethoscope may indicate

pneumonia, tumor (some sort of consolidation process is happening in the lung tissue)

Primary hyper-aldosteronism would elicit a profound decline in

potassium

vibration

pressing on the chest with the flat of the hands while repeatedly tensing the hand and arm muscles to facilitate movement of secretions to larger airways.

A client with RA would use splints during sleep for

prevention of deformities

CKD chronic kidney disease

prior to hemodialysis lab= hypocalcemia due to hyperphosphatemia, hyperkalemic & hypernatremic

Classic symptom of chronic bronchitis

productive cough with grayish-white sputum, usually occurring in the AM

Vasopressin is used to

promote vasoconstriction in an effort to reduce bleeding

older adults

protein found in urine slightly rises as a result of kidney change or UTI w/asymptomatic bacteriuria and pyuria as a result of incomplete bladder emptying

chronic pancreatitis

pulmonary disease state characterized by the presence of airflow obstruction caused by chronic bronchitis or emphysema; clinical use of the term indicates the presence of chronic bronchitis and/or emphysema; includes asthma, chronic bronchiectasis, chronic bronchitis, and emphysema.

This would be a great concern to the nurse, caring for a patient on corticosteroids

purulent sputum

Chemo affects

rapidly growing cells in the body, both good and bad

mechanical receptors

receptors located in lungs, upper airways, chest wall, and diaphragm that are stimulated by irritants, muscle stretching, and alveolar wall distortion.

It is common to see this in post-op TURP urinary drainage:

reddish-pink color

lactulose (Cephulac)

reduce blood ammonia by excreation of ammonia by stools 2 -3 soft stools per day

cancer

reduce fats increase fruits, vegetables and fiber ie bran flakes, skim milk, orange slices

When the dermal layer of the skin is destroyed, this is lost:

regenerative function of the skin

Hypoglycemic reactions, such as sweating, dizziness, and trembling are those related to the

release of epinephrine as a compensatory response to the low BS in the body

SIADH occurs when the posterior pituitary gland

releases too much ADH

When inspecting someone's eye with an ophthalmoscope, the nurse should

remember to use her opposite eye to look into the patient's eye

Axillary Node Dissection

removal of 12-20 nodes to treat breast cancer

For a client retaining fluid, what is the first dietary modification you would teach?

restrict salt and fluid intake

Cushing Syndrome

results from hypersecreation glucocorticoids in the adrenal cortex often develope diabetes mellitus - monitor serum glucose levels generialized edema low calorie, low carbohydrate, low sodium diet

chest percussion

rhythmic percussion of a patient's chest with cupped hands to loosen retained respiratory secretions.

On a modified chest lead one (MCL I), negative polarity is to left as positive polarity is to _________.

right

jaundice

serium amylase & lipase 2 times higher than normal indicate pancreatic injury frothy tea colored urine clay colored stools complaints of puritis

pancreatitis

serium amylase & lipase 2 to 5 times higher than normal hypercalcemia 40 ~ 75% = positive trousseau sign = carpal spasm severe boring pain

Best indicator of glomerular filtration?

serum creatinine

HGB is less than 6. This indicates ________ anemia.

severe

Pallor, Glossitis, Chest Pain, Cheilitis, Dyspnea at Rest, Tachypnea all indicate

severe anemia

Decorticate posturing indicates

severe dysfunction of the cerebral cortex

The patient with SBO is at risk for

severe electrolyte imbalances

Neutropenia

severely low neutrophils, making patient susceptible to infections

The treatment for Addison's Crisis is directed toward

shock management, high-dose hydrocortisone replacement

crackle

short, low-pitched sounds consisting of discontinuous bubbling caused by air passing through airway intermittently occluded by mucus, unstable bronchial wall, or fold of mucosa; evident on inspiration and, at times, expiration; similar sound to blowing through a straw under water.

dyspnea

shortness of breath; difficulty breathing that may be caused by certain heart conditions, strenuous exercise, or anxiety.

Pap smear

should be continued through menapause to test for vaginal and cervical cancer

vecuronium bromide (Norcuron)

skeletal muscle relaxatant ND: impaired communication R/T paralysis of skeletal muscles

This would be necessary for full-thickness burns

skin-grafting (because the dermal layer has been destroyed, and with it, the regenerative tissue)

seconel

sleep aide 15g=1g

Buerger's disease is strongly related to

smoking

If a Cushing Syndrome patient undergoes surgery to correct, the nurse caring for him post-operatively should be sure to prioritize to

stabilize electrolyte imbalances first

Precautions for HIV

standard

How should a nurse attempt to help a patient relieve a feeling of urgency

start with the least invasive interventions first, then move along

older adults

stooped posture results in upper torso becomming center of gravity

If a urinary drainage catheter in a post-op TURP patient shows minimal output for the last couple of hours, the nurse should

stop the CBI and irrigate the catheter

kidney stone

strain all urine most important encourage urine

Bladder control is a common problem for patients with MS, so if this patient has urinary retention, the nurse should

teach techniques of intermittent self-catheterization

Alopecia associated with chemo is

temporary

When a patient is diagnosed with endometriosis, it's important for the nurse to tell the patient

that it is NOT life-threatening

Eight hours after TURP surgery, a post-op client has reddish pink urinary drainage. This indicates

the CBI is infusing correctly

Dullness heard on percussion below the umbilicus would indicate that

the bladder contains fluid

The best time to draw a trough level is

the closest time to the next administration as possible

The rate of CBI infusion is based on

the color of its drainage

If hypocalcemia is suspected in a thyroidectomy patient, it indicates that

the parathyroid was inadvertently removed during the surgery

The larger the tumor,

the poorer the prognosis

allergic rhinitis

the reaction of the nasal mucosa to a specific allergen.

LLQ pain occurs with diverticulitis because

the sigmoid colon is the most common area for diverticula

rhinoplasty

the surgical reconstruction of the nose.

elastic recoil

the tendency for the lungs to recoil or reduce in volume after being stretched or expanded.

postural drainage

the use of various positions to promote gravity drainage of bronchial secretions; coughing usually expels secretions of the trachea.

meniere syndrome (8th crainal nerve)

tinnitus, vertigo, eharing difficulties

If the cause of PID pelvic pain cannot be identified, the plan of care focuses on

treating the associated pain

After a TURP, it's important to communicate to the patient that he should NOT

try to void around the catheter

Tumor staging designates

tumor size and spread of cancer cells

panlobular emphysema

type of emphysema involving distention and destruction of the entire primary respiratory lobule; usually associated with "1-antitrypsin deficiency; also called chronic hypertrophic, diffuse, generalized, panlobular, or vesicular emphysema.

centrilobular emphysema

type of emphysema often associated with chronic bronchitis in which respiratory bronchioles enlarge, the walls are destroyed, and the bronchioles become confluent; characterized by enlargement of air spaces in the proximal part of the acinus, primarily at the level of the respiratory bronchioles.

Smoking has been associated with __________ formation.

ulcer

Tenesmus is associated specifically with

ulcerative colitis

Bell palsy (7th crainal nerve)

unilateral facial weakness and paralysis

Most breast cancer is found in

upper, outer quadrant

A complication of BPH

urinary retention

When treating anemia, a patient pay require IM iron injections. Because iron has a tendency to stain, the nurse should

use separate needles for withdrawing and injecting

To prevent deformities, a client with RA would be advised to

use splints at nighttime

In a postmenopausal client, this may indicate presence of endometrial cancer

vaginal bleeding

Estrogen deficiency causes

vaginal tissues to become dry and thinner

To reduce risk for hemolytic transfusion reaction in a patient getting 1 unit of PRBCs, the nurse should

verify the blood type and Rh factor with another nurse

fremitus

vibration of the chest wall produced by vocalization.

Drugs like Questran, that treat hyperlipidemia, reduce the absorption of

vitamin K and other fat-soluble vitamins such as: A, D, & E

tidal volume

volume of air exchanged with each breath.

SIADH inappropriate antidiuretic hormone secreation

water retention & dilutional hyponatremia POC=quiet enviroment, deep tendon reflex assessment, neurologic checks, daily weights 1kg=1L

If a patient taking Lasix reports difficulty sleeping, the nurse should determine

when the client is taking the med, since a diuretic will often cause the patient to have to use the bathroom, which could interrupt sleep

angina pectoris

when walking=1.assist to seated position; 2. sublingual nitroglycerin; 3.oxygen 4. wheelchair to room


Conjuntos de estudio relacionados

Chapter 63: Assessment and Management of Patients With Eye and Vision Disorders

View Set

DBA 120: Database Programming I - Chapter 2 (uCerfity)

View Set